You are on page 1of 472

UNIVERSIDAD DE CONCEPCIN FACULTAD DE CIENCIAS FSICAS Y MATEMTICAS Departamento de Fsica de la Atmsfera y del Ocano FSICA: INTRODUCCIN A LA MECNICA Proyecto

de Docencia 02-16 Prof. Juan Inzunza B. Concepcin 2002

2002 FSICA: INTRODUCCIN A LA MECNICA. Universidad de Concepcin Registro Propiedad Intelectual N 127.438 I.S.B.N. 956-8029-35-4 Primera Edicin Ago sto 2002 Impresin: Talleres Direccin de Docencia Edmundo Larenas 64-A Barrio Unive rsitario Concepcin IMPRESO EN CHILE / PRINTED IN CHILE

Fsica: Introduccin a la Mecnica Juan Inzunza B. Licenciado en Fsica, Universidad de Concepcin, Chile Doctor en Ciencias de la Atmsfera, Universidad de Buenos Aires, A rgentina Profesor Asociado, Universidad de Concepcin Barrio Universitario, Concep cin, Chile Julio de 2002.

A mis hijos, Claudia Alicia y Juan Carlos

PROLOGO El ensear es para m la mejor forma de aprender. En la enseanza de la fsica e l objetivo es ensear a pensar y razonar, y para eso se debe estimular el aprendiz aje por problemas, no slo tericos, sino que tambin prcticos. El aprendizaje por prob lemas debe tender a terminar con la clase magistral y fomentar el trabajo en gru po de los alumnos, con el propsito de que aprendan por s solos. Por ello, este cur so de Fsica estar disponible en formato pdf en la pgina web www2.udec.cl/~jinzunza/ fisica, para que los alumnos puedan de forma ms expedita obtener sus documentos p ara estudiar de manera autnoma. Lo ideal sera que la asistencia al aula fuera para aclarar las dudas y resolver los problemas que se le presentan durante su autoe studio. En el mundo globalizado, en la era de las comunicaciones, el estudiante debe estar preparado para aprender a travs de su propio esfuerzo investigativo, s obre la base de problemas que se le plantean y que el alumno debe resolver indiv idual o en grupo. Los alumnos pueden trabajar de forma autnoma y despus relacionar se con el profesor y el resto de sus compaeros, pero para ello deben realizar un trabajo previo de estudio y tener preguntas para plantearlas o respuestas para d arlas a los otros compaeros que preguntan. Esto supone un cambio de mentalidad, u n cambio cultural en los alumnos, donde l debe aprender a aprender, y eso hay que producirlo, no se produce solo. El alumno esta acostumbrado a una forma de trab ajo en la cual viene a la Universidad a or al profesor, donde el primero es el qu e sabe y el otro el que aprende, no a trabajar en forma autnoma. Hay que evitar q ue el alumno pase seis o ms horas diarias, cinco das a la semana, escuchando, y lu ego se va a tratar de aprender a su casa; lo ms sensato sera que dedicara un alto porcentaje de ese tiempo directamente a aprender. Para eso hay que tratar de cen trar la actividad docente en el aprendizaje y no en la enseanza, donde el profeso r debiera preocuparse de que los alumnos retengan lo expuesto y no solamente tra tar de cumplir el programa de estudio. Este texto a nivel bsico de Introduccin a l a Mecnica y Calor, se basa en la experiencia de varios aos de docencia de pregrado y posgrado en el Departamento de Geofsica de la Universidad de Concepcin. En part icular se trat de escribir las clases de la asignatura de Fsica, parte de Mecnica y Calor, la rama de la fsica que se ocupa de describir el movimiento y las transfo rmaciones de energa producidas por variaciones de temperatura, realizadas durante v

los ltimos aos en las aulas. Est diseado para alumnos que realizan un primer curso d e fsica universitaria de las carreras de ciencias bsicas, ingenieras, tecnolgicas, p edagogas y en general para toda carrera que requiera un curso de este nivel. Se p rofundiza la descripcin de algunos fenmenos en particular, con el uso de matemticas de nivel intermedio, como clculo diferencial e integral elemental, pero en todos los casos esta descripcin se puede obviar si el alumno no tiene la formacin en es as herramientas matemticas, sin que sta pierda su validez. La descripcin de los fenm enos fsicos se complementa con figuras esquemticas, en un intento por dar la mayor claridad posible al problema. En el texto se ha pretendido hacer la descripcin n ecesaria, evitando escribir ms de lo que se requiere, para no cansar al alumno co n lectura de prrafos extensos. Cada tema tratado se complementa con ejemplos sele ccionados, resueltos detalladamente, que tienen como objetivo reforzar la compre nsin de la teora. Al final de cada captulo se plantean un nmero considerado suficien te de problemas, muchos de ellos originales preparados por el autor, especialmen te para hacer notar la aplicacin de los contenidos tericos a situaciones reales en diferentes reas de la fsica. La dificultad de su resolucin se puede considerar en general de nivel apropiado a un primer curso de Fsica universitaria, aunque siemp re se presenta alguno de elevada dificultad. Se dan los resultados de un nmero im portante de problemas y aquellos que no tienen respuesta es porque su resultado va a depender de los valores que el alumno asigne a las variables o porque su re solucin es similar a otro problema que ya tiene respuesta, por lo que el alumno s e puede asegurar que su resultado ha sido obtenido por un procedimiento correcto . Los resultados se dan al final de cada problema, evitando as el engorroso proce so de ir a las ltimas pginas del texto a ver la respuesta, generalmente del proble ma impar. Juan C. Inzunza Concepcin, Chile Diciembre de 2006. vi

CONTENIDOS. CAPTULO 1. INTRODUCCIN A LA FSICA. CAPTULO 2. MOVIMIENTO EN UNA DIMENSIN. CAPTULO 3. MOVIMIENTO EN DOS DIMENSIONES. CAPTULO 4. DINAMICA DE LA PARTICULA. CA PTULO 5. TRABAJO Y ENERGIA. CAPTULO 6. TORQUE Y EQUILIBRIO DE CUERPO RIGIDO. CAPTUL O 7. MOMENTO LINEAL Y CHOQUES. CAPTULO 8. DINAMICA DE ROTACIN. CAPTULO 9. LEY DE GR AVITACIN UNIVERSAL. CAPTULO 10. MECANICA ELEMENTAL DE FLUIDOS. CAPTULO 11. MOVIMIEN TO OSCILATORIO. CAPTULO 12. TEMPERATURA, DILATACION TERMICA Y GASES. CAPTULO 13. C ALOR Y LA PRIMERA LEY DE LA TERMODINAMICA. CAPTULO 14. MECANISMOS DE TRANSFERENCI A DE CALOR. CAPTULO 15. SEGUNDA LEY DE LA TERMODINAMICA Y ENTROPIA. vii

viii

INDICE. CAPTULO 1. INTRODUCCIN A LA FSICA. 1.1 Introduccin 1.2 Definiciones 1.3 El mtodo cien tfico 1.4 Sistemas de magnitudes y unidades 1.5 Mltiplos, submltiplos y prefijos 1. 5.1 Orden de magnitud 1.5.2 Estimacin 1.5.3 Transformacin de unidades 1.5.4 Anlisis dimensional 1.6 Sistemas de referencia 1.6.1 Coordenadas cartesianas o rectangu lares 1.6.2 Coordenadas polares 1.7 Conceptos bsicos de vectores 1.7.1 Igualdad d e vectores 1.7.2 Multiplicacin de un vector por un escalar 1.7.3 Vectores especia les 1.7.4 Adicin de vectores y algunas de sus propiedades 1.7.5 Representacin de l os vectores en coordenadas cartesianas 1.7.6 Igualdad de vectores en componentes 1.7.7 Suma, resta y multiplicacin por un escalar 1.7.8 Producto escalar entre ve ctores 1.7.9 Producto vectorial de vectores Problemas CAPTULO 2. MOVIMIENTO EN UN A DIMENSION. 2.1 Definiciones 2.2 Velocidad y aceleracin 2.2.1 Velocidad media 2. 2.2 Velocidad instantnea 2.2.3 Aceleracin media 2.2.4 Aceleracin instantnea 2.3 Desc ripcin del movimiento en una dimensin con aceleracin constante 2.4 Clculo grfico de x y v 2.5 Cuerpos en cada libre 2.5.1 Efectos de g en las personas Problemas CAPTULO 3. MOVIMIENTO EN DOS DIMENSIONES. 3.1 Descripcin del movimiento en dos dimensione s 3.2 Movimiento de proyectiles 3.3 Movimiento circunferencial 3.4 Velocidad y a celeracin angular 3.4.1 Cinemtica de rotacin 3.4.2 Relacin entre las variables angul ares y lineales. 3.5 Movimiento relativo Problemas CAPTULO 4. DINAMICA DE LA PART ICULA. 4.1 Introduccin 4.2 Primera ley de Newton 4.3 Concepto de masa ix 13 13 16 18 19 21 22 24 24 24 25 25 26 28 28 29 29 30 30 31 32 32 33 36 39 39 42 42 43 4 4 44 47 55 59 62 64 75 75 77 84 89 90 91 94 99 105 105 109 110

4.4 Segunda ley de Newton 4.5 Peso 4.6 Tercera ley de Newton 4.7 Fuerza de roce 4.8 Fuerza centrpeta 4.8.1 La descripcin de peralte 4.9 Breve descripcin de aplicac iones de algunas fuerzas en la medicina 4.9.1 Fuerza peso 4.9.2 Fuerza muscular 4.9.3 Fuerza de roce Problemas CAPTULO 5. TRABAJO Y ENERGIA. 5.1 Trabajo realizad o por una fuerza constante 5.2 Trabajo realizado por una fuerza variable 5.3 Ene rga cintica 5.4 Potencia 5.5 Fuerzas conservativas y no conservativas 5.6 Energa po tencial 5.7 Conservacin de la energa mecnica 5.8 Energa y la mquina humana 5.8.1 Cmo mina la mquina humana? 5.8.2 Articulaciones artificiales Problemas CAPTULO 6. TORQ UE Y EQUILIBRIO DE CUERPO RIGIDO. 6.1 Torque de una fuerza 6.2 Equilibrio de un cuerpo rgido 6.2.1 Centro de gravedad 6.2.2 Centro de masa 6.3 Aplicaciones del t orque al cuerpo humano Problemas CAPTULO 7. MOMENTO LINEAL Y CHOQUES. 7.1 Momento lineal 7.2 Impulso 7.3 Conservacin del momento lineal 7.4 Choques 7.4.1 Ejemplos de choques en una dimensin 7.5 Choques en dos dimensiones Problemas CAPTULO 8. DI NAMICA DE ROTACIN. 8.1 Energa cintica de rotacin 8.2 Relacin entre torque y aceleraci angular 8.3 Trabajo, energa y potencia en el movimiento de rotacin 8.4 Movimiento de rodadura de un cuerpo rgido 8.5 Momento angular de una partcula 8.6 Rotacin de un cuerpo rgido en torno a un eje fijo 8.7 Conservacin del momento angular Problem as CAPTULO 9. LEY DE GRAVITACIN UNIVERSAL. 9.1 La Ley y la fuerza gravitacional 9. 2 Fuerza gravitacional y peso x 111 112 114 121 126 128 131 131 131 132 135 143 143 146 149 150 153 154 157 160 162 163 164 171 171 176 177 177 182 186 193 193 194 199 201 202 205 209 215 215 217 222 226 229 231 235 239 247 247 248

9.3 Energa potencial de la fuerza gravitacional 9.3.1 Velocidad de escape 9.4 Las leyes de Kepler 9.4.1 La tercera ley de Kepler 9.4.2 La segunda ley de Kepler y la conservacin del momento angular 9.5 El campo gravitacional Problemas CAPTULO 1 0. NOCIONES DE MECANICA DE FLUIDOS. 10.1 Estructura de la materia 10.1.1 Estados de la materia 10.1.2 Plasma 10.1.3 Fluido 10.2 Densidad 10.3 Presin 10.4 La ecua cin hidrosttica 10.4.1 El barmetro 10.5 Ley de Pascal 10.6 Principio de Arqumedes 10 .7 Nociones elementales de dinmica de fluidos 10.8 Ecuacin de continuidad 10.9 Ecu acin de Bernoulli Problemas CAPTULO 11. MOVIMIENTO OSCILATORIO. 11.1 Movimiento ar mnico simple 11.2 Masa sujeta a un resorte 11.3 Energa en el movimiento armnico sim ple 11.4 El pndulo 11.4.1 Pndulo simple 11.4.2 Pndulo fsico 11.4.3 Pndulo de torsin 1 .5 Oscilaciones amortiguadas 11.6 Oscilaciones forzadas Problemas CAPTULO 12. TEM PERATURA, DILATACION TERMICA Y GASES 12.1 Temperatura y ley cero de la termodinmi ca. 12.2 Termmetros y escalas de temperatura 12.3 Termmetro de gas y escala Kelvin 12.4 Escalas de temperatura Celsius y Fahrenheit 12.5 Dilatacin trmica de slidos y lquidos 12.6 Descripcin macroscpica de un gas ideal 12.7 Teora cintica de los gases 12.8 Interpretacin molecular de la temperatura Problemas CAPTULO 13. CALOR Y LA PR IMERA LEY DE LA TERMODINAMICA 13.1 Definiciones 13.2 Calor 13.3 Capacidad calrica y calor especfico 13.4 Calor latente y cambios de estado 13.4.1 Vaporizacin o eva poracin 13.4.2 Condensacin 13.4.3 Fusin o derretimiento xi 252 255 258 259 261 264 266 271 271 273 273 274 274 276 277 280 281 282 285 285 288 294 299 299 305 308 310 310 313 316 317 319 322 329 329 330 332 337 338 342 346 350 354 363 363 364 366 371 371 373 373

13.4.4 Solidificacin 13.4.5 Sublimacin 13.4.6 Deposicin 13.4.7 Ebullicin 13.5 Trabaj o en procesos termodinmicos 13.6 Primera ley de la termodinmica 13.6.1 Casos parti culares 13.7 Procesos termodinmicos 13.7.1 Proceso isobrico 13.7.2 Proceso isovolu mtrico 13.7.3 Proceso adiabtico 13.7.4 Proceso isotrmico 13.8 Capacidad calrica de u n gas ideal 13.9 Proceso adiabtico de un gas ideal Problemas CAPTULO 14. MECANISMO S DE TRANSFERENCIA DE CALOR 14.1 Calor y temperatura 14.2 Conduccin de calor 14.3 Conveccin 14.4 Radiacin 14.4.1 Espectro de radiacin 14.4.2 Penetracin de la radiacin electromagntica 14.4.3 Leyes de radiacin Problemas CAPTULO 15. SEGUNDA LEY DE LA T ERMODINAMICA Y ENTROPIA. 15.1 Mquinas trmicas 15.1.1 Mquina trmica 15.1.2 Eficiencia trmica 15.2 Segunda Ley de la Termodinmica 15.2.1 Forma de Kelvin-Planck de la se gunda ley de la termodinmica 15.2.2 Enunciado de Clausius de la segunda ley de la termodinmica 15.3 Procesos reversibles e irreversibles 15.4 Mquina de Carnot 15.4 .1 Eficiencia de una mquina de Carnot 15.5 Escala de temperatura absoluta. 15.6 B ombas de calor y refrigeradores 15.7 Entropa 15.7.1 Entropa en un proceso reversib le de un gas ideal 15.7.2 Entropa en la conduccin de calor 15.7.3 Entropa en una ex pansin libre 15.7.4 Entropa en la transferencia de calor irreversible Problemas AP ENDICES A. lgebra B. Geometra C. Trigonometra D. Derivadas e integrales E. Datos co munes en el sistema solar y terrestre F. Factores de conversin de unidades de med ida G. Letras Griegas 373 373 373 373 378 383 384 385 385 386 386 387 389 396 399 407 407 408 412 414 415 417 419 424 429 430 430 431 431 432 433 434 435 439 441 441 443 446 447 448 450 453 461 461 463 465 468 470 471 473 xii

Cap. 1 Introduccin a la Fsica CAPTULO 1. INTRODUCCIN A LA FSICA 1.1 INTRODUCCION. Los adelantos de la ciencia han provocado muchos cambios en el mundo. Por ejemplo, desde Aristteles en el 350 AC y hasta hace 500 aos se crea que la Tierra era plana y que estaba en el centro de l universo, hace 70 aos no se conoca la televisin, los aviones jet ni la forma de p revenir las picaduras dentales, hace pocos aos se descubri la clonacin de seres viv os, recientemente se descifr el cdigo del genoma humano (dicen que Dios esta hecho un diablo por esto). La ciencia no es nueva, data de la prehistoria. El ser hum ano ha estado sobre la Tierra desde hace 100 mil aos y desde entonces ha empezado a hacer ciencia. Por ejemplo en el comienzo se descubrieron las primeras regula ridades y relaciones en la naturaleza. Una de las regularidades era la forma de los patrones de las estrellas que aparecan en el cielo nocturno. Otra evidente er a el ciclo del clima a lo largo del ao, distinguindose claramente el comienzo de l a temporada de lluvias o la de calor. La gente aprendi a usar estos ciclos para h acer predicciones y surgieron los primeros pronsticos del tiempo. De este modo fu eron aprendiendo ms y ms acerca del comportamiento de la naturaleza. Todos estos c onocimientos forman parte de la ciencia, pero la parte principal esta formada po r los mtodos que se usan para adquirir esos conocimientos. La ciencia es una acti vidad humana, formada por un conjunto de conocimientos. La ciencia es el equival ente contemporneo de lo que se llamaba filosofa natural. La filosofa natural era el estudio de las preguntas acerca de la naturaleza que an no tenan respuesta. A med ida que se iban encontrando esas respuestas, pasaban a formar parte de lo que ho y llamamos ciencia. La ciencia hizo sus mayores progresos en el siglo XVI, cuand o se descubri que era posible describir la naturaleza por medio de las matemticas. Cuando se expresan las ideas de la ciencia en trminos matemticos no hay ambigedad, es mas fcil verificarlos o refutarlos por medio del experimento. La ciencia cont empornea se divide en el estudio de los seres vivos y en el estudio de los objeto s sin vida, es decir, en ciencias de la vida y en ciencias fsicas. Las ciencias d e la vida se dividen en reas como la biologa, zoologa y la botnica. Las ciencias fsic as se dividen en reas como la fsica, geologa, astronoma y qumica. 13

Cap. 1 Introduccin a la Fsica La fsica es mas que una rama de las ciencias fsicas: es la ms fundamental de las ci encias. Estudia la naturaleza de realidades bsicas como el movimiento, las fuerza s, energa, materia, calor, sonido, luz y el interior de los tomos. La qumica estudi a la manera en que esta integrada la materia, la manera en que los tomos se combi nan para formar molculas y la manera en que las molculas se combinan para formar l os diversos tipos de materia que nos rodea. La biologa es an mas compleja, pues tr ata de la materia viva. As, tras la biologa esta la qumica y tras la qumica esta la fsica. Las ideas de la fsica se extienden a estas ciencias mas complicadas, por es o la fsica es la mas fundamental de las ciencias. Podemos entender mejor la cienc ia en general si antes entendemos algo de fsica es lo que vamos a prender en este curso! El entender la naturaleza se busca por diferentes formas: la ciencia, el arte, la religin, cuyas orgenes datan de miles de aos. Estas formas son distintas, pero sus dominios se traslapan. La ciencia investiga los fenmenos naturales y el arte es la creacin de los objetos o eventos que estimulan los sentidos, pero amba s son comparables debido a que son esfuerzos que muestran como son las cosas y c uales son posibles. Por otra parte, los objetivos de la ciencia y la religin son diferentes, ya que esta ltima se ocupa del propsito de la naturaleza. Las creencia s y ceremonias religiosas generan convivencia humana, sin ocuparse directamente de los mtodos de la ciencia. En este sentido son diferentes, como las manzanas co n las peras, pero no se contradicen, son complementarias, de manera que no es ne cesario elegir entre ambas, se pueden adoptar ambas, entendiendo que tratan aspe ctos distintos de la experiencia humana. Una persona realmente culta posee conoc imientos tanto de la religin, como del arte y de la ciencia. En este captulo se da una breve explicacin de algunas definiciones de conceptos usados en el curso. Se hace una descripcin de los sistemas de unidades de medida, de las magnitudes fsic as fundamentales y derivadas, se definen los mltiplos, submltiplos y los prefijos. Se hace notar la necesidad de expresar los valores numricos de las magnitudes en ciencias en notacin cientfica, se explica como expresar los valores numricos dando slo su orden de magnitud o haciendo una estimacin de su valor. Se dan reglas de a nlisis dimensional, lo que proporciona un mtodo para determinar la forma funcional de las leyes fsicas y permite verificar si est bien planteada. Se definen los sis temas de referencias y de coordenadas y finalmente se hace un breve repaso del lg ebra vectorial y se presentan algunos ejemplos bsicos. 14

Cap. 1 Introduccin a la Fsica La figura 1.1 tal vez la conozcan: es una imagen de nuestra Tierra, sobre la cua l haremos la mayora de las aplicaciones de este curso. Los colores sobre los ocano s representan los valores de la temperatura de la superficie del mar, siendo may ores los tonos en rojo y menores los tonos en azul. En la imagen se observa clar amente la presencia del fenmeno de El Nio en el Pacifico sur. Se representa tambin un esquema de las nubes en la atmsfera con tonos de color gris claro. En Chile se observa un frente ubicado entre la novena y dcima regiones. Figura 1.1. Imagen de satlite modificada de la Tierra. Este es nuestro planeta, al que le estamos dando un muy mal trato, con todos los desperdicios y contaminantes que estamos arrojando a los ros, lagos, ocanos, tier ra y atmsfera. No olvidemos que los recursos de nuestra Tierra son finitos y no r enovables, por lo que a nosotros nos corresponde cuidar estos recursos, para dej arlos de la mejor forma a las futuras generaciones, que tambin querrn vivir en un ambiente limpio. Las mediciones ya indican que la 15

Cap. 1 Introduccin a la Fsica humanidad est consumiendo los recursos de la Tierra mas rpidamente de lo que esta es capaz de renovarlos, por lo que es clara la tendencia a que los recursos natu rales se agoten. Lo peor de todo es que la distribucin de los recursos no es equi tativa, ya que una minora de empresas y pases mas ricos se enriquecen mas y la may or parte de la poblacin mundial se empobrece mas, incluyendo un importante porcen taje de la poblacin que nada tiene. Lo ms que podemos hacer nosotros como profesio nales y habitantes de la Tierra, es crear conciencia para no seguir daando nuestr o ambiente, que nos permite la vida. Evitemos que el ser humano evolucione rpidam ente a una nueva especie, que se podra llamar Homo Furioso, que al final de este siglo se pregunte en que pensaran esos prehistricos Homo Sapiens de principios de si glo que nos dejaron el planeta en estas lamentables condiciones? 1.2 DEFINICIONES. En esta seccin se dan las definiciones de algunos trminos usados en ciencias y de temas relacionados, que usaremos durante el curso, sin pretend er profundizar en el contenido terico del concepto definido. Fsica: Es una ciencia fundamental que estudia y describe el comportamiento de los fenmenos naturales q ue ocurren en nuestro universo. Es una ciencia basada en observaciones experimen tales y en mediciones. Su objetivo es desarrollar teoras fsicas basadas en leyes f undamentales, que permitan describir el mayor nmero posible de fenmenos naturales con el menor nmero posible de leyes fsicas. Estas leyes fsicas se expresan en lengu aje matemtico, por lo que para entender sin inconvenientes el tratamiento del for malismo terico de los fenmenos fsicos se debe tener una apropiada formacin en matemti cas, en este curso basta un nivel bsico de matemticas. Teora cientfica: Sntesis de un a gran cantidad de informacin que abarca diversas hiptesis probadas y verificables de ciertos aspectos del mundo natural. Ningn experimento resulta aceptable a men os que sea reproducible, es decir que produzca un resultado idntico independiente mente de cuando, donde y por quien sea realizado. Los resultados de los distinto s experimentos se renen para formar una teora. Una teora es la sntesis de todas las observaciones realizadas en los experimentos, que debera hacer posible predecir e l resultado de nuevos experimentos antes de que se realicen. Pero no se debe esp erar que una teora explique ciertos fenmenos de una vez por todas, sino 16

Cap. 1 Introduccin a la Fsica mas bien los coordine dentro de un conjunto sistemtico de conocimientos. La valid ez de una teora puede probarse nicamente con el experimento. Una teora cientfica no debe contener elemento alguno metafsico o mitolgico, se deben eliminar los mitos y prejuicios. Hoy en da se debe tener especial cuidado, puesto que nuestro mitos c ontemporneos gustan de ataviarse con ropajes cientficos, pretendiendo con ello alc anzar gran respetabilidad. Los charlatanes siempre buscan mencionar el nombre de algn gran cientfico en un intento por hacer crebles sus charlataneras. Mecnica. Es u na rama de la fsica. Su objetivo es describir (con la cinemtica) y explicar (con l a dinmica) el movimiento de los cuerpos. Cinemtica. Describe el movimiento de los cuerpos sin preocuparse de las causas que lo producen. Dinmica. Describe el movim iento de los cuerpos considerando las causas que lo producen, y las causas del m ovimiento son las fuerzas. Hiptesis: Suposicin bien fundamentada, considerada como un hecho cuando se demuestra experimentalmente. Hecho: Acuerdo entre observador es competentes sobre una serie de observaciones de un fenmeno particular. Ley: Co mprobacin de una hiptesis sin ninguna contradiccin. Una ley fsica se considera como tal cuando todos los experimentos obedecen esa ley, si en algn caso no se cumple, deja de ser ley fsica. Son las leyes terrestres vlidas en todo el Universo? Hay qu e usarlas y despus evaluar su resultado. No se debe pretender buscar una nueva le y para explicar algn fenmeno en el cual las leyes ya existentes no parecen encajar satisfactoriamente, porque esto conduce al caos lgico. Aunque se debe estar disp uesto a aceptar nuevas leyes naturales si su adopcin demuestra ser necesaria. Cie ncia: Mtodo para dar respuestas a preguntas tericas. La ciencia descubre hechos y formula teoras. Tecnologa: Mtodo para resolver problemas prcticos, usa tcnicas y proc edimientos para aplicar los descubrimientos de la ciencia. 17

Cap. 1 Introduccin a la Fsica Modelo: Concepto introducido por los cientficos para ayudarse a visualizar posibl es procesos dentro de un sistema fsico. Un modelo se usa para representar la real idad fsica y debe tener en cuenta dos aspectos conflictivos entre s: a) tiene que ser lo bastante simple para como para ser elaborado con mtodos matemticamente rigu rosos, b) debe ser realista para que los resultados obtenidos sean aplicables al problema considerado. La sencillez del modelo, su belleza matemtica, es incompat ible con la fidelidad al problema real. Lo bello raramente es fiel y lo fiel rar amente es bello. Matemticas: Es el lenguaje de las ciencias, es lo que establece una conexin entre la teora y el experimento. Las leyes Fsicas se expresan en lengua je matemtico, en general de nivel muy avanzado. Religin: Se ocupa del propsito de l a naturaleza, no se preocupa por usar los mtodos de la ciencia, tiene que ver con la Fe y la adoracin de un ser supremo, que es Dios. Ciencia y religin no son cont radictorias, son complementarias. No es necesario elegir entre ambas, se pueden adoptar las dos. 1.3 EL MTODO CIENTFICO. El mtodo cientfico es un mtodo efectivo para adquirir, organi zar y aplicar nuevos conocimientos. Su principal fundador fue Galileo (1564-1642 ). Se basa en la formulacin de hiptesis y en la recopilacin de pruebas objetivas qu e traten de probar la veracidad de tales hiptesis establecidas previamente. El mto do cientfico puede dividirse a grandes rasgos en varios pasos: a. b. c. d. e. Obs ervar el medio natural. Hacerse una pregunta sobre el comportamiento del medio. Formular una hiptesis y derivar de ella predicciones que puedan ser demostradas. Planear un experimento que pueda verificar esa hiptesis. Analizar los datos obten idos de ese experimento. Si los datos coinciden con las derivaciones de la hiptes is, se podr decir que sta funciona y es vlida en ese contexto. A partir de esa hipte sis demostrada, elaborar una Teora. Nuevamente acudir a la Naturaleza para contra starla. f. g. 18

Cap. 1 Introduccin a la Fsica h. Si la Teora se cumple y demuestra, a partir de ella se formular una Ley, que trata r de describir el fenmeno. Antes de Galileo, la mayor parte de los experimentos no seguan este orden de pens amiento, sino que se basaban en la observacin del medio y emisin de teoras, sin may or comprobacin posterior de stas. La novedad que trajo consigo el mtodo cientfico fu e que se trabajaba con hiptesis que deban ser demostradas. Todo ello supuso un gra n avance para la fsica como ciencia, puesto que se empez a observar la naturaleza y a afirmar expresiones, hoy en da tan comunes como parece que va a llover. Este mto do no siempre ha sido la clave de los descubrimientos, en muchos casos gran part e del progreso de la ciencia se ha debido a resultados obtenidos por error o por casualidad. 1.4 SISTEMAS DE MAGNITUDES Y UNIDADES. Medir una magnitud consiste en compararla con una cantidad arbitraria fija de la magnitud. Una medicin se expresa con un nm ero seguida de un smbolo de la unidad usada. Existen medidas directas e indirecta s, por ejemplo el largo y el ancho de una sala son medidas directas, pero la sup erficie de la sala es una medida indirecta. Gran parte de la Fsica tiene que ver con la medida de cantidades fsicas tales como distancia, tiempo, volumen, masa, t emperatura, etc. Las leyes Fsicas se expresan en trminos de cantidades bsicas que r equieren una definicin clara, llamadas magnitudes fsicas fundamentales. En mecnica las magnitudes fsicas fundamentales son tres: longitud, tiempo y masa. Se llaman magnitudes fsicas fundamentales porque estn definidas en forma independiente de cu alquier otra magnitud fsica. Para que sean tiles deben ser invariables y reproduci bles y se debe definir una unidad de medida nica para la magnitud fsica, llamada p atrn de medida. El Sistema Internacional (SI) de unidades determina el conjunto d e patrones de medida. En este sistema, las unidades de medida de las magnitudes fsicas fundamentales en Mecnica, son las que se dan en la tabla 1.1. Este se conoc e tambin como el sistema MKS (abreviaturas de metro, kilogramo y segundo). Tambin existe el sistema CGS cuyas unidades de medida son el centmetro, gramo y segundo, y el sistema ingls de ingeniera, que es extre19

Cap. 1 Introduccin a la Fsica madamente confuso, por lo que no lo usaremos en este curso. El SI es el que se u sa mayoritariamente en todas las reas de las ciencias. La definicin operacional ac tual de las magnitudes fsicas fundamentales se da a continuacin. Tabla 1.1. Unidades de medida de las magnitudes fsicas fundamentales en mecnica. M agnitud Fsica Unidad de medida Smbolo Longitud Metro m Tiempo Segundo s Masa Kilog ramo kg Longitud: Se han desarrollado muchos sistemas de medicin de longitud, pero se han abandonado por razones de precisin. Desde 1983, la unidad de longitud, el metro, se define como la distancia recorrida por la luz en el vaco durante un tiempo de 1/299792458 segundos. De paso esta definicin establece que la rapidez de la luz en el vaco es de 299 792 458 m/s. Tiempo: En 1967 se defini el segundo como unidad de tiempo igual a 9 192 631 770 periodos de la radiacin de tomos de cesio 133. Co n un reloj atmico de cesio, se puede medir la frecuencia de su radiacin con una pr ecisin de una parte en 1012, lo que equivale a una incertidumbre menor que un seg undo cada 30000 aos. Masa: Desde 1987 se considera como unidad de masa, el kilogr amo, que se define como la masa de una aleacin de platino e iridio que se conserv a en el Laboratorio Internacional de Pesas y Medidas en Sevres, cerca de Pars, Fr ancia. Este patrn es confiable porque dicha aleacin es muy estable. Las otras magn itudes fundamentales de la Fsica, que con las anteriores suman siete en total, es tn indicadas en la tabla 1.2. En ciencias se usan muchas otras magnitudes fsicas, que se obtienen como una combinacin de las magnitudes fsicas fundamentales. Se lla man magnitudes fsicas derivadas, porque se derivan de las magnitudes fsicas fundam entales. Por ejemplo: 20

Cap. 1 Introduccin a la Fsica rea = longitud por longitud, se mide en m2 aceleracin = longitud/tiempo al cuadrad o, se mide en m/s2 fuerza = masa por aceleracin, se mide en Newton, N = kg m/s2 d ensidad = masa/volumen, se mide en kg/m3, etc. Tabla 1.2. Unidades de medida de las magnitudes fsicas fundamentales. Magnitud Fsi ca Unidad de medida Temperatura Kelvin Corriente elctrica Ampere Intensidad lumin osa Candela Cantidad de sustancia Mol Smbolo K A Cd mol 1.5 MULTIPLOS, SUBMULTIPLOS Y PREFIJOS. Teniendo en cuenta que la Fsica estudia e l comportamiento del universo, los valores numricos de las magnitudes fsicas varan en un rango muy amplio, desde cantidades muy pequeas a muy grandes. Por ejemplo, para comprender el origen del Universo, a los astrofsicos y cosmlogos les preocupa actualmente saber que paso entre el Big Bang y el minsculo instante 10-43 s!, o c omo determinar bien la edad del Universo cuyas ltimas mediciones dan un valor de 1.45x1010 aos, con una incertidumbre de un par de miles de millones de aos. La Tie rra tiene una edad de 4600 millones de aos. Especialistas han estudiado la cronol oga de la Biblia para calcular cuanto tiempo ha pasado desde los das del Edn, suman do la edad de Adn y sus descendientes. En 1650 el arzobispo irlands James Ussher p ropuso que Dios creo la Tierra el 22 de octubre del ao 4004 antes de nuestra era, valor que no concuerda con las mediciones. Los valores numricos de la fsica puede n ser muy complicados de leer en su forma tradicional, por lo que generalmente s e expresan en potencias de 10, que es la notacin cientfica. Ejemplos de algunos va lores comunes se muestran en la tabla 1.3. 21

Cap. 1 Introduccin a la Fsica Tabla 1.3. Algunos valores numricos de magnitudes fsicas conocidas. Sol Humano Ele ctrn Distancia Tierra - Sol Cancha de ftbol Dimetro ncleo atmico Edad de la Tierra Ed ad de estudiante UdeC Duracin choque nuclear 2 x 1030 70 9.1 x 10-31 1.5 x 1011 9 0 10-14 1.5 x 1017 5 x 108 10-22 Masa (kg) Longitud (m) Tiempo (s) Si el exponente de la potencia de 10 es positivo (o negativo) el valor de la mag nitud fsica es un mltiplo (o submltiplo). Para medir magnitudes muy grandes o muy p equeas se expresan los valores en potencias de 10 y se usan los prefijos del SI q ue es el nombre que se le da a la potencia de 10. Existen algunas unidades de me dicin que tienen nombres especiales, como por ejemplo el ao luz que es la distanci a que recorre la luz en un ao, igual a 9.45 x 1015 m, o el Angstrom que es igual a 10-10 m. En la tabla 1.4 se dan los nombres de los prefijos del Sistema Intern acional. 1.5.1 Orden de magnitud. El orden de magnitud es la potencia de 10 ms cercana al valor verdadero de una magnitud fsica conocida cuyo valor numrico se conoce. Para indicarla se usa el smbolo vrgula, ~. Cuando se compara entre magnitudes fsicas sim ilares, se dice que una magnitud fsica difiere de la otra en un orden de magnitud , cuando es mayor o menor en un factor de 10. Ejemplo 1.1. El orden de magnitud de 1 es cero 100, el orden de magnitud de 10 es uno 101, el orden de magnitud de 100 es dos 102, etc. Ejemplo 1.2. a) Determinar el orden de magnitud de la masa de la Tierra, cuyo valor es aproximadamente 6 x 1024 kg. b) Si la masa del Sol 1030 kg, en cuantos rdenes de magnitud difiere de la masa de la Tierra? 22

Cap. 1 Introduccin a la Fsica Solucin: a) considerando que 6 es un valor mas cercano a 10 = 101 que a 1 = 100, su orden de magnitud es 6 101, por lo tanto el orden de magnitud de la masa de l a Tierra es 6 x 1024 101x1024 1025 kg 10 Ykg del orden de 25. b) Si la masa del Sol 1030 kg, en cuantos rdenes de magnitud difiere de la masa de la Tierra? Solucin : masa del Sol 10 30 = 25 = 10 5 masa de la Tierra 10 Por lo tanto la masa del Sol es 5 rdenes de magnitud mayor (cien mil veces mas gr ande) que la masa de la Tierra. Tabla 1.4 Prefijos del Sistema Internacional. Potencia 10x -24 -21 -18 -15 -12 9 -6 -3 -2 -1 1 2 3 6 9 12 15 18 21 24 Prefijo yocto zepto atto. femto pico nano micro mili centi deci deca hecto kilo mega giga tera peta exa zeta yota Smbolo y z a f p n m c d da h k M G T P E Z Y 23

Cap. 1 Introduccin a la Fsica 1.5.2 Estimacin. Hacer una estimacin es asignar un valor numrico razonable a una magnitud Fsica cono cida, cuyo valor verdadero, en el momento de usar esa magnitud, no se conoce. Ejemplo 1.3. Estimar la edad de los alumnos del curso de Fsica I. Solucin: Conside rando que los alumnos ingresan a la universidad a la edad aproximada de 18 aos, q ue el curso de Fsica I lo realizan en el segundo semestre, que algunos alumnos in gresan a la carrera tiempo despus de egresar de la enseanza media y que es probabl e que el curso de fsica no lo estn cursando en el semestre que corresponde, se pue de considerar que la edad de los alumnos del curso de Fsica I varia entre 18 y 22 aos, por lo que se puede estimar como edad de cualquier alumno en 20 aos. Su orde n de magnitud es 10 aos. 1.5.3 Transformacin de unidades. Muchos clculos en Fsica requieren convertir unidades de un sistema a otro. Las uni dades pueden convertirse sustituyndolas por cantidades equivalentes. En toda resp uesta numrica de los problemas siempre debe escribirse las unidades en el resulta do final. Ejemplo 1.4. Transformar 18 km/hora a m/s. Solucin: Se sabe que 1h = 3600 s y que 1 km = 1000 m, entonces: 18 1000m km 1hr m =5 hr 3600 s 1km s 1.5.4 Anlisis dimensional. Se usa para verificar que todos los trminos de una ecuacin tengan las mismas dimen siones, lo que garantiza que la ecuacin est planteada en forma 24

Cap. 1 Introduccin a la Fsica correcta. Cuando se hace el anlisis dimensional, los trminos no se operan con el lg ebra corriente, por ejemplo las unidades de medida no se suman o restan, solo se comparan sus unidades entre trminos de la ecuacin a dimensionar, generalmente se usa el smbolo [ ] en cada trmino al hacer el anlisis. Ejemplo 1.5. Hacer el anlisis dimensional para el siguiente modelo fsico v 2 = vo2 + 2ax , donde v se mide en m/s, x en m y a en m/s2. Solucin: se escriben las unidades de medida en cada trmino de la ecuacin, considera ndo que las unidades no se suman ni restan y que el 2 es un nmero sin unidades de medida que no multiplica a la unidad de medida: v 2 = vo2 + 2ax 2 m 2 m 2 m m 2 m = + 2 [m] = 2 = Por lo tanto la expresin es dimensionalmente consistente. 1.6 SISTEMAS DE REFERENCIA. En mecnica se tratan problemas relacionados con la descripcin del movimiento de un objeto en el espacio, por lo que se requiere un mtodo para conocer la posicin de ese objeto. Para esto se definen los sistemas de coordenadas y marcos de referen cia. Un sistema de coordenadas usado para indicar las posiciones en el espacio c onsta de: 1. Un punto de referencia fijo O, llamado origen. 2. Un conjunto de ej es o direcciones con una escala apropiada. 3. Instrucciones sobre como identific ar un punto en el espacio respecto al origen y a los ejes. 1.6.1 Coordenadas cartesianas o rectangulares. Un sistema de coordenadas frecuentemente usado es el sistema de coordenadas cart esiano o rectangular, que se muestra en la figura 1.2, con ejes x sa25

Cap. 1 Introduccin a la Fsica liendo del plano de la figura, eje y horizontal y eje z vertical. En este sistem a un punto P arbitrario se identifica con tres coordenadas identificadas por (x, y,z), con los valores positivos de los ejes hacia fuera del plano de la figura, hacia la derecha y hacia arriba, respectivamente en cada eje, como se indica en la figura 1.2. Es el espacio comn en el que vivimos, se llama espacio tridimensio nal porque tiene tres dimensiones, para indicarlo usamos en smbolo 3D. En ocasion es bastan dos o una coordenadas para fijar la posicin del objeto, estos se llaman espacio bidimensional (2D) o unidimensional (1D), respectivamente. Figura 1.2. Coordenadas cartesianas. 1.6.2 Coordenadas polares. Otro sistema de coordenadas conocido es el de las coordenadas polares (r,) (figur a 1.3), donde r es la distancia desde el origen al punto (x,y), generalmente lla mado radio, y el ngulo entre el eje x y r, por convencin, considerado positivo cua ndo es medido en sentido antihorario desde el eje x hacia r. La relacin entre las coordenadas cartesianas y polares es

26

x = r cos , y = rsen . Se deja como ejercicio al alumno demostrar

ue sus relaciones inversas son:

Cap. 1 Introduccin a la Fsica tan = y , x r = x2 + y2 Figura 1.3. Coordenadas polares. De paso aprovechemos de recordar el teorema de Pitgoras y las funciones trigonomtr icas bsicas seno, coseno y tangente, ue se definen para un tringulo rectngulo, com o el ue se muestra en la figura 1.4, estas son: r 2 = x2 + y2 sen = c teto opuesto y = hipotenus r cos =

t n = c teto opuesto y = c teto dyecente x 27

c teto dy cente x = hipotenus

Figur 1.4. Un tringulo rectngulo. 1.7 CONCEPTOS BSICOS DE VECTORES. L s m gnitudes fsic s con l s que tr t remos en el curso pueden ser esc l res o v ectori les. L s m gnitudes fsic s esc l res qued n complet mente definid s medi n te un nmero y sus respectiv s unid des de medid , por ejemplo l densid d del gu de 1 gr/cm3 o l temper tur del ire de 20 C, son un esc l r. P r l s m gnitu des fsic s vectori les debe especific rse su m gnitud (un nmero con sus unid des), su direccin (un nmero que puede ser un ngulo si el esp cio es bi o tridimension l) y su sentido (que indic h ci donde se dirige o punt el vector), por ejempl o un velocid d de 80 km/h h ci el noreste. Un vector se represent grfic mente como un tr zo dirigido (flech ) y se simboliz medi nte letr s m yscul s o minscul s, con un flech sobre r r l letr o escrit en negrit , como V o V , r o r , OP o OP . L longitud de l flech indic l m gnitud rel tiv del vector, el p unto desde donde se comienz dibuj r el vector se ll m punto de plic cin, l direccin se mide desde lgn eje de referenci , gener lmente horizont l, el sentido est d do por l punt de l flech y l rect sobre l cu l se ubic el vector se ll m lne de ccin. En l figur 1.5, el vector A tiene m gnitud A, su punto de plic cin es O y su direccin es gr dos sobre l horizont l. 1.7.1 Igu ld d de vectores. Dos o ms vectores son igu les si: ) punt n en l mism direccin, b) si sus r r r r m gnitudes son igu les. En l figur 1.6, = b = c = d independientemente de l ubic cin de los vectores en el esp cio. 28

C p. 1 Introduccin

l Fsic

Figur 1.5. Represent cin de un vector. Figur 1.6 Igu ld d de vectores. 1.7.2 Multiplic cin de un vector por un esc l r. El result do de multiplic r un vector por un esc l r es un vector, de magnitud d istinta y de direccin igua (o contraria) a vector origina . En a figura r r r r 1.7 se muestra que B = 2b y D = 2 3 d . Figura 1.7. 1.7.3 Vectores especia es. Vector nu o: es un vector de magnitud igua a cero (0 ). 29

C p. 1 Introduccin

l Fsic

Cap. 1 Introduccin a a Fsica

1.7.4 Adicin de vectores y a gunas de sus propiedades. Los vectores se pueden sumar en forma geomtrica por diversos mtodos, ta es como o s que se muestran en a figura 1.8, a) e mtodo de po gono o b) e mtodo de para e ogramo. Figura 1.8. a) Mtodo de po gono, b) mtodo de para e ogramo. Adems os vectores cump en con as siguientes propiedades de gebra: Conmutatividad de a suma: a + b = a + b. Asociatividad de a suma: a + b + c = (a + b) + c = a + (b + c). Distributividad de a mu tip icacin por un esca ar en a suma de vectores. Conmutatividad de producto: a b = b a , a a = a2. Asociati vidad de producto: a ( b + c) = a b +a c Inverso aditivo: si a + b = 0, entonce s b es e inverso aditivo de a y se escribe b = a. La resta de vectores es un c aso especia de adicin, donde e vector restando se suma con su inverso aditivo: a b = a +( b). La divisin entre vectores no est definida. 1.7.5 Representacin de os vectores en coordenadas cartesianas.

Las componentes vectoria es de un vector son aque as que sumadas dan como resu tado e vector origina . Las componentes vectoria es de un vector en e espacio se ca cu an a o argo de un conjunto de 3 neas mutuamente perpen30

Vector unitario: vector de magnitud igua

a uno (1).

Cap. 1 Introduccin a a Fsica dicu ares que se cortan en un mismo punto, es decir en neas para e as a os ejes de un sistema de coordenadas cartesiano. Los vectores unitarios y as component es vectoria es de vector A en estas direcciones se designan por j i , , k y por Ax, Ay, Az, respectivamente, ta que: r A = Ax + Ay + Az k i j En e p ano (x, y) de a figura 1.9, se tiene: Vector: Componentes: Magnitud: Di reccin: r A = Ax + Ay i j Ax = A cos, Ay = A sen 2 2 A = Ax + Ay t n = Ay/Ax Figur 1.9. Componentes de un vector. 1.7.6 Igu ld d de vectores en componentes. Dos vectores son igu les si tod s sus componentes son igu les, esto es, A = B si Ax = Bx, Ay = By y Az = Bz. 31

1.7.7 Sum , rest y multiplic cin por un esc l r.

+ Bz k i j i j r r A + B = ( Ax + Bx ) + + Ay + Az k Bx + B y + Bz k i j i j j

( ) ( ) ( ) ) ( A = (Ax ) + (Ay )j + (Az )k i r 1.7.8 Producto esca ar entre vectores.

= = k k = 1 i i j j = k = k = 0 i j i j r r A B = Ax Bx + 32

r r A B = AB cos donde A y B es l m gnitud y es el ngulo entre los vectores A y B. Aplic do tores unit rios y l s componentes de un vector, se tiene:

E producto esca ar entre vectores da como resu tado un esca ar, se B, y se define como:

ee A punto

Se oper sobre l s componentes esc l res nlog idimension l se re liz n tres oper ciones esc l como se indic , donde representa un esca ar: r r A + B = Ax + Ay + Az k + Bx + B y + B y ) + ( Az + Bz )k i j r r A B = Ax = ( Ax Bx ) + (Ay B y ) + ( Az Bz )k i

s de los vectores. P r el c so tr res por c d oper cin vectori l, vec

C p. 1 Introduccin

l Fsic

1.7.9 Producto vectori l de vectores. El producto vectori l entre vectores d como result do un vector, se lee A cruz B, y se define como: r r r r C = A B, con C = ABsen donde A y B es l m gnitud y es el ngulo entre los vectores A y B, y l direccin d e C est d d por l regl de l m no derech o del tornillo derecho, C es un ve ctor perpendicul r l pl no form do por A y B. El producto vectori l se c lcul resolviendo el siguiente determin nte:

j Ay By k Az Bz

Solucin: ) en l figur 1.10 se dibuj el di gr m vectori l. 33

Ejemplo 1.6. Un g to se mueve st l posicin P2 en (10,2) en coorden d s c rtesi n s. C gnitud l v ri cin y (d) su

en el pl no (x,y) desde l posicin P1 en (-3,5) m h m. ( ) Dibuj r los vectores de posicin y escribirlos lcul r (b) l v ri cin de l posicin del g to, (c) m direccin.

= k

k = 0 i i j j

Aplic do

i r r A

B = Ax Bx

vectores unit rios, se obtiene que: = k, k = , k = i j j i i j

C p. 1 Introduccin

l Fsic

Figur 1.10. Ejemplo 6. Posiciones: r j r1 = x1 + y1 i r r2 = x2 + y2 i j v r1 = 3 5 i j r r2 = 10 + 2 i j b) La variacin de a posicin es a diferencia entre as posiciones de objeto, r e sto es a posicin fina menos a posicin inicia denotada por r . r r r r = r2 r1 = (10 + 2 ) ( 3 5 ) = 13 + 7 m i j i j i j c) Magnitud: r = ( 13 )2 + ( 7 )2 = 14 ,8 m d) Direccin: tan = r 7 = 28.3 13 Ejemplo 1.7: Una hormiga camina por el borde de un CD de 6 cm de radio, rodeando la mitad del disco. Calcular: (a) la variacin de su posicin, (b) cunto camina?, (c) su variacin de posicin si completa el crculo. Solucin: Usando el sistema de referencia de la figura 1.11, donde i es la posicin inicial, ue se elige en el origen, y f la posicin final. 34

C p. 1 Introduccin

l Fsic

Cap. 1 Introduccin a la Fsica

r r r r r ri = 0 + 0 , rf = 12 + 0 i j i j r r = 12 cm i Figura 1.11. b) Se pide distancia d recorrida desde i hasta f por el borde (por ejemplo el su perior) del disco, si P es el permetro, entonces: d= 1 1 P = 2R = R = 6 cm = 18.8 cm 2 2 se observa que d r r r c) Hay que calcular r despus que la hormiga ha dado una vuelta completa. r r r r = rf ri r r r r rf = ri = 0 r = 0 0 = 0 cm i j 35

a) r = rf ri , de

a figura 11

Ca . 1 Introduccin a la Fsica PROBLEMAS. 1.1 Escribir usando refijos, en unidades del Sistema Internacional: longitud del ec uador, radios del ncleo y tomo, segundos de un milenio, edad de la Tierra, volumen de una ulga, masa del Sol, distancia de la estrella ms cercana a la Tierra (des us del Sol). El Sol es un adulto joven de a enas casi 5 mil millones de aos, escrib a la edad del Sol sin y con refijos del Sistema Internacional. (Cuando el Sol s e a ague, se acabar la fuente de energa que mantiene todos los rocesos sobre la T ierra y or lo tanto la vida sobre ella.) R: 1.57x1017 s. La energa que la Tierra recibe del Sol es del orden de 220 watts/m2, estimar la cantidad de energa sobre toda la su erficie terrestre. Ex resar el resultado con refijos. Estimar la ca ntidad de kilmetros que tu has caminado desde que naciste a la fecha. Estimar el nmero de inos y su valor en esos ara un bosque de inos t ico de la 8 Regin. Si d urante un evento de lluvia en la zona cayeron 25 mm de agua, esto es 25 lt/m2, e stime la cantidad de agua que cay sobre la Baha Conce cin. A cuantas casas se odra a bastecer con agua durante todo un da con esa cantidad? Transformar 10 m/s a km/h, 300000 km/h a m/s, 250 Glt a m3, 1.25 kg/m3 a gr/cm3, 500 hPa a atm, 4500 m2 a cm2. La Tierra tiene una edad de 4600 millones de aos y el ser humano ha estado s obre ella desde hace unos 150 mil aos. Si la edad la Tierra la hacemos equivalent e a un da, cuntos segundos tiene el ser humano sobre la Tierra? Para las ex resione s x = At + Bt 3 y v = A + 3 Bt 2 donde x se mide en m, t en s y v en m/s, determ ine las unidades de medida de A y de B. 1.2 1.3 1.4 1.5 1.6 1.7 1.8 1.9 36

1.13 Si A = 4 + 3 y B = + 5 , calcula su oducto escala , vecto ial y el i j i j ngulo que fo man los vecto es. Dibuja todos los vecto es. 1.14 Pa a los siguientes vecto es: V1 = 2 + 3 , V2 = 3 + 1.5 + 2 k , i j i j V3 = 2.5 7 alcula la magnitud y di eccin de cada vecto . i j 1.15 Pa a los vecto es del oblema 1.14 calcula : a) su suma, b) 3V2 V1, c) 5V3 + V2, d) 2V1 +3V2 0.5V3. Dibuja los vecto es y los esultados. 1.16 Pa a los v ecto es del oblema 1.14, calcula a) el oducto escala ent e cada a de vec to es, f) el oducto vecto ial ent e cada a . 1.17 El vecto F1 tiene una magn itud de 5 unidades y el vecto F2 tiene una magnitud de 10 unidades. Ambos vecto es fo man un ngulo de 120 ent e si. Calcula su oducto escala y vecto ial. 1.18 Demost a que: A B = Ax Bx + Ay B y + Az Bz 1.19 Demost a que: = = k k = 0 i i j j 1.20 Demost a que: j i i j 37

= k ,

y T = 2 l / g son dimensionalmente co ectas, donde x, h y l son longitudes, v y v0 son velocidad (m/s), a y g acele acin (m/s2), T tiem o (s), esin (kg/ms2), y densidad (kg/m3). 1.11 Un vecto de 5 unidades se o ienta en di eccin ositiva del eje x, y ot o de 3 unidades se o ienta en 230. Dete mine la suma y la esta d e estos vecto es, g fica y analticamente. 1.12 El vecto A se extiende desde el o igen hasta un unto que tiene coo denadas ola es (8,60) y el vecto B se extiend e desde el o igen hasta un unto que tiene coo denadas ola es (3,340). Calcula su oducto escala , vecto ial y el ngulo que fo man los vecto es.

Ca . 1 Introduccin a la Fsica 2 1.10 Demuestre que las ecuaciones

+ ( 1 / 2 )v 2 + gh = cte , v 2 = v0 + 2ax

Ca . 2 Movimiento en una dimensin. CAPITULO 2. MOVIMIENTO EN UNA DIMENSION. La cinemtica es la ama de la mecnica que estudia la geomet a del movimiento. Usa las magnitudes fundamentales longitud, e n fo ma de camino eco ido, de osicin y de des lazamiento, con el tiem o como a met o. La magnitud fsica masa no inte viene en esta desc i cin. Adems su gen como magnitudes fsicas de ivadas los conce tos de velocidad y acele acin. Pa a conoce el movimiento del objeto es necesa io hace lo es ecto a un sistema de efe enci a, donde se ubica un obse vado en el o igen del sistema de efe encia, que es q uien hace la desc i cin. Pa a un objeto que se mueve, se ueden distingui al men os t es ti os de movimientos dife entes: t aslacin a lo la go de alguna di eccin v a iable e o definida, otacin del cue o al ededo de algn eje y vib acin. Gene al mente el movimiento de t aslacin en el es acio est acom aado de otacin y de vib acin del cue o, lo que hace que su desc i cin sea muy com leja. Po esto, se conside a un estudio con sim lificaciones y a oximaciones, en el cual se o one un mo delo sim le a a estudia cada movimiento en fo ma se a ada,. La ime a a oxim acin es conside a al cue o como una a tcula, la segunda es conside a slo el mov imiento de t aslacin, una te ce a a oximacin es conside a el movimiento en una s ola di eccin. 2.1 DEFINICIONES. Antes de hace la desc i cin del movimiento, es necesa io defin i algunos conce tos y va iables fsicas que se usa n en este cu so. Cinemtica: desc ibe el movimiento de los cue os en el unive so, sin conside a las causas que lo oducen. Movimiento: es el cambio continuo de la osicin de un objeto en el t anscu so del tiem o. Pa tcula: el conce to intuitivo que tenemos de a tcula co es onde al de un objeto muy equeo que uede tene fo ma, colo , masa, etc., como o ejem lo un g ano de a ena. El conce to fsico abst acto es una idealizacin de un objeto conside ado como un unto matemtico sin dimensiones, que tend slo osicin , masa y movimiento de t aslacin. Esto significa que cualquie 39

Ca . 2 Movimiento en una dimensin. objeto uede se conside ado como a tcula, inde endiente de su tamao, conside and o su masa concent ada en un unto que lo e esenta. Ejem los de objetos que se ueden conside a como una a tcula son un tomo, una ho miga, un avin, la Tie a, e tc., en este ltimo caso se justifica si se estudia su movimiento de t aslacin en t o no al Sol. Posicin: es la ubicacin de un objeto ( a tcula) en el es acio, elativ a a un sistema de efe encia. Es un vecto y se denota o :

(2.1) j donde x, y y z son los valo es de la osicin en cada di eccin, e i , y k son los vecto es unita ios en la di eccin de cada eje x, y y z, es ectivamente. En un a dimensin es sim lemente = xi . Es una de las va iables bsicas del movimiento, junto con el tiem o, en el SI se mide en met os. La osicin se uede dibuja en u n sistema de efe encia en una y dos dimensiones como se muest a en la figu a 2. 1a y 2.1b es ectivamente: Figu a 2.1a: Posicin en una dimensin. Figu a 2.1b: Posicin en dos dimensiones.

40

Des ula ble iga ial

lazamiento: el des lazamiento se define como el cambio de osicin de una a tc en el es acio ( a a indica cambios o dife encias finitas de cualquie va ia en fsica se usa el smbolo delta, ). Es independiente de la trayectoria que se s para cambiar de posicin. Para determinarlo se debe cor r nocer la posicin inic ri y final rf de la partcula en movimiento. E1 des-

= xi + y + zk j

Cap. 2 Movimiento en una dimensin. plazamiento es un vector, que puede ser positivo, negativo o cero, en el SI se m ide en metros; se dibuja en el esquema de la figura 2.2. En una dimensin y en dos dimensiones, el desplazamiento es: r x = ( x f xi )i (2.2) r = rf i = ( x f i + y f ) ( xi i + yi ) j j

Figu a 2.2. Vecto des lazamiento en dos dimensiones. T ayecto ia: es la cu va geomt ica que desc ibe una a tcula en movimiento en el e s acio, y se e esenta o una ecuacin de la t ayecto ia. En una dimensin es una ecta y = cte, a alela al eje x; en dos dimensiones uede se una a bola y = a + bx2 o una ci cunfe encia x2 + y2 = 2 u ot a cu va. Distancia: es la longitud que se ha movido una a tcula a lo la go de una t ayecto ia desde una osicin inic ial a ot a final. Su valo num ico en gene al no coincide con el valo num ico del des lazamiento, exce to en casos muy a ticula es. Tiem o: Qu es el tiem o? No es fcil defini fsicamente el conce to de tiem o. Es ms sim le habla de inte valo de tiem o, que lo odemos defini como la du acin de un evento, o si conside amos l a osicin y sus cambios, odemos deci que el tiem o es lo que ta da una a tcula en move se desde una osicin inicial a ot a final. 41

Ca . 2 Movimiento en una dimensin. 2.2 VELOCIDAD Y ACELERACION. Pa a desc ibi el movimiento debemos defini ot as va iables cinemticas, que son la velocidad y la acele acin. 2.2.1 Velocidad media. Pa a una a tcula que se mueve en di eccin del eje x, desde la osicin inicial xi que en un instante inicial ti se encuent a en el unto P, hasta la osicin final xf que en un instante final tf se encuent a en el unto Q, el des lazamien to de la a tcula en el inte valo de tiem o t = t f ti es x = x f xi . Se elige el sistema de efe encia que se muest a en la figu a 2.3. Se define la com onente x de la velocidad media vmx de la a tcula como el cambio de osicin en un inte valo de tiem o o la ex esin: x x f xi vmx = = t t f ti

(2.3)

De su definicin se obtiene que la unidad de medida de la velocidad media en el SI es el cuociente ent e la unidad de medida de longitud y de tiem o, esto es m/s, que se lee met os o segundo. La velocidad media es inde endiente de la t ayec to ia en el movimiento desde P a Q, es un vecto y uede se ositiva, negativa o ce o, segn el signo o valo del des lazamiento (ya que t > 0 siempre). En una di mensin, si la posicin x aumenta con el tiempo (xf > xi) x r > 0, entonces vmx > 0 , y la partcula se mueve en direccin positiva del eje x, y viceversa si x < 0. 42

Figu a 2.3 Sistema de efe encia en una dimensin

a a defini la velocidad media.

Cap. 2 Movimiento en una dimensin. Una interpretacin geomtrica de la velocidad media se puede ilustrar en un grfico x/ t llamado grfico posicin - tiempo. La recta PQ es la hipotenusa del tringulo de lad os x y t, que se muestra en la figura 2.4. La pendiente de la r recta PQ, que tien e el mismo valor numrico que la vmx , est dada por la tangente del ngulo que form l pendiente con el eje horizont l, cuyo v lor es: t n = x = pendiente t Figura 2.4a Figura 2.4b Notar que el grfico de la figura 2.4 no es un sistema de referencia en dos dimens iones, a pesar de tener dos ejes, ya que el eje horizontal no es de posicin, sino de tiempo.

2.2.2 Velocidad instantnea. Es la velocidad de la partcula en un instante determin ado. Si se considera que el intervalo de tiempo t se puede hacer cada vez ms y ms p equeo, de tal manera que el instante final tf tiende a coincidir con el instante inicial ti, entonces se dice que el intervalo de tiempo tiende a cero, o sea t 0. En el r lmite cuando t 0, r tambin tiende a cero, por lo que la partcula se encuent a en una posicin instantnea. Por lo tanto se puede definir el vector ver locidad i nstantnea v de la siguiente forma: 43

Cap. 2 Movimiento en una dimensin. r r r r dr v = lim = t 0 t dt (2.4) La velocidad instantnea, que llamaremos simplemente velocidad, puede ser positiva (negativa) si la partcula se mueve en direccin positiva (negativa) del eje x, o c ero, en este caso se dice que la partcula est en reposo. La velocidad tiene la mis ma interpretacin geomtrica que la velocidad media y en la figura 2.4b se ilustra e n el grfico x/t una curva de pendiente positiva, que representa una velocidad pos itiva. Rapidez. Se define como rapidez instantnea v a la magnitud o valor numrico del vec tor velocidad, por lo tanto es siempre positiva. 2.2.3 Aceleracin media. Lo normal es que la velocidad de una partcula en movimient o vare en el transcurso del tiempo, entonces se dice que la partcula tiene acelera cin. Se define la aceleracin media am como el cambio de velocidad en un intervalo de tiempo, lo que se escribe como: r r r r v v f vi am = = t t f ti (2.5) La acele acin media es un vecto , su unidad de medida en el SI es el esultado de dividi la unidad de medida de velocidad y de tiem o, esto es (m/s)/s, que se l ee m/s2. 2.2.4 Acele acin instantnea. Es la acele acin a de la a tcula en un instante dete m inado. De mane a anloga a la definicin de la velocidad, se esc ibe: 44

Ca . 2 Movimiento en una dimensin. v dv a = lim = t 0 t dt

(2.6) Como vector, si la aceleracin es positiva (negativa) apunta en direccin positiva ( negativa) del eje x, independientemente de la direccin del movimiento de la partcu la. Puede existir una aceleracin positiva o negativa y la partcula puede estar aum entando su velocidad, y viceversa. En el esquema de la figura 2.5 se muestra par a algunos casos el sentido de la aceleracin para diferentes valores y signos de l a velocidad. Figura 2.5 Esquema de diferentes sentidos de la aceleracin. Si la aceleracin es constante, entonces la rapidez promedio se puede calcular com o el promedio aritmtico entre los distintos valores de rapidez de la forma: 1 (vi + v f ) 2 vm = Una interpretacin geomtrica de la aceleracin se obtiene del grfico rapidez versus ti empo o grfico v/t, donde la pendiente de la curva representa el valor numrico de l a aceleracin, como se ve en la figura 2.6. Si la rapidez, esto es la pendiente de la curva, es positiva (negativa), la aceleracin es positiva (negativa). En el grf ico se observa una curva con pendiente positiva que dismi45

Cap. 2 Movimiento en una dimensin. nuye su valor hasta cero, que representa un movimiento con aceleracin positiva, p ero disminuyendo su valor, luego la pendiente se hace negativa, aumentando negat ivamente su valor y lo mismo ocurre con la aceleracin. tan = v = pendiente = a t Figura 2.6 Grfico rapidez versus tiempo. La aceleracin tambin se puede escribir como:

r r r r dv d dx d 2 x = = a= dt dt dt dt 2 que corresponde a la segunda derivada de la posicin respecto al tiempo. La aceler acin tambin puede variar en el tiempo, pero esa variacin no tiene significado fsico de importancia, por lo que no se le da un nombre en particular. Aunque da/dt pod ra representar o llamarse algo as como sacudn o empujn. Tambin puede existir un d( dt y as hasta el infinito. Ejemplo 2.1: Una partcula se mueve en direccin x > 0 durante 10 s con rapidez cons tante de 18 km/h, luego acelera hasta 25 m/s durante 5 s. Calcular: a) su despla zamiento en los primeros 10 s, b) la aceleracin media en cada intervalo de tiempo , c) la rapidez media del movimiento. 46

Cap. 2 Movimiento en una dimensin. Solucin: Datos t1 = 10 s, vi = 18 km/h = 5 m/s, t2 = 5 s, vf = 25 m/s a) v = x m x = vt = 5 10 s = 50m t s b) para t1: vi = cte => a = 0 para t2: a = vi + v f 2 v (25 5)m / s m = =4 2 t 5s s (5 + 25)m / s m = 15 2 s c) vm = = 2.3 DESCRIPCIN CINEMTICA DEL MOVIMIENTO EN UNA DIMENSIN CON ACELERACIN CONSTANTE. E1 movimiento de una partcula se describe por completo si se conoce su posicin en cualquier instante. Para encontrar leyes que expliquen los diferentes cambios de los cuerpos en el tiempo, se deben registrar los cambios y describirlos. Alguno s cambios son difciles de describir, como por ejemplo los movimientos de una nube , formada por billones de gotitas de agua que se mueven al azar y pueden evapora rse o unirse para formar gotas ms grandes, o bien los cambios de opinin de una muj er. Describir el movimiento significa poder responder a la pregunta en que posicin se encuentra el cuerpo en movimiento en cualquier instante de tiempo? Si r la a celeracin a vara en el tiempo el movimiento puede ser muy complejo y difcil de anal izar. Un caso simple de movimiento es aquel que se realiza en una direccin con ac eleracin constante. Si la aceleracin es constante, entonr r ces la a = am , lo que significa que la velocidad cambia de manera uniforme en todo el movimiento. Con sideremos primero el caso de una partcula que se mueve en direccin del eje x con l a magnitud de la aceleracin a constante. Si v0 es el valor de la velocidad o rapi dez en el instante inicial t0, y v su valor en el instante t, de la definicin de a se tiene: 47

Cap. 2 Movimiento en una dimensin. a= v t t dv dv = adt v dv = t adt = a t dt dt 0 0 o v v0 = a (t t 0 ) r r r v (t ) = v0 + a (t t 0 ) (2.7) La ecuacin 2.7 e mite dete mina la velocidad v = v(t) de una a tcula que se m ueve en una di eccin con acele acin a constante, a a cualquie instante t > t0. C omo v0, a y t0 son valo es conocidos, se obse va que v es una funcin lineal del t iem o t, o lo tanto el g fico a idez ve sus tiem o o g fico v/t es de la fo ma que se muest a en la figu a 2.7a. Pa a a < 0, y a a el caso de una a tcula que est disminuyendo su a idez, los g ficos v/t y a/t se muest an en la figu a 2.7b. Figu a 2.7a. G ficos v/t y a/t, a a a > 0. Figu a 2.7b. G ficos v/t y a/t, a a a < 0. 48

Ca . 2 Movimiento en una dimensin.

v= dx dx = vdt dx = vdt dt Si inicialmente, para t = to, la partcula se encuentra en la posicin xo y en cualq uier instante t se encuentra en la posicin x, la velocidad en funcin del tiempo es v( t ) = v0 + a( t t0 ) , eem lazando en la integ al, con los lmites de integ a cin co es ondientes queda: x x0 t 1 dx = t [v0 + a( t t0 )]dt = v0 ( t t0 ) + a( t t0 )2 2 0 Esc ita en fo ma vecto ial, se obtiene: 1 x x0 = v0 ( t t0 ) + a( t t0 )2 2 Como xo, vo y a son los valo es conocidos a a t = to, se deduce que x es slo fun cin del tiem o, as la ecuacin que desc ibe la osicin de una a tcula en movimiento e n funcin del tiem o x = x(t) es: 1 x = x0 + v0 ( t t0 ) + a( t t0 )2 2

(2.8) 49

El valo de la endiente al valo num ico de la ) es la ecuacin de una elocidad a a obtene la

de la tangente a la cu va v(t) en el g fico v/t es igual acele acin. Pa a el movimiento con acele acin constante v(t ecta. Conocida v = v(t) se uede usa la definicin de la v osicin de la a tcula en cualquie instante.

Ca . 2 Movimiento en una dimensin.

Figura 2.8 Grfico x/t Las ecuaciones x = x(t), v = v(t) y a = cte., forman el conjunto de ecuaciones c inemticas, que ermiten describir el movimiento sim le de una artcula que se muev e con aceleracin constante en una direccin, y como con esas ecuaciones se ueden d eterminar los valores de esas variables ara la artcula en cualquier instante, e l movimiento queda com letamente descrito. Para el caso articular de un movimie nto con ra idez constante, la aceleracin de la artcula es cero, y las ecuaciones del movimiento 2.7 y 2.8 se reducen a: r r r x r r v = Ejem lo stante, 50 = x0 + v0 ( t t0 ) v0 = cte. 2.2: Demostrar que si la aceleracin de una se tiene que v 2 = vo + 2ax .

artcula en movimiento 2 es con

La ecuacin 2.8 es la ex esin que e mite dete mina el valo de a tcula en cualquie instante, conocido los valo es iniciales. El em o es una a bola, ya que la ecuacin x = x(t) es cuad tica en la tangente a la cu va en cualquie instante t e esenta el valo velocidad de la a tcula (figu a 2.8). Esta ecuacin x(t) tambin in de itine a io.

la osicin de la g fico osicin/ti t. La endiente de num ico de la se conoce como ecuac

Cap. 2 Movimiento en una dimensin. Solucin: De v( t ) = vo + a( t to ) , se des eja t t0 = v v0 , a eem lazando en x = x0 + v0 ( t t0 ) +

1 a( t t0 )2 , 2 ( v v0 ) 1 v v0 + a x x0 = v0 a 2 a 2 2 v0 v v0 ( v 2 2vv0 + v0 ) , + x x0 = a a 2a 2 2 2 dividiendo 2

o 2a

2 a( x x0 ) = 2v0 v 2v0 + v 2 2vv0 + v0 = v 2 v0 v 2 = v02 + 2 ax Esta es una expresin escalar independiente del tiempo, no es una ecuacin general, por lo que no se puede usar en cualquier problema, es de utilidad restringida ya que slo permite obtener la magnitud de las variables que contiene. Ejemplo 2.3. un mvil parte desde el reposo en el instante t = 5 s y acelera hacia la derecha a razn de 2 m/s2 hasta t = 10 s. A continuacin mantiene su velocidad c onstante durante 10 s. Finalmente frena hasta detenerse, lo que logra hacer 3 se gundos ms tarde. a) Determinar a qu distancia del punto de partida se encuentra en t = 10 s. b) Con qu velocidad se mueve en ese instante? c) A qu distancia de la par tida se encuentra cuando empieza a frenar? d) Dnde se detiene respecto al punto de partida? e) Escriba las ecuaciones correspondientes a: a(t), v(t), x(t) para ca da etapa del movimiento. Solucin: Se puede elegir el SR como el cliente guste; un a posibilidad se ilustra en la figura 2.9, donde inicialmente se ubica a la partc ula en el origen O y se empieza a medir el tiempo desde el instante inicial 5 s. a) Se pide evaluar x(t) para t = 10 s, con las condiciones xo = 0, vo = 0, ao = 2m/s2, to = 5s, t1 = 10s, en el tramo A 51

Cap. 2 Movimiento en una dimensin. 1 x( t ) = x0 + v0 ( t t0 ) + a0 ( t t0 )2 2 x( 10 ) = 0 + 0 + 1 m 2 2 ( 10 5 )2 s 2 = 25m 2 s Figu a 2.9

v( t ) = v0 + a0 ( t t0 ) v( 10 ) = 0 + 2 m ( 10 5 )s = 10 m/s s2 c) Piden evalua x(t) a a t = 20 s, usando esquema y datos del t amo B: x( t ) = x10 + v10 ( t t1 ) + x( 20 ) = 25m + 10 1 a1 ( t t1 )2 2 m ( 20 10 )s + 0 = 125 m s d) Aqu se ide calcula x(t) a a t = 23 s, se conoce vf = 0, t3 =23 s, e o no s e conoce a2, o lo que se debe calcula . 1 x( t ) = x20 + v20 ( t3 20 ) + a2 ( t 20 )2 2 clculo de a2: 52

b) Aho a hay que calcula

v(t) en t = 10 s, usando la ecuacin:

Ca . 2 Movimiento en una dimensin. v = v2 + a2 ( t t 2 ) en el t amo C 0 = v2 + a2 ( t3 20 ) a2 = v2 t3 20 Pe o v2 = cte en el t amo B v2 = 10 m/s a= 10 m / s 10 m = ( 23 20 )s 3 s2 x( t ) = 125 + 10( 23 20 ) x( 23 ) = 140 m 1 10 ( 23 20 )2 = 140 m 2 3 e) Ecuaciones de movimiento: Pa a el t amo A: 1 x( t ) = x0 + v0 ( t t0 ) + ao ( t t0 )2 2 Con xo = 0, vo = 0, ao = 2m/s2, to = 5s x( t ) = 1 ao ( t 5 )2 x( t ) = ( t 5 )2 2 v( t ) = v0 + a0 ( t t0 ) v( t ) = 2( t 5 ) Las ecuaciones a a los t amos B y C las uede deduci el alumnos de los esulta dos obtenidos en c) y d), donde basta eem laza los valo es en las funciones de osicin y a idez en funcin de t. Ejem lo 2.4. Un auto ing esa en Conce cin al uente nuevo a San Ped o con una a idez de 54 km/h, la que mantiene constante mient as eco e el uente. En el mis mo instante en San Ped o ot o auto ing esa lentamente al uente con una a idez inicial de 10.8 km/h hacia Conce cin, acele ando a 1 m/s2. Si la longitud del ue nte es de 1838 m. Calcula a) la osicin donde se c uzan, b) la a idez del auto de San Ped o en el instante en que se c uzan, qu comenta io uede hace de este e sultado? 53

Ca . 2 Movimiento en una dimensin. Solucin: Datos: toA = toB = 0, xoA = 0, xoB = 1838m voA = 54 km 1h 1000m m = 15 , aA = 0 s h 3600 s 1km voB = 10.8 km/h = 3 m/s , aB = 1m/s2 El esquema de la figu a 2.10, muest a el sistema de efe encia elegido: Figu a 2.10. a) El movimiento es en una dimensin con a =cte, las ecuaciones a a cada mvil (A e n Conce cin, B en San Ped o) son: 1 2 x A = x0 A + v0 A (t t0 ) + a A (t t0 ) x A = v0 At x A = 15t 2 v A = v0 A + a A (t t0 ) v A = v0 A v A = 15 m/s 1 1 2 xB = x0 B + v0 B (t t0 ) + a B (t t0 ) xB = 1838 3t t 2 2 2 vB = v0 B + a B (t t0 ) vB = 3 t Cuando se c uzan: xA = xB, entonces 15t = 1838 3t 0 ,5t 2 0.5t 2 + 18t 1838 = 0 54

Ca . 2 Movimiento en una dimensin. t=

El proceso de integracin es grficamente equivalente a encontrar el rea bajo la curv a y = f(x). Se puede usar esta propiedad de las integrales para calcular grficame nte el valor del desplazamiento x y el cambio de rapidez v de una partcula en movim iento. De la definicin de velocidad se tiene: v= x t dx dx = vdt x dx = t v(t )dt dt o 0 x = t v(t )dt 0 t donde v(t) es la velocidad en cualquier instante. Si se conoce la forma analtica de v(t) se puede calcular la integral, pero si no se conoce, se puede evaluar grf icamente y por definicin de integral, la expresin anterior se interpreta como (ver figura 2.11a): desplazamiento = rea bajo la curva v/t Considerando primero el ca so en que la partcula se mueve con rapidez constante vo (significa que su acelera cin es cero), entonces del grfico v/t, que se 55

b) vB (45.2 ) = 3 45.2 = le a du ante todo ese tiem n mucho la mxima e mitida 2.4 CALCULO GRFICO DE x Y

48.2m/s = 173.5 km/h El automvil de San Ped o no uede ace o, o que alcanza a una a idez muy alta, su e ando e y osible de alcanza . v.

18

18 2 + 4(0.5)(1838) t1 = 45.2 s, t 2 = 40.6 s 1 x(45.2 ) = 15( 45.2 ) = 678 m

Cap. 2 Movimiento en una dimensin. muestra en la figura 2.11a, el desplazamiento es el rea del rectngulo de lados vo y t, esto es: desplazamiento = rea rectngulo x = vo t , con vo = cte. Figura 2.11 a) izquierda, b) derecha. Considerando ahora el caso en que la partcula se mueve con rapidez v(t) funcin lin eal del tiempo (en este caso la aceleracin es constante), o sea v(t) = vo + a(t to), el desplazamiento x de la partcula durante el intervalo de tiempo desde to a t es igual al rea bajo la recta v(t) de la figura 2.11b: desplazamiento = rea rec tngulo + rea tringulo x = vo t + 1 vt 2 1 a( t )2 2 x = vo t + De manera similar se obtiene el calculo grfico para el cambio de rapidez. Conside rar una partcula que se mueve con rapidez vo en el instante inicial to y con rapi dez v en el instante t, que aumenta su aceleracin linealmente con el tiempo, o se a a(t) = ao + k(t - to), donde ao es el valor inicial de la aceleracin 56

Cap. 2 Movimiento en una dimensin. y k representa el valor de la pendiente de la recta en el grfico aceleracin versus tiempo, que debe tener unidad de medida de m/s3. En este caso estamos extendien do la descripcin del movimiento al caso de una partcula con aceleracin variable, de jando de lado la restriccin impuesta al principio de este captulo. El cambio de ra pidez v de la partcula durante el intervalo de tiempo desde to a t es igual al rea bajo la recta a(t) de la figura 2.12: cambio de rapidez = rea rectngulo + rea tringu lo v = ao t + 1 at 2 Como se propuso, a es una funcin lineal de t de la forma a(t) = ao +k(t - to), en tonces a(t) - ao = k(t - to), o bien a = kt, reemplazando se tiene: v = ao t + 1 k ( t )2 2 Observar que en este caso se tiene un mtodo para describir un movimiento con acel eracin variable (en este caso linealmente) en el tiempo. Figura 2.12 Ejemplo 2.5: En la figura 2.13 se muestra el grfico rapidez/tiempo para una partcu la que se mueve en direccin positiva del eje x. a) calcular el desplazamiento de la partcula, b) hacer el grfico aceleracin/tiempo, c) determinar las ecuaciones de movimiento en cada intervalo de tiempo, d) calcular su posicin en los instantes 5 , 10 y 20 segundos. 57

Cap. 2 Movimiento en una dimensin. Figura 2.13 Ejemplo 5.

Solucin. a) El desplazamiento es igual al rea (A) bajo la curva v/t, que es conven iente calcular por intervalos de tiempo, entonces: 1 m 0 t < 5s : A1 = x1 = 20 (5s = 50m 2 s m 5 t < 10 s : A2 = x2 = 20 (5s ) = 100m s 1 m m 10 t 20 + 10 (10s ) = 150m 2 s s xT = x1 + x2 + x3 = 50 + 100 + 150 = 300 m b) Los valores de la aceleracin que se pueden calcular de la pendiente del grfico v/t en cada intervalo de tiempo, se indican en el grfico a/t de la figura 2.14. Figura 2.14. Ejemplo 5, parte b). c) Determinacin de las ecuaciones de movimiento, suponiendo que xo = 0 para to = 0. 58

Cap. 2 Movimiento en una dimensin. 0 t < 5s : 1 2 1 0 x( t ) = vot + at 2 x( t ) = 2t 2 2 1 2 x( t ) = x( 5 ) + vo (t 5 ) + a(t 5 ) x( t ) = 50 + 20(t 5 ) 2 x( t ) = x( 10 ) + vo (t 10 ) + a(t 10 ) 2 1 2 x( t ) = 150 + 20(t 10 ) (t ) 2

5 t < 10s : 10 t 20s : d) La osicin en los instantes edidos (y en cualquie ot o tiem o) se uede calc ula con las ecuaciones de movimiento ante io es a a t = 5s: x(t) = 2t2 x(5) = 2(5)2 = 50 m para t = 10s: x(t) = 50+20(t-5) x(10 )=50+20(10-5) = 150 m para t = 20s: x(t) = 150+20(t-10)- (t-10)2 x(20) = 300 m Ej ercicio: calcular la posicin en los instantes 2.5, 8 y 15 segundos. 2.5 CUERPOS EN CADA LIBRE. Un caso particular de movimiento en una dimensin, es aquel de los objetos que se mueven libremente en direccin vertical cerca de la superficie de la Tierra, que s e conoce como movimiento de cada libre. Galileo (1564 1642), fsico y astrnomo itali ano, fue el primero en estudiar el movimiento de cada libre, al observar que dos cuerpos diferentes, al dejarlos caer desde la torre inclinada de Pisa, llegaban al suelo casi al mismo tiempo. Experimentalmente se demuestra que todos los cuer pos que se dejan caer cerca de la superficie de la Tierra, lo hacen con una acel eracin aproximadamente constante. Esta aceleracin, que se llama aceleracin de grave dad, es producida por una fuerza que existe entre cuerpos con masa, llamada fuer za de atraccin gravitacional, cuyo origen ser explicado en el Captulo 9. 59

Cap. 2 Movimiento en una dimensin. r La aceleracin de gravedad, que se denota por g es un vector que apunta hacia el centro de la Tierra, su magnitud aumenta levemente al aumentar la latitud, es d ecir desde el ecuador hacia los polos, y disminuye al aumentar la altura sobre l a superficie terrestre. Su valor medio en la superficie de la Tierra es aproxima damente de 9.8 m/s2. Se dice que un objeto est en cada libre cuando se mueve bajo la influencia slo de l a aceleracin de gravedad, despreciando la resistencia (es otra fuerza que se resi ste al movimiento y que tambin ser estudiada ms adelante) que el aire opone a los c uerpos en movimiento, sin importar la velocidad inicial del objeto. Todos los cu erpos que se lanzan hacia arriba o hacia abajo, o se dejan caer, lo hacen librem ente una vez que se dejan en libertad. La aceleracin que adquieren es siempre la aceleracin de gravedad, vertical hacia abajo, cualquiera sea la direccin inicial d el movimiento. Como el movimiento de cada libre es en una dimensin, con aceleracin constante, se puede adoptar como direccin del movimiento al eje vertical y. Por l o tanto se pueden aplicar las ecuaciones para el movimiento en una dimensin, toma ndo al eje y en la direccin del movimiento de cada, por convencin positivo hacia ar riba. Con esta convencin, un movimiento de cada libre de ascenso o de descenso tie ne una aceleracin g negativa. Tambin se debe tener en cuenta que si el cuerpo asci ende (desciende) su velocidad ser positiva (negativa) en este sistema de referenc ia. De est forma las ecuaciones de movimiento 2.7 y 2.8 se transforman en las ecu aciones para cada libre: 1r r r r 2 y = yo + voy g (t to ) 2 (2.9) v y = voy g (t to )

(2.10) Los g ficos osicin/tiem o, velocidad/tiem o y acele acin/tiem o a a una a tcula q ue se lanza ve ticalmente hacia a iba, desde una osicin inicial yo, que no tien e o que se el suelo, son los que se muest an en la figu a 2.15 60

Ca . 2 Movimiento en una dimensin.

v(t ) = vo gt = 0 vo = gt t = 20m/s = 2s 10m/s 2 b) Se pide evaluar y(t) para t = 2 s 1r 1 r r r y = y o + voy (t t o ) g (t t o ) 2 y = vo t gt 2 2 2 y max = y (2) = (20m/s )(2 s ) 1 2 10m/s 2 (2 s ) = 20m 2 Figu a 2.16 ( ) 61

a) Cuando la

ied a alcanza la mxima altu a v = 0:

Ejem lo 2.6: Tito lanza una ied a hacia a iba desde la te aza de un edificio de 50 m de alto, con una a idez inicial de 20 m/s. Cuando est cayendo la ied a asa justo o el costado del edificio. Calcula a) el tiem o a a que la ied a alcance su altu a mxima, b) la altu a mxima, c) el tiem o que ta da en asa o el unto inicial, d) la velocidad de la ied a en ese instante, e) el tiem o que ta da en llega al suelo, f) la velocidad en ese instante. Solucin: Conside ando un sistema de efe encia que se muest a en la figu a 2.16, con el eje y ositiv o ve tical hacia a iba y el o igen yo = 0 donde comienza el movimiento de la i ed a, con to = 0 y vo = 20 m/s.

Figu a 2.15. G ficos y/t, vy/t y a/t, a a a =

Ca . 2 Movimiento en una dimensin. c) Cuando asa o y = vo t 1 2 1 gt = 0 vo gt t = 0 2 2 2v 1 (2)(20) gt = 0 t = o = = 4s 2 g 10 t1 = 0 y vo el unto inicial y = 0

d) Hay que evalua v a a t = 4s v(t ) = vo gt v(4) = 20 (10)(4) = 20 e) En esta osicin y = 50 m 1 y = vo t gt 2 50 = 20t 5t 2 2 t 2 4t 10 = 0 t1 = 5.7 s y t 2 = 1.7 s Se des el tiem o negativo, o que fsicamente no es osible. f) v(t ) = vo gt v(5.7) = 2 0 (10)(5.7) = 37 m s m s

2.5.1 Efectos de g en las e sonas. La ca acidad de una e sona a a so o ta un a acele acin de ende tanto de la magnitud como de la du acin de sta. Debido a la in e cia de la sang e y de los ganos dilatables, las acele aciones equeas tienen o ca im o tancia si du an slo f acciones de segundo. El lmite de tole ancia se encue nt a ce cano a 10g y de ende de la esistencia est uctu al de los cue os. La ma yo a de las e sonas han ex e imentado acele aciones ve ticales mode adas en los ascenso es. La sang e ci cula o vasos dilatables de mane a que cuando el cue o es acele ado hacia a iba, la sang e se acumula en la a te infe io de ste. Cu ando la acele acin es hacia abajo, aumenta el volumen de sang e en la a te su e io del cue o, a su vez los ganos inte nos no se mantienen 62

Ca . 2 Movimiento en una dimensin. te su e io del cue o, a su vez los ganos inte nos no se mantienen gidos en su sitio y su des lazamiento du ante la acele acin uede oduci sensaciones desag adables. Cuando un avin des ega, ate iza o ealiza gi os muy idos, est sometido a acele aciones de hasta 9g. El g ado de tole ancia de un humano a esta acele ac in de ende ent e ot os facto es del eso, edad y condicin fsica de la e sona. A mo do de ejem lo, un iloto que en tie a esa 80 kilos, cuando es sometido a este valo de acele acin siente e entinamente que su eso es al ededo de 720 kilos. Esta misma acele acin hace que la sang e fluya hacia los ies del iloto, esto di sminuye el eto no venoso al co azn con lo cual la esin baja y el iloto uede e de la visin tem o almente, a a luego e de la conciencia. Tambin existen acel e aciones negativas du ante el vuelo en la cual el iloto ex e imenta la acele a cin en osicin inve tida. En ese caso la acele acin hace que la sang e fluya al ce eb o, el iloto suf e de alidez y su visin se to na oja. Estudios han dete mina do que los humanos ueden so o ta hasta 9g de acele aciones ositivas y 3g a a acele aciones negativas. Un iloto que viaja en aviones mode nos que incluso al canzan velocidades ce canas a la del sonido, od a detene se sin elig o en una d istancia a oximada de 200 m, e o si esta velocidad fuese unas 100 veces mayo (valo es que ueden se alcanzados en viajes inte laneta ios), la distancia de f enado que necesita a a a no oduci efectos nocivos en sus t i ulantes debe s e de a oximadamente 16000km. La azn de esta dife encia est en que la cantidad t otal de ene ga que se disi a du ante la desacele acin es o o cional al cuad ado de la velocidad, lo que es suficiente a a aumenta la distancia unas 10000 vece s. Po esta azn se han c eado ocedimientos y a a atos es eciales a a otege a los ilotos del cola so ci culato io que a a ece du ante acele aciones ositi vas. P ime o, si el iloto a ieta sus msculos abdominales en g ado ext emo y se inclina hacia adelante a a com imi el abdomen, uede evita la acumulacin de s ang e en los g andes vasos abdominales, evitando as la e dida de conciencia. Ade ms se han diseado t ajes anti g ara revenir el estancamiento de sangre en la arte ms baja del abdomen y las iernas. Este ti o de traje a lica una resin ositiva en iernas y abdomen, inflando com artimientos de aire a medida que aumenta la a celeracin ositiva. Adems el cuer o humano resenta de 1 a 2 cm de tejido blando e xterno, lo que aumenta la distancia de desaceleracin y or lo tanto disminuye la fuerza de im acto, or ejem lo, durante una cada. 63

Ca . 2 Movimiento en una dimensin. PROBLEMAS. 2.1

Cuando Carlos viaja en una auto ista, asa or la marca de 260 km. Des us sigue m ovindose hasta la marca de 150 km. y luego se devuelve hasta la marca 175 km. Cul e s su des lazamiento resultante res ecto a la marca de 260 km.? R: 85 km. Un gato negro se encuentra en una osicin final de 3.6 m en direccin 240 res ecto a x, des us de realizar un des lazamiento de 120 cm en 135 res ecto de x. Determine su osi cin inicial. R: 4.1m, 256.5 . La luz del Sol llega a la Tierra en 8.3 min. La ra id ez de la luz es de 3 x 108m/s. Calcular la distancia de la Tierra al Sol. R: 1.5 x 1011 m. Usted y un amigo conducen recorriendo 50 km. Usted viaja a 90 km/h y su amigo a 95 km/h. Cunto tiem o tiene que es erarlo su amigo al final del viaje? R: 1.8 min. Ana conduce calle abajo a 55 km/h. Re entinamente un nio atraviesa la calle. Si Ana demora 0.75 s en reaccionar y a licar los frenos, cuntos metros alc anza a moverse antes de comenzar a frenar? R: 11 m. Las condiciones de movimient o de una artcula que se mueve en direcr r r cin x son x o = 7i m, v o = 3i m/s, a = 4i m/s 2 , en el instante inicial t0 = 0. a) Escribir las ecuaciones vectoriale s de la osicin y velocidad del cuer o en cualquier instante. b) Calcular la osi cin del cuer o res ecto al origen a los 10 s de iniciado el movimiento. c) Averig uar si el cuer o se detiene en algn instante. R: b) 223i m, c) no. Una artcula se mueve a lo largo del eje x de acuerdo con la ecuacin x(t)=(3t2 2t+3)m. Calcular a ) la ra idez romedio entre t = 2s y t = 3s, y b) la velocidad instantnea en t = 2s y t = 3s, c) la aceleracin romedio entre t = 2s y t = 3s y d) la aceleracin in stantnea en t = 2s y t = 3s. Una artcula se mueve a lo largo del eje x de acuerdo con la ecuacin x(t)=2+3t t2, donde x est en metros y t en segundos. Para t=3s, ca lcular a) la osicin de la artcula , b) su velocidad c) su aceleracin. R: a) 2m, b ) 3m/s, c) 2m/s2. 64 2.2 2.3 2.4 2.5 2.6 2.7 2.8

Ca . 2 Movimiento en una dimensin. 2.9 Las ecuaciones de movimiento ara dos artculas A y B que se mueven en la misma d ireccin son las siguientes (x en m y t en s). x A (t ) = 3.2t 2 6t 20 x B (t ) = 29 + 8.5t 4.1t 2 Calcular: a) el instante ara el cual las osiciones de A y B c oinciden, b) las velocidades de A y B en el instante en que se encuentran en la misma osicin.R: a) 3.8s, b) 18.3 m/s, 22.7 m/s. 2.10 Un electrn en un tubo de rayos catdicos acelera de 2x104m/s hasta 6x106m/s en 1.5cm. a) Cunto tiem o tarda el electrn en recorrer esta distancia? b) Cul es su ace leracin? 2.11 Un electrn tiene una velocidad inicial de 3x105m/s. Si ex erimenta u na aceleracin de 8x1014 m/s2, a) Cunto tardara en alcanzar una velocidad de 5.4x105 m/s, y b) qu distancia recorre en ese tiem o? 2.12 Determine la velocidad final de un rotn que tiene una velocidad inicial de 2.35 x 105 m/s, y es acelerado uni formemente en un cam o elctrico a razn de 1.10x1012 m/s2 durante 1.5x10 7s. R: 7.0 x 104 m/s. 2.13 Un jet su ersnico que vuela a 145 m/s acelera uniformemente a razn de 23.1 m/s2 durante 20s. a) Cul es su velocidad final? b) La ra idez del sonido en el aire es 331 m/s. Cuntas veces mayor es la velocidad final del avin com arada con la del sonido? R: a) 607 m/s, b) 1.83 veces la ra idez del sonido. 2.14 Dos autos A y B se mueven en lnea recta en direccin ositiva del eje x. En el instante inicial A est en re oso y acelera con 2m/s2. El movimiento de B es con ra idez c onstante de 20m/s. Calcular: a) la distancia que recorren en un minuto, b) el ti em o que demorara A en igualar la ra idez de B, c) la distancia que los se ara cu ando sus ra ideces son iguales, d) la aceleracin que debera ejercerse sobre B ara que udiera detenerse en 4 s. R: a) 3600m, 1200 m, b) 10 s, c) 100 m, d) 5 m/s2. 65

Ca . 2 Movimiento en una dimensin. 2.15 Un auto que se mueve con aceleracin constante recorre en 6 s la distancia de 60 m que se ara dos untos; su ra idez al asar or el segundo unto es de 14 m /s. Calcular: a) la aceleracin del auto, b) su velocidad al asar or el rimer unto, c) la osicin donde se encontraba en re oso. R: a) 4/3 m/s2, b) 6 m/s, c) 14 .4m. 2.16 Dos autos viajan a lo largo de una carretera recta. En el instante t = 0h, el auto A tiene una osicin xA = 48 km y una ra idez constante de 36 km/h. Ms tarde en t=0.5h, el auto B est en la osicin xB=0 km con una ra idez de 48 km/h. Res onda las siguientes reguntas: rimero, grficamente, haciendo una grfica de o sicin versus tiem o; segundo, algebraicamente, escribiendo las ecuaciones ara la s osiciones xA y xB en funcin del tiem o t. a) Cul es la lectura del cronmetro cuan do el auto B sobre asa al auto A? b) En qu osicin A es alcanzado or B? c) Cunto tie m o transcurre desde que A estaba en su unto de referencia hasta que B lo alcan za? R: a) 6 h, b) 260 km, c) 7.3 h. 2.17 Un auto y un tren se mueven al mismo ti em o a lo largo de trayectorias aralelas a 25m/s. Debido a una luz roja el auto ex erimenta una aceleracin uniforme de 2.5m/s2 y se detiene. Permanece en re oso durante 45s, des us acelera hasta una velocidad de 25m/s a una tasa de 25m/s2. A q u distancia del tren est el auto cuando alcanza la velocidad de 25m/s, su oniendo que la velocidad del tren se ha mantenido en 25m/s? 2.18 Una artcula arte desde el re oso de la arte su erior de un lano inclinado y se desliza hacia abajo c on aceleracin constante. El lano inclinado tiene 2m de largo, y la artcula tarda 3s en alcanzar la arte inferior. Determine a) la aceleracin de la artcula, b) s u velocidad en la arte inferior de la endiente, c) el tiem o que tarda la artc ula en alcanzar el unto medio del lano inclinado, y d) su velocidad en el unt o medio. R: a) 0.44m/s2, b) 1.3m/s, c) 2.1s, d) 0.94m/s. 2.19 Dos trenes ex reso s inician su recorrido con una diferencia de 5 min. A artir del re oso cada uno es ca az de alcanzar una velocidad mxima de 160km/h des us de acelerar uniformeme nte en una distancia de 2km. a) Cul es la aceleracin de cada tren? b) A que distanci a est el rimer tren cuando el segundo inicia su trayecto? c) Qu tan se arados se e ncuentran cuando ambos viajan a mxima velocidad? 66

Ca . 2 Movimiento en una dimensin.

2.20 Un automvil que se mueve a una velocidad constante de 30m/s ierde velocidad re entinamente en el ie de una colina. El auto ex erimenta una aceleracin const ante de 2 m/s2 (o uesta a su movimiento) mientras efecta el ascenso. a) escriba ec uaciones ara la osicin y la velocidad como funciones del tiem o considerando x = 0 en la arte inferior de la colina, donde vo = 30m/s. b) Determine la distanc ia mxima recorrida or el auto des us de que ierde velocidad. R: a) 30t t2, 30 2t b) 225m. 2.21 Paco manejando a 30m/s entra en un tnel de una sola ista. Des us o bserva una camioneta que se mueve des acio 155m adelante viajando a 5m/s. Paco a lica sus frenos ero uede desacelerar slo a 2m/s2, debido a que el camino est hme do. Chocar? Si es as, calcular a qu distancia dentro del tnel y en qu tiem o ocurre e choque. Si no choca, calcular la distancia de mximo acercamiento entre el auto d e Paco y la camioneta. R: 11.4s, 212m. 2.22 Una bala indestructible de 2cm de la rgo se dis ara en lnea recta a travs de una tabla que tiene 10cm de es esor. La ba la entra en la tabla con una velocidad de 420m/s y sale con una velocidad de 280 m/s. a) Cul es la aceleracin romedio de la bala a travs de la tabla? b) Cul es el ti m o total que la bala est en contacto con la tabla? c) Qu es esor de la tabla se re querira ara detener la bala? 2.23 Un africano que se encuentra a 20 m de un len h ambriento arranca con una ra idez constante de 36 km/hr, alejndose en lnea recta d el len, que est inicialmente detenido. El len tarda 2 segundos en reaccionar cuando em ieza a erseguir al africano con una aceleracin de 4 m/s2, siem re en lnea rec ta hacia el africano, que huye hacia un rbol que se encuentra ms adelante en la mi sma recta. a) Hacer un esquema ilustrativo de la situacin. b) Cul debe ser la mxima distancia a la que debe estar el rbol ara que el africano ueda subirse justo an tes que el len lo alcance? c) Calcular la ra idez con la que el len llega al rbol. R: b) 116m, c) 30.4 m/s. 2.24 Un camin se mueve a 90 km/hr en una carretera recta . Cuando se encuentra a 70 m de un rbol atravesado en la carretera, el conductor se da cuenta de ello, tardando 0.5 s en reaccionar y resionar los frenos del ca min que le im rimen una aceleracin de 5 m/s2. Determinar si el 67

Ca . 2 Movimiento en una dimensin. camin choca o no con el rbol cruzado en la carretera. R: si a 25.5 km/h. 2.25 Dos autos se a roximan uno al otro; ambos se mueven hacia el oeste, uno a 78 km/h y el otro a 64 km/h. a) Cul es la velocidad del rimer auto relativa al (en el siste ma de referencia del) segundo auto? b) Cambian su velocidad relativa des us de que el uno sobre asa al otro? R: a) 14km/h, oeste, b) no. 2.26 En la figura 2.17 se muestra el grfico ra idez/tiem o ara una artcula que se mueve en direccin del ej e x. a) Dibujar el grfico osicin/tiem o, b) calcular el des lazamiento de la artc ula, c) hacer el grfico aceleracin/tiem o, d) calcular su osicin en los instantes 5, 10, 20, 25, 30 y 40 segundos, e) calcular el cambio de ra idez en los interva los 0 y 5, 5 y 20, 20 y 25, 25 y 40 segundos. Figura 2.17. Problema 2.26. 2.27 Dos autos viajan a lo largo de una lnea en la misma direccin, el que va adela nte a 25m/s y el otro a 30m/s. En el momento en que los autos estn a 40m de dista ncia, el conductor del auto delantero a lica los frenos de manera que el vehculo acelera a 2 m/s2. a) Cunto tiem o tarda el carro ara detenerse? b) su oniendo que el carro trasero frena al mismo tiem o que el carro delantero, Cul debe ser la ace leracin negativa mnima del auto trasero de manera que no choque con el auto delant ero? c) Cunto tiem o tarda en detenerse el auto trasero? R: a) 1.25s, b) 2.3m/s2 c ) 13.1s. 68

Ca . 2 Movimiento en una dimensin. 2.28 Un automovilista conduce or un camino recto a una velocidad constante de 1 5m/s. Cuando asa frente a un olica motociclista estacionado, ste em ieza a acele rar a 2 m/s2 ara alcanzarlo. Su oniendo que el olica mantiene esta aceleracin, d etermine a) el tiem o que tarda el olica en alcanzar al automovilista, encuentre b) la velocidad y c) el des lazamiento total del olica cuando alcanza al automo vilista. 2.29 Dos objetos se conectan mediante una barra rgida de longitud L. Los objetos deslizan a lo largo de rieles er endiculares, como se muestra en la fi gura 2.18. Si el que est en el eje x se desliza hacia la izquierda con ra idez co nstante vo, calcular la ra idez del otro cuando = 60. R: 0.58vo.

2.30 Un tren vi j de l siguiente m ner : en los primeros 60 minutos se despl z con velocid d v, en los siguientes 30 minutos llev un velocid d de 3v, en lo s 90 minutos que le siguen vi j con un velocid d v/2; en los 120 minutos fin l es, se mueve con un velocid d de v/3. ) Dibuje l grfic velocid d-tiempo p r este recorrido. b) Qu dist nci recorre el tren en el vi je? c) Cul es l velocid d promedio del tren en el vi je completo? 2.31 Un tren puede minimiz r el tiempo t entre dos est ciones celer ndo r zn de 1= 0.1 m/s2 por un tiempo t1 y despus experiment un celer cin neg tiv 1 = -0.5 m/s2 cu ndo el m quinist us los f renos dur nte un tiempo t2. Puesto que l s est ciones estn sep r d s slo por 1km, el tren nunc lc nz su velocid d mxim . Encuentre el tiempo mnimo de vi je t y e l tiempo t1. R: 155s, 129s. 69

Figur

2.18 Problem 2.29.

C p. 2 Movimiento en un dimensin. 2.32 Cu ndo un semforo c mbi verde, un uto rr nc con un celer cin const nt e de 6 m/ss. En el inst nte en que comienz celer r es sobrep s do por un c m in con un velocid d const nte de 21 m/s. ) Qu dist nci recorre el uto ntes de lc nz r el c min? b) Qu velocid d tendr el uto cu ndo lc nce el c min? R: 150 m, b ) 42 m/s 2.33 El conductor de un uto que vi j 90 km/h sbit mente ve l s luces de un b rrer que se encuentr 40 m del nte. Tr nscurren 0.75 s ntes de qu l plique los frenos; l celer cin medi dur nte l fren d es 10 m/s2. ) Determine si el c rro choc contr l b rrer . b) Cul es l r pidez mxim l cu l puede vi j r el uto p r no choc r contr l b rrer ? Supong celer cin const nte. R: ) Si, golpe l b rrer , b) 22 m/s. 2.34 Con el fin de proteger su limento de o sos, un boy scout elev su p quete de comid , de m s m, con un cuerd que l nz sobre l r m de un rbol de ltur h. El scout c min lejndose de l cuerd ver tic l con velocid d const nte vs mientr s sostiene en sus m nos el extremo libre . ) H cer un esquem de l situ cin. b) Demuestre que l velocid d vp del p quet e de comid es x( x 2 + h 2 ) 1 / 2 v s , donde x es la distancia que el muchacho ha caminado alejndose de la cue da ve tical. c) Demuest e 2 que la acele acin a del aquete de comida es h 2 ( x 2 + h 2 ) 3 / 2 v s . d) Qu valo es de la acele ac in y la velocidad se tienen des us que l se aleja de la cue da ve tical? e) A qu valo es se a oximan la velocidad y la acele acin cuando la distancia x contina aument ando? 2.35 Un objeto se mueve en un medio donde ex e imenta una acele acin de es istencia al movimiento o o cional a su a idez, esto es a = kv, donde k es un a constante ositiva igual a 0.5 s 1. a) Calcula la a idez y osicin del objeto en cualquie instante. b) Si a a t = 0 el objeto se encuent a en el o igen mov indose con una a idez de 10 m/s, calcula la osicin donde se detiene. R: b) 20 m . NOTA: En algunos oblemas de cada lib e, se usa g = 10 m/s2 70

Ca . 2 Movimiento en una dimensin. 2.36 Un ast onauta deja cae una luma a 1.2 m de la su e ficie de la Luna. Si l a acele acin de la g avedad en la Luna es 1.62 m/s2, cunto tiem o em lea la luma e n llega a la su e ficie? R: 1.2 s. 2.37 Una ied a cae lib emente desde el e o so du ante 8 s. a) Calcule la velocidad de la ied a a los 8 s. b) Cul es el des l azamiento de la ied a du ante ese tiem o? R: a) 78 m/s, hacia abajo, b) 310 m. 2. 38 Un estudiante deja cae una oca al agua desde un uente de 12 m de altu a. Cul es la a idez de la oca cuando llega al agua? R: 15.5 m/s. 2.39 Un globo meteo olgico flota a una altu a constante sob e la Tie a cuando deja cae un aquete. a) Si el aquete choca cont a el iso a una velocidad de 73.5 m/s, Qu distancia e co i el aquete? b) Du ante cuanto tiem o cay el aquete? R: a) 276 m, b) 7.5 s. 2 .40 Una elota se lanza ve ticalmente hacia a iba con una a idez inicial de 10 m/s desde una altu a de 10 m es ecto al suelo. Dete mine a) su osicin en el u nto ms alto, b) su velocidad cuando asa o el unto inicial, c) su velocidad y acele acin justo antes de gol ea el suelo. R: a) 15m 2.41 Un globo inflado con a i e caliente se eleva ve ticalmente con una a idez constante de 5 m/s. Cuando e st a 50 m sob e el suelo, se deja cae un aquete desde el globo. a) Calcula el tiem o que ta da el globo en llega a los 50 m. b) Cunto tiem o demo a el aquete en llega al suelo des us que se ha soltado? c) Cul es la velocidad del aquete jus to antes de llega al suelo? d) Re eti b) y c) a a el caso en que el globo des ciende a 5 m/s desde una altu a de 50 m. R: a) 10s, b) 3.7s, c) 32 m/s. 2.42 Un g lobo sonda meteo olgico se lanza desde la su e ficie de la tie a con una velocid ad inicial ve tical hacia a iba de magnitud 18 km/h, la que mantiene constante du ante 15 min. A a ti de ese instante se comienza a com o ta como a tcula li b e. Calcula : a) la altu a mxima que alcanza, b) su velocidad justo antes de lle ga nuevamente al suelo. R: a) 4501.25m. 71

Ca . 2 Movimiento en una dimensin. 2.43 Se deja cae una ied a desde el bo de de un acantilado. Una segunda ied a se lanza hacia abajo desde el mismo luga un segundo ms ta de con una a idez in icial de 15 m/s. a) Si ambas ied as gol ean el suelo simultneamente, dete mine l a altu a del acantilado. b) Calcula la velocidad de cada ied a justo antes de llega al suelo. R: a) 20m, b) 20 y 25 m/s. 2.44 Un cohete a te del e oso y sub e con acele acin neta constante ve tical hacia a iba de 5 m/s2 du ante un minuto . A a ti de ese momento deja de acele a y sigue subiendo, e o com o tndose co mo a tcula lib e. Dete mina : a) la altu a que alcanza el cohete du ante el im e minuto, b) su velocidad en ese instante, c) la altu a mxima que alcanza, d) el tiem o total de vuelo. R: a) 9000m, b) 300m/s, c) 13.5km, d) 142s. 2.45 Se lanz a una elota ve ticalmente hacia a iba con una a idez inicial de 10 m/s. Un se gundo ms ta de se lanza una ied a ve ticalmente hacia a iba con una a idez ini cial de 25 m/s. Dete mina a) el tiem o que ta da la ied a en alcanza la misma altu a que la elota, b) la velocidad de la elota y de la ied a cuando se enc uent an a la misma altu a, c) el tiem o total que cada una est en movimiento ante s de eg esa a la altu a o iginal, d) la altu a mxima de las dos. R: a)0.2s, b) 2 y 23m/s, c) 2 y 6s. 2.46 Anglica deja cae una elota de tenis desde la te aza de un edificio, y un segundo des us ti a ve ticalmente hacia abajo ot a elota co n una a idez de 20 m/s. Calcula la altu a mnima del edificio a a que la segund a elota ueda alcanza a la ime a. R: 11.25m 2.47 Una elota es lanzada ve ti calmente hacia a iba desde el suelo con una velocidad inicial de 15m/s. Calcula : a) el tiem o que la elota ta da en alcanza su altu a mxima, b) la altu a mxima , c) la velocidad y la acele acin de la elota a a t = 2s. R: a)1.5s, b)11.5m, c ) 4.6m/s, g. 2.48 La altu a de un helic te o sob e el suelo est e esentada o h = 3t3, donde h est en met os y t en segundos. Des us de 2s, el helic te o deja cae una equea valija con la co es ondencia. Cunto tiem o ta da la valija en llega a l suelo? R: 8s 72

Ca . 2 Movimiento en una dimensin. 2.49 Una elota se deja cae al suelo desde una altu a de 2m. En el ime ebot e la elota alcanza una altu a de 1.85m, donde es at a ada. Encuent e la velocid ad de la elota a) justo cuando hace contacto con el suelo y b) justo cuando se aleja del suelo en el ebote. c) Igno e el tiem o que la elota mantiene contact o con el suelo y dete mine el tiem o total que necesita a a i del unto en que se suelta al unto donde es at a ada. R: a) 6.3 m/s, b) 6m/s, c) 1.25s. 2.50 U na elota de tenis que se deja cae al iso desde una altu a de 1.2 m, ebota ha sta una altu a de 1 m. a) Con qu velocidad llega al iso? b) Con qu velocidad deja e l iso al ebota ? c) Si la elota de tenis est en contacto con el iso du ante 0 .01 s, calcula su acele acin du ante este tiem o, com ela con g. R: a) 4.85 m/s, b) 4.43 m/s, c) +930 m/s2, 93g. 2.51 Una ulga salta 20 cm en un salto ve tical. a) Calcula su a idez inicial. b) Si ha alcanzado esa a idez encogiendo y lue go esti ando sus atas una longitud del o den de 1 mm, calcula su acele acin ini cial. c) La distancia de acele acin en una e sona adulta es del o den de 50 cm, si una e sona salta a con la misma acele acin que una ulga, a que altu a llega a? R: a) 2m/s, b) 2000m/s2, c) 2.52 Cuando las anas saltan, t icamente acele an en una distancia ve tical de unos 10 cm, y ueden alcanzan altu as de hasta 30 cm, medidas desde el suelo. Calcula : (a) la velocidad de des egue de la ana, y (b ) la acele acin media que ella siente ent e que comienza el salto y el momento de l des egue. Su onga una acele acin constante. 73

Ca . 3 Movimiento en dos Dimensiones CAPITULO 3. MOVIMIENTO EN DOS DIMENSIONES. En gene al e1 movimiento de los objet os ve dade os se ealiza en el es acio eal t idimensional. E1 movimiento de una a tcula que se ealiza en un lano es un movimiento en dos dimensiones, si el m ovimiento se ealiza en el es acio, se oduce en t es dimensiones. En este ca tu lo se estudia la cinemtica de una a tcula que se mueve sob e un lano. Ejem los d e un movimiento en dos dimensiones son el de un cue o que se lanza al ai e, tal como una elota, un disco gi ando, el salto de un cangu o, el movimiento de la netas y satlites, etc. El movimiento de los objetos que gi an en una bita cuya t ayecto ia es una ci cunfe encia, se conoce como movimiento ci cunfe encial; es u n caso de movimiento en dos dimensiones, que tambin es estudiado en este ca tulo. El vuelo de una mosca, el de un avin o el movimiento de las nubes se oduce en t es dimensiones.

3.1 DESCRIPCIN DEL MOVIMIENTO EN DOS DIMENSIONES. Continuamos est ingiendo el es tudio del movimiento al caso de una a tcula que se mueve con acele acin constante , es deci que su magnitud y di eccin no cambian du ante el movimiento. E1 vecto osicin de una a tcula que se mueve en el lano xy es una funcin del tiem o, se e sc ibe como: (t ) = x(t )i + y (t ) j Po definicin, la velocidad de la a tcula en movimiento en el lano xy es, el cam bio de osicin en el t anscu so del tiem o y se uede dete mina o : d dx dy v= j = vxi + v y j = i+ dt dt dt es deci , v ( t ) = v x ( t ) + v y ( t ) i j 75

Ca . 3 Movimiento en dos Dimensiones donde vx y vy son las com onentes de la velocidad en la di eccin x e y. Si la ace le acin es constante, sus com onentes ax en la di eccin x, y ay en la di eccin y, t ambin lo son. A licando las ecuaciones cinemticas de la velocidad deducidas a a e l movimiento en una dimensin, inde endientemente en cada di eccin x e y, a a una a tcula que en el instante inicial to se mueve con velocidad inicial v o = v ox i + v oy se obtienen las com onentes de la j velocidad en funcin del tiem o : v x = vox + a x ( t to ) v y = voy + a y ( t to )

eem lazando en la ex esin de v ( t ) , se obtiene la velocidad en cualquie i nstante t: v (t ) = [vox + a x (t t o )]i + v oy + a y (t t o ) j v (t ) = (v ox i + v oy ) + (a x i + a y )(t t o ) j j [ ] v ( t ) = vo + a( t to )

(3.1) De mane a simila eem lazando las en cada di eccin x e y, a a una ncuent a en la osicin inicial o a a tcula, en cualquie instante x = x o + v ox (t t o ) + 1 a x (t t o ) 2 2 76 ex esiones de la osicin en funcin del tiem o a tcula que en el instante inicial to j se e = x o i + y o se obtiene la osicin ( t ) de l t:

Ca . 3 Movimiento en dos Dimensiones

(3.2) Se concluye que el movimiento bidimensional con acele acin constante es equivalen te a dos movimientos inde endientes en las di ecciones x e y con acele aciones c onstantes ax y ay. A esta o iedad se le llama inci io de inde endencia del m ovimiento. 3.2 MOVIMIENTO DE PROYECTILES. Cualquie objeto que sea lanzado en el ai e con una velocidad inicial vo de di eccin a bit a ia, se mueve desc ibiendo una t ayecto ia cu va en un lano. Si a a esta fo ma comn de movimiento se su one que: a) la acele acin de g avedad es co nstante en todo el movimiento (a oximacin vlida a a el caso en que el des lazami ento ho izontal del cue o en movimiento sea equeo com a ado con el adio de la Tie a) y b) se des ecia el efecto de las molculas de ai e sob e el cue o (a o ximacin no muy buena a a el caso en que la a idez del cue o en movimiento sea alta), entonces a este ti o de movimiento se le llama movimiento de oyectil y se oduce en dos dimensiones. Se elige el sistema de coo denadas (x, y) t adici onal como se ve en la figu a 3.1, donde se dibuja la t ayecto ia de una a tcula en movimiento en dos dimensiones, junto con los vecto es velocidad y acele acin d e g avedad. Su oniendo que en el instante inicial t = to el oyectil se encuent a en la osicin inicial (xo, yo) movindose con una velocidad inicial vo que fo ma un ngulo r con l horizont l, b jo l ccin de l celer cin de gr ved d g , l s ecu ciones p r l posicin del cuerpo en movimiento en dos dimensiones, se puede n escribir, p rtir de l ecu cin gener l de posicin 3.2, p r c d componente x e y por sep r do. Pero del grfico (x, y) de l figur 3.1 se pueden obtener l s r componentes de l velocid d inici l vo , de m gnitud vo, y l s componentes de r l celer cin de m gnitud g: 77

1 y = y o + voy (t t o ) + a y (t t o ) 2 2 1 a (t t o ) 2 2

(t ) = o + vo (t t o ) +

C p. 3 Movimiento en dos Dimensiones

Reempl z ndo en l s componentes de l ecu cin 3.2, se obtiene: x = x o + v o cos (t t o ) 1 y = y o + v o sen (t t o ) g (t t o ) 2 2 (3.3) Pa a las com onentes de la velocidad se obtiene: v x = vo cos v y = vo sen g (t t o ) (3.4) 78

Figur 3.1 Sistem de referenci

p r

el movimiento de un proyectil.

v ox = v o cos , v oy = v o sen ,

x = 0,

y = g

Ca . 3 Movimiento en dos Dimensiones Como no hay acele acin en la di eccin ho izontal x, la com onente x de la velocida d es constante, y como la acele acin en la di eccin ve tical y es g, las com onent es de la osicin y de la velocidad en esa di eccin son idnticas a las ecuaciones a a cada lib e, con = 90. Entonces el movimiento de proyectil se compone de l supe rposicin de un movimiento en direccin x con velocid d const nte y un movimiento en direccin y de c d libre: es el principio de superposicin del movimiento. L ecu c in de l tr yectori , esto es l curv geomtric que describe el cuerpo dur nte el movimiento del proyectil, se puede obtener despej ndo el p rmetro t - to de l e cu cin en x y reempl z ndo en l ecu cin p r y: x xo vo cos t to = ( x xo ) 1 ( x xo ) 2 g 2 y = y o + vo sen vo cos 2 vo cos 2 y = y o + t n ( x xo ) g ( x xo ) 2 2 2 2vo cos (3.5) que es l ecu cin de un p rbol , por lo t nto l tr yectori del proyectil es p r blic y qued tot lmente conocid si se conoce vo y . L velocid d del proyectil es siempre t ngente l tr yectori en cu lquier inst nte, por lo que l direcc in y l m gnitud de l velocid d en cu lquier inst nte se puede c lcul r en form geomtric de l s ecu ciones: t n = vy vx 2 , v = vx + v 2 y Ejemplo 3.1: P r un proyectil que se l nz en el inst nte inici l to = 0 desde r el origen, con un velocid d inici l vo form ndo un ngulo con l horizont l, c lcul r: ) l ltur mxim , b) l dist nci horizont l. 79

C p. 3 Movimiento en dos Dimensiones

Figur 3.2 Ejemplo 1. ) Cu ndo el proyectil lc nz su mxim ltur , l componente y de l velocid d e s cero y que no sigue subiendo, dems eso signific que l velocid d en es posi cin es horizont l, entonces de vy se obtiene: v y = vo sen gt = 0 t= vo sen g

y = y mx y mx v 1 v = vo sen o sen g o sen g 2 g 2 2 vo sen 2 = 2g b) P r determin r l dist nci horizont l, conocido t mbin como lc nce horizont l, us mos l condicin que en es posicin el proyectil se encuentr en (x,y) = (x, 0), s que igu l ndo l ecu cin p r y cero se obtiene: 80

que es el tiempo que t rd en lleg r

ltur mxim . Reempl z ndo en y

Solucin: l

situ cin se puede gr fic r en el esquem de l

figur 3.2.

C p. 3 Movimiento en dos Dimensiones 0 = vo sent t=2 vo sen g 1 2 gt 2

Como consecuencia de esta expresin para la distancia horizontal, se puede obtener el alcance mximo para una velocidad inicial vo conocida, este se produce cuando sen2 = 1, entonces sen 2 = 1 2 = 90 = 45

E1 lc nce mximo se produce p r un ngulo de l nz miento igu l 45, como se muestr en l figur 3.3 . Adems p r cu lquier ngulo distinto de 45 se puede obtener un mismo lc nce p r dos ngulos complement rios, t les como = 30 y = 60, situ cin que se ilustr en l figur 3.3b. )

b) Figur 3.3. ) Alc nce mximo, b) igu l lc nce p r ngulos complement rios. 81

que es el tiempo que demor el proyectil en lleg r l posicin (x,0), se observ que es el doble del tiempo que demor en lleg r l ltur mxim . Reempl z ndo este tiempo en x se obtiene l dist nci horizont l x o lc nce: 2 vo vo x = vo cos 2 sen = g g sen2

C p. 3 Movimiento en dos Dimensiones Ejemplo 3.2. Se l nz un proyectil de m ner que l dist nci horizont l que rec orre es el doble de su ltur mxim , c lcul r su ngulo de l nz miento Solucin: D do x = 2ymx, se pide c lcul r . De los result dos obtenidos en el ejempl o 1 p r ltur mxim y dist nci horizont l, se tiene: 2 vo = sen 2 y 2g y m x 2 vo x = sen2 g x = 2 y m x 2 2 vo vo sen2 = 2 sen 2 2g g

sen2 = sen 2 Us ndo l identid d trigonomtric sen2 = 2 sen cos y sep r ndo sen2 en sus f ctores, se obtiene l expresin: 2sen cos = ( sen )( sen ) 2 cos = sen de donde se concl ue: t n = 2 = 63.4 . Ejemplo 3.3. Se l nz un pelot desde l terr z de un edificio, con un r pide z inici l de 10 m/s en un ngulo de 20 deb jo de l horizont l, y demor 3s en lleg r l suelo. C lcul r ) l dist nci horizont l que recorre l pelot b) l lt ur desde donde se l nz, c) el tiempo que t rd en lleg r 10 m deb jo del punto de l nz miento, d) l ecu cin de l tr yectori . Solucin: se debe h cer un esquem en un sistem de referenci con l inform cin que se d en el enunci do del eje mplo; uno propi do puede ser el que se muestr en l figur 3.4, pero dej mos e n cl ro que este no es el nico posible, por ejemplo, se puede c mbi r el origen O y ubic rlo donde comienz el 82

C p. 3 Movimiento en dos Dimensiones movimiento y no en el suelo, como en este c so (y no es neces rio dibuj r el edi ficio).

Reempl z ndo los d tos inici les en l s ecu ciones gener les p r el movimiento de proyectil (ec. 3.3), se tiene:

x = xo + vo (cos )t x = 10(cos 20)t = 9.4t y = y o vo sent 5t 2 y = y o 10( se )t 5t 2 a) Pa a t =3s, eem lazando en x, x = 9.4 3 = 28.2m b) En t =3s la elota llega al suelo donde y = 0, eem lazando en y, 0 = y o 10sen20 3 5 3 2 yo = 55.2 m c) Se pide calcular t cuando y = yo 10 = 45.2 m, reemplazando en y: 45.2 = 55.2 10( sen20)t 5t 2 83

Figur 3.4 Sistem de referenci

p r

el ejemplo 3.

Ca . 3 Movimiento en dos Dimensiones 5t 2 + 3.4t 10 = 0 t= 3 .4 (3 .4) 2 + 4 5 10 3.4 14 .5 = 10 10

t1 = 1.1s y t 2 = 1 .8 s El valo vlido es t1, el tiem o t2 negativo es un esultado matemtico co ecto, e o no es fsicamente osible. d) Pa a encont a la ecuacin de la t ayecto ia y = y( x), es conveniente des eja t de la ecuacin x = 9.4 t t = x/9.4; y reemplazar est e valor de t en la ecuacin para y: y = 55.2 3.4t 5t 2 x 5x 2 y = 55.2 3.4 9.4 (9.4) 2 y ( x) = 55.2 0.36 x 0.056 x 2 Eje cicio: dibuja la ecuacin de la t ayecto ia usando Excel, a a ello da valo es a x en el ango 0 < x < 28 y calcula los valo es de y. 3.3 MOVIMIENTO CIRCUNFERENCIAL. Ot o caso a ticula de movimiento en dos dimensiones es el de una a tcula que s e mueve desc ibiendo una t ayecto ia ci cunfe encial, con velocidad v. Pa a un o bjeto que se mueve en una t ayecto ia ci cunfe encial, si la a idez v es consta nte, el movimiento se llama ci cunfe encial unifo me. Si en el instante inicial ti el objeto tiene una velocidad inicial vi y un instante oste io tf tiene una velocidad final vf, como la a idez es constante entonces vi = vf y cambia slo l a di eccin de la velocidad. Se uede calcula la acele acin media am de la a tcula usando su definicin: 84

Ca . 3 Movimiento en dos Dimensiones v v f vi = am = t t De la figura 3.5 se puede obtener v geomtricamente. En la circunferencia (figura 3 .5a) la longitud del arco s, subtendido por el ngulo, es aproximadamente igual al la do del tringulo ue une los puntos de vi y vf. Observando ue los tringulos de lad os r(s)r en la circunferencia y de lados vi(v)vf de la figura 3.5b son semejantes, entonces como vi = vf, se tiene la siguiente relacin de semejanza de tringulos:

r v v = v = s r s v Reemplazando este valor de v en la magnitud de la aceleracin me , se obtiene: am = v v s = t r t Figura 3.5 a) izquierda, b) derecha. 85

Cap. 3 Movimiento en dos Dimensiones Si t es muy pequeo, tendiendo a cero, s y v tambin lo son, y v se hace perpendicular v, por lo tanto apunta hacia el centro de la circunferencia. En el lmite cuando t 0 , am a y se puede escribir: v s v v2 s v a = lim = lim = va= t 0 r t r t 0 t r r Entonces en el movimiento circunferencial con rapidez constante, la aceleracin ap unta hacia el centro de la circunferencia (ya que en el lmite v apunta hacia el ce ntro), por lo que se llama aceleracin centrpeta ac (tambin se usan los nombres cent ral o radial) y el vector con su magnitud es:

(3.6) donde es un vecto unita io adial di igido desde el cent o de la ci cunfe enc ia hacia fue a, que se muest a en la figu a 3.5a. Pa a el caso en que du ante el movimiento ci cunfe encial de la a tcula cambia l a velocidad tanto en di eccin como en magnitud, la velocidad siem e es tangente a la t ayecto ia (figu a 3.6), e o aho a la acele acin ya no es adial, sino que fo ma un ngulo cualquie a con la velocidad. En este caso es conveniente esc ibi la acele acin en dos com onentes vecto iales, una adial hacia el cent o a y ot a tangente a la t ayecto ia at, entonces a se esc ibe como: a = a + a t = a ( ) + a t t ,

donde es un vecto unita io tangente a la t ayecto ia, en la di eccin del mot vim iento. En esta ecuacin, la com onente adial de la acele acin es la acele acin cent eta o iginada o el cambio en la di eccin de la velocidad y la 86

v2 ac =

v2 r ac = (

Ca . 3 Movimiento en dos Dimensiones com onente tangencial es oducida o o lo tanto su valo num ico es: at = dv dt Figu a 3.6 Entonces la acele acin total en el movimiento ci cunfe encial es:

2 (3.7) En la figu a 3.7 se ven los vecto es unita ios a a un movimiento ci cunfe encia l. Obse va que en el caso del movimiento ci cunfe encial unifo me v = cte, entonces dv/dt = 0 y a = a = ac . Y si no cambia la di eccin de v , , a = 0, el movimiento es en una dimensin con a = at = dv / dt . Aunque esta dedu ccin fue ealizada a a el movimiento ci cunfe encial, es vlida a a cualquie t a yecto ia cu va, conside ando el adio de cu vatu a de la t ayecto ia desde el u nto donde se miden las va iables hasta el cent o de cu vatu a de la t ayecto ia en ese unto. 87

v dv

a =

+ t

dt

el cambio en la magnitud de la velocidad,

Ca . 3 Movimiento en dos Dimensiones Figu a 3.7 Ejem lo 3.4. Calcula la a idez o bital de la t aslacin te est e al ededo del Sol y la acele acin cent eta co es ondiente.

Solucin: la distancia media ent e el Sol y la Tie a es dST = 149.6 x 106 km. La Tie a com leta una vuelta en to no al Sol en un ao o 365.242199 das, entonces la a idez o bital es: x 2 = t t 2 d TS 2 1.496 1011 m m km v= = 2.98 10 4 = 29.8 365.24 24 3600 s s s x = x 0 + v(t t 0 ) x x 0 = v(t t 0 ) v = Notar que la Tierra tiene una rapidez de traslacin enorme en su movimiento en tor no al Sol, es uno de los objetos mas veloces que cualquier otro que se mueva sob re la superficie terrestre. Pero su aceleracin centrpeta es muy pequea (comparada c on g por ejemplo), como se obtiene del calculo siguiente: (2.98 10 4 ) 2 v2 v2 m ac = = = = 5.9 10 3 2 11 d TS 1.496 10 s 88

Ca . 3 Movimiento en dos Dimensiones 3.4 VELOCIDAD Y ACELERACIN ANGULAR. Una a tcula que gi a ubicada en un unto P a una distancia del o igen, desc ib e una ci cunfe encia en to no al o igen. La osicin de la a tcula se uede ex es a en coo denadas ola es ( ,), donde la nica coordenada ue cambia en el tiempo e s el ngulo . Si la partcula se mueve desde el eje x positivo, donde = 0 hasta un pu nto P, el arco de longitud s recorrido por la partcula, y el ngulo, como se ve en la figura 3.8, se definen como: s = r = s r (3.8) Se observa ue el ngulo es una variable adimensional, pero se le asigna como unid ad de medida el nombre del ngulo, llamado radian, con smbolo rad. De la ecuacin 3.8 , se define un radian como el ngulo subtendido por un arco de circunferencia de i gual longitud ue el radio de la misma. Como en una circunferencia, s = 2 , y 2 ( ad) = 360, se uede encont a la elacin ent e adianes y g ados: (rad ) = 2 360 De a u se deduce ue el valor en grados de un radian es 1 rad = 57.3, y ue por ej emplo, 45 = /4 ad. Figu a 3.8 89

Ca . 3 Movimiento en dos Dimensiones Cuando una a tcula se mueve desde P hasta Q segn la figu a 3.9, en un inte valo d e tiem o t, el radio se mueve un ngulo , ue es el desplazamiento angular. De manera anloga al movimiento lineal, se definen la rapidez angular y aceleracin angular c omo: = d , dt = d dt Sus unid des de medid son r d/s y r d/s2, record ndo que el r di n no es un un id d de medid , por lo que en el nlisis dimension l se obtienen p r est s v ri bles l s dimensiones de 1/s y 1/s2. De l definicin de est s v ri bles se deduce dems que p r l rot cin de un cuerpo lrededor de un eje, tod s l s p rtcul s tie nen l mism velocid d ngul r y l mism celer cin ngul r.

3.4.1 Cinemtica de rotacin. El desplazamiento, velocidad y aceleracin angular son anlogos a sus similares vari ables lineales. As las ecuaciones cinemticas del movimiento de rotacin con acelerac in angular constante tienen la misma forma ue las correspondientes al movimiento lineal haciendo los reemplazos x por , v por y a por , por lo que l s ecu ciones cinemtic s del movimiento ngul r son: 90

Figur 3.9 Despl z miento

ngul r desde P a Q.

C p. 3 Movimiento en dos Dimensiones = o + o (t t o ) + (t t o ) 2 1 2 (3.9) = o + (t to ) (3.10) 3.4.2 Relacin ent e las va iables angula es y lineales. Pa a toda a tcula que gi a desc ibiendo una t ayecto ia ci cunfe encial, existe una elacin ent e las magnitudes angula es con las co es ondientes lineales. Si la a tcula eco e una distancia lineal s, movindose un ngulo sobre una trayectori a circunferencial de radio r, tiene una velocidad ue por ser tangente a la tray ectoria se llama velocidad tangencial, y tiene aceleracin tangencial y centrpeta, entonces las relaciones entre las variables son:

s = r v= ds d (r ) d = =r v = r dt dt dt dv d (r ) d at = = =r at = r dt dt dt v = r 2 r (3.11)

91

= c + t2

L m gnitud de l

celer cin en el movimiento circunferenci l es:

C p. 3 Movimiento en dos Dimensiones Por ltimo se debe decir que se us comnmente como unid d de medid de l v ri cin ngul r el trmino revolucin, que corresponde un vuelt complet , 360 2 ( ad). Y a velocidad angula se usan las vueltas o evoluciones o minuto, con unidad d e medida ev/min. Siem e se debe tene en mente que las vueltas o evoluciones son medidas de ngulo, o lo tanto son un nme o adimensional.

12

Ejemp o 3.6. Ca cu ar a rapidez angu ar, a ve ocidad tangencia y ace eracin ce ntrpeta a) en un punto sobre e ecuador para a rotacin terrestre, b) para a tras acin de a Tierra en torno a So . So ucin: a) a Tierra da una vue ta en 23 hora s 56' 4" o un da y su radio medio es 6371 km. Para un punto sobre e ecuador se t iene: =

b) La traslacin de la Tierra en torno al Sol se completa en un ao y la distancia m edia de la Tierra al Sol es aproximadamente 150106 km: 92

v2

ad 2 m 6 ac = = 2 RT = 7.27 10 5 6.371 10 m = 3.37 10 RT s s2

vt = RT = 7.27

10 5

rad 2 2 = = 7.27 2

10 5 Tda 86400 s s ad m (6.371

10 6 m) = 463.3 s s

ev ev 1 min 2 ( ad )

ad = 12

= 1.26 1.26s 1 min min 60s 1rev s

Ejem lo 3.5. T ansfo ma 12

ev/min a ad/s. Solucin:

Cap. 3 Movimiento en dos Dimensiones =

v2 ad 3 m 11 ac = = 2 R = 1.99 10 7 1.5 10 m = 6 10 R s s2 Ejemplo 3.7. Un disco de 10 cm de radio que gira a 30 rev/min demora un minuto e n detenerse cuando se lo frena. Calcular: a) su aceleracin angular, b) el nmero de revoluciones hasta detenerse, c) la rapidez tangencial de un punto del borde de l disco antes de empezar a frenar, d) la aceleracin centrpeta, tangencial y total para un punto del borde del disco. Solucin: Datos: r = 0.1m, t = 1 min = 60 s. Pri mero se transforman las 30 rev/min a rad/s. o = 30

(a) Usando las ecuaciones de cinemtica de otacin: = o + (t t o ) , se des eja , ndo se detiene = 0: 0 = o + t = o t =

= o + o (t t o ) + (t t o )2 eem lazando los datos, se obtiene: 1 2 93

(b) Se

ide calcula , usando la ecuacin

3.14

ad s ad = 0.05 2 60 s s

ev 2 ( ad ) 1 min ad

vt = RST = 1.99

10 7

= 3.14 min 1 ev 60s s

2 2 ad = = 1.99 2

10 7 Tao 365

86400 s

ad m km

1.5

1011 m = 2.98

10 4 = 29.8

Ca . 3 Movimiento en dos Dimensiones o = 3.14 rad 1 2 60S 0.05 (60) = 94.2rad s 2 1rev = 94.2rad = 15rev 2 ( ad )

(d) La acele acin cent eta, tangencial y total es: v 2 (0.314 ) m ac = = = 0.98 2 0.1 s 2

m s2 0.98 m s2

(0.98)2 + (0.005)2 3.5 MOVIMIENTO RELATIVO. P r un p rtcul en movimiento, observ dores ubic dos en sistem s de referenci diferentes medirn v lores distintos de l s v ri bles cinemtic s, unque el movimie nto es el mismo. Por ejemplo, un objeto que se dej c er desde un vehculo en movi miento: el observ dor en el vehculo que dej c er el objeto lo ve c er vertic lme nte, pero un observ dor en tierr lo ve moverse como movimiento p r blico en dos dimensiones. Es un mismo movimiento visto en form diferente por observ dores en sistem s de referenci diferentes, se ll m movimiento rel tivo, se produce en dos dimensiones. P r describir el movimiento rel tivo consider mos observ dores en dos sistem s de referenci : un sistem de referenci (x,y) fijo respecto l Tierr con origen O y otro sistem de referenci (x,y) que se mueve respecto l fijo, con 94

c +

t2 =

at = = 0.1

0.05 = 0.005

(c) Se s s

uede calcula la a idez con la ecuacin: v =

v = 0.1m

3.14 ad m = 0.314

C p. 3 Movimiento en dos Dimensiones origen O, como se ve en l figur 3.10, donde los ejes x y x estn superpuestos. Sup ong mos dems que el sistem de referenci mvil se mueve en lne r rect en direccin x con velocid d const nte u respecto l sistem de referenci fijo. Figur 3.10. Vectores de posicin de un p rtcul en movimiento rel tivo. L posicin de l p rtcul P en movimiento respecto l sistem de referenci fijo s er r y respecto l sistem de referenci mvil ser r. Si en to = 0 mbos orgenes coinc iden, xo = 0, y como u = cte, l posicin del sistem de referenci mvil en el inst nte t ser: r r r 1r x = x0 + ut + t 2 2 r r x = ut Del diagrama de vectores de la figura 3.10, se obtiene que la posicin de la partcu la cumple la siguiente relacin vectorial: r r r r = x + r ' r r r r = ut + r ' De esta expresin se puede obtener la velocidad de la partcula 95

Cap. 3 Movimiento en dos Dimensiones r r dr dr ' r = +u dt dt r r r v = v '+u

r r a = a' Se concluye que dos observadores ubicados en sistemas de referencia diferentes m iden velocidades diferentes para la partcula, pero si la velocidad del sistema de referencia mvil es constante, los dos miden la misma aceleracin de la partcula en movimiento. Usaremos la siguiente notacin: si P es la partcula, F el sistema de re ferencia fijo y M el sistema de referencia mvil, entonces la velocidad vPF de la partcula respecto al sistema de referencia fijo es igual a la velocidad vPM de la partcula respecto al sistema de referencia mvil ms la velocidad vMF del sistema de referencia mvil respecto al sistema de referencia fijo, esto es: r r r vPF = vPM + vMF (3.12) 96

dv dv du = + dt dt dt

du = 0 , entonces como u = cte dt

Entonces, s igual a l sistema se conoce btene de

la velocidad v de la partcula medida en el sistema de referencia fijo e la velocidad v es ecto al sistema de efe encia mvil ms la velocidad u de de efe encia mvil es ecto al sistema de efe encia fijo. Esta ecuacin como la t ansfo macin galileana de velocidades. La acele acin se uede o ivando la velocidad

Cap. 3 Movimiento en dos Dimensiones Ejemplo 3.8. La rapidez del agua de un ro es 5 km/h uniforme hacia el este. Un bo te que se dirige hacia el norte cruza el ro con una rapidez de 10 km/h respecto a l agua. a) Calcular la rapidez del bote respecto a un observador en la orilla de l ro. b) Calcular la direccin donde debe dirigirse el bote si se quiere llegar jus to al frente en la orilla opuesta. c) Calcular ahora su rapidez respecto a la ti erra. Solucin: El sistema de referencia fijo es la tierra, el sistema de referenc ia mvil el ro y la partcula es el bote, entonces: vPM = 10 km/h vMF = 5 km/h vPF = ? : rapidez del bote (partcula) respecto al agua (SR mvil) : rapidez del agua (SR mvi l) respecto a tierra (SR fijo) : rapidez del bote (partcula) respecto a tierra (S R fijo) a) Es conveniente hacer el diagrama de vectores de velocidades, que se muestra e n la figura 3.11a: a. b. Figura 3.11 Ejemplo 8. La magnitud de la velocidad del bote respecto a tierra vPF, que tiene una compon ente a favor de la corriente, se puede calcular del tringulo rectngulo de vectores de la figura 3.11a 2 2 2 v PF = v PM + v MF 2 v PF = 10 2 + 5 2 = 125 v PF = 11.2 km h 97

Cap. 3 Movimiento en dos Dimensiones su direccin es: tan = v MF 5 1 = = = 26.6 NE v PM 10 2 b) Si uiere llegar justo al frente desde donde sale, como la corriente del ro lo arrastra hacia el este, haciendo el diagrama de vectores, figura 3.11b, se obse rva ue debe apuntar en direccin h ci el noroeste, entonces: sen = v MF 5 1 = = = 30 v PM 10 2 c) Ahor , l r pidez vPF es: 2 2 2 2 2 2 v PM = v MF + v PF v PF = v PM v MF 2 v PF = 10 2 5 2 = 75 v PF = 8.7 km h Como debe remar con una componente de la velocidad en contra de la corriente, la velocidad resultante del bote en este caso es menor que en la parte a), donde u na componente de la velocidad es a favor de la corriente. 98

Cap. 3 Movimiento en dos Dimensiones PROBLEMAS. 3.1. Se dispara un proyectil desde el piso con velocidad r v = (12i + 24 ) m / s . a) Cul es la velocidad despus de 4 s? b) Cules j es la posicin del punto en el cual la altura es mxima? c) Cul es la distancia horizontal? R: a) 12i-15j m/s, b) 30i+3 0j m. 3.2. Desde el borde de un acantilado se lanza una piedra horizontalmente con una rapidez de 15 m/s. El acantilado est 50 m de altura respecto a una playa horizon tal. a) En que instante la piedra golpear la playa bajo el acantilado?, b) Dnde golp ea? c) Con qu rapidez y ngulo golpear la playa? d) Encontrar la ecuacin de la trayect oria de la piedra. R: a) 3.16s, b) 47.4m, c) 35m/s, 65 , d) y=50-(x2/45). 3.3. Un baln de ftbol que se patea a un ngulo de 50 con la horizontal, recorre una distancia horizontal de 20 m antes de chocar contra el suelo. Calcular a) la rapidez inic ial del baln b) el tiempo que permanece en el aire y c) la altura mxima que alcanz a. R: a) 14.2m/s, b) 2.2s, c) 6m. 3.4. Se lanza horizontalmente una pelota desde la parte superior de un edificio que tiene 35 m de alto. La pelota choca contra el piso en un punto que se encuentra a 80 m de la base del edificio. Calcular: a) el tiempo que la pelota se encuentra en el aire, b) su rapidez inicial y c) l a velocidad justo antes de que choque contra el suelo. R: a) 2.6s, b) 30 m/s, c) 30i-26j m/s. 3.5. Se lanza una piedra de manera que la distancia horizontal que recorre es el triple de su altura mxima, calcular su ngulo de lanzamiento. R: 53. 1 . 3.6. En el prximo partido de Chile con la seleccin de Micomicon, el Che Copete d eber patear un tiro libre desde un punto a 25m del arco cuya altura es 2.5m. Cuan do patea, la pelota sale del csped con una rapidez de 20m/s en un ngulo de 20 sobre la cancha. Suponiendo que la pelota no sufre ninguna alteracin de su trayectoria , a) se convierte o no el gol? b) Con qu velocidad cruza por el arco? c) Obtenga la ecuacin de la trayectoria de la pelota. (Por cuanto perder Chile con los Micomico nes). R: a) si, pasa a 0.25m del suelo, b) 18.8i-6.5j m/s. 99

Cap. 3 Movimiento en dos Dimensiones 3.7. Se lanza un cohete formando un ngulo de 60 con la horizontal con una rapidez inicial de 100 m/s. El cohete se mueve a lo largo de su direccin inicial de movim iento con una aceleracin de 30 m/s2 durante 3s. En ese instante deja de acelerar y empieza a moverse como un proyectil. Calcular: a) la altura mxima alcanzada por el cohete; b) su tiempo total de vuelo, c) la distancia horizontal. R: a) 1730m , b) 38s, c) 3543m. 3.8. Un proyectil se dispara desde cierta altura y0 en un ngu lo de 45 , con la intencin que golpee a un mvil que se mueve con velocidad constante de 21 m/s hacia la derecha, que se encuentra ubicado a 70 m del origen sobre el eje x en el instante del disparo. Si el proyectil impacta al mvil al cabo de 10 s, calcular a) la rapidez inicial del proyectil, b) su posicin inicial, c) su alt ura mxima desde el suelo. R: a) 39.6m/s, b) 220m, c) 259.2m. 3.9. Katy le lanza u n chicle (nuevo) desde una altura de 1.5 m a Pepe, que se encuentra separado a 3 m de Katy. El chicle pasa un segundo despus a una altura de 1 m por donde est Pep e, pero como l estaba pajareando no lo toma. a) Hace un esquema de la situacin en u n SR. b) Calcula la velocidad inicial que Katy le im ime al chicle. c) A qu dist ancia det s de Pe e cae el chicle?, en este caso qu se debe su one ? d) Dete mina la ecuacin de la t ayecto ia del chicle de Katy. R: b)3i+4.5j m/s, c)0.45m. 3.10 . Lucho se encuent a a 5m de una a ed ve tical cuando lanza una elota de bsquet bol desde 2.25m de altu a, con una velocidad inicial de 10i +10j m/s. Cuando la elota choca con la a ed, la com onente ho izontal de la velocidad de la elot a se invie te y la com onente ve tical no cambia su di eccin ( e o si su magnitud ). a) Hace el esquema de la situacin. b) A que distancia de Lucho toca el suelo l a elota? R: b) 12m det s. 3.11. Un t en se mueve con a idez constante de 54 km/ h. Desde una ventana del t en ubicada a 2 m del suelo, un cab ochico ti a un obj eto ho izontal y e endicula mente a la di eccin de movimiento del t en, con una a idez de 5 m/s. Calcula la osicin donde cae el objeto es ecto al unto de l anzamiento. R: 3.15i+9.45j+0k m. 100

Ca . 3 Movimiento en dos Dimensiones

3.12. Se a unta un ifle ho izontalmente a t avs de su mi a hacia el cent o de un blanco g ande que esta a 200 m. La velocidad inicial de la bala es de 500 m/s. a) En dnde gol ea la bala en el blanco? b) Calcula el ngulo de elevacin del can a a da en el cent o del blanco. R: a) 0.8m debajo de la altu a del ifle, b) 0.23. 3 .13. Un can dis a a un oyectil con una a idez inicial vo inclinado en un ngulo . Si el ngulo se c mbi , el alcance del proyectil aumenta en una distancia D. Dem uestre que 2 vo (sen2 sen2 ) D= g 3.14. L dist nci horizont l mxim l que puede p te r l pelot un rquero es 120 m. En un s que desde el rco, golpe l pelot con l mism r pidez inici l con l que lc nz es dist nci mxim , pero form ndo un ngulo de 25 con l horizo nt l. C lcul r que dist nci del rco lleg r l pelot con un chute del rquero . 3.15. Un pulg puede s lt r un ltur vertic l h. ) Cul es l dist nci horiz ont l mxim que puede recorrer? b) Cul es su perm nenci en el ire en mbos c sos? 3.16. Un c min se mueve l norte con un velocid d const nte de 10 m/s en un tr mo de c mino horizont l. Un c brochico que p se en l p rte posterior del c min dese l nz r un pelot mientr s el c min se est moviendo y tr p rl despus de que el c min h y recorrido 20 m. ) Despreci ndo l resistenci del ire, qu ngulo d e l vertic l deber ser l nz d l pelot ? b) Cul debe ser l r pidez inici l de l pelot ? c) Cul es l form de tr yectori de l pelot vist por el c brochico ? d) Un person sobre l tierr observ que el much cho l nz l pelot y l t r p . En este m rco de referenci fijo del observ dor, determine l form gener l de l tr yectori de l pelot y l velocid d inici l de est . 3.17. Un c broc hico tir un pelot l ire lo ms fuerte que puede y luego corre como un liebre p r poder tr p r l pelot . Si su r pidez mxim en el l nz miento de l pelot es 20 m/s y su mejor tiempo p r recorrer 20 m es 3 s, c lcul r l ltur de l pelot p r que pued tom rl . 101

C p. 3 Movimiento en dos Dimensiones

3.18. Un pelot de golf s le desde el piso en un ngulo y golpe un rbol un ltur H del suelo. Si el rbol se encuentr un dist nci horizont l D del punto de l nz miento, ) demuestre que t n = 2H/D. b) C lcul r l r pidez inici l de l pelot en trminos de D y H. 3.19. Un p rtcul comienz gir r desde el reposo h st un r pidez ngul r de 15 r d/s en 3 segundos. C lcul r ) su celer cin n gul r, b) el nmero de vuelt s en ese tiempo. 3.20. Un rued de biciclet de 30 c m de r dio comienz gir r desde el reposo con un celer cin ngul r const nte de 3 r d/s2. Despus de 10 segundos c lcul r: ) su r pidez ngul r, b) el despl z miento ngul r, c) l r pidez t ngenci l de un punto del borde, d) su celer cin tot l p r un punto del borde. R: ) 30 r d/s, b) 150 r d, c) 9 m/s, d) 270 m/s 2. 3.21. Busque l inform cin neces ri p r c lcul r l celer cin centrpet l ni vel del m r de un punto sobre el Ecu dor, en Concepcin, en 45 de l titud sur y en el Polo Sur. R: 0.034 m/s2, 0.027 m/s2, 0.024 m/s2, 0. 3.22. L rbit de l Lun lrededor de l Tierr es proxim d mente circul r, con un r dio promedio de 3.8 4 x 108 m. L Lun complet un revolucin en torno l Tierr y en torno su ej e en 27.3 d s. C lcul r ) l r pidez orbit l medi de l Lun , b) l r pidez ng ul r, c) celer cin centrpet . R: ) 1023m/s, b) 2.7x10-6 r d/s, c) 2.7x10-3 m/s2. 3.23. C lcul r l r pidez orbit l medi de l Tierr en torno l Sol y su r pid ez ngul r en torno su eje de rot cin. R: 29.8km/h, 7.27x10-5 r d/s. 3.24. A l p rtcul del extremo de un pndulo de l rgo un metro se l h ce gir r de form t l que su movimiento describe un circunferenci en un pl no horizont l. Cu ndo el pndulo se h desvi do 30 de l vertic l, l p rtcul complet un vuelt c d 3 se gundos. C lcul r ) su r pidez ngul r b) su r pidez t ngenci l, c) su celer cin centrpet . R: ) 2.1 r d/s, b) 1.05 m/s, c) 2.2 m/s2. 3.25. Un centrfug cuyo t mbor tiene 50 cm de dimetro, comienz gir r desde el reposo h st lc nz r un r pidez ngul r de 1000 rpm en 10 s. ) C lcul r su celer cin ngul r. b) Si des pus de los 10 s gir con r 102

C p. 3 Movimiento en dos Dimensiones pidez const nte dur nte 5 minutos, c lcul r el nmero de vuelt s que d c d minut o. c) c lcul r l r pidez t ngenci l, celer cin centrpet y t ngenci l en l s p r edes del t mbor. d) Si despus de los 5 minutos t rd 20 s en detenerse, c lcul r su celer cin ngul r. R: )10.5r d/s, b)103, d)-5.2/s2. 3.26. Un disco comienz gir r desde el reposo con celer cin ngul r const nte h st un r pidez ngul r de 12 r d/s en 3 s. C lcul r: ) l celer cin ngul r del disco, b) el ngulo qu e describe. R: ) 4r d/s2, b) 18r d. 3.27. Un motor elctrico h ce gir r un disco r zn de 100 rev/min. Cu ndo se p g el motor, su celer cin ngul r es 2 r d/s2. C lcul r: ) el tiempo que demor el disco en detenerse, b) el nmero de vuelt s que gir en ese tiempo. R: ) 5.2 s, b) 27.5 r d. 3.28. Un disco comienz gir r desde el reposo con celer cin ngul r const nte de 5 r d/s2 por 8 s. Luego el disco se llev l reposo con un celer cin ngul r const nte en 10 revoluciones. C lcul r: ) su celer cin ngul r, b) el tiempo que demor en detenerse. R: ) 1 2.7 r d/s2, b) s. 3.29. Un volante de 2 m de dimet o, comienza a gi a desde el e oso con acele acin angula constante de 4 ad/s2. En el instante inicial un un to P del bo de del volante fo ma un ngulo de 57.3 con la ho izontal. Calcula a a el instante 2 s: a) su a idez angula , b) la a idez lineal de P, c) la acele acin lineal de P, d) la osicin de P. R: a) 8 ad/s, b) 8 m/s, d) 9 ad. 3.30. Un disco de 8 cm de adio, gi a con una a idez angula constante de 1200 ev/min. Calcula : a) la a idez angula del disco, b) la a idez lineal de un unto a 3 cm del disco, c) la acele acin adial de un unto en el bo de del disco, d) la di stancia total eco ida o un unto del bo de en 2 s. R: a) 126 ad/s b) 3.8 m/ s, c) 1.26 km/s2, d) 20.1m. 3.31. La osicin de una a tcula que se mueve en el l ano xy va a con el tiem o segn la ecuacin = a cos( t)i + a sen( t)j, en donde y a se miden en m, en s 1 y t en s. a) Demuest e que la t ayecto ia de la a tcula e s una ci cunfe encia que tiene a m de adio y su cent o est en el o igen. b) dete mine los vecto es de velocidad y de acele acin c) 103

Ca . 3 Movimiento en dos Dimensiones

Demuest e que el vecto acele acin siem e a unta hacia el o igen (o uesto a ) y tiene una magnitud de v2/ . 3.32. Su e man (no el Va gas, el de ve dad), que le anda echando el ojo a Luisa Lane, vuela hacia el no este, donde se encuent a el la, con una a idez de 54 km/h es ecto al ai e. El viento so la hacia el no oes te a 7.5 m/s es ecto de tie a. a) Calcula la a idez de Su e man es ecto de tie a. b) Su e man, que no a ob Fsica I, no se encuent a con Luisa o qu? R: a) 6 0.3 km/h, b) o que se desva 26.6. 3.33. Un cndo (no el Rojas, sino uno de ve dad) vuela hacia el este con una a idez de 12 km/h es ecto del ai e, en esencia de un viento que so la hacia el no este (a 45) con una a idez de 5 m/s. a) Calcu la la a idez esultante del cndo . b) Qu distancia se desva cada minuto es ecto a la di eccin este? R: a) 27.8km/h, b) 212m. 3.34. El iloto de un avin se o ienta hacia el oeste en esencia de un viento que so la hacia el su a 75 km/h. Si la a idez del avin es ecto al viento es 500 km/h, a) Cul es su a idez es ecto a l a tie a? b) en qu di eccin se desva el avin? c) en qu di eccin debe di igi se el a a i hacia el oeste? d) En este caso cul se su a idez es ecto a la tie a? R: a ) 506 km/h, b) 8.5, c) 8.6, d) 494.3 km/h. 3.35. El iloto de una avin obse va que la b jula indica que va di igindose hacia el oeste. La a idez del avin es ecto al ai e es de 150 km/h. Si existie a un viento de 30 km/h hacia el no te, calcule la velocidad del avin es ecto a la Tie a. R: 153km/h, 11.3NW. 3.36. Un escado desea c uza un o de 1 km de ancho, el cual tiene una co iente de 5 km/h hacia el no te. El escado est sob e el lado oeste. Su bote se im ulsa con una a idez de 4 km/h es ecto del agua. a) En qu di eccin debe a unta a a hace el c uce en un tiem o mnimo?, b) Cunto tiem o le toma a a c uza ?, c) Dete mine la velocidad del bote con es ecto a un obse vado estaciona io en la Tie a, d) Encuent e el des lazamiento final co iente abajo. R: a) este, b) 15min, c) 6.4km/h, 51.3 NE, d) 1.25 km. 104

Ca . 4 Dinmica de la a tcula. CAPTULO 4. DINMICA DE LA PARTCULA. 4.1 INTRODUCCIN. En este ca tulo se sigue conside ando un modelo a a hace el est udio de la dinmica slo a a el caso de a tculas. Un modelo se usa a a e esenta la ealidad fsica y debe tene en cuenta dos as ectos conflictivos ent e s: a) ti ene que se lo bastante sim le a a como a a se elabo ado con mtodos matemticame nte igu osos, b) debe se ealista a a que los esultados obtenidos sean a lic ables al oblema conside ado. Estos dos as ectos hacen que la sencillez del mod elo, su belleza matemtica, sea incom atible con la fidelidad al oblema eal. La dinmica estudia el movimiento de los cue os conside ando las causas que lo od ucen. Es una ama de la Mecnica que aba ca casi toda la Mecnica Clsica. En la Mecnic a Clsica se est inge el estudio a los cue os ( a tculas) g andes com a ados con el tamao de un tomo (~10 10 m) y a a velocidades equeas com a adas con la de la l uz (~3x108 m/s). Isaac Ne ton (1642 1727) es el inci al c eado de la Mecnica C lsica. La Mecnica Relativista estudia el movimiento de las a tculas subatmicas, que se mueven a muy altas velocidades, es ms gene al que la Mecnica Clsica a la que in cluye como caso a ticula . Su c eado fue A. Einstein (1879 1955). En los ime os estudios, Galileo Galilei (1564 1642), hizo un g an avance en la com ensin d el movimiento. Las ideas de Galileo e an evoluciona ias a a su oca, l o uso l a teo a cientfica que la Tie a gi aba en to no al Sol, teo a cont a ia a las doct inas de la iglesia que im onan la c eencia que la Tie a e a el cent o del Unive so, sin tene fundamentos a a hace esa afi macin. Quienes se o onan a esas c een cias e an seve amente castigados, con enas tales como mo i quemado en la hogue a u ot as ba ba ies im uestas o la eligin catlica. Galileo se encont en esa si tuacin elig osa, o lo que no udo ublica sus esultados y fue obligado a et acta se blicamente. Poste io mente, la inquisicin es aola o icio que todas sus unive sidades a oba an y estudia an la tesis de Galileo. Du ante el Jubileo 200 0 la Iglesia Catlica tuvo que edi e dn al mundo cientfico o no habe c edo en l a teo a de Galileo y le idi e dn a Galileo mismo. Pe o un filsofo contem o neo de G alileo, Gio dano B uno (1548 1600) tuvo un final t gico, ya que 105

Ca . 4 Dinmica de la a tcula. mu i en Roma en 1600 quemado en la hogue a de la Inquisicin, o defende las mism as ideas de Galileo. En la actualidad, la Iglesia Catlica contina con sus ideas e t g adas y dictato iales o que, o ejem lo, ace ta la tesis abo tiva de la ldo a del da des us, a esa de que se ha demost ado cientficamente que no es abo tiva, o se o ona a la a obacin de leyes como la Ley del Divo cio, o one t abas a a la ealizacin del og ama Jo nadas de Conve sacin, Afectividad y Sexualidad, JOCAS, de educacin sexual en los Liceos. Sin emba go la iglesia se esiste a ace ta las sanciones en cont a de sus sace dotes que son acusados de abusos deshonestos, y los defiende Cmo eso va a se algo ace table? Ojal que no se deba es e a ot os 50 0 aos a a que la iglesia econozca este nuevo e o . Antes de Galileo la mayo a d e los filsofos ensaba que se necesitaba una influencia exte na a a mantene a un cue o en movimiento. C ean que un cue o se encont aba en su estado natu al cuan do estaba en e oso, y que a a que el cue o se movie a en lnea ecta con veloci dad constante, tenia que move lo continuamente algn agente exte no, de ot a mane a natu almente se detend a. Pa a oba esa idea, Galileo em ez o encont a una fo ma de libe a a un cue o de toda influencia exte na. En la natu aleza eso no se uede log a , o que an cue os muy alejados de un cue o de ueba ueden ej e ce una influencia sob e l y cambia su movimiento. Pe o se uede hace que las influencias exte nas sean muy equeas (es el modelo) y ensa que ealmente no e xisten a a tene una idea de cmo se a el movimiento. La ex e iencia de Galileo fu e desliza un bloque de made a sob e una su e ficie bajo una influencia exte na ( o ejem lo la mano que lo em uja), si se elimina la influencia exte na el bloq ue se detiene, o eso los filsofos ensaban que e manentemente tenia que esta actuando la influencia exte na a a mantene el movimiento. Pe o si se elige com o cue o una esfe a y se hace desliza sob e una su e ficie muy lisa, al one la en movimiento lo ha con mucha facilidad sin ninguna influencia exte na, (el con tacto ent e las dos su e ficies es ot a influencia exte na que se des ecia). En el caso que no exista ninguna influencia exte na sob e un cue o des us que se l o one en movimiento, nunca ms se detend a. A la influencia exte na que hace que u n cue o este detenido o en movimiento se le llama una fue za. Qu es fue za? En la vida cotidiana se conside a fue za a una sensacin comn asociada con la dificultad a a move o levanta un cue o. En Fsica se identifica una fue za o el efecto que oduce. Uno de los efectos de una fue za es cambia el 106

Ca . 4 Dinmica de la a tcula.

estado de e oso o de movimiento del cue o, ms conc etamente, una fue za cambia la velocidad de un objeto, es deci oduce una acele acin. Cuando se a lica una fue za sob e un cue o y no se oduce movimiento, entonces uede cambia su fo ma, an si el cue o es muy gido. La defo macin uede o no se e manente. Entonces los efectos de la fue za neta son dos: cambia el estado de movimiento de un cu e o o oduci una defo macin, o ambas cosas simultneamente. No malmente sob e un cue o ueden actua va ias fue zas, entonces el cue o acele a cuando el efect o de la fue za neta que acta sob e l no es ce o. Se llama fue za neta o fue za es ultante a la suma de todas las fue zas que actan sob e un cue o. Si la fue za ne ta es ce o, la acele acin es ce o, el movimiento es con velocidad igual a ce o (c ue o detenido) o con velocidad constante. Cuando un cue o est en e oso o se mu eve con velocidad constante, se dice que est en equilib io. Pa a una fue za usa e mos el smbolo F. Se ueden distingui dos g andes clases de fue zas: fue zas de c ontacto, e esentan el esultado del contacto fsico ent e el cue o y sus al ede do es, o ejem lo move un ca o o esti a un eso te; y fue zas de accin a dist ancia que actan a t avs del es acio sin que haya contacto fsico ent e el cue o y s us al ededo es, o ejem lo la fue za con que la Tie a at ae a los cue os que caen en cada lib e. Todas las dife entes fo mas de fue zas se encuent an dent o d e esas dos g andes clasificaciones. Pa a desc ibi el mundo, la fsica contem o nea ecu e a cuat o inte acciones o fue zas fundamentales, que actan sob e las a tc ulas de mate ia (y sob e las anti a tculas), vehiculadas o a tculas llamadas ve cto es de inte accin, que son: fotn (inte accin elect omagntica), bosn (inte accin d ), glun (inte accin fue te) y g avitn (inte accin g avitacional). 1) Fue zas elect o magnticas de at accin o e ulsin ent e a tculas ca gadas en e oso o en movimiento, ex lica la cohesin de los tomos, es mucho ms intensa que la fue za g avitacional. 2) Fue zas nuclea es intensas ent e a tculas subatmicas, es onsable de la existe ncia del ncleo atmico asegu a la cohesin inte na de los constituyentes del ncleo atmi co, otones y neut ones, y es es onsable de un g an nme o de eacciones y de de sinteg aciones; es la de mayo magnitud (102 103 veces la fue za elect omagntic a). 107

Ca . 4 Dinmica de la a tcula. 3) Fue zas nuclea es dbiles de co to alcance, ige algunos ocesos adiactivos, establece la estabilidad de algunos ncleos, es va ios denes de magnitud (1012) me no que la fue za elect omagntica. 4) Fue za de at accin g avitacional ent e cue os debido a sus masas, ent e ot as cosas hace que caigan las manzanas y que suba la ma ea, es la fue za de meno magnitud com a ada con las ot as. Pa a que el c once to de fue za sea exacto se debe establece un mtodo a a medi la. Una fue za se uede medi o el efecto que oduce. Po ejem lo se uede usa la defo mac in que una fue za oduce en un eso te, como en la figu a 4.1. Si se a lica una fue za ve ticalmente a un eso te y se esti a una unidad (figu a 4.1a), le asign amos a la fue za una magnitud unita ia de valo F. Se a lica aho a ot a fue za a l mismo eso te ho izontalmente (figu a 4.1b), oducindole un esti amiento de do s unidades, la magnitud de la fue za se de 2F. Si se a lican simultneamente las d os fue zas, el eso te se inclina, como en la figu a 4.1c, y se esti a 5 veces. L a fue za equivalente que oduce ese esti amiento del eso te es la suma vecto i al de F y 2F. Es deci , la fue za es un vecto . Figu a 4.1 a) izquie da, b) cent o, c) de echa.

El inst umento a a medi fue zas se llama dinammet o, es un eso te que se esti a sob e una escala. Si se a lica una fue za de una unidad sob e el dinammet o, el eso te se esti a hasta que eje ce una fue za igual y cont a ia a la 108

Ca . 4 Dinmica de la a tcula. a licada. En la escala se mide el ala gamiento del eso te y se le asigna una un idad de fue za. De esa mane a se calib a el dinammet o y se usa a a medi fue za s, o ejem lo se a lica una fue za sob e el dinammet o y si se esti a 2.5 unidad es, entonces la fue za a licada es 2.5 veces la unidad de fue za. Este ocedimi ento es vlido a a equeos ala gamientos del eso te, ya que si la fue za es muy i ntensa, se uede defo ma y no volve a su fo ma o iginal. 4.2 PRIMERA LEY DE NEWTON. Antes de 1600 los filsofos afi maban que el estado nat u al de la mate ia e a el e oso. Galileo fue el ime o que tuvo una idea disti nta del movimiento haciendo ex e imentos. Esencialmente sus ex e imentos consista n en analiza en fo ma semi cuantitativa el movimiento de los cue os, t atando de elimina toda influencia exte na que lo alte a , concluyendo que el estado nat u al de los cue os no es el e oso, sino el esisti se a una acele acin. Poste i o mente, Ne ton, que naci el ao en que mu i Galileo, e feccion los ex e imentos de Galileo ealizando cuidadosas mediciones ex e imentales, lo que le e miti fo mul a las aho a conocidas t es Leyes del Movimiento de Ne ton. La ime a Ley de Ne ton se uede enuncia de la siguiente mane a: Un cue o en e oso e manece en e oso y uno en movimiento continua en movimiento con velocidad constante, a meno s que acte una fue za sob e el cue o que alte e su estado de e oso o de movimie nto. En otros trminos se enuncia de la siguiente forma: si la suma de fuerzas que acta so re un cuerpo es cero, su aceleracin es cero. Esto significa que la partcula se encuentra en equili rio de traslacin, y se cumple la condicin: F = 0 a = 0 r r (4.1) Es importante darse cuenta que esta ley no ha sido probada real y verdaderamente , ya que no es posible eliminar totalmente las fuerzas que actan sobre un cuerpo. Es una generalizacin de la experiencia. 109

Cap. 4 Dinmica de la partcula. La primera Ley de Ne ton se conoce tambin como Ley de Inercia, porque define un s istema de referencia inercial. Un sistema de referencia inercial es aquel en el cual si sobre un cuerpo no acta fuerza alguna, este se mueve con velocidad consta nte. En este sistema de referencia se cumple la primera Ley de Ne ton. La Tierra no es un sistema de referencia inercial porque tiene una aceleracin de 5.9 x 103 m/s2 por su traslacin alrededor del Sol y una aceleracin por rotacin en torno a s u eje, que en el ecuador vale 3.4 x 10-2 m/s2. Como estos son valores pequeos com parados con g, se puede suponer que la tierra es un sistema de referencia inerci al. En la naturaleza no existen los sistemas de referencia inercial. Un marco de referencia inercial que se mueve con velocidad constante respecto a las estrell as muy lejanas, aparentemente fijas, es la mejor aproximacin a un sistema de refe rencia inercial. Para nuestros efectos, en la mayora de los casos consideraremos a la tierra como un sistema de referencia inercial, ya que para los objetos que se mueven distancias cortas comparadas con el radio terrestre sobre la superfici e, se pueden despreciar los movimientos de la Tierra. 4.3 CONCEPTO DE MASA. Qu efecto tendr una misma fuerza sobre cuerpos diferentes? No es lo mismo golpear con el pe una pelota que un adoqun. La masa es la propiedad d el cuerpo que determina el efecto de una fuerza aplicada sobre l. Cuando se quier e cambiar el estado de movimiento de un cuerpo, este se resiste al cambio. La in ercia es la propiedad de la materia que hace que se resista a cualquier cambio d e su movimiento, ya sea en su direccin o rapidez. Por ejemplo, los pasajeros de u n automvil que acelera sienten contra la espalda la fuerza del asiento, que vence su inercia y aumenta su velocidad. Cuando ste frena, los pasajeros tienden a seg uir movindose y se mueven hacia delante, por lo que deben apoyarse en el asiento delantero para no salir del suyo. Si se realiza un giro, un paquete situado sobr e el asiento se desplazar lateralmente, porque la inercia del paquete hace que ti enda a seguir movindose en lnea recta. La masa es el trmino que se usa para cuantif icar la inercia. Como mide la resistencia de un cuerpo a cambiar su estado de mo vimiento o de reposo, se le llama masa inercial, y est determinada por la razn ent re la fuerza neta sobre el cuerpo y su aceleracin. 110

Cap. 4 Dinmica de la partcula. Otro mtodo para encontrar la masa consiste en comparar la fuerzas gravitacionales ejercidas sobre dos objetos, uno de ellos de masa desconocida y el otro de masa conocida. El objeto de masa desconocida se coloca en uno de los platillos de un a balanza y en el otro platillo el conocido. Cuando los dos brazos estn balancead os la fuerza gravitacional es la misma sobre cada uno de ellos. Entonces las mas as de los cuerpos son iguales; cuando la masa se mide de esta forma se llama mas a gravitacional. Experimentos muy precisos indican que ambas masas, inercial y g ravitacional, son iguales. La masa es una propiedad del cuerpo, es independiente del medio que la rodea y del mtodo usado para medirla, para un cuerpo determinad o tiene el mismo valor en cualquier lugar del universo. Es un escalar por lo que cumple las reglas de la aritmtica comn, en el SI se mide en kg. 4.4 SEGUNDA LEY DE NEWTON. Cuando la fuerza neta que acta sobre un cuerpo no es c ero, el cuerpo se mueve con una aceleracin en la direccin de la fuerza. Experiment almente se demuestra que para una masa fija, si aumenta el valor de la fuerza, s u aceleracin aumenta proporcionalmente; por ejemplo si F aumenta a 2F la acelerac in a aumenta a 2a. Por otra parte, si se aplica una fuerza fija, pero se aumenta el valor de la masa, la aceleracin del cuerpo disminuye proporcionalmente al aume nto de masa, por ejemplo si m aumenta a 2m la aceleracin a disminuye a ()a. Lo opu esto se observa si en lugar de considerar aumento de fuerza o de masa, se consid eran disminuciones. La Segunda Ley de Ne ton se enuncia basndose en estos resulta dos experimentales, resumiendo esas observaciones en el siguiente enunciado: La a celeracin de un cuerpo es directamente proporcional a la fuerza resultante que ac ta sobre el cuerpo e inversamente proporcional a su masa. Escrita en trminos matemti cos, si F es la fuerza neta que acta sobre un cuerpo de masa m, la Segunda Ley de Ne ton se expresa como: r 111

Cap. 4 Dinmica de la partcula. r r r dv F = ma = m dt (4.2) Esta ecuacin fundamental muy sencilla y completa, encierra razonamientos fsicos mu y profundos, producto de la experiencia, se conoce como la ecuacin fundamental de movimiento. Permite describir el movimiento y la mayor parte de los fenmenos de la Mecnica Clsica, (excepto los cambios de opinin de una mujer que se rigen por una fuerza de voluntad o se producen por motivos de fuerza mayor, son aleatorios, c aticos e impredecibles). Como la Mecnica Clsica es vlida para cuerpos grandes que se ueven con v << c, la misma restriccin vale para las Leyes de Newton. La Segunda L ey de Newton es una expresin vectorial y equivale a tres ecuaciones escalares, un a en cada direccin x, y y z, F x = max , F y = ma y , F z = maz . La Segunda Ley de Newton se puede usar para definir la unidad de medida de una f uerza. En el sistema internacional, la unidad de medida de fuerza se llama Newto n, que se sim oliza por N, se define como la fuerza necesaria para mover una mas a de un kg producindole una aceleracin de un m/s2, entonces 1 N = 1 kg m/s2. Se o serva que la primera Ley de Newton es un caso particular de la segunda ley cuand o la fuerza neta es cero, ya que en ese caso la aceleracin de e ser cero, por lo tanto es una consecuencia de la segunda ley. 4.5 PESO. Todos los cuerpos que se dejan en li ertad cerca de la superficie terr estre caen con la aceleracin de gravedad. Lo que los hace caer es la fuerza funda mental de atraccin gravitacional con que la Tierra atrae a cualquier cuerpo con m asa. Si dos partculas que tienen masas m1 y m2 estn separadas una distancia r 112

Cap. 4 Dinmica de la partcula. medida desde sus centros, como se ve en la figura 4.2, la fuerza de atraccin grav itacional FG ejercida por la masa m1 so re la masa m2 tiene una magnitud: FG = G m1m2 r2 donde G = 6.672 x 10 11 N m2/kg2. El cuerpo a su vez ejerce una fuerza de atracc in so re la Tierra, pero como la masa de cualquier o jeto so re la Tierra es much o menor que la masa de la Tierra, el movimiento que el cuerpo le imprime a la Ti erra no se aprecia. A la fuerza de atraccin gravitacional que la Tierra ejerce so re un cuerpo en sus cercanas se le llama peso del cuerpo, se sim oliza con P. Es un vector fuerza dirigido hacia el centro de la Tierra, en la direccin de g, se mide en N. Figura 4.2 Fuerza de atraccin gravitacional entre masas. Cuando un cuerpo que es dejado en li ertad en las cercanas de la superficie terre stre, cae con la aceleracin de gravedad, es la fuerza peso P la que le imprime al cuerpo una aceleracin g, entonces de la Segunda Ley de Newton, el peso es: r r F = ma r r P = mg 113

Cap. 4 Dinmica de la partcula. Si se quiere evitar raria al peso, para d de la fuerza peso los pesos es igual g= P1 P = 2 m1 m2 El peso depende de g, vara con la ubicacin geogrfica y disminuye con la altura, por lo tanto no es una propiedad del cuerpo y no se debe confundir con la masa. Una balanza que es un instrumento para comparar fuerzas, se usa en la prctica para c omparar masas. Generalmente se dice que un kilo de azcar pesa 1 kg, aunque el kilogra mo es una unidad de masa, no de fuerza. 4.6 TERCERA LEY DE NEWTON. Cada vez que un cuerpo ejerce una fuerza so re otro c uerpo, este reacciona ejerciendo una fuerza so re el primero. Las fuerzas en cad a cuerpo son de igual magnitud, y actan en la misma lnea de accin, pero son de sent ido contrario, como se ve en la figura 4.2. Esto significa que no es posi le que exista una fuerza aislada, es decir, no existe un cuerpo aislado en la naturale za, cualquier fuerza individual es un aspecto de una interaccin mutua entre dos c uerpos, que puede ser por contacto directo o por accin a distancia. Esta propieda d de las fuerzas fue demostrada experimentalmente y expresada por Newton en su T ercera Ley de Movimiento, que se enuncia como sigue: Si dos cuerpos interactan, la fuerza que el cuerpo 1 ejerce so re el cuerpo 2 es igual y opuesta a la fuerza que el cuerpo 2 ejerce so re el cuerpo 1. Escrita en trminos de una ecuacin se pued e escri ir: 114 que un cuerpo caiga, se debe ejercer una fuerza igual y cont que la fuerza neta sea cero. De aqu se obtiene que la magnitu es P = mg. Como g es la misma para dos cuerpos, la relacin de a la relacin de las masas de los cuerpos, o sea:

Cap. 4 Dinmica de la partcula. r r F12 = F21 (4.3) donde F12 (F21) es la fuerza que ejerce el cuerpo de masa m1 (m2) so re el cuerp o de masa m2 (m1). Si una de las fuerzas que intervienen en la interaccin entre d os cuerpos se llama accin, la otra reci e el nom re de reaccin, por esto la Tercer a Ley de Newton se conoce tam in con el nom re Ley de Accin y Reaccin. Las fuerzas de accin y reaccin actan siempre en pareja y so re cuerpos diferentes. Si actuaran so re el mismo cuerpo no existira el movimiento acelerado, porque la resultante s iempre sera cero. Entonces, para que una pareja de fuerzas se consideren como fue rzas de accin y reaccin, de en cumplir los siguientes requisitos simultneamente: de en tener igual magnitud, la misma direccin, sentido opuesto, actuar en cuerpos d iferentes y actuar en parejas. De las tres leyes de Newton, slo la segunda y la t ercera son independientes, ya que la primera es una consecuencia de la segunda, cuando la velocidad es constante o la aceleracin es cero. Al aplicar las leyes de Newton se de en identificar todas las fuerzas externas que actan so re un cuerpo y di ujar un diagrama de cuerpo li re. Un diagrama de cuerpo li re es un esquem a donde se muestra el cuerpo aislado o un punto que lo representa, en el que se di ujan todas las fuerzas aplicadas so re el cuerpo. So re este esquema se elige un sistema de referencia conveniente para aplicar las leyes de Newton. Cuando s e considera un sistema mecnico con varios cuerpos, se de e hacer el diagrama de c uerpo li re y aplicar las leyes de Newton para cada componente del sistema. La f uerza que produce una superficie so re un cuerpo que se encuentra apoyado en la superficie se llama fuerza normal N, las fuerzas que ejercen cuerdas y ca les so re un cuerpo se llaman fuerza de tensin T. A menos que se diga lo contrario, las cuerdas y poleas que formen parte de un sistema mecnico se considerarn de masa de sprecia le comparada con la masa de los cuerpos en estudio y las cuerdas y ca le s se considerarn inextensi les, esto significa que sirven slo para cam iar la dire ccin de la tensin cuando pasan por una polea; se dice que son ideales. 115

 

 

 

Cap. 4 Dinmica de la partcula. Ejemplo 4.1. Un loque de 50N de peso se u ica so re un plano inclinado en un ngu lo de 30 con l horizont l. El bloque se sujet con un cuerd ide l que se encue ntr fij en l p rte superior del pl no inclin do, como se muestr en l figur 4.3 . C lcul r l tensin de l cuerd y l fuerz norm l. Solucin: Se identific n l s fuerz s que ct n sobre el cuerpo, est s son: Fuerz de tr ccin de l Tierr , que es su peso P Fuerz de l cuerd que lo sostiene, que es l tensin T Fuerz que el pl no ejerce sobre el cuerpo, que es l norm l N El di gr m de cuerpo l ibre (DCL) del bloque se muestr en l figur 4.3b.

r F = 0 F x = 0, F y =0 Del diagrama de cuerpo libre se obtiene: eje x: -T + P sen = 0 eje y: N P cos = 0 Despej ndo T y N, y reempl z ndo los v lores numricos, se obtiene: 116

Como el sistem est en equilibrio, se plic l primer eccin x e y:

Ley de Ne ton en c d dir

Figur

4.3. Ejemplo 1, ) izquierd , b) derech .

C p. 4 Dinmic de l p rtcul . T = P sen = 50 sen 30 = 25 N N = P cos = 50 cos 30 = 43.2 N Ejemplo 4.2. El sistem de l figur 4.4 se encuentr en equilibrio. Los c bles form n ngulos de 30 y 60 con l horizont l y el bloque pes 100 N. C lcul r l ten sin en los c bles. Solucin: Se h ce un di gr m de cuerpo libre p r el bloque (fi gur 4.4b) y en el nudo de unin de l s cuerd s (figur 4.4c). Figur 4.4 Ejemplo 2 ) izquierd , b) centro, c) derech .

F = 0 F r x = 0, F y =0 Del DCL del bloque y en el nudo se obtienen las ecuaciones: bloque: nudo: eje y: T1 P = 0 eje x: -T3 cos60 + T2 cos30 = 0 eje y: T3 sen60 + T2 sen30 T1 = 0 (1) (2) (3) 117

Como el sistem est en equilibrio, se plic l primer

Ley de Ne ton:

Cap. 4 Dinmica de la partcula. De la ecuacin (1) se obtiene: T1 = P T1 = 100 N De la ecuacin (2): T3 cos60 = T2 c os30 Reemplazando en las ecuacin (3): T2 cos 30 sen60 + T2 sen30 = 100 cos 60 T3 = T2 cos 30 cos 60 T2 (cos 30 tan 60 + sen30 ) = 100 Finalmente: T3 = 50 cos 30 cos 60 2T2 = 100 T2 = 50 N T3 = 86.6 N Ejemplo 4.3. Si un bloque de masa m se ubica sobre un plano sin roce, inclinado un ngulo con l horizont l, como se muestr en l figur 4.5 , p rtiendo del repo so, resb l r un dist nci D lo l rgo del pl no. Describir su movimiento. Soluc in: como el sistem est en movimiento, se plic l segund Ley de Ne ton, en comp onentes: r r F = m F x = max , F y = ma y Las fuerzas aplicadas sobre el cuerpo de masa m son la fuerza de atraccin de la T ierra, que es su peso P y la fuerza normal N del plano sobre el cuerpo. Del diag rama de cuerpo libre (figura 4.5b), considerando que el bloque resbala en direcc in del plano, o sea en direccin x, tiene slo ax y no ay, se obtiene: 118

Cap. 4 Dinmica de la partcula. Figura 4.5. Ejemplo 3: a) izquierda, b) derecha. eje x: P sen = m x eje y: N P cos = m y = 0 (1) (2) Despej ndo x de (1) y N de (2), consider ndo que P = mg, se obtiene: x = g sen N = mg cos Se concluye que l celer cin del bloque en direccin del pl no inclin do es l componente de g en es direccin. Estudi ndo hor el movimiento del bloque , consider ndo que p rte del reposo y se desliz un dist nci D, se puede c lcu l r l r pidez con que lleg l b se del pl no. Si se consider que el movimie nto del bloque comienz desde el reposo, se puede us r: v2 = v2o + 2 x x v2 = 0 + 2 (g sen )D v = 2gDsen ecu cin vlid solo p r este c so p rticul r. Esto complet l descripcin del movim iento del bloque sobre el pl no inclin do. 119

C p. 4 Dinmic de l p rtcul .

Figur 4.6 Ejemplo 4: ) izquierd , b) centro, c) derech .

F x = max , F y = ma y Como no se conoce la direccin del movimiento, podemos suponer que el cuerpo de ma sa M sube por el plano inclinado, lo que determina el sentido de la aceleracin de l sistema, entonces del DCL para M (figura 4.6b) y para m (figura 4.6c), se obti ene: Para M eje x: T - Mg sen = M eje y: N - Mg cos = 0 (2) (1) P r m eje y: T mg = -m (3)

120

De (3) se despej T y se reempl z

en (1):

r r F = m

Solucin: El sistem est en movimiento, por lo que se plic l segund on c d m s :

Ley de Ne t

Ejemplo 4.4. En el sistem mecnico de l figur 4.6 , el bloque de m s sobre el pl no liso inclin do en un ngulo . L pole por donde cuelg e de m s m conect do M es ide l y l cuerd se consider inextensible s despreci ble. C lcul r l celer cin de l s m s s M y m y l tensin .

M se ubic otro bloqu y de m de l cuerd

C p. 4 Dinmic de l p rtcul .

M + m = g(m - M sen) a= m Msen m+M g

m Msen T = mg m m+M T= mM g ( 1 + sen ) m+M g 4.7 FUERZA DE ROCE. Cuando un cuerpo es arrojado sobre una superficie comn o cuando un objeto se muev e a travs de un medio viscoso como agua o aire, despus de cierto tiempo se detiene , porque experimenta una resistencia a su movimiento debido a la interaccin del c uerpo con el medio que lo rodea. Esa resistencia cambia la velocidad del cuerpo, por lo tanto se mide con una fuerza. Una fuerza de resistencia de esa naturalez a se llama fuerza de roce o de friccin. Son muy importantes en la vida cotidiana, ya que por ejemplo nos permiten caminar y son necesarias para que se realice el movimiento de vehculos. La fuerza de roce es paralela a la superficie en el punt o de contacto entre dos cuerpos y tiene direccin opuesta al movimiento, nunca ayu dan al movimiento. Las evidencias experimentales indican que esta fuerza se prod uce por la irregularidad de las superficies, de modo que el contacto se realiza slo en unos cuantos puntos, como se ve en una vista amplificada de las superficie s que se muestra en la figura 4.7. La fuerza de roce a escala microscpica es ms co mpleja de lo que aqu se presenta, ya que corresponde a fuerzas electrostticas entr e tomos o molculas en los puntos donde las superficies estn en contacto. 121

Se observ que el signo de depende del trmino m - M sen. Ahor or de l tensin reempl z ndo el v lor de en T:

mg - ma - Mg sen = M

se c lcul el v l

T = mg m

Cap. 4 Dinmica de la partcula. Si se tiene un bloque en reposo sobre una mesa horizontal y se aplica una pequea fuerza F (figura 4.8), que se puede medir con un dinammetro, el cuerpo no se move r. En esta situacin la fuerza de roce equilibra la fuerza aplicada (figura 4.8a). La fuerza de roce que acta sobre los cuerpos en reposo se llama fuerza de roce es ttico, FE. La mxima fuerza de roce esttica es igual a la mnima fuerza necesaria para iniciar el movimiento. Figura 4.7 La irregularidad de la superficie produce la fuerza de roce. Si aumenta la fuerza aplicada F (figura 4.8b) hasta que el bloque se mueve, ento nces aumenta la fuerza de roce. Cuando el bloque est apunto de moverse, la fuerza de roce esttico es mxima. Al aumentar la fuerza aplicada a un valor mayor que FEm ax, entonces comienza el movimiento y el bloque acelera hacia la derecha. Cuando el bloque est en movimiento, la fuerza de roce se hace menor que la FEmax, en es te caso se llama fuerza de roce cintica FC. La fuerza aplicada no equilibrada con la FC produce la aceleracin del cuerpo (figura 4.4b). Si la fuerza aplicada es i gual a la FC el bloque se mueve con velocidad constante. Si deja de actuar la fu erza aplicada, entonces la fuerza de roce, que continua actuando, se opone al mo vimiento hasta detener al bloque. Experimentalmente se encuentra que para dos ti pos de superficies dadas, las fuerzas de roce esttica y cintica son aproximadament e independientes del tamao del rea de las superficies en contacto y son proporcion ales a la fuerza normal N. La fuerza de roce esttico, FE , es opuesta a la fuerza aplicada y la constante de proporcionalidad con la normal se llama coeficiente de roce esttico, E, entonces la magnitud de la fuerza de roce esttico es: 122

Cap. 4 Dinmica de la partcula. Figura 4.8 a) izquierda, b) derecha. FE E N Cuando el bloque est apunto de moverse, la fuerza de roce esttico es mxima, FEmx, lo mismo que el coeficiente de roce es mximo, Emx, entonces: FEmx = E max N La fuerza de roce cintico es opuesta al movimiento, es aproximadamente independie nte de la velocidad con que se mueven las superficies, para velocidades pequeas, si la velocidad aumenta hasta valores muy altos, comienza a sentirse el efecto de la friccin con el medio donde se mueve el cuerpo. La constante de proporcionalida d con la normal se llama coeficiente de roce cintico, C, entonces la magnitud de l a fuerza de roce cintico es: FC = C N Las expresiones de FC y FE son empricas, no representan leyes fsicas fundamentales . Los coeficientes de roce esttico E y cintico C son constantes adimensionales. Sus valores dependen de la naturaleza de las superficies en contacto y en general pa ra un par de superficies dadas Emx > C. Algunos valores de los coeficientes de roce se dan en la ta la 4.1. 123

Cap. 4 Dinmica de la partcula. El grfico de la magnitud de la fuerza aplicada F versus la fuerza de roce se mues tra en la figura 4.9. Cuando el cuerpo no est en movimiento, la fuerza de roce es ttico se equili ra con la fuerza aplicada, hasta que el loque esta a punto de mo verse, donde la fuerza FE alcanza su valor mximo. Luego que comienza el movimient o del loque, surge la fuerza de roce cintico FC, que disminuye rpidamente a un va lor constante menor que la fuerza de roce esttico mxima FEmx, independientemente de l valor de la fuerza aplicada. Ta la 4.1 Algunos valores de coeficientes de roce. Superficies Madera madera Ac ero acero Vidrio vidrio Caucho concreto Co re vidrio Hielo hielo Articulaci ones humanas E 0.25 0.5 0.74 0.94 0.15 0.68 0.1 0.01 C 0.2 0.57 0.40 0.06 0.53 0.0 3 0.003 Figura 4.9. Grfico de la fuerza de roce. Ejemplo 4.5. En el sistema mecnico de la figura 4.10a, se aplica una fuerza F inc linada un ngulo sobre el cuerpo de m s m, ubic do sobre l mes horizont l con c oeficiente de roce . L pole por donde cuelg otro bloque de m s M no tiene roc e y l cuerd se consider inextensible y de m s despreci ble. C lcul r l cel er cin de l s m s s y l tensin de l cuerd . 124

C p. 4 Dinmic de l p rtcul .

F x = ma x , F y = ma y Como no se conoce la direccin del movimiento, podemos suponer que el cuerpo de ma sa M desciende y tira a m hacia la derecha, lo que determina el sentido de la ac eleracin del sistema, entonces del DCL para m (figura 4.10b) y para M (figura 4.1 0c), en cada direccin x e y, se obtiene: Figura 4.10. Ejemplo 5. a) izquierda, b) centro, c) derecha. Para m eje x: T - Fcos - FR = m eje y: N + Fsen - mg= 0 (1) (2) P r M eje y: T - Mg = -M (3) Adems se s be que por definicin, l fuerz de roce es: FR = N. De (2) se despej N y se reempl z en FR: N = mg - Fsen De (3) se despej T: FR =(mg - Fsen) (4) (5)

Ahor (4) y (5) se reempl z n en (1), lo que permite despej r l 125

T = Mg - M

celer cin

r r F = m

Solucin: El sistem est en movimiento, por lo que se plic l segund on c d m s :

Ley de Ne t

C p. 4 Dinmic de l p rtcul . Mg - M - Fcos - (mg - Fsen) = m a= y la tensin T (M m )g F (cos sen ) M +m T = Mg M (M m )g F (cos sen ) M +m 4.8 FUERZA CENTRPETA. Un p rtcul que se mueve sobre un tr yectori circul r de r dio R con r pidez c onst nte, se encuentr sometid un celer cin r di l de m gnitud v2/R. Por l segund ley de Ne ton, sobre l p rtcul ct un fuerz en l direccin de , h ci el centro de l circunferenci , cuy m gnitud es: Fc = m c = m v2 R Por ser proporcion l l celer cin centrpet , l fuerz Fc se ll m fuerz centrp et . Su efecto es c mbi r l direccin de l velocid d de un cuerpo. Se puede sent ir est fuerz cu ndo se h ce gir r un objeto t do un cuerd , y que se no t el tirn del objeto. L s fuerz s centrpet s no son diferentes de otr s fuerz s y conocid s, su nombre se debe que punt h ci el centro de un tr yectori c ircunferenci l. Cu lquier de l s fuerz s y conocid pueden ctu r como fuerz centrpet si producen el efecto correspondiente, como ser l tensin de un cuerd , un fuerz de roce, lgun componente de l norm l, l fuerz gr vit cion l en el c so de movimientos de pl net s y s tlites, etc. Ejemplo 4.6. Un cuerpo de m s m, sujeto l extremo de un cuerd de longitud L, que describe un tr yectori circul r en el pl no horizont l, gener un superf icie cnic (figur 4.11 ), por lo que se ll m pndulo cnico. C lcul r l r pidez y el perodo de revolucin de l m s . 126

C p. 4 Dinmic de l p rtcul . Figur 4.11 Ejemplo 6. ) izquierd , b) derech .

Solucin: L p rtcul est sometid un celer cin centrpet , y l fuerz centrpet c rrespondiente est d d por l componente de l tensin de l cuerd en direccin r di l h ci el centro de l circunferenci . De l segund Ley de Ne ton

r r x = ma x , F y = ma y aplicada al DCL de m que se muestra en la figura 4.11b), se tiene: eje x: T sen = m = m v2/r eje y: T cos - mg = 0 Despej ndo T de l ecu cin del eje y y reempl z ndo en l ecu cin del eje x, mg v2 sen = m cos r v2 t n = rg De l geometr de l figur , r = L sen, reempl z ndo se puede despej r l r pidez de m: 127

F = m

C p. 4 Dinmic de l p rtcul .

t = = 2 L cos g

P r un cuerpo como un vehculo o un v gn de tren que se mueven describiendo un tr yectori curv de r dio r, sobre el vehculo debe ctu r un fuerz centrpet p r evit r que contine movindose en lne rect y se s lg de l pist ; est es l fuer z p r h cer que el vehculo gire por l pist curv . L fuerz centrpet neces ri l d el roce de los neumticos o l s pest s de l s rued s del tren. P r no ten er que confi r en el roce o reducir el desg ste de los rieles y pest s, l c rre ter o l v pueden inclin rse, como en l figur 4.12 . A l inclin cin de l pis t o v se le ll m ngulo de per lte, . En este c so l componente de l norm l dir igid h ci el centro de curv tur proporcion l fuerz neces ri p r m ntener l mvil en l pist . P r un pist curv de r dio r, con ngulo de per lte , p r l que se consider l fuerz de roce FR, l fuerz centrpet corresponde l s c omponentes de l norm l y de l fuerz de roce h ci el centro de curv tur de l pist . Son est s componentes l s que producen l celer cin centrpet que m ntie ne l vehculo de m s m sobre l pist . Del di gr m de cuerpo libre de l figur 128

4.8.1 L

descripcin del per lte.

Se puede observ r que el periodo es independiente del v lor de l m s lo.

m del pndu

v 2 = gLsen (t n ) v = P r c lcul r el periodo , e que x = vt, con x = 2 2 = v 2Lsen Lgsen (t

gLsen (t n ) es o es el iempo que demora en dar una vuel a, se sab , entonces: n )

C p. 4 Dinmic de l p rtcul .

v2 eje x : Nsen FR cos = m r eje y : N cos FR sen mg = 0 Mul iplicando por c u cin en x y por sen l ecu cin en y, y sumndol s, se obtiene: v2 FR = m cos gsen r Casos particulares. a) Si no se considera el roce, y la ecuacin anterior se reduce a: v2 cos gsen = 0 r v2 tan = rg 129

Figur 4.12 ) Angulo de per lte en un quierd ).

pist

curv (izquierd ), b) DCL de m (iz

4.12b, se puede c lcul r l fuerz de roce neces ri p r que el vehculo no se s lg de l pist , por l segund ley de Ne ton, se obtiene:

C p. 4 Dinmic de l p rtcul .

Se observ que el ngulo de per lte depende de l r pidez y del r dio de l tr yec tori curv y es independiente de l m s del vehculo. P r un cierto v lor del r dio, no existe un ngulo que s tisf g l ecu cin p r tod s l s r pideces, por lo t nto l s curv s se per lt n p r un r pidez medi . Por ejemplo, si v = 72 km/ hr = 20 m/s, y r = 100 m, se obtiene: 20 2 = rct n 100 9.8 = 22.2 b) P en que la curva o va no tiene peralte, = 0, l expresin p r FR se reduce : v2 F R = m r L r pidez mxim que puede tener el mvil l gir r sobre un c rreter o v sin per lte, corresponde quell en l cu l est punto de resb l r h ci fuer , en este c so debe ctu r l FRmx p r obtener l r pidez mxim , que no se debe super r p r que el vehculo no se s lg de l pist : FR m x = E m x N = E m x mg v 2 max E max mg = m v max = E max rg r Este tratamiento completa una descripcin bsica para entender como se deben inclina r las vas de trenes o carreteras en las curvas, para que los vehculos al entrar en las curvas no se salgan de su pista para evitar accidentes. 130

Cap. 4 Dinmica de la partcula. 4.9 BREVE DESCRIPCIN DE APLICACIONES DE ALGUNAS FUERZAS EN LA MEDICINA. 4.9.1 Fue rza peso. La fuerza de gravedad que ejerce la Tierra sobre los objetos cerca de su superficie se conoce como el peso del cuerpo. Esta fuerza es la que hace que todos los cuerpos en cada libre caigan con g. La fuerza de gravedad sobre un cuer po extenso, requiere una especial consideracin, porque acta sobre cada partcula del objeto, la suma de todas estas fuerzas representa el peso del cuerpo. El punto donde se considera que acta esta fuerza total de gravedad se denomina centro de g ravedad del cuerpo (c.g.) Si el cuerpo es simtrico, el centro de gravedad se ubic a en el centro geomtrico, y puede estar localizado dentro o fuera del cuerpo. Si el objeto es asimtrico tal como el brazo de una persona, que se muestra en la fig ura 4.13, el c.g. se ubicar ms cerca de su parte ms masiva y si adems el objeto es f lexible, como el cuerpo humano, la posicin del centro de gravedad vara si el objet o cambia de forma, por ejemplo el c.g. estando parado es diferente que estando i nclinado, en el primer caso se ubica cerca del ombligo (dentro del cuerpo) y en el segundo caso incluso puede estar fuera del cuerpo. 4.9.2 Fuerza muscular. La postura y el movimiento de los animales estn controlados por fuerzas producidas p or los msculos. Un msculo consta de un gran nmero de fibras cuyas clulas son capaces de contraerse al ser estimuladas por impulsos que llegan a ellas procedentes de los nervios. Un msculo est generalmente unido en sus extremos a dos huesos difere ntes por medio de tendones (figura 4.13). Los dos huesos estn enlazados por una c onexin flexible llamada articulacin. La contraccin del msculo produce dos pares de f uerzas que actan sobre los huesos y los msculos en el punto donde estn ligados los tendones. La fuerza mxima que puede ejercer un msculo depende del rea de su seccin t ransversal, y en el hombre es de unos 30 a 40 N/cm2. Esto es, para producir una fuerza muscular de 600N se necesita un msculo con una seccin transversal 15 a 20 c m2. El estudio del funcionamiento de las fuerzas musculares para producir movimi ento y equilibrio en el hombre recibe el nombre de Kinesiologa o biomecnica. Es de particular importancia para atletas y terapeutas fsicos, quienes necesitan saber qu fuerzas se requieren para producir movimientos especficos del cuerpo. 131

Cap. 4 Dinmica de la partcula. Figura 4.13 Msculos del brazo y ubicacin del centro de gravedad. 4.9.3 Fuerza de roce. Si un objeto se mueve dentro de un fluido la fuerza de roc e se denomina fuerza de roce viscoso, y su valor es pequeo si se compara con el r oce entre superficies slidas. Por lo tanto el uso de lquidos lubricantes como el a ceite, que se interpone entre las superficies en contacto, disminuye bastante el roce. Anlogamente, una capa de aire suministra un soporte casi sin roce para los vehculos aerodeslizantes o para mesas experimentales de aire. Al caminar o correr, no advertimos roce en las rodillas ni en las articulaciones de las piernas. Estas y muchas otras articulaciones se encuentran bien lubricad as mediante el lquido sinovial, que pasa a travs del cartlago que las reviste cuand o ellas se mueven (figura 4.14). Este lubricante tiende a ser absorbido, cuando la articulacin est en reposo, aumentando entonces el rozamiento y facilitando el m antener una posicin fija. Esto constituye un excelente ejemplo 132

Cap. 4 Dinmica de la partcula. de la sabia ingeniera biolgica empleada por la naturaleza. El roce, por un lado li mita la eficiencia de mquinas y motores, pero por otro lado, hacemos uso del roce en un gran nmero de situaciones, como en el frenar de automviles, las correas tra nsportadoras, al escribir, caminaretc. Figura 4.14 Lubricacin de articulaciones por el lquido sinovial. Ejemplo 4.7. La figura 4.15 muestra la forma del tendn del cuadriceps al pasar po r la rtula. Si la tensin T del tendn es 1400 N. Calcular la a) la magnitud y b) la direccin de la fuerza de contacto F ejercida por el fmur sobre la rtula. Solucin. El diagrama de fuerzas correspondiente a la rtula, se muestra en la misma figura 4.15. Como el sistema est en equilibrio, se aplica la primera ley de Ne t on, que en componentes se escribe de la siguiente forma: Figura 4.15 Ejemplo 7. 133

Cap. 4 Dinmica de la partcula.

F cos 1400 cos 37 o 1400 cos 80 o = 0 Fsen + 1400sen37 o 1400sen80 o = 0 De la primera ecuacin se ob iene: De la segunda ecuacin se ob iene: F cos = 1361.2 N Fsen = 536.2 N

F = 536.2 2 + 1361.2 2 F = 1463N Y su direccin es: tg = Fsen 536.2 = = 0.39 F cos 1 361.2 = 215 o Por lo t nto l fuerz de compresin F que ejerce el hueso sobre l rtul tiene un v lor de 1463 N y ct en un ngulo de 21,5 respecto l horizont l. 134

Los v lores obtenidos corresponden t nto su m gnitud es:

l s componentes rect ngul res de F, por lo

Fx = 0 F cos T cos 37 o T cos 80 o = 0 Fy = 0 Fsen + Tsen37 o Tsen80 o = 0 Reemplazando los valores de la fuerza T se ne:

ie

C p. 4 Dinmic de l p rtcul . PROBLEMAS. 4.1. Este libro de Fsic , est poy do en el extremo superior de un resorte vertic l, que su vez est p r do sobre un mes . P r c d componente del sistem libro -resorte-mes -tierr : ) dibuj r el di gr m de cuerpo libre, b) identific r tod os los p res de fuerz s de ccin y re ccin. 4.2. De cuerdo con l leyend , un c b llo prendi l s leyes de Ne ton. Cu ndo se le pidi que tir r un c rret , se neg rotund mente rgument ndo que si l tir b l c rret h ci del nte, de cuerdo co n l tercer ley de Ne ton h br un fuerz igu l h ci trs. De est m ner , l s fuerz s est r n b l nce d s y de cuerdo con l segund ley de Ne ton, l c rret no celer r . Pero como usted es m s di bl zo que el c b llo, s be que l c rret se mueve Cmo podr usted r zon r con este misterioso c b llo, p r h cerlo entend er? 4.3. Dos lumnos ubic dos en los bordes opuestos de un c mino recto tir n un c rro por el c mino, con fuerz s de 160 N y 200 N, que form n un ngulo de 30 y 60 respectiv mente, con l direccin del c mino. ) C lcul r l m gnitud de l fuer z result nte y l direccin en l que se mover el c rro. b) C lcul r l fuerz nec es ri p r que el c rro se muev en l direccin del c mino. R: ) 256.1N, -21.3, b) F2 = 128N. 4.4. Un fuerz dependiente del tiempo, F = (8i 4tj) N (donde t es t en segundos), se plic un objeto de 2 kg inici lmente en reposo. ) En qu tiem po el objeto se mover con un velocid d de 15 m/s? b) A qu dist nci est de su posic in inici l cu ndo su velocid d es 15 m/s? c) Cul es l posicin del objeto en este ti empo? R: ) 3s, b) 20.1m, c) 18i-9j m 4.5. Tres fuerz s F1 = (-2i + 2j)N, F2 = ( 5i 3j)N, y F3 = (-45i)N que ct n sobre un objeto le producen un celer cin de v lor 3 m/s2. ) Cul es l direccin de l celer cin? b) Cul es l m s del objeto? c) Si el objeto est inici lmente en reposo, c lcul r su velocid d despus de 10s? R: ) 1.4, b) 14 kg, c) 30 m/s. 135

C p. 4 Dinmic de l p rtcul . 4.6. C lcul r l tensin en c d cuerd en los sistem s que se muestr n en l s fig ur s 4.13, 4.14 y 4.15. L s m s s son de m kg y l inclin cin de los pl nos es gr dos. H cer tod s l s suposiciones neces ri s. 4.7. Un m s de 5kg cuelg de un cuerd de 1m de longitud que se encuentr sujet un techo. C lcul r l fuerz horizont l que plic d l m s l desve 30 cm de l vertic l y l m nteng e n es posicin. R: 15.7 N. Figur 4.13 Figur 4.14 Figur 4.15 4.8. Un r de 2 x 10-4 kg est suspendid de un hebr delg d de tel r . L ten sin mxim que soport l hebr ntes de romperse es 2.1 x 10-3 N. C lcul r l cel er cin mxim con l cu l l r puede subir por l hebr con tod segurid d. R: 0. 5m/s2. 4.9. Un fuerz F plic d sobre un m s m1 le produce un celer cin de 3m/s2. L mism fuerz plic d un m s m2 le produce un celer cin de 1m/s2. ) C lcul r el v lor de l proporcin m1/m2. b) Si se combin n m1 y m2, c lcul r l celer cin producid por F. R: ) 1/3, b) 0.75 m/s2. 4.10. L velocid d promed io de un molcul de nitrgeno en el ire es cerc n 6.7x102m/s y su m s proxim d mente de 4.68x10-26kg. ) Si se requieren 3x10-13s p r que un molcul de nit rgeno golpee un p red y rebote con l mism r pidez pero en direccin opuest , c l cul r l celer cin promedio de l molcul dur nte ese interv lo de tiempo. b) C l cul r l fuerz promedio que ejerce l molcul sobre l p red. R: ) 4.5x1015 m/s 2, b) 2.1x10-10 N. 136

C p. 4 Dinmic de l p rtcul . 4.11. Sobre el pl net X un objeto pes 12 N. En el pl net Y, donde l m gnitud de l celer cin de c d libre es 1.6g, el objeto pes 27 N. C lcul r: ) l m s del objeto y b) l celer cin de c d libre en el pl net X? R: ) 1.7 kg, b) 7m/ s2. 4.12. Los instrumentos de un globo sond meteorolgico tienen un m s de 1 kg . ) El globo se suelt y ejerce un fuerz h ci rrib de 5 N sobre los instru mentos. Cul es l celer cin del globo y de los instrumentos? b) Despus de que el gl obo h celer do dur nte 10 segundos, los instrumentos se suelt n. Cul es velocid d de los instrumentos en el momento en que se suelt n? c) cul es l fuerz net qu e ct sobre los instrumentos despus de que se suelt n? d) En qu momento l direccin de su velocid d comienz ser h ci b jo? 4.13. Un m no ejerce un fuerz hor izont l de 5 N p r mover h ci l derech dos bloques en cont cto entre s uno l l do del otro, sobre un superficie horizont l sin roce. El bloque de l izqu ierd tiene un m s de 2 kg y el de l derech de 1 kg. ) Dibuj r el di gr m de cuerpo libre p r c d bloque. C lcul r: b) l celer cin del sistem , c) l celer cin y fuerz sobre el bloque de 1 kg, d) l fuerz net ctu ndo sobre c d cuerpo. R: b) 5/3 m/s2, c) 5/3 m/s2, 5/3N, d) 5 N. 4.14. Dos bloques de m s s M y 3M ubic do l derech de M, que estn sobre un mes horizont l lis se unen entre s con un v rill de l mbre horizont l, de m s despreci ble. Un fuerz h orizont l de m gnitud 2Mg se plic sobre M h ci l izquierd . ) H cer los di gr m de cuerpo libre. b) C lcul r l celer cin del sistem . c) C lcul r l tens in del l mbre. R: b) 5 m/s2, c) 15M (N). 4.15. Dos bloques de 1 y 2 kg, ubic dos sobre pl nos lisos inclin dos en 30, se conect n por un cuerd liger que p s por un pole sin roce, como se muestr en l figur 4.15. C lcul r: ) l cele r cin de c d bloque, b) l tensin en l cuerd . 4.16. Respecto l problem nteri or, si l celer cin cu ndo los pl nos son rugosos fuer de l c lcul d en ese p roblem , c lcul r: ) el coeficiente de roce, b) l tensin en l cuerd . 137

C p. 4 Dinmic de l p rtcul . 4.17. Un trineo de 50 kg de m s se empuj lo l rgo de un superficie pl n cu biert de nieve. El coeficiente de roz miento esttico es 0.3, y el coeficiente de roz miento cintico es 0.1. ) Cul es el peso del trineo? b) Qu fuerz se requiere p r que el trineo comience moverse? c) Qu fuerz se requiere p r que el trineo s e muev con velocid d const nte? d) Un vez en movimiento, qu fuerz tot l debe p licrsele l trineo p r celer rlo 3 m/s2? 4.18. Pepe nd esqui ndo, cu ndo en lgn momento sube 5 m deslizndose por l pendiente de un cerrito nev do en sus es ques, s liendo desde l cim ubic d 3 m de ltur respecto l horizont l, co n un r pidez de 10 m/s. El coeficiente de roce entre l nieve y los esques es 0. 1. ) C lcul r l r pidez con l cu l el esqui dor comienz subir l pendiente . b) Determine l dist nci horizont l que vuel Pepe cu ndo s le de l punt de l cerro. R: ) 13 m/s, b) 12.8 m. 4.19. Dos bloques de m s s 1 y 2 kg (figur 4. 16) cuelg n de los extremos de un cuerd liger y flexible que p s por un pol e sin roce, sujet l techo; el sistem se ll m mquin de At ood. Si en el inst nte inici l los cuerpos se encuentr n en reposo y 1 y 2 m respectiv mente del suelo, ) dibuj r el di gr m de cuerpo libre p r c d bloque. b) Escribir l s ecu ciones de movimiento p r c d cuerpo. c) Determin r l posicin y l velocid d de c d cuerpo un segundo despus de empez r moverse. d) C lcul r el v lor de l tensin de l cuerd cu ndo el sistem est en movimiento. R: c) 8/3 m; 1/3 m; 1 0/3 m/s, d) 13.3 N. 4.20. El bloque de m s m de l figur 4.17 p rte del reposo , deslizndose desde l p rte superior del pl no inclin do 30 con l horizont l. El coeficiente de roce cintico es 0.3. ) C lcul r l celer cin del bloque mientr s se mueve sobre el pl no. b) C lcul r l longitud del pl no si el bloque s le co n un r pidez de 5 m/s. c) Si el bloque c e l suelo un dist nci horizont l de 3 m desde el borde del pl no, determine el tiempo tot l del movimiento. R: ) 2.4 m/s2, b) 5.2 m, c) 2.8 s. 4.21. En el sistem de l figur 4.18, se plic un fuerz F sobre m. El coeficiente de roce es entre c d cuerpo y los pl nos. Deducir l expresin de l m gnitud de F p r que el sistem se muev : ) con r pi dez const nte, b) con celer cin const nte. R: b) Mg(cos+sen)+mg+ (m+M). 138

C p. 4 Dinmic de l p rtcul .

4.22. En el sistem de l figur 4.19, l fuerz F p r lel l pl no inclin do e mpuj l bloque de m s m h cindolo subir sobre el pl no, de coeficiente de roce . C lcul r en funcin de m, F, g, y , l celer cin del bloque. R: F/m -g(cos + sen). 23. Un fuerz F se plic un pequeo bloque de m s m p r h cerlo moverse lo l rgo de l p rte superior de un bloque de m s M y l rgo L. El coeficiente de roce es entre los bloques. El bloque M desliz sin roce en l superficie horizon t l. Los bloques p rten del reposo con el pequeo en un extremo del gr nde, como s e ve en l figur 4.20. ) C lcul r l celer cin de c d bloque rel tiv l su perficie horizont l. b) C lcul r el tiempo que el bloque m demor en lleg r l o tro extremo de M, en funcin de L y l s celer ciones. R: ) (F-mg)/m, mg/(m+M), b) [2L/( 1- 2)]1/2. 4.24. En el sistem de l figur 4.21, el br zo del pndulo es de longitud l y l cuerd de l rgo L. ) C lcul r l r pidez t ngenci l p r que e l sistem gire en torno l eje de rot cin que p s por l b rr vertic l, de modo que l cuerd que sostiene l m s m forme un ngulo de 30 con l vertic l. b) C lcul r l tensin de l cuerd . c) Si el sistem d un vuelt en 30 s, determin r el ngulo que form l cuerd con l vertic l. R: ) [(l+Lsen) gt n]1/2, b) mg/cos. Figur 4.16 Figur 4.17 Figur 4.18 139

C p. 4 Dinmic de l p rtcul . Figur 4.19 Figur 4.20 Figur 4.21 4.25. P r que un s tlite teng un rbit circul r con r pidez const nte, su cele r cin centrpet debe ser invers mente proporcion l l cu dr do del r dio r de l rb it . ) Demuestre que l r pidez t ngenci l del s tlite es proporcion l r -1/2. b) Demuestre que el tiempo neces rio p r complet r un rbit es proporcion l r3/2. 4.26. Un bloque de m s M se ubic sobre un pequeo pl no inclin do un ngulo sin roce, que tiene su extremo inferior fijo un eje vertic l que puede gir r. En lgn momento el eje gir con el pl no con r pidez const nte. Demostr r que si l m s sciende desde l b se del pl no, su r pidez cu ndo h subido un dist n ci L es v = gLsen . 4.27. L m s m1 sobre un mes horizont l sin friccin se con ect l m s m2 por medio de un pole mvil y un pole fij sin m s s (figur 4.22). ) Si 1 y 2 son m gnitudes de l s celer ciones de m1 y m2, respectiv m ente, determin r un rel cin entre est s celer ciones. Determin r expresiones p r : b) l s tensiones en l s cuerd s, y c) l s celer ciones 1 y 2 en funcin de m1, m2 y g. 4.28. C lcul r l fuerz F que debe plic rse sobre un bloque A de 2 0 kg p r evit r que el bloque B de 2 kg c ig (figur 4.23). El coeficiente de friccin esttico entre los bloques A y B es 0.5, y l superficie horizont l no pres ent friccin. R: 480N. 140

C p. 4 Dinmic de l p rtcul . Figur 4.22 Figur 4.23 4.29. Demuestre que l r pidez mxim que un mvil puede tener en un c rreter sin per lte es vm x = Rg , donde es el coeficiente de roce y R el r dio de l curv . 4.30. C lcul r el ngulo de per lte de un c rreter en un curv de r dio 150m, p r que un c min de 15 tonel d s pued gir r con un r pidez de 70km/hr, sobre un p vimento cubierto de esc rch . R: 14. 4.31. L figur 4.24 muestr l c bez de un p ciente en tr ccin de cuello sobre un pl t form mvil sin roce. Se tienen l s siguientes fuerz s: F fuerz ejercid por l vend sobre l c bez , Fc fuerz ejercid por el cuello sobre l c bez , N fuerz ejercid por l mes sobre l c bez , P peso de l c bez . ) Dibuj r el di gr m de fuerz s correspondiente l c bez . b) Indic r l re ccin c d un de l s fuerz s nteriores. c) Sobre q uin ct l fuerz gr vit cion l? d) En l b se qu leyes se obtiene el v lor de l tensin en l s vrtebr s del cuello? e) Cul es el v lor de l tensin en el cuello? Figur 4.24 Figur 4.25 141

C p. 4 Dinmic de l p rtcul . 4.32. El tendn del bceps de l figur 4.25 ejerce un fuerz F de 70 N sobre el n tebr zo. El br zo p rece dobl do, de t l m ner que est fuerz form un ngulo d e 40 con el ntebr zo. H ll r l s componentes de F: ) P r lel l ntebr zo (fue rz est biliz dor ), b) Perpendicul r l ntebr zo (fuerz de sostn). 4.33. C lcu l r l fuerz tot l plic d l c bez del p ciente por el dispositivo de tr c cin de l figur 4.26. 4.34. L figur 4.27 represent l c bez de un nio inclin d sobre un libro. L c bez pes 30N y est sostenid por l fuerz muscul r ejer cid por los extensores del cuello y por l fuerz del cont cto Fm ejercid en l rticul cin tl ntooccipit l. D do que el mdulo de Fm es 45 N y que est dirigido 35 por deb jo de l horizont l, c lcul r: ) l m gnitud y b) l direccin de Fc. Fm Fg Fc Figur 4.26 Figur 4.27 142

C p. 5 Tr b jo y Energ . CAPITULO 5. TRABAJO Y ENERGIA. El problem fund ment l de l Mecnic es describir como se movern los cuerpos si se conocen l s fuerz s plic d s sobre l. L form de h cerlo es plic ndo l segund Ley de Ne ton, pero si l fuerz no es const nte, es decir l celer cin no es const nte, no es fcil determin r l velocid d de l cuerpo ni t mpoco su posicin, por lo que no se est r resolviendo el problem . L os conceptos de tr b jo y energ se fund ment n en l s Leyes de Ne ton, por lo qu e no se requiere ningn principio fsico nuevo. Con el uso de est s dos m gnitudes fs ic s, se tiene un mtodo ltern tivo p r describir el movimiento, esp ci lmente ti l cu ndo l fuerz no es const nte, y que en est s condiciones l celer cin no es const nte y no se pueden us r l s ecu ciones de l cinemtic nteriormente est udi d s. En este c so se debe us r el proceso m temtico de integr cin p r resolve r l segund Ley de Ne ton. Ejemplos de fuerz s v ri bles son quell s que v r n con l posicin, comunes en l n tur lez , como l fuerz gr vit cion l o l s fuer z s elstic s. 5.1 TRABAJO REALIZADO POR UNA FUERZA CONSTANTE. Si l fuerz F que ct sobre un p rtcul es const nte (en m gnitud y direccin) el movimiento se re liz en lne re ct en l direccin de l fuerz . Si l p rtcul se despl z un dist nci x por ef ecto de l fuerz F (figur 5.1), entonces se dice que l fuerz h re liz do tr b jo W sobre l p rtcul de m s m, que en este c so p rticul r se define como: W=Fx

143

Figur 5.1 Fuerz horizont l const nte que re liz

un despl z miento x.

C p. 5 Tr b jo y Energ . Si l fuerz const nte no ct en l direccin del movimiento, el tr b jo que se re liz es debido l componente x de l fuerz en l direccin p r lel l movimie nto, como se ve en l figur 5.2 . L componente y de l fuerz , perpendicul r l despl z miento, no re liz tr b jo sobre el cuerpo. Figur 5.2 Fuerz const nte que form un ngulo con el despl z miento x.

W = ( F cos ) x De cuerdo l ecu cin nterior, se pueden obtener los siguientes conclusiones: ) si = 0, es decir, si l fuerz , como en l figur 5.1, o un componente de l fuerz , es p r lel l movimiento, W = (F cos 0) x = F x; b) si = 90, es decir, s i l fuerz o un componente de l fuerz es perpendicul r l movimiento, W = (F cos90) x = 0, no se re liz tr b jo; c) si l fuerz plic d sobre el cuerpo n o lo mueve, no re liz tr b jo y que el despl z miento es cero; d) si 0 < < 90, es decir, si l fuerz tiene un componente en l mism direccin del despl z mien to, el tr b jo es positivo; e) si 90 < < 180, es decir, si l fuerz tiene un com ponente opuest l direccin del despl z miento, el tr b jo es neg tivo.

De est s conclusiones se deduce que el tr b jo, p r ede expres r de l siguiente form : 144

un

fuerz const nte, se pu

Si es el ngulo medido desde el despl z miento x h ci l r b jo W es hor :

fuerz F, el v lor del t

C p. 5 Tr b jo y Energ . r r W = F r El tr b jo es un m gnitud fsic esc l r, obtenido del producto esc l r de los ve ctores fuerz y posicin. De l expresin nterior, por l definicin de producto esc l r, qued cl ro que el tr b jo puede ser positivo, neg tivo o cero. Su unid d d e medid en el SI es N m que se ll m Joule, smbolo J. Otr s fuerz s ct n sobre e l cuerpo de m s m (peso, roce, norm l, etc.), por lo que l ecu cin nterior se refiere slo l tr b jo de l fuerz F en p rticul r; l s otr s fuerz s t mbin pued en re liz r tr b jo. En l figur 5.2 l s fuerz s peso y norm l no re liz n tr b jo y que son perpendicul res l despl z miento y l fuerz de roce re liz tr b jo neg tivo, y que siempre se opone l despl z miento. El tr b jo tot l sobre l p rtcul es l sum esc l r de los tr b jos re liz dos por c d un de l s fu erz s. Ejemplo 5.1: Con un fuerz de 250 N que form un ngulo de 60 con l horizont l se empuj un c j de 50 kg, en un superficie sper horizont l (figur 5.2 ). L c j se mueve un dist nci de 5m con r pidez const nte. C lcul r: ) el tr b jo re liz do por c d fuerz , b) el coeficiente de roce. Solucin: L s fuerz s que c t n sobre l c j son F, norm l, roce y peso, el di gr m de cuerpo libre se mues tr en l figur 5.2b. Figur 5.2b. Ejemplo 5.1

r r ) L definicin de tr b jo es W = F r , que se 145

plic

c d

fuerz

C p. 5 Tr b jo y Energ .

WF = (F cos) x = 250 (cos60) 5 = 625 J WN = (N cos90) x = 0 WP = (mg cos270) x = 0 WR = (FR cos180) x, Como no se conoce el v lor de l fuerz de roce, se debe c lcul r, del DCL y pl ic ndo l primer ley de Ne ton, y que l c j se mueve con r pidez const nte, se obtiene: Eje x: Eje y: F cos - FR = 0 F sen + N - mg = 0 (1) (2) De (1) FR = F cos = 250 cos60 = 125 N, reempl z ndo en el tr b jo, WR = 125 cos180 5 = -625 J b) Por definicin, FR = N, despej ndo N de (2) se tiene N = mg - F sen, en tonces: FR = (mg Fsen ) = FR mg Fsen =

5.2 TRABAJO REALIZADO POR UNA FUERZA VARIABLE. i una fuerza variable F es moviendo a un obje o a lo largo del eje x desde una posicin inicial a o ra final, ya no se puede usar la expresin an erior para calcul ar el rabajo realizado por la fuerza. En es e caso se puede hacer que el 146

125 = 0.44 50

10 250 sen60

P r F: P r N: P r

mg: P r FR:

Cap. 5 Trabajo y Energa. cuerpo experimen e pequeos desplazamien os dx, en onces la componen e Fx de la fu erza en la direccin del desplazamien o se puede considerar aproximadamen e cons a n e en ese in ervalo dx y se puede calcular un rabajo dW en ese pequeo desplazam ien o como: dW = Fx dx i se calcula el rabajo o al en el desplazamien o desde la posicin inicial a la final, es e es igual a la suma de odos los pequeos raba jos dW, es o es: W = dW W = x Fx dx i xf Matemticamente, el valor de la integral es numricamente igual al rea bajo la curva de Fx versus x (figura 5.3). Si actan ms de una fuerza sobre el cuerpo, el trabajo resultante es el realizado por la componente de la fuerza resultante en direccin del desplazamiento, entonces en trminos del producto escalar en tres dimensiones , el trabajo total es: WTOTAL = r rf ri F dr r r r (5.1) Figura 5.3 147

  

 

Cap. 5 Trabajo y Energa. Ejemplo 5.2: Calcular rabajo realizado por un resor e. Un sis ema fsico comn en el que la fuerza vara con la posicin, es el de un cuerpo co nec ado a un resor e. i el resor e, orien ado en direccin del eje x, se deforma desde su configuracin inicial, es decir se es ira o se comprime, por efec o de al guna fuerza ex erna sobre el resor e, ins an neamen e ac a una fuerza F producida por el resor e con ra el obje o que ejerce la fuerza ex erna, cuya magni ud es: F= kx donde x es la magni ud del desplazamien o del resor e desde su posicin no deforma da en x = 0 y k una cons an e posi iva, llamada cons an e de fuerza del resor e, que es una medida de la rigidez (dureza) del resor e. Es a ecuacin se llama Ley de Hooke, y es vlida para pequeos desplazamien os, ya que si el resor e se es ira demasiado, puede deformarse y no recuperar su forma original. El signo nega ivo indica que la direccin de es a fuerza es siempre opues a al desplazamien o, como se ilus ra en la figura 5.4, donde F represen a la fuerza producida por el resor e. Figura 5.4 148

 

  

 

 

 

Cap. 5 Trabajo y Energa. i el cuerpo se desplaza desde una posicin inicial a la final, el rabajo realiza do por el resor e es: W = x ( kx )dx = xf i 1 2 1 2 kxi kx f 2 2 Por ejemplo, para un resor e de k = 100 N/m, que se es ira 10 cm (= xf), el rab ajo que realiza la fuerza del resor e para recuperar su posicin inicial no deform ada (xi = 0) es 0.5 J. 5.3 ENERGA CINTICA. Cuando se hace rabajo con ra el roce, se observa que en la superficie de los cu erpos en con ac o se produce un aumen o de empera ura. Es porque se ha producid o una ransformacin desde movimien o a calor, es decir que se ha producido una r ansferencia de energa de movimien o a energa calrica. En o ras ransformaciones se produce energa en forma de luz, sonido, elc rica, nuclear, e c. En las ransformac iones se miden cambios de energa cuando se realiza rabajo, aparecen las fuerzas que realizan rabajo, por lo an o el rabajo es una medida de las ransferencia s de energa. El concep o de energa se puede generalizar para incluir dis in as for mas de energa conocidas como cin ica, po encial, calrica, elec romagn ica, e c. De e s a forma, la mecnica de los cuerpos en movimien o se relaciona con o ros fenmenos na urales que no son mecnicos por in ermedio del concep o de energa. El concep o de energa invade oda la ciencia y es una de las ideas unificadoras de la Fsica. C uando una fuerza ac a sobre un cuerpo, le produce una aceleracin duran e su despla zamien o. El rabajo realizado por la fuerza para mover al cuerpo es: r rf i WTOTAL = r r F dr r r Por la segunda Ley de New on se iene: 149

 

 

 

 

 

 

  

Cap. 5 Trabajo y Energa. r r r r r r r dv dv dr dv r r F = ma = m d = m dr d reemplazando en el rabajo o al, se ob iene: r rf r ri WTOTAL = r r v r r 1 1 2 r dv r mv r dr = m v v dv = mv 2 mv0 r dr 2 2 0 La can idad mv2, se llama energa cin ica, Ec, es energa que se ob iene por el movimi en o, es siempre posi iva porque la rapidez es al cuadrado. 1 Ec = mv2 2 (5.2) Por lo an o, el rabajo realizado por la fuerza resul an e sobre una par cula es igual al cambio de energa cin ica, enunciado que se conoce como el Teorema del Tr abajo y la Energa. Cuando la rapidez es cons an e, no hay variacin de energa cin ica y el rabajo de la fuerza ne a es cero. La unidad de medida de la energa cin ica es el Joule, J. 5.4 POTENCIA. Para fines prc icos in eresa ambin conocer la rapidez con la cual se realiza ajo. Es a informacin la en rega la po encia, que se define como la rapidez de nsferencia de energa. i se aplica una fuerza ex erna a un cuerpo y se realiza abajo dW en un in ervalo de iempo d , la po encia ins an nea P (cuidado de no nfundir con el peso de un cuerpo) se define como: 150

rab ra r co

  

 

 

 

= mv dr ,

 

    

 

 

Cap. 5 Trabajo y Energa. P= dW d

La unidad de medida de la po encia en el I es J/s, que se llama Wa , smbolo W ( cuidado de no confundir con el rabajo). Como dW = F dr, se puede escribir a po tencia como: r r F dr r r = F v P= dt (5.3)

151

Ejemp o 5.3: Un mueb e de 40 kg que se encuentra inicia mente e reposo, se empu ja con una fuerza de 130 N, desp azndo o en nea recta una distancia de 5 m a o argo de un piso horizonta de coeficiente de roce 0.3 (figura 5.1). Ca cu ar: a) e trabajo de a fuerza ap icada, b) e trabajo de roce, c) a variacin de ener ga cintica, d) a rapidez fina de mueb e, e) a potencia fina de a fuerza ap i cada.

Se puede definir una nueva unidad de energa en trminos de a unidad de potencia, amada ki o att hora. Un ki o att hora (kWh) es a energa uti izada durante una h ora con una potencia constante de 1 kW. E va or de un kWh es: 1 kWh = 1000 W 36 00 s = 3.6 x 106 J. E kWh es unidad de energa, no de potencia. Por ejemp o, para encender una ampo eta de 100 W de potencia se requieren entregar e 3.6 x 105 J de energa durante una hora, equiva ente a 0.1 kWh. Notemos que esta es una unida d de medida que nos indica que a energa es una magnitud fsica que, aunque abstrac ta, tiene va or comercia , se puede vender y comprar, ya que por ejemp o, todos os meses pagamos por una determinada cantidad de ki o att hora o energa e ctrica para nuestros hogares, en cambio no se pueden comprar 50km/h de rapidez, pero si compramos energa en forma de gaso ina para hacer que un vehcu o pueda moverse.



Cap. 5 Trabajo y Energa.

a)

WF = (130N)(5m) = 650J FR = N = mg r r WR = FR r = FR (cos180) x = mgx WR = 0.340105 = 600 J WTota = Ec WF +WN + P = Ec, pero WN = WP = 0, ya que las fuerzas normal y peso son perpendiculares al desplazamiento, entonces: Figura 5.5 Problema 5.3 b) c) Ec = WF +WR = 650 600 = 50 J d) Para calcular la rapidez final, usamos el resultado anterior EC = 1 2 1 2 1 2 mv f mv0 = mv v f = 2 2 2 vf = 2 E C = m 2 50 m = 1.6 40 s 2EC m e) Usando la definicin de potencia: r r Pf = F v = F cos 0 v f = Fv

152

Pf = 130

1.6 = 208( wa



r r W = F r = F cos 0

x = Fx

So ucin: E diagrama de cuerpo ibre para e muestra en a figura 5.5.

mueb e de masa m de a figura 5.1 se

Cap. 5 Trabajo y Energa. 5.5 FUERZA CON ERVATIVA Y NO CON ERVATIVA . e llaman fuerzas conserva ivas aquellas para las cuales el rabajo realizado po r las fuerzas para mover un cuerpo en re dos pun os por cualquier rayec oria ar bi raria, no depende de la rayec oria que une los pun os. Las fuerzas que depen den de la posicin son conserva ivas, por ejemplo: la gravi acional, els ica, elec romagn ica, e c. uponer que una par cula se mueve, por la accin de una fuerza, des de una posicin inicial P has a o ra posicin final Q, por rayec orias arbi rarias 1 y 2, como se ve en la figura 5.6a. i la fuerza es conserva iva, en onces el rabajo para mover la par cula desde P a Q slo depende de las coordenadas inicial y final de la par cula, es o es: WPQ (por rayec oria 1) = WPQ (por rayec oria 2) Figura 5.6a Figura 5.6b i ahora la par cula se mueve desde P has a Q por la rayec oria 1 y luego regres a desde Q has a P por la rayec oria 2 (figura 5.6b), se observa que en el regre so, WQP (por rayec oria 2) = WPQ (por rayec oria 2), en onces: WPQ(por rayec oria 1) = WQP(por rayec oria 2) WPQ(por rayec oria 1) + WQP(por rayec oria 2) = 0 153

 

 

 

 

 

 

  

 

 

 

Cap. 5 Trabajo y Energa. En onces, si la par cula se mueve desde una posicin inicial, realiza un circui o d onde regresa a la misma posicin inicial, el rabajo realizado por una fuerza cons erva iva en una rayec oria cerrada es cero. Por el con rario, las fuerzas no co nserva ivas o fuerzas disipa ivas son aquellas para las cuales el rabajo realiz ado por las fuerzas para mover una par cula en re dos pun os, depende de la raye c oria que se realice para unir los pun os. Para las fuerzas no conserva ivas se iene que, WPQ(por rayec oria 1) WPQ(por rayec oria 2). Las fuerzas de roce, que siempre se oponen al desplazamien o, son no conserva ivas o disipa ivas, el rabajo de es as fuerzas es nega ivo y le hacen perder energa al sis ema. 5.6 ENERGA POTENCIAL. El rabajo realizado por una fuerza conserva iva es independien e de la rayec o ria y de la rapidez con la que se mueve la par cula. En es e caso el rabajo es sl o funcin de las coordenadas, por lo que se puede asociar con una variacin de energa funcin de la posicin, similar al caso de la energa cin ica que es funcin de la veloc idad. Las fuerzas que son funcin de la posicin generan energa de posicin, a la que s e llama energa po encial. El rabajo realizado por la fuerza se almacena como ene rga po encial en el obje o en movimien o. e define la energa po encial EP, a aque lla que puede ob enerse en vir ud de la posicin del cuerpo, al que el rabajo re alizado por la fuerza conserva iva en re dos posiciones, es igual a la disminucin de la energa po encial, es o es, el rabajo realizado por una fuerza conserva iv a es igual al valor nega ivo del cambio de energa po encial asociada con la fuerz a: r r W = F dr = E P = E Pi E Pf r rf r ri e puede elegir una posicin de referencia inicial y medir las diferencias de ener ga po encial respec o a ese pun o y definir una funcin energa po encial en cualquie r posicin r como: 154

 

 

 

 

 

 

 

 

 

Cap. 5 Trabajo y Energa. r r r r r E P (r ) = r F dr + E Pi r i El valor de EPi generalmen e no se conoce, por lo que se elige una posicin arbi r aria, donde por convencin se le asigna el valor cero a la energa po encial inicial , EPi = 0, ya que por su definicin, slo iene significado fsico el cambio de energa po encial. Es a posicin arbi raria se llama nivel de referencia y puede ser cualq uiera; generalmen e se oma como nivel de referencia la superficie de la Tierra o cualquier o ra posicin convenien e, pero una vez que se ha elegido no debe camb iarse. Con es a eleccin, se define la energa po encial en una posicin r como: r r r E P ( r ) = F dr (5.4) Para las fuerzas no conserva ivas no exis e una funcin de energa po encial, ya que el rabajo, que depende de la rayec oria, no es funcin de la posicin inicial y f inal de la par cula. Ejemplo 5.4. Calcular la energa po encial de la fuerza peso. e calcular el rabajo y la energa po encial para una par cula que se deja caer lib remen e desde una posicin inicial yi a o ra posicin final yf (figura 5.7). La fuer za que produce el movimien o de la par cula es la gravi acional, que para cada lib re es el peso P = mg, en onces el rabajo es: r r r y r W = r F dr = y mg ( ) dy ( ) j j r f f i i W = mgy f mgyi Es o demues ra que la fuerza gravi acional es conserva iva, ya qu e el rabajo realizado por esa fuerza depende slo de las posiciones inicial y fin al de la par cula. 155

 

 

  

Cap. 5 Trabajo y Energa. Figura 5.7. Ejemplo 5.4.

La variacin de energa po encial de la par cula es: E P = W = (mgy f mgyi ) = mgyi f Como las posiciones inicial y final son arbi rarias, se define la energa po enc ial de la fuerza gravi acional, o simplemen e energa po encial gravi acional Eg, vlida en las condiciones de cada libre, por la expresin: E g = mgy (5.5) Ejemplo 5.5. Calcular la energa po encial de la fuerza els ica. O ra fuerza conserva iva es la que ejerce un resor e deformado sobre un cuerpo f ijo a l. El rabajo realizado por la fuerza els ica del resor e sobre el cuerpo ya se calcul, y es: W = x ( kx )dx = xf i 1 2 1 2 kxi kx f = E P = E Pi E Pf 2 2 156

 

Cap. 5 Trabajo y Energa. Es o permi e definir la energa po encial els ica EE almacenada en un resor e como: 1 2 kx 2 EE = (5.6) La energa po encial els ica es cero cuando el resor e no es deformado, es mxima cua ndo alcanza su deformacin mxima y es siempre posi iva ya que es proporcional a x2. 5.7 CON ERVACIN DE LA ENERGA MECNICA. Cuando una par cula se mueve por la accin de una fuerza conserva iva, por el eore ma del rabajo y la energa se iene que el rabajo realizado por la fuerza es igu al a la variacin de energa cin ica de la par cula: W = Ec Pero como la fuerza es cons ervativa, entonces W = -EP, donde EP puede ser la energa potencial gravitacional, elstica o cualquier otra forma de energa potencial mecnica. Igualando ambas expresi ones del trabajo se obtiene: E c = E P E c + E P = 0 ( Ec + E P ) = 0 esta ecuacin se puede escribir tambin de la siguiente forma: Eci + EPi = Ecf + E Pf 157

Cap. 5 Trabajo y Energa. Se puede definir la energa mecnica total como la suma de la energa cintica y la ener ga potencial: E = Ec + E P entonces se obtiene la ley de conservacin de la energa mecnica, que se escribe como : Ei = E f E = cte (5.7) La ley de conservacin de la energa mecnica establece que la energa mecnica total de u n sistema permanece constante si las nicas fuerzas que realizan trabajo sobre el sistema son conservativas. Cuando una cantidad fsica no cambia, decimos que se co nserva. Decir que la energa se conserva significa que la cantidad total de energa de un sistema natural no cambia, no se puede crear ni destruir energa, slo se pued e convertir de una forma a otra. Es una de las leyes fundamentales de la Fsica, d educida a partir de una de las leyes fundamentales de la mecnica, la segunda ley de Ne ton. Si las fuerzas presentes en un sistema mecnico no son conservativas, c omo ocurre en los sistemas reales, la energa aparentemente no se conserva, porque se transforma en otro tipo de energa. Por ejemplo, la fuerza de roce se dice que es disipativa porque disipa energa, que se transforma en calor en la superficie de contacto entre los cuerpos. En efecto, se puede aplicar el teorema del trabaj o y la energa tomando en cuenta la existencia de las fuerzas no conservativas. Si WNC es el trabajo sobre una partcula de todas las fuerzas no conservativas y WC el trabajo de todas las fuerzas conservativas, entonces: WNC + WC = Ec Como WC = -EP entonces: 158

Cap. 5 Trabajo y Energa. W NC = EC + E P W NC = ( ECf ECi ) + ( E Pf E Pi ) W NC = ( ECf + E Pf ) ( ECi + E Pi ) = E f Ei W NC = E f Ei Es decir, el rabajo realizado por odas las fuerzas no conserva ivas es igual a l cambio de energa mecnica o al del sis ema. Ejemplo 5.6. Conservacin de la energa en el movimien o de cada libre. Aplicando el principio de conservacin de la energa para un cuerpo en cada libre, se ob iene a la siguien e expresin: E = c e E ci + E gi = E cf + E gf 1 2 1 mvi + mgy i = mv 2 + mgy f f 2 2 Si se conoce la rapidez inicial y la posicin inicial y final de la partcula, se pu ede calcular su rapidez final: v f = vi + 2 g ( y i y f ) expresin equivalen e a la ob enida por m odos cinem icos. Ejemplo 5.7. Para el sis ema de la figura 5.8, donde el cuerpo de masa m desliza desde una al ura h por la superficie curva sin roce, calcular la compresin mxima del resor e de cons an e k, cuando la masa choca con l. olucin: si no hay roce, s e conserva la energa mecnica, en onces: E = c e E ci + E gi + E Ei = E cf + E gf + E Ef 159

  

 

 

Cap. 5 Trabajo y Energa. 1 2 1 1 1 mvi + mgyi + kxi2 = mv 2 + mgy f + kx 2 f f 2 2 2 2 Figura 5.8 Ejemplo 5.7 Eligiendo el punto inicial i en la parte superior de la pista curva y el punto f inal f en la posicin de la mxima compresin del resorte (figura 5.8), la energa cintic a inicial y final es cero, porque m parte del reposo, vi = 0, y en la compresin mx ima del resorte vf = 0 ya que se detiene; la energa gravitacional inicial es mgyi = mgh, ya que yi = h y la final es cero en el suelo, porque se considera que la altura yf es cero; la energa elstica inicial es cero porque en esa posicin no hay resorte, entonces queda: mgh = 1 2 kx x = 2 2mgh k donde x es la compresin mxima del resorte. 5.8 ENERGIA Y LA MAQUINA HUMANA. La magnitud Fsica tal vez ms importante en la descripcin de la naturaleza es la Ene rga. Es un concepto difcil de definir; no siempre se advierte y cambia de aspecto con facilidad asombrosa. Las formas bajo las cuales se presenta la energa, suelen ser tan diferentes que la humanidad demor siglos en reconocerla. Su importancia principal radica en su permanencia; veremos que puede afirmarse que la energa es una magnitud increable e indestructible. Esta calidad de permanencia constituye un concepto unificador importante, porque 160

Cap. 5 Trabajo y Energa.

fenmenos tan diversos como el funcionamiento de un motor y el movimiento del cuer po humano, puede analizarse en funcin del paso continuo de energa de una a otra de sus formas y su simultnea transferencia de un cuerpo a otro. Son diversas las fo rmas bajo las cuales puede presentarse la energa la energa: un cuerpo por el slo he cho de estar en movimiento posee energa cintica; el mismo cuerpo u otro en virtud de su posicin respecto a un cierto nivel de referencia tiene energa potencial grav itacional; un cuerpo elstico que ha sido deformado posee energa potencial elstica. La lista de formas de energa no termina aqu. Se dice que los cuerpos que rotan, po seen energa de rotacin; los que vibran, energa vibracional; las ondas como las onda s marinas transportan energa ondulatoria; las ondas luminosas, energa luminosa; lo s cables elctricos transportan energa elctrica; en el interior del tomo tenemos ener ga atmica, energa nuclear; en las reacciones qumicas estamos en presencia de energa q umica, etc. Es un hecho comprobado que hay muchos casos en los que aparentemente no se mantiene constante la suma de la energa cintica y potencial de un cuerpo o c uando se aplican fuerzas externas sobre l, el trabajo realizado no se invierte en su totalidad en aumentar la energa cintica o potencial. Por ejemplo, si dejamos c aer un objeto al suelo, llega con cierta velocidad (con cierta energa cintica), pe ro al llegar se detiene y pierde su energa cintica sin que gane energa potencial. S i arrastramos un cuerpo por el suelo, movindolo con velocidad aproximadamente con stante, en realidad tenemos que realizar una fuerza y por tanto, al haber despla zamiento, un trabajo, pero este trabajo no se emplea en aumentar la energa potenc ial, porque el cuerpo se desplaza horizontalmente, ni energa cintica, porque la ve locidad no aumenta. Qu ha pasado con la energa cintica en el primer caso? Qu ha ocurr do, en el segundo, con la energa que en forma de trabajo se le suministr al cuerpo ? La respuesta es: la energa que ha desaparecido se ha transformado en energa inter na del suelo o del cuerpo que se mueve. Ntese que no decimos que se ha transformad o en calor, como se podra esperar, sino en energa interna. En otro curso de fsica se blar del calor y veremos la razn de esta distincin. En conclusin, vivimos rodeados d e energa. No slo la energa intrnseca de las molculas, tomos y ncleos, sino tambin m staciones de la energa a escala macroscpica como resultado de la organizacin parcia l del movimiento molecular, tal como la energa del viento en una tormenta, la ene rga del agua en una catarata, de un ro o de las mareas, la energa del vapor produci do 161

Cap. 5 Trabajo y Energa.

en un volcn o en el interior de la Tierra, etc. Uno de los grandes problemas es d isear los medios para que esa energa pueda aprovecharse bajo control en la forma q ue nos interese, esto es, como energa til. Sin embargo, slo sabemos transformar en energa til una pequesima fraccin de la energa a nuestro alrededor, debido en gran par e a la falta de organizacin en la materia y a que, para producir cierta organizac in molecular, es necesario a su vez invertir cierta energa. 5.8.1 Cmo camina la mquina humana? El movimiento del cuerpo humano se explica con l os mismos principios de fuerza y trabajo que describen todo movimiento. Las mquin as simples, en forma de palancas, dan la capacidad para caminar y correr. Los si stemas de palanca del cuerpo son complejos, pero en un modelo se pueden consider ar cuatro partes bsicas que se muestran en la figura 5.9: 1 una barra rgida (un hu eso), 2 una fuente de fuerza (un msculo), 3 un punto de apoyo (articulaciones mvil es entre los huesos) y 4 una resistencia (peso del cuerpo u objeto que se levant a o mueve). Los sistemas de palanca del cuerpo humano no son muy eficientes, por esto caminar y correr requiere energa (se queman caloras) y ayuda a que las perso nas bajen de peso. Figura 5.9. Modelo del sistema de palanca del cuerpo humano. 162

Cap. 5 Trabajo y Energa. Cuando una persona camina, la cadera acta como punto de apoyo y se mueve a travs d el arco de un crculo centrado en el pie. El centro de masa del cuerpo se mueve co mo una resistencia alrededor del punto de apoyo en el mismo arco. La longitud de l radio del crculo es la longitud de la palanca formada por los huesos de la pier na. Los atletas de marcha incrementan su rapidez balanceando las caderas hacia a rriba para aumentar este radio. 5.8.2 Articulaciones artificiales. Se han logrado grandes avances en el diseo y s ustitucin de articulaciones lesionadas por articulaciones artificiales. Debido a las inmensas tensiones de las articulaciones en los brazos y las piernas, loa ma teriales con los cuales se elaboran las partes artificiales y las uniones deben ser extremadamente fuertes. El titanio es un material comn usado para elaborar ar ticulaciones artificiales. Pero ahora se est desarrollando y probando la resisten cia de plsticos ligeros y materiales similares a los huesos. Las uniones permanen tes en las articulaciones artificiales generalmente se hacen por medio de cement os especiales, por fijacin biolgica con un sistema de ajuste preciso. En la fijacin b iolgica se usa un ma erial poroso que permi e al hueso crecer den ro de la pared ar ificial. Huesos de ajus e preciso son hechos de manera an exac a que encajan e n su si io alrededor de los huesos na urales. in impor ar el m odo usado, las ar iculaciones ar ificiales deben ser capaces de sopor ar las cargas normales. Las ar iculaciones de la cadera y el codo son las reas que sopor an el mayor esfuerz o. La ar iculacin redondeada de la cadera sopor a la mayor par e del peso del cue rpo y es esencial para caminar. Aunque el codo no es una ar iculacin que sopor e mucho peso, es el pun o de apoyo de la palanca del an ebrazo y debe sopor ar esf uerzos significa ivos. Por ejemplo al sos ener un peso de 10N (1kg) en la palma de la mano con el codo formando un ngulo de 90, sobre el se ejerce una fuerza de 9 0N (9kg). 163

 

Cap. 5 Trabajo y Energa. PROBLEMA . 5.1. Una par cula de 4 kg. se mueve desde el origen has a la posicin C que iene c oordenadas x=5m e y=5m con la influencia de la fuerza de gravedad, la cual ac a e n la direccin y nega iva (figura 5.10). Calcule el rabajo realizado por la grave dad al ir de O a C a lo largo de las siguien es rayec orias: a) OAC, b) OBC, c) OC. R: 200 J. 5.2. Una fuerza que ac a sobre una par cula que se mueve sobre el plano r horizon al xy es dada por F = 2 y + x 2 N , en donde x e y es n en i j m. La par cula se mueve desde el origen has a una posicin final C de coordenadas x=5m e y=5m, como en la figura 5.10. Calcular el rabajo efec uado por la fuerza a l o largo de a) OAC, b) OBC, c) OC. d) F es conserva iva o no conserva iva? R: a) 1 25 J, b) 50 J, c) 66.7 J d) No. ( ) Figura 5.10 Problemas 5.1 y 5.2 r i j 5.3. Una sola fuerza cons an e F = 3 + 5 N ac a sobre una par cula de 4 kg. a ) Calcule el rabajo efec uado por es a fuerza si la par cula se mueve desde el o rigen has a un pun o cuyo vec or de posicin es r r = 2 3 m. Es e resul ado depende de la rayec oria? Explicar b) i j Cul es la rapidez de la par cula en r si su rapi dez en el origen es 4 m/s. c) Cul es el cambio en la energa po encial de la par cula ? R: a) 9 J, b) 3.4 m/s, c) 9 J. ( ) ( ) 5.4. El Nico recibe un servicio del Fea con una pelo a de enis de 50 gr, la cual al llegar a la raque a del Nico con una rapidez de 200 km/hr, la hunde 2 cm, se de iene y sale nuevamen e disparada ( odo eso ocurre en 164

  

 

 

 

 

 

Cap. 5 Trabajo y Energa. un in ervalo de iempo muy pequeo). Calcular: a) la energa cin ica de la pelo a an es que golpee la raque a, b) el rabajo realizado sobre la pelo a duran e el gol pe, c) la fuerza media sobre la pelo a. R: a) 77J, c) 3850N 5.5. obre un cuerpo de 2 kg que se mova inicialmen e con una rapidez de 5 m/s hacia la derecha, en u na superficie horizon al, se aplica una fuerza de 10 N inclinada 30 respec o a la horizon al. El desplazamien o mien ras se ejerce la fuerza fue de 5 m, y el coe ficien e de roce es 0.25. Calcular a) el rabajo realizado por cada fuerza sobre el cuerpo, b) la variacin de energa cin ica, c) la velocidad final del cuerpo. R: b) 24.5 J, c) 7 m/s. 5.6. obre un cuerpo de masa M que se mova inicialmen e con una rapidez v0 hacia la derecha, en una superficie horizon al de coeficien e de roce , se aplica una fuerza de magni ud F inclinada sobre l horizont l. El despl z miento mientr s se ejerce l fuerz fue D. C lcul r: ) el tr b jo re liz do por F sobre el cuerpo, b) el tr b jo re liz do por l fuerz de roce, c) l v ri cin de energ cintic , d) l r pidez fin l del cuerpo. Expres r los result dos en funcin de los v lores conocidos M, v0, , F, y D. R: b) -(Mg-Fsen)D, d) 2 v o + (2 D M )[F cos (Mg Fsen )] .

5.7. Un fuerz F p r lel un pl no inclin do en 37, se plic sobre un bloque de m s 50 kg. El bloque se mueve con un r pidez const nte de 10 m/s h ci rri b del pl no, un dist nci de 20 m. El coeficiente de roce cintico entre el bloq ue y el pl no inclin do es 0.2. C lcul r el tr b jo efectu do sobre el bloque po r l s fuerz s ) F, b) roce y c) de gr ved d. R: ) 7.5 kJ, b) 1.6 kJ, c) 6 kJ. 5. 8. Un bloque de 5 kg. se pone en movimiento subiendo por un pl no inclin do en u n ngulo de 30 respecto l horizont l, con un r pidez inici l de 8 m/s. El bloqu e lc nz el reposo despus de recorrer 3 m lo l rgo del pl no inclin do spero. D etermine: ) el c mbio en l energ cintic . b) el c mbio en l energ potenci l. c ) l fuerz de roce sobre el bloque. d) el coeficiente de roce cintico. R: ) 160 J, b) 73.5 J, c) 28 N, d) 0.7. 165

 

 

C p. 5 Tr b jo y Energ . 5.9. Algunos lumnos de Fsic , despus de s ber el result do de su primer cert men, se premi n subiendo v ri s veces l cerro del EULA. ) Cunto tr b jo re liz n en n subid s? b) Comp r r l potenci cu ndo suben el cerro corriendo con l potenc i cu ndo b j n lent mente. c) Un kilo de gr s entreg unos 10 kWh de energ , si se convierte gr s en energ con un rendimiento del 20%, un cerro de que ltur tendr n que subir p r b j r 2 kilos de peso? R: ) n(mgh), c) si m=72 kg, 20 km. 5.10. Por un seccin unit ri del S lto del L j fluye gu r zn de Q kg/s. Supo niendo que de l potenci gener d por l c d del gu en el s lto se provech un 58%, Cunt s mpollet s de 100 W se podr n encender con es potenci ? R: depende de v lores estim dos. 5.11. Se tiene un sistem form do por 5 esferit s de m s M unid s por cuerd s tens s de m s despreci ble, sep r d s L entre s, coloc do i nici lmente en form horizont l. C lcul r el tr b jo neces rio p r poner un un tod s l s esferit s en posicin vertic l. R: 10 MgL. 5.12. Un bloque de m s m se suelt desde l p rte superior de un pist lis form d por un cu dr nte cnc vo de circunferenci de r dio R por l cu l desliz . Cu ndo lleg l extremo in ferior choc con un resorte de const nte k que se encuentr ubic do sobre un su perficie horizont l. C lcul r: ) l energ cintic del cuerpo justo ntes de choc r con el resorte, b) l compresin mxim del resorte. R: ) mgR, b) 2mgR / k . 5.1 3. Un esfer de 0.5 kg desliz por un riel curvo p rtir del reposo en el punt o A de l figur 5.11. El tr mo de A B no tiene roce y el de B C si tiene ro ce. ) C lcul r l r pidez de l esfer en B. b) Si l esfer lleg l reposo en C, c lcul r el tr b jo por el roce en el tr mo BC. R: ) 4.5 m/s, b) 2.5 J. 5.14 . Un bloque de m s m comienz moverse desde un ltur H sobre un pl no incli n do en 30. Al lleg r l p rte inferior del pl no, el bloque se desliz por un superficie horizont l. Si el coeficiente de friccin en mb s superficies es , c l cul r l dist nci horizont l que desliz r el bloque ntes de lleg r l reposo. 166

C p. 5 Tr b jo y Energ . Figur 5.11 Problem 5.13. 5.15. Desde el extremo superior de un pl no inclin do respecto l horizont l, de coeficiente de roce de , desliz desde el reposo, un bloque de m s M. El bloq ue se mueve un longitud L ntes de comprimir un resorte de const nte K ubic d o en l p rte inferior del pl no. ) C lcul r, en funcin de los v lores conocidos M, L, K, , y g, l r pidez del bloque justo ntes de toc r l resorte. b) Deduci r (no resolverl ) l expresin que permite c lcul r l mxim compresin del resorte. 5.16. Desde l b se de un pl no inclin do 30 respecto l horizont l, se l nz e n subid un cuerpo de 1 kg. El cuerpo recorre 0.5 m y despus comprime 0.1 m un re sorte de const nte 100 N/m ubic do en l p rte superior del pl no ntes de deten erse. ) Si el pl no es liso, determine l r pidez inici l del cuerpo. b) Si l r pidez con l que el cuerpo inici l subid del pl no fuer el doble de l c l cul d en ) y el coeficiente de roce entre el cuerpo y el pl no fuer de 0.2, cun to se comprimir el resorte? c) y si l r pidez se reduce l mit d? R: ) 2.64 m/ s, b) 0.38 m. c) no h y compresin. 5.17. Un bloque de 1kg que cuelg por el cost do de un mes se conect por un cuerd que p s por un pole ide l un resor te de const nte 100 N/m, ubic do horizont lmente sobre l mes , fijo en el otro extremo. Se sostiene inici lmente l bloque en reposo m nteniendo l resorte sin estir r y luego se suelt . C lcul r: ) el estir miento mximo del resorte. b) l r pidez del bloque cu ndo el resorte se h estir do l mit d del l rg miento mx imo. R: ) 0.2 m, b) 1 m/s. 5.18. Un pelot describe un circunferenci vertic l en el extremo de un cuerd . Si l energ tot l de l pelot perm nece const nt e, demuestre 167

C p. 5 Tr b jo y Energ . que l tensin en l cuerd en l p rte ms b j es m yor que l tensin en el punto ms lto en seis veces el peso de l pelot . 5.19. A l m s de 1 kg de un pndulo de 1 m de longitud, se l impuls con un r pidez inici l de 2 m/s en su posicin ms b j . Cu ndo l cuerd form un ngulo de 30 con l vertic l, c lcul r: ) l v ri cin de energ gr vit cion l de l m s , b) l r pidez de l m s , c) l ltur mxim lc nz d por l m s por sobre su posicin ms b j . R: ) 1.3J, b) 1.2m/s, c) 0. 2m. 5.20. T rzn de m s M, p r impresion r J ne, se b l nce de un li n de l ongitud L (como un pndulo) lc nz ndo un r pidez vo en su posicin ms b j , esto es cu ndo l li n se encuentr vertic l. Luego, cu ndo l li n form un ngulo con l vertic l, c lcul r en funcin de los v lores conocidos M, L, vo, y g: ) l r pidez de T rzn, b) l tensin en l li n . c) ltur mxim lc nz d por T rzn desde su posicin ms b j . 5.21. L esfer de m s m de un pndulo de longitud L se m ntien e inici lmente en posicin vertic l. Cu ndo sopl un viento con un fuerz const n te F no conserv tiv , demuestre que si l esfer comienz moverse 2L desde el reposo, l ltur mxim que lc nz es H = . 2 1 + (mg F ) 5.22. Un m s de peso P se m rr un hilo de pesc que puede soport r h st u n peso de 4P. Si l m s se suelt desde el reposo en l posicin horizont l, c lc ul r el ngulo respecto l vertic l l cu l se rompe el hilo. 5.23. Se l nz un pelot en un ngulo respecto l horizont l, desde un ltur h, con un r pidez inici l vo. Us r el mtodo de l energ p r c lcul r, cu ndo su ltur es h/2 l velocid d de l pelot . 5.24. Un proyectil de 1 kg se l nz desde l superficie con un r pidez inici l de 180 km/h en un ngulo de 30 sobre el suelo. C lcul r ) el tr b jo p r que lc nce su ltur mxim , b) su energ cintic cu ndo se encuent r en su ltur mxim , c) l potenci medi entre l superficie y su ltur mxim . 168

C p. 5 Tr b jo y Energ . 5.25. Un bloque de 0.5 kg. se mueve h ci l derech sobre un superficie horizo nt l sper y choc contr un resorte horizont l, de const nte 100 N/m. L r pidez del bloque justo ntes del choque es 10 m/s. Despus que el resorte h ce rebot r l bloque h ci l izquierd , su r pidez justo cu ndo dej el resorte es 5 m/s. Si el coeficiente de r zon miento cintico entre el bloque y l superficie es de 0 .4, determine: ) el tr b jo re liz do por l friccin mientr s el bloque se encue ntr en cont cto con el resorte y b) l mxim compresin del resorte. 5.26. Se colo c un bloque de m s 0.25 kg sobre un resorte vertic l de const nte k=5000 N/m y se empuj h ci b jo, comprimiendo el resorte un dist nci de 0.1 m. Cu ndo e l bloque se suelt , dej el resorte y continu su c mino h ci rrib . A qu ltur mxim por encim del punto de liber cin lleg el bloque? R: 10 m. 5.27. Se conect n dos m s s por un cuerd liger que p s por un pole de m s despreci ble, sin friccin, como se muestr en l figur 5.12. Un m s de 5 kg se liber desde el reposo, de un ltur de 2.5 m sobre el suelo. Utiliz ndo l ley de l conser v cin de l energ determin r: ) l velocid d fin l de l m s de 5 kg, b) l vel ocid d de l m s de 3 kg justo cu ndo l m s de 5 kg choc con el piso, c) l ltur mxim l cu l se elev r l m s de 3 kg. R: b) 4.5 m/s, c) 5 m. 5.28. El coeficiente de friccin entre el objeto de 3 kg y l superficie de l mes que se ve en l figur 5.13, es 0.4. cul es l r pidez de l m s de 5 kg que cuelg , cu ndo h c do un dist nci vertic l de 1 m? R: 3.1 m/s. 5.29. Un bloque de 2 kg so bre un pl no spero inclin do en 37, se conect un resorte ligero de const nte 10 0 N/m (figur 5.14). El bloque se suelt del reposo cu ndo el resorte no est esti r do y se mueve 20 cm h ci b jo del pl no ntes de detenerse. C lcul r el coef iciente de roce. R: 0.12. 5.30. Supong que el pl no inclin do del sistem descr ito en el problem nterior es liso. El bloque se liber p rtir del reposo con el resorte inici lmente no estir do. ) Cunto se despl z h ci b jo del pl no ntes de qued r en reposo? b) cul es l celer cin del bloque l lleg r 169

C p. 5 Tr b jo y Energ .

Figur 5.12. Figur 5.13. Problem 28 Figur 5.14. Problem 29 170

su punto ms b jo? Su celer cin es const nte? c) Describ nerg que ocurren dur nte el descenso del bloque.

l s tr nsform ciones de e

C p. 6 Torque y equilibrio. CAPTULO 6. TORQUE Y EQUILIBRIO DE CUERPO RGIDO. En gener l un cuerpo puede tener t res tipos distintos de movimiento simultne mente. De tr sl cin lo l rgo de un t r yectori , de rot cin mientr s se est tr sl d ndo, en este c so l rot cin puede s er sobre un eje que p se por el cuerpo, y si l vez este eje est gir ndo en t orno un eje vertic l, l rot cin del eje del cuerpo rot nte se le ll m movim iento de precesin (por ejemplo un trompo), y de vibr cin de c d p rte del cuerpo mientr s se tr sl d y gir . Por lo t nto el estudio del movimiento puede ser en gener l muy complejo, por est r zn se estudi c d movimiento en form independ iente. Cu ndo un cuerpo est en rot cin, c d punto tiene un movimiento distinto de otro punto del mismo cuerpo, unque como un todo se est moviendo de m ner simil r, por lo que y no se puede represent r por un p rtcul . Pero se puede represe nt r como un objeto extendido form do por un gr n nmero de p rtcul s, c d un con su propi velocid d y celer cin. Al tr t r l rot cin del cuerpo, el nlisis se s implific si se consider como un objeto rgido y se debe tener en cuent l s dime nsiones del cuerpo. Cuerpo rgido. Se define como un cuerpo ide l cuy s p rtes (p rtcul s que lo form n) tienen posiciones rel tiv s fij s entre s cu ndo se somete fuerz s extern s, es decir es no deform ble. Con est definicin se elimin l p osibilid d de que el objeto teng movimiento de vibr cin. Este modelo de cuerpo rg ido es muy til en much s situ ciones en l s cu les l deform cin del objeto es des preci ble. El movimiento gener l de un cuerpo rgido es un combin cin de movimient o de tr sl cin y de rot cin. P r h cer su descripcin es conveniente estudi r en fo rm sep r d esos dos movimientos. 6.1 TORQUE DE UNA FUERZA. Cu ndo se plic un fuerz en lgn punto de un cuerpo rgido, el cuerpo tiende re liz r un movimiento de rot cin en torno lgn eje. L propied d de l fuerz p r h cer gir r l cuerpo se mide con un m gnitud fsic que ll m mos torque o momento de l fuerz . Se prefiere us r el nombre torque y no momento, porque este ltimo se emple p r referirnos l momento line l, 171

C p. 6 Torque y equilibrio. l momento ngul r o l momento de inerci , que son tod s m gnitudes fsic s difer entes p r l s cu les se us el mismo trmino. An liz remos cu lit tiv mente el ef ecto de rot cin que un fuerz puede producir sobre un cuerpo rgido. Consideremos como cuerpo rgido un regl fij en un punto O ubic do en un extremo de l regl , como se muestr en l figur 6.1, sobre el cu l pued tener un rot cin, y des crib mos el efecto que lgun fuerz de l mism m gnitud ctu ndo en distintos puntos, produce sobre l regl fij en O. L fuerz F1 plic d en el punto pr oduce en torno O un rot cin en sentido ntihor rio, l fuerz F2 plic d en e l punto b produce un rot cin hor ri y con m yor r pidez de rot cin que en , l fuerz F3 plic d en b, pero en l direccin de l lne de ccin que p s por O, no produce rot cin (se puede decir que F3 empuj l regl sobre O, pero no l mueve ), F4 que ct inclin d en el punto b produce un rot cin hor ri , pero con menor r pidez de rot cin que l que produce F2; F5 y F6 plic d s perpendicul res l regl , s liendo y entr ndo en el pl no de l figur respectiv mente, no produce n rot cin. Por lo t nto existe un c ntid d que produce l rot cin del cuerpo rgido rel cion d con l fuerz , que es lo que definimos como el torque de l fuerz . Figur 6.1 Se define el torque de una fuerza F que ac a sobre algn pun o del cuerpo rgido, en una posicin r respec o de cualquier origen O, por el que puede pasar un eje sobre el cual se produce la ro acin del cuerpo rgido, al produc o vec orial en re la po sicin r y la fuerza aplicada F, por la siguien e expresin: 172

Cap. 6 Torque y equilibrio. r r =r F r (6.1) El orque es una magni ud vec orial, si es el ngulo entre r y F, su v lor numrico, por definicin del producto vectori l, es: = r ( Fsen ) (6.2) su direccin es siempre perpendicul r l pl no de los vectores r y F, cuyo di gr m vectori l se muestr en l figur 6.2, su sentido est d do por l regl del p roducto vectori l, l regl del sentido de v nce del tornillo o l regl de l m no derech . En l regl de l m no derech los cu tro dedos de l m no derech punt n lo l rgo de r y luego se gir n h ci F tr vs del ngulo , l direccin d el pulg r derecho estir do d l direccin del torque y en gener l de cu lquier pr oducto vectori l.

Por convencin se consider el torque positivo (neg tivo) si l rot cin que produci r l fuerz es en sentido ntihor rio (hor rio); esto se ilustr en l figur 6. 3. L unid d de medid del torque en el SI es el Nm (igu l que p r tr b jo, per o no se ll m joule). 173

Figur

6.2

C p. 6 Torque y equilibrio. Figur 6.3

El torque de un fuerz depende de l m gnitud y direccin de F y de su punto de plic cin respecto un origen O. Si l fuerz F p s por O, r = 0 y el torque es cero. Si = 0 o 180, es decir, F est sobre l lne de ccin de r, Fsen = 0 y el torqu es cero. F sen es l componente de F perpendicul r r, slo est componente re li z torque, y se le puede ll m r F. De l figur 6.3 t mbin se ve que r = r sen es l dist nci perpendicul r desde el eje de rot cin l lne de ccin de l fuerz , r se le ll m br zo de p l nc de F. Entonces, l m gnitud del torque se puede e scribir como: = r (Fsen ) = F (rsen ) = rF = r F Ejemplo 6.1: C lcul r el torque respecto l origen, producido por un fuerz F = (4i - 5j) N, que se plic un objeto en l posicin r = (2i + j) m. Solucin: Apl ic ndo l definicin de producto vectori l, se obtiene: i r r r = r F = x Fx j y Fy k Fz i j 1 k 0 z = 2 4 5 0 r = 0i 0 14k = 14kNm j 174

Cap. 6 Torque y equilibrio. Ejemplo 6.2: Calcular el orque ne o por los pun os A y por B en el sis ema de l a figura 6.4, donde F1 = 10 N, F2 = 5 N, F3 = 15 N, a = 50 cm, b = 1 m. Figura 6.4 Ejemplo 6.2. olucin: el orque ne o es la suma de los orques realizados por cada fuerza. Los pun os A y B se consideran ejes de ro acin en forma independien e, por supues o no simul neamen e, por lo an o los orque se calculan en forma separada en cada pun o. Para ro acin en orno al pun o A, considerando el sen ido de la ro acin que produce cada fuerza, lo que le da el signo al orque, se iene: A = F1 r1 sen45 + F2 r2 sen60 F3 r3 sen20 los valores de las dis ancias son: r1 =0, r2 = a = 0.5 m, r3 = b = 1 m. A = (10)(0) sen45 + (5)(0.5) sen60 (15)(1) sen20 = 3 Nm Para ro acin en orno al pun o B, considerando el sen ido de la ro acin: B =+ F1 r1 sen45 + F2 r2 sen60 F3 r3 sen20 ahora los valores de las dis ancias son: r1 = a = 0.5 m, r2 =0, r3 = b a = 0.5 m . B = (10)(0.5) sen45 + (5)(0) sen60 (15)(0.5) sen20 = 1 Nm 175

 

 

 

Cap. 6 Torque y equilibrio. 6.2 EQUILIBRIO DE UN CUERPO RGIDO. Por definicin una par cula puede ener solo movi mien o de raslacin. i la resul an e de las fuerzas que ac an sobre una par cula e s cero, la par cula es movindose con velocidad cons an e o es en reposo; en es e l imo caso se dice que es en equilibrio es ico. Pero el movimien o de un cuerpo rgi do en general es de raslacin y de ro acin. En es e caso, si la resul an e an o d e las fuerzas como de los orques que ac an sobre el cuerpo rgido es cero, es e no endr aceleracin lineal ni aceleracin angular, y si es en reposo, es ar en equilibr io es ico. La rama de la mecnica que es udia el equilibrio es ico de los cuerpos se llama es ica. Para que un cuerpo rgido es e en equilibrio es ico se deben cump lir dos requisi os simul neamen e, llamados condiciones de equilibrio. La primera condicin de equilibrio es la Primera Ley de New on, que garan iza el equilibrio de raslacin. La segunda condicin de equilibrio, corresponde al equilibrio de ro a cin, se enuncia de la siguien e forma: la suma vec orial de odos los orques ex e rnos que ac an sobre un cuerpo rgido alrededor de cualquier origen es cero. Esto se traduce en las siguientes dos ecuaciones, consideradas como las condiciones de equili rio de un cuerpo rgido: 1 condicin de equili rio: F = 0 F1 + F2 + L + Fn = 0 r r r r (6.3) 2 condicin de equilibrio: r r r r = 0 1 + 2 + L + n = 0 (6.4) Como es as ecuaciones vec oriales son equivalen es a seis ecuaciones escalares, resul a un sis ema final de ecuaciones con seis incgni as, por lo que limi aremos el anlisis a si uaciones donde odas las fuerzas que ac an sobre un cuerpo rgido, es n en el plano xy, donde ambin obviamen e se encuen ra r. Con es a res riccin se iene que ra ar slo con res ecuaciones escalares, dos de la primera condicin de equilibrio y una de la segunda, en onces el sis ema de ecuaciones vec orial (6. 3) y (6.4) se reduce a las siguien es ecuaciones escalares: 176

 

     

   

 

    

   

    

 

 

 

 

 

 

 

Cap. 6 Torque y equilibrio. Fx = 0, Fy = 0, O =0 Cuando se ra an problemas con cuerpos rgidos se debe considerar la fuerza de gra vedad o el peso del cuerpo, e incluir en los clculos el orque producido por su p eso. Para calcular el orque debido al peso, se puede considerar como si odo el peso es uviera concen rado en un solo pun o, llamado cen ro de gravedad. e han pregun ado alguna vez por qu no se cae la Torre de Pisa?, o por qu es imposible oc ar e los dedos de los pies sin caer e cuando es as de pie apoyado con los alone s con ra la pared? Por qu cuando llevas una carga pesada con una mano, ex iendes y levan as el o ro brazo? Para responder a es o debemos definir los concep os de cen ro de masa y de cen ro de gravedad y su aplicacin al equilibrio es ico. 6.2.1 Cen ro de gravedad. Debido a que un cuerpo es una dis ribucin con inua de masa, en cada una de sus pa r es ac a la fuerza de gravedad. El cen ro de gravedad es la posicin donde se pued e considerar ac uando la fuerza de gravedad ne a, es el pun o ubicado en la posi cin promedio donde se concen ra el peso o al del cuerpo. Para un obje o sim rico homogneo, el cen ro de gravedad se encuen ra en el cen ro geom rico, pero no para un obje o irregular. 6.2.2 Cen ro de masa. Es la posicin geom rica de un cuerpo rgido donde se puede considerar concen rada o da su masa, corresponde a la posicin promedio de odas las par culas de masa que f orman el cuerpo rgido. El cen ro de masa de cualquier obje o sim rico homogneo, se ubica sobre un eje se sime ra. Cuando se es udia el movimien o de un cuerpo rgido se puede considerar la fuerza ne a aplicada en el cen ro de masa y analizar el m ovimien o del cen ro de masa como si fuera una par cula. Cuando la fuerza es el p eso, en onces se considera aplicado en el cen ro de gravedad. Para casi odos lo s cuerpos cerca de la superficie erres re, el cen ro de masa es equivalen e al cen ro de gravedad, ya que aqu la gravedad es prc icamen e cons an e, es o es, si g es cons177

    

  

 

 

 

  

 

 

     

 

 

Cap. 6 Torque y equilibrio.

an e en oda la masa, el cen ro de gravedad coincide con el cen ro de masa. Exi s en m odos de clculo in egral para calcular es as dos posiciones, pero aqu no las de allaremos. Ahora se pueden responder las pregun as an eriores. Respec o a la Torre de Pisa, la respues a a la pregun a de porque no se cae, es porque su cen ro de gravedad es geom ricamen e den ro de su base, que se llama rea de sus en acin. Si la torre contina inclinndose hasta que su centro de gravedad caiga fuera del rea de sustentacin, entonces se derrum ar. Pero se le han puesto apoyos en su ase pa ra evitar que continu inclinndose. Las otras preguntas ahora las puedes responder tu. Para aplicar las condiciones de equili rio, es recomenda le seguir las sigui entes instrucciones, que corresponde a di ujar el DCL del cuerpo rgido: a) Aislar al cuerpo rgido del sistema con un lmite imaginario. ) Di ujar los vectores que representen las fuerzas en el punto de aplicacin donde las fuerzas efectivamente actan. c) Elegir un sistema de coordenadas conveniente para descomponer las fuerz as, donde di ujar la componente perpendicular a la posicin. d) Elegir un eje de r otacin O adecuado en el cuerpo rgido, donde se anulen los torques de (algunas) fue rzas desconocidas.

Ejemplo 6.3: Una arra uniforme de longitud L y peso P est articulada en A en una pared. Un alam re fijo en la pared a una distancia D so re la articulacin, sujet a a la arra por el extremo superior, como se muestra en la figura 6.5a. El alam re permanece horizontal cuando se cuelga un cuerpo de peso p en el extremo supe rior de la arra. Calcular la tensin del alam re y la fuerza de reaccin en la arti culacin de la arra. Solucin: se elige como eje de rotacin la articulacin de la arr a en la pared, en el punto A, se identifican las fuerzas que actan so re la arra , se di uja el DCL de la arra (figura 6.5 ) y se aplican las condiciones de equ ili rio. 178

  

  

 

  

  

     

Cap. 6 Torque y equili rio. Figura 6.5 Ejemplo 6.3 a) izquierda, 1 condicin de equili rio: r F = 0 Fx = 0 y eje x: FAx T = 0 eje y: FAy P - p = 0 F (1) (2) y =0 2 condicin de equilibrio:

A = 0 T + p + P = 0 +T cos L p sen L P sen (L/2) = 0 (3) De l geometr de l figur se obtienen sen y s en trminos de los v lores conocidos D y L: cos = D , sen = L L2 D 2 L que se reemplazan en (3), luego se despeja T: T= ( p + P / 2) D 179 L2 D 2

 

) derecha.

 

Cap. 6 Torque y equilibrio. Ahora se calculan FAx y FAy de las ecuaciones (1) y (2). De (1): De (2): FAx = T ( p + P / 2) = L2 D 2 D FAy = P + p Ejercicio: calcular el vec or fuerza en A, su magni ud y direccin. Ejemplo 6.4. En el sis ema de la figura 6.6a, una fuerza horizon al F, cuya lnea de accin pasa por el cen ro de un ambor de radio R y peso P, se aplica sobre el ambor, para hacerlo subir por un escaln de al o R/2. Hacer las suposiciones nece sarias para calcular el valor de la: a) fuerza F, b) fuerza del borde del escaln en A, c) direccin de la fuerza en A. Figura 6.6 Ejemplo 6.4 a) izquierda, b) derecha. olucin: e conocen slo el peso P y el radio del cilindro R. Hay que calcular la f uerza aplicada F y la fuerza del borde del escaln en A, FA. Las condiciones de equilibrio son: 1 condicin r F = 0 y 2 condicin A = 0 180

Cap. 6 Torque y equilibrio. e hace el DCL (figura 6.6b), se elige como eje de ro acin el pun o A, y al aplic ar las condiciones de equilibrio se ob iene: eje x: F FAx = 0 eje y: N P + FAy = 0 (1) (2) (3) A : Pd Nd F ( R / 2) = 0 donde d es la dis ancia perpendicular, o brazo de palanca, desde A has a las fue rzas peso P y normal N, y el brazo de palanca de F es R/2. De la geome ra de la f igura, se calcula d: R2 3 2 R 2 2 2 R = +d d = R = R 4 4 2 2 2 d= 3 R 2d = 3R 2 De (3) se obtiene el valor de la fuerza aplicada: (P N )d = FR (P N ) 2 F = 3 (P N ) De (1) : De (2): FAx = 3 (P N ) FAy = P N FR 3 R= 2 2 El vector fuerza es: r FA = FAx i + FAy = 3 (P N )i + (P N ) j j 181

 

Cap. 6 Torque y equilibrio. u magni ud:

r v 2 2 FA = 3(P N ) + (P N ) FA = 2(P N ) FAy FAx = Direccin de FA: an = (P N ) = 3 (P N ) 1 = 30 3

6.3 APLICACIONES DEL TORQUE AL CUERPO HUMANO. L tcnic p r c lcul r el v lor de l s fuerz s sobre cuerpos en equilibrio, pued e ser plic d l cuerpo hum no, donde existen fuerz s en msculos, huesos y rtic ul ciones, que permiten l s diferentes postur s y movimientos. El torque produci do por l fuerz de gr ved d jueg un p pel import nte en el equilibrio de un cu erpo. L fuerz de gr ved d produce un torque cero en torno l centro de gr ved d (c.g.) El c.g. de un person en posicin firme est sobre un lne vertic l que to c el suelo 3 cm del nte de los tobillos (figur 6.7 ). Si se inclin p r toc r l punt de los pies, su c.g. tiende moverse h ci del nte, ms ll del re de cont cto, perdindose el equilibrio. P r evit r esto, sus piern s y n lg s se mue ven h ci trs, con lo cu l el cuerpo vuelve est r en equilibrio (figur 6.7b). Los centros de gr ved d de l m yor de l s p rtes del cuerpo no estn encim de l s rticul ciones de poyo y h cen f lt fuerz s muscul res p r m ntener el equ ilibrio. Es s que p r m ntener el equilibrio y evit r que el cuerpo vuelque h c i del nte teniendo como eje l rticul cin del tobillo, se necesit un fuerz plic d por el msculo del tendn de Aquiles que v unido l tobillo (figur 6.7c). El problem de m ntener el equilibrio cu ndo c min mos es n m yor. Al lev nt r un pie del suelo, el c.g. del cuerpo tiene que despl z rse por encim del pie p oy do. Esto exige que todo el cuerpo se muev l ter lmente. Es s que l c min r el cuerpo se mueve de un l do otro p r m ntener el c.g. sobre su re de poyo, en continuo movimiento. Un buen est bilid d se obtiene teniendo el c.g. de un objeto en un posicin deb jo de su re de sustent cin. P r un cu drpedo, el re de poyo es el re que h y entre l s p t s, lo cu l h ce que el nim l teng gr n es t bilid d. Si el c.g. est re lmente deb jo del re de poyo se logr un gr n est bilid d. A lo l rgo de l evolucin, los nim les h n des rroll do postur s c d v ez ms inest bles. L ines182

Not r que no se conoce N, se puede suponer que N = 0 cu ndo F es l fuerz mnim p r h cer subir l t mbor.

C p. 6 Torque y equilibrio.

Figur 6.7 ) b) c)

L column vertebr l hum n const de 24 vrtebr s sep r d s por discos impregn do s de un fluido. Cu ndo un person se g ch p r recoger unque se un objeto l ivi no, se produce un gr n fuerz sobre el disco s cro lumb r que sep r l ltim vrtebr del s cro, el hueso que sostiene l column vertebr l. Si este disco se debilit puede deform rse o romperse y ejercer presin sobre los nervios prximos p roduciendo gr ndes dolores. P r comprender por qu est fuerz es t n gr nde pode mos us r un modelo que tr t l column como un b rr con pivote que correspond e l s cro (figur 6.8 ). Los diversos msculos de l esp ld los represent remos como un r solo msculo que produce un fuerz T . Si l esp ld est horizont l, el n gur lo que form respecto l column es proxim d mente 12. P Represent el pes o del torso, c bez y br zos, que corresponde proxim d mente l 65% del peso to t l del r cuerpo. Obsrvese que como el ngulo es pequeo, l lne de ccin de T p s c rc del pivote (s cro), por lo cu l su dist nci perr pendicul r es peque . El pe so P ct en ngulo recto respecto l column y su dist nci perpendicul r es muc ho m yor. Por lo t nto, p r que se equilir r bren los torques, l fuerz muscul r T debe ser mucho m yor que el peso P . 183

t bilid d permite los nim les moverse ms rpid euromuscul r complejo p r m ntener el equilibrio. mente inest ble que un nio le cuest m s de un uscul r suficiente p r m ntenerse en pie sin yud

mente, pero requiere un control n L posicin hum n es t n mecnic o des rroll r el control neurom .

C p. 6 Torque y equilibrio. r Como T es gr nde, t mbin lo es su componente horizont l, por lo t nto l r fuer z R debid l s cro debe tener un componente de igu l v lor y sentido r opuest o. L fuerz debid l s cro t mbin debe ser m yor que el peso P . Ejemplo 6.5. Re licemos los clculos p r un person que pes 700 N (m s de 70kg ). El v lor de P es 65% de 700 = 455N. Se supone que P y T ct n un dist nci del s cro de y 2/3 del l rgo l de l column (figur 6.8 ). P r determin r el v lor de T y R se plic n l s condiciones de equilibrio. Figur 6.8 ). b)

2 condicin de equilibrio, consider ndo el eje O en el hueso s cro: L 2 3P O = 0 T + P = 0 Tsen12 3 L P 2 = 0 T = 4 sen12 T= 3 455 = 1641N 4 1 condicin de equilibrio: Fx Fy = 0 : Rx Tx = 0 Rx = Tx Rx = Tcos12 =1641 cos12 = 1605N = 0 : Ry + Ty P = 0 Ry = P - Ty = 455 - 1641 sen12 = 114N 2 2 Luego: R = Rx + R y = 1605 2 + 114 2 = 1610 N 184

Cap. 6 Torque y equilibrio. Tales fuerzas en los msculos y en el disco son potencialmente peligrosas, pues el valor de dichas fuerzas son grandes an sin levantar un cuerpo. Si se flexionan l as rodillas manteniendo la espalda vertical, los centros de gravedad de todos lo s pesos estn aproximadamente en la vertical del sacro, por lo tanto sus torques r especto al sacro son pequeos y los msculos no deben realizar gran fuerza (figura 6 .8b). La fuerza sobre el disco respectivo es entonces aproximadamente, igual al peso que sostiene. El diagrama de la figura 6.9 ilustra los valores de presin (fu erza) sobre el tercer disco lumbar, en atmsferas, si la persona est de pie (A), de pie y sostiene 20kg (B), levantando correctamente un bulto de 20kg (C), levanta ndo incorrectamente un bulto de 20kg (D). Notar como aumenta la fuerza lumbar en l os dis in os casos. Figura 6.9 185

 

Cap. 6 Torque y equilibrio. PROBLEMA . 6.1 a) Es imar las longi udes y masas de los pun eros del reloj del Campanil. b) Cal cular el orque en orno al eje de ro acin de los pun eros debido a su peso, cuan do la hora marca las: 14:00, 16:45, 18:00, o ra a su gus o. Hacer odas las supo siciones necesarias para es imar el orque que deben ejercer las races de un pino radia a D. Don, para evi ar que el pino se vuelque, cuando en un emporal de in vierno se inclina por efec o de la fuerza ejercida por el vien o. Y si la plan a es un rosal? Una fuerza F = (2i + 3j) N se aplica a un obje o que es ar iculado alrededor de un eje fijo alineado con el eje z. i la fuerza se aplica en la pos icin r = (4i + 5j) m, calcular: a) el vec or orque ne o en orno a z, b) la magn i ud del orque ne o y c) su direccin. La figura 6.10 mues ra las fuerzas F1=40 N , F2=30 N, F3=50 N, F4=60 N aplicadas a un cuerpo rgido que puede girar en orno de un eje que pasa por O. Calcular el orque resul an e. R: 10.8 Nm. Calcular e l orque ne o sobre la rueda producido por las fuerzas F1=8 N, F2=10 N, F3=15 N, que se indican en la figura 6.11, alrededor de un eje que pase por su cen ro, s i a = 10 cm, b = 20 cm y = 30. 6.2 6.3 6.4 6.5 Figur 6.10 Problem 6.4 Figur 6.11 Problem 6.5 6.6 Dos fuerz s F1 y F2 ct n lo l rgo de los l dos de un tringulo equiltero de l do , como se muestr en l figur 6.12. Encuentre un tercer fuerz F3 que plic d en el vrtice lo l rgo del l do produzc un torque neto en torno O igu l cero. R: F3 = F1 + F2 (m gnitudes). 186

 

 

 

C p. 6 Torque y equilibrio. 6.7

Figur 6.12 Problem 6.6 Figur 6.13 Problem s 6.7 y 6.8 6.8 P r el sistem de l figur 6.13, c lcul r el v lor de x t l que l fuerz norm l en O se l mit d de l fuerz norm l en Q. ) Desprecie el peso de l vig . b) Considere el peso P de l vig . Un t bln uniforme de 6m de longitud y 30kg de m s , desc ns horizont lmente sobre un nd mio. Si 1.5m del t bln sobres le por un extremo del nd mio. Cunto puede c min r un pintor de broch gord de 70kg por l p rte sobres liente ntes de que el t bln se vuelque? R: 0.64 m. 6.9 6.10 Un t bln uniforme de 5 m de l rgo y 150 kg est rticul do en A. En B est sos tenido por un cuerd ubic d 1.5 m del extremo inferior del t bln, form ndo un ngulo de 90 con el t bln, como se ve en l figur 6.14. C lcul r l tensin de l cu erd y l fuerz de l rticul cin en A. R: 643 N, -514 + 1114j N. 6.11 El t bln un iforme de l figur 6.15, de 5 m de l rgo y peso P est rticul do en A e inclin d o gr dos con l horizont l. En el extremo opuesto est sostenido por un cuerd qu e form un ngulo de 90 con el 187

Un vig uniforme de peso P y longitud L, t dos pesos, P1 sobre O y P2 l derech 13. C lcul r el v lor de x p r el cu l l poyo Q de t l m ner que l fuerz en O

que se poy en los puntos O y Q sopor de Q, como se muestr en l figur 6. vig qued r equilibr d en el punto de se cero. R: [(P1 + P)D + LP1]/P2.

C p. 6 Torque y equilibrio.

Figur 6.14 Problem 6.10 Figur 6.15 Problem 6.11 6.12 Un esc ler homogne de m s M desc ns contr un p red vertic l sin fricc in, en un ngulo de con l vertic l. El extremo inferior se poy sobre un piso hor izont l con un coeficiente de friccin . Un pintor de broch gord de m s 2M inten t subir l esc ler . C lcul r l fr ccin de l longitud L de l esc ler subir el pintor ntes de que l esc ler empiece resb l r. R: (1.5 ctg 0.25)L. 6.13 Un t bln uniforme de 5m de longitud y 50N de peso, pern do en A es sostenido por un cuerd en su extremo superior, como se muestr en l figur 6.16. Un c rg de 100 N cuelg del t bln en un punto un dist nci x de A. Si l resistenci de r uptur de l cuerd es 50 N, c lcul r el v lor de x. Considere = 30 y = 60 . R: 1.2 9 m. 6.14 Un poste uniforme de 1200 N se sostiene por un ca le, como en la figur a 6.17. El poste se sujeta con un perno en A la parte inferior y en la parte sup erior se cuelga un cuerpo de 2000 N. Encuentre la tensin en el ca le de soporte y las componentes de la fuerza de reaccin en el perno en A. R: 1465 N, (1328 + 2581 j) N. 6.15 Una fuerza F, cuya lnea de accin pasa por el orde superior de un tam o r de radio R y peso P, se aplica so re el tam or, para hacerlo su ir por un esca ln de alto R (figura 6.18). Calcular: a) la fuerza F, ) la fuerza del vrtice del e scaln en A, c) la direccin de la fuerza en A. R: a) 3 3 P , ) 10 9 P , c) tan = 3 . ( ) ( ) 188

t bln, sosteniendo un peso P. C lcul r: A. R: ) 0.6 P, b) (0.47 + 1.14j)P.

) l

tensin de l cuerd , b) l fuerz en

C p. 6 Torque y equilibrio. 6.16 Un cilindro de m s M y r dio r desc ns sobre un pl no inclin do sujet do por un cuerd t ngente l cilindro y p r lel l superficie del pl no. El pl no est inclin do en un ngulo con l horizont l, como se muestr en l figur 6.1 9. C lcul r: ) el v lor mnimo del coeficiente de friccin esttico, en trminos de , p r que el cilindro no resb le h ci b jo del pl no inclin do, b) l tensin en l cuerd en trminos de M, g y . R: ) 0.5t n, b) 0.5Mg sen. Figur 6.16 Problem 6.13 Figur 6.17 Problem 6.14 Figur 6.18 Problem 6.15 Figur 6.19 Problem 6.16 6.17 El ntebr zo de l figur 6.20, est con respecto l br zo 90 y sostiene en l m no un cuerpo de peso 70 N. Despreci ndo l peso del ntebr zo: Cul es el torq ue producido por el peso de 70N lrededor de l rticul cin del codo (punto O)? Cul es el torque lrededor de O producido 189

C p. 6 Torque y equilibrio. por l fuerz Fm ejercid sobre el ntebr zo por el bceps? Cul es l r m gnitud de Fm ? 6.18 Repetir el problem nterior suponiendo que el ntebr zo y l m no jun tos pes n 35N y que su centro de gr ved d est 15 cm de O. 6.19 Con el ntebr zo en posicin horizont l, t l como p rece en l figur 6.21, l m no ejerce un fu erz de 90N sobre l b l nz . H ll r l s m gnitudes de l s fuerz s Fm y Fc que e jercen sobre el ntebr zo el trceps y el hmero, (desprecie el peso del ntebr zo). 6.20 Repetir el problem nterior suponiendo que el ntebr zo y l m no juntos pes n 25N y que su centro de gr ved d est 18 cm de 0. r Figur 6.20 Problem s 6.17 y 6.18 Figur 6.21 Problem s 6.19 y 6.20 6.21 Un person puede ejercer un fuerz mxim de 400 N sobre el p r to que se muestr en l figur 6.22. Si el p r to est 28 cm del codo, y el bceps est unido 5 cm del codo, Cules son l s m gnitudes de l s fuerz s ejercid s por: el bceps, el hmero. 6.22 L figur 6.23 nos muestr un tlet prep r do p r h cer un tiburn . Pes 750N y su centro de gr ved d est loc liz do por encim de un punto P que h y en el suelo. Este punto est 0,9 m de l punt de sus pies y 0,6m de sus ho mbros, Cules son l s fuerz s ejercid s por el suelo sobre l s m nos y pies del tl et ? 6.23 En el ejercicio que p rece en l figur 6.24 el torque lrededor de l rodill ejercido por l pes sujet l tobillo, v r con l elev cin de l 190

C p. 6 Torque y equilibrio.

Figur 6.22 Problem 6.21 Figur 6.23 Problem 6.22. 6.24 Cu ndo un person est g ch d , el msculo de l esp ld unido un punto d os tercios del s cro (eje) en un ngulo de 12, m ntiene l espin dors l, de l rgo l, en posicin horizont l (figur 6.8). Si l p rte superior del cuerpo pes 450 N , c lcul r l tensin T del msculo de l esp ld y l fuerz R de l espin en el s cro, cu ndo l person lev nt con los br zos un peso de 200 N. R: 3066 N. Figur 6.24 Problem 6.23 Figur 6.25 Problem 6.25 6.25 Considere el modelo mecnico de l figur 6.25 en el cu l l b rr AB represe nt l column vertebr l. El c ble CD represent l grupo de msculos de l esp ld que m ntienen l person en l posicin incli191

piern . C lcul r el torque p r

l s cu tro posiciones que p recen en l figur .

C p. 6 Torque y equilibrio. n d . El punto A represent el punto fin l de poyo de l column . Si l person tiene un m s de 60 Kg, y lev nt un cuerpo de m s 25 Kg, r determine l fuer z de tensin T que ejercen los msculos y l fuerz de r compresin Fc que se ejerce en el punto de unin de l column . Supong que 1 es el peso del tronco, cuyo pun to de plic cin es el punto G y v le 260 N y 2 es el peso combin do de c bez , b r zo y objeto, ct en el punto B y v le 590 N. 6.26 El msculo deltoides lev nt e l br zo h st l posicin horizont l, figur 6.26. Si el peso del br zo es 35N, c lcul r: el v lor de l tensin T ejercid por el msculo, el v lor de l s componente s de R de l fuerz ejercid por l rticul cin del hombro. Figur 6.26 Problem 6.26. 192

C p.7 Momento line l y choques CAPITULO 7. MOMENTO LINEAL Y CHOQUES. Cmo puede un k r tec p rtir un montn de l dr illos?, por qu un porr zo es m s doloroso sobre el cemento que sobre el p sto?, por qu cu ndo se s lt desde un lug r lto es conveniente flexion r l s rodill s l lleg r l suelo?. P r entender y responder est s pregunt s h y que record r el concepto de inerci . Todos s bemos que es ms fcil detener un pelot peque que un gr nde que se muev con l mism velocid d por qu?. Est s cciones estn rel cion d s con l inerci (m s ) de los objetos en movimiento, y est ide de inerci en movimiento est incluid en el concepto de momento, trmino que se refiere los objetos que se mueven. 7.1 MOMENTO LINEAL. El concepto de momento line l se us p r denot r l inerci en movimiento. El momento line l p de un p rtcul de m s m que se mueve con ve locid d v, se define como el producto de l m s de un objeto por su velocid d: r r p = mv (7.1) P r un p rtcul en movimiento en el esp cio, l s componentes del momento line l en c d direccin x, y y z son: p x = mv x , p y = mv y , p z = mv z El momento line l (much s veces mencion do solo como momento) es un m gnitud fsi c vectori l porque l velocid d es un vector, su direccin es lo l rgo de v, su unid d de medid en el SI es kg m/s. De est definicin se observ que el momento line l de un cuerpo en movimiento puede ser gr nde si su m s es gr nde, como e n el c so de l pelot ms gr nde mencion d en el primer prr fo, si su velocid d e s gr nde, o mb s lo son. Si un cuerpo est en 193

C p.7 Momento line l y choques reposo, su momento line l es cero. Puesto que el movimiento es producido por fue rz s, si l m s es const nte, se puede rel cion r el momento line l con l fuer z F que ct sobre l p rtcul us ndo l segund Ley de Ne ton: r r r r dv d ( mv ) = F = ma = m dt dt r r dp F= dt Esta ltima ecuacin dice que la fuerza neta sobre una partcula es igual a la rapidez de cambio del momento lineal de la partcula. Para el caso particular en que la f uerza neta es cero, esto es para una partcula en equilibrio de traslacin, el momen to lineal resultante de la partcula debe ser constante, ya que: r r dp r F =0 = 0 p = cte. dt 7.2 IMPULSO. Si cambia el momento lineal de una partcula, su velocidad vara, y si la masa es co nstante, como casi siempre es el caso, entonces hay aceleracin, que necesariament e debe ser producida por una fuerza. Mientras mayor sea la fuerza, mayor el camb io de velocidad, y por lo tanto mayor el cambio de momento lineal. Pero hay otro factor importante a considerar: el tiempo durante el cual se ejerce la fuerza. El cambio de momento lineal es mayor si se aplica la misma fuerza durante un int ervalo de tiempo largo que durante un intervalo de tiempo corto. Estas afirmacio nes se pueden demostrar escribiendo la ecuacin de momento lineal de la siguiente forma: r r dp = Fdt 194

Cap.7 Momento lineal y choques Esta ecuacin se puede integrar para obtener la variacin de momento p de la partcula. Si el momento cambia desde un valor inicial pi en el instante inicial ti a un v alor final pf en el instante final tf, integrando la ecuacin anterior, se obtiene : t r r r r p f pi = p = F dt f ti La cantidad integral de la fuerza por el intervalo de tiempo, se define como el impulso I de la fuerza F en el intervalo de tiempo dt, es decir el impulso I es un vector definido por la expresin: r t r r I = t Fdt = p f i (7.2) Cuanto mayor sea el impulso, mayor ser el cambio de momento de la partcula. Esta e xpresin se llama el teorema del impulso y del momento, que se expresa como: el im pulso de la fuerza neta es igual al cambio de momento lineal de la partcula. Este teorema es equivalente a la segunda Ley de Newton. El impulso es una magnitud v ectorial cuyo valor numrico, por definicin de integral, es igual al rea bajo la cur va F vs t, como se ilustra en la figura 1, tiene la misma unidad de medida que e l momento lineal. En general la fuerza puede variar en forma complicada con el t iempo (figura 7.1), por lo que es conveniente definir una fuerza promedio en el tiempo, Fm, que se puede considerar como una fuerza constante que dar el mismo im pulso a la partcula que la fuerza F actuando durante el intervalo de tiempo t. De nuestros conocimientos de estadstica, sabemos que el valor medio de alguna variab le, se define como: r 1 t r Fm = Fdt t t f i Despejando la integral y reemplazando en la definicin del impulso se puede escrib ir: 195

Cap.7 Momento lineal y choques r r r I = Fm t = p (7.3) Figura 7.1 Esta expresin se llama la aproximacin del impulso, supone que una de las fuerzas q ue actan sobre la partcula lo hace en un tiempo muy corto y es de magnitud mucho m ayor que cualquier otra fuerza presente. Esta aproximacin es muy til cuando se tra baja con choques (evento en Fsica que luego definiremos), donde las fuerzas son m uy intensas y de muy corta duracin, en este caso se les da el nombre de fuerzas i mpulsivas o de impacto. Ahora se pueden responder las preguntas formuladas al co mienzo de esta captulo. En la del salto, al flexionar las rodillas se aumenta el tiempo durante el cual varia el momento, por lo que se reducen las fuerzas que s e ejercen sobre los huesos respecto al valor que tendran si cayeras con las piern as extendidas, lo que evita posibles lesiones. Las respuestas a las otras pregun tas se deja como reflexin para el alumno. Ejemplo 7.1. En un saque, el Chino Ros golpea su pelota (la de tenis) de 50 gr co n la raqueta, proporcionndole una fuerza impulsiva. Suponiendo que la pelota sale de la raqueta en un ngulo de 2,5 y recorre 10 m para llegar a la misma altura en el otro sector de la cancha, calcular: a) el impulso, b) la duracin del golpe si la deformacin de la pelota por el golpe fue de 1 cm, c) la fuerza media sobre la pelota. 196

Cap.7 Momento lineal y choques

Solucin. a) Clculo del impulso, por su definicin: I = p = p f pi = m(v f vi ) donde vi = 0 es la rapidez de la pelo a jus o an es del golpe y vf es la rapidez con l a que sale de la raque a despus del golpe, que no se conoce, pero que se puede ca lcular con los ecuaciones de cinem ica, sabiendo que la pelo a recorre x = 10 m y sale con una inclinacin = 2.5. Us ndo l expresin de l dist nci horizont l mxim p r un proyectil, se tiene: 2 vo xg 10 10 2 x= sen2 vo = = vo = 33.9 m/s sen 2 sen5 g v f = 33.9 m/s reemplazando en el impulso, se obtiene: I = mv f m 0 = mv f = (0.05 gr )(33.9 m/ s) I = 1.7 kg m/s b) i la pelo a se deform 1cm duran e el golpe, considerando qu e cuando comienza la deformacin la vi = 0, suponiendo que duran e la deformacin la a = c e, la duracin del golpe sera: v2 f = vi2 (33.9 )2 = 57460.5m/s 2 + 2ax a = = 2x 2(0.01) v2 f vf a = 33.9m/s 57460.5 m/s 2 v f = vi + at t = t = 5.9 10 4 s 197

 

Cap.7 Momen o lineal y choques c) El clculo de la fuerza media se puede hacer con la ecuacin: I = Fm t Fm = I t Fm = 1.7 kgm / s = 2881.5 N 5.9 10 4 s

olucin: a) en la figura 7.2 se mues ra el esquema de la si uacin. El momen o line al inicial y final es: momen o inicial: momen o final: r pi = mvi j r p f = mv f j Los valores de las velocidades inicial y final se pueden calcular usando el prin cipio de conservacin de la energa. Inicial: 0 + mghi = mvi2 +0 vi = 2 ghi Figura 7.2 Ejemplo 7.2 Final: mvf2 +0 = 0 + mghf v f = 2 gh f = 2 g (3 4 )hi = Por lo tanto, el momento inicial y final es: (3 2)ghi r r j pi = m 2 ghi , p f = m (3 2) ghi j Reemplazando los valores numricos, se iene: 198

Ejemplo 7.2. Una pelo a de a ver icalmen e despus de men o de la pelo a an es y fue de 0.01 s, calcular la

100 g que se deja caer desde una al ura h = 2m, rebo golpear el suelo has a h (figura 7.2). a) Calcular el mo despus de golpear el suelo, b) si la duracin del golpe fuerza media ejercida por el piso sobre la pelo a.

Cap.7 Momen o lineal y choques

r r r r r r p p f p i r = I = Fm t = p Fm = t t r 0.54 (0.63 ) j j Fm = = 11 7.3 CON ERVACIN DEL MOMENTO LINEAL. La segunda ley de New on afirma que para acelerar un obje o hay que aplicarle un a fuerza. Ahora vamos a decir lo mismo, pero con o ro lenguaje: para cambiar el momen o de un obje o hay que aplicarle un impulso, impulso que es producido por una fuerza. En ambos casos hay un agen e ex erno que ejerce la fuerza o el impul so, las fuerzas in ernas no se consideran. Cuando la fuerza ne a es cero, en onc es el impulso ne o es cero, y por lo an o no hay cambio del momen o lineal o a l. En onces se puede afirmar que si sobre un sis ema no se ejerce fuerza ne a, e l momen o o al del sis ema no puede cambiar. Para probar an osada afirmacin, co nsideremos un sis ema mecnico formado solo por dos par culas que in erac an en re s, pero que es n aisladas de los alrededores, y que ejercen fuerzas en re ellas (es as fuerzas pueden ser gravi acionales, els icas, elec romagn icas, nucleares, e c .), sin consideran o ras fuerzas ex ernas al sis ema. i en un cier o ins an e el momen o de la par cula 1 es p1 y el momen o de la par cula 2 es p2, y si F12 e s la fuerza sobre la par cula 1 producida por la par cula 2 y F21 es la fuerza sob re la par cula 2 producida por la par cula 1, como se mues ra en la figura 7.3, en onces se puede aplicar la segunda Ley de New on a cada par cula: r r dp1 F12 = d r r dp 2 F21 = d y 199

   

 

 

 

 

 

  

 

pi =

0.63 kgm/s, pf = 0.54 kgm/s b) Usando la aproximacin del impulso:

  

Cap.7 Momen o lineal y choques Figura 7.3 Por la ercera Ley de New on, F12 y F21 son un par de accin y reaccin, en onces: r r F12 + F21 = 0 r r dp1 dp 2 d r r + = ( p1 + p 2 ) = 0 d d d r r p1 + p2 = c e. (7.4) e concluye que el momen o lineal o al es cons an e. Cuando una can idad fsica n o cambia, decimos que se conserva, por lo an o el momen o o al se conserva. No hay caso alguno en que el momen o de un sis ema pueda cambiar si no se aplica u na fuerza ex erna. Es a es una de las leyes fundamen ales de la mecnica, conocida como ley de conservacin del momen o lineal. Como es una ecuacin vec orial, equiva le a res ecuaciones escalares, una para cada componen e x, y y z: p1x + p 2 x = c e p1 y + p 2 y = c e p1z + p 2 z = c e i pi1 y pi2 son el momen o en el ins an e inicial de las par culas 1 y 2 y pf1 y pf2 son el momen o en el ins an e final, en onces la conservacin del momen o se escribe como: 200

  

  

 

  

 

 

 

 

Cap.7 Momen o lineal y choques r r r r pi1 + pi 2 = p f 1 + p f 2 r r r r m1vi1 + m 2 vi 2 = m1v f 1 + m2 v f 2 La conservacin de la energa mecnica o al se cumple slo cuando las fuerzas sobre el sis ema aislado son conserva ivas. El momen o lineal para un sis ema de par culas se conserva sin impor ar la na uraleza de las fuerzas in ernas que ac an sobre e l sis ema aislado, por lo que el principio de conservacin del momen o lineal es ms general y comple o que el de la conservacin de la energa, es una de las leyes ms i mpor an es de la mecnica, deducido a par ir de las Leyes de New on. Como el sis e ma es aislado, las fuerzas in ernas que ac an son de accin y reaccin, en es e caso el momen o se conserva, por lo que el principio de conservacin del momen o lineal es un enunciado equivalen e a la ercera Ley de New on. No ar como in ervienen las res Leyes de New on en es e anlisis. Aunque la an erior deduccin del principi o de conservacin del momen o lineal fue formulada en es e anlisis para dos par cula s que in erac an en re s, se puede demos rar que es vlida para un sis ema de n par c ulas y para una dis ribucin con inua de masa, aplicada al movimien o del cen ro d e masa del sis ema de par culas o de la dis ribucin de masa. 7.4 CHOQUE . La ley de conservacin del momen o lineal se puede aplicar muy claramen e en lo qu e en Fsica se conoce como choque o colisin. e usa el rmino choque para represen a r, en escala macroscpica, un even o en el que dos par culas in erac an y permanecen jun as duran e un in ervalo de iempo muy pequeo, produciendo fuerzas impulsivas en re s. e supone que la fuerza impulsiva es mucho ms grande que cualquier o ra fuerza ex erna. En escala a mica iene poco sen ido hablar del con ac o fsico; cua ndo las par culas se aproximan en re si, se repelen con fuerzas elec ros icas muy in ensas sin que lleguen a ener con ac o fsico. Cuando dos o mas obje os chocan sin que ac en fuerzas ex ernas, el momen o lineal o al del sis ema se conserva. Pero la 201

 

     

 

 

 

 

 

 

  

 

   

  

Cap.7 Momen o lineal y choques energa cin ica en general no se conserva, ya que par e de es a se ransforma en en erga rmica y en energa po encial els ica in erna de los cuerpos cuando se deforman duran e el choque. De acuerdo a lo expues o, exis en diferen es procesos duran e los choques, por lo que es os se pueden clasificar en res ipos: a) Cuando dos o mas obje os chocan sin deformarse y sin producir calor, se llama choque els ic o. En es e caso se conserva an o el momen o lineal como la energa cin ica del sis ema. b) Cuando los obje os que chocan se deforman y producen calor duran e el c hoque, se llama choque inels ico. En es e caso se conserva el momen o lineal, per o no la energa cin ica del sis ema. c) Un choque se dice perfec amen e inels ico cu ando los obje os se deforman, producen calor y permanecen unidos despus del choqu e, por lo que sus velocidades finales son las mismas, y an es vlida la conservacin del momen o lineal. 7.4.1 Ejemplos de choque en una dimensin. La ley de conservacin del momen o lineal es il de aplicar cuando duran e un choqu e se producen fuerzas impulsivas. e supone que las fuerzas impulsivas son mucho mayor que cualquier o ra fuerza presen e y como es as son fuerzas in ernas, no cambian el momen o lineal o al del sis ema. Por lo an o, el momen o lineal o al del sis ema jus o an es del choque es igual al momen o lineal o al del sis e ma jus o despus del choque y el momen o o al se conserva. Pero en general la ene rga cin ica no se conserva. Ejemplo 7.3: dos par culas de masas m1 y m2 que se mueven en la misma lnea de accin , con velocidades vi1 y vi2, chocan en forma comple amen e inels ica. Despus del c hoque ambas par culas se mueven jun as; de erminar la velocidad final vf del sis ema. olucin: upongamos que inicialmen e las par culas se mueven en el mismo sen ido, y si en es e caso lo consideramos hacia la derecha como se mues ra en 202

  

 

  

  

 

 

 

 

  

 

 

 

 

 

 

   

Cap.7 Momen o lineal y choques la figura 7.4, la velocidad inicial de m1 debe ser mayor que la de m2, dando com o resul ado una velocidad final del conjun o hacia la derecha. Figura 7.4 Choque comple amen e inels ico en una dimensin. En es e choque comple amen e inels ico, el momen o lineal del sis ema se conserva , y como el movimien o es en una dimensin (por ejemplo, la direccin del eje x), en onces de la figura 7.4, se ob iene: pan es del choque = pdespus del choque pi = p f pi1 + pi 2 = p f 1 + p f 2 m1vi1 + m2 vi 2 = (m1 + m2 )v f vf = m1vi1 + m2 vi 2 m1 + m2 Existen otras opciones respecto a la direccin que pueden tener las velocidades in iciales de las partculas para que se produzca el choque: que las dos se muevan en sentidos contrarios, en cuyo caso, independientemente del valor de las velocida des, se producir el choque, ya que se mueven sobre la misma lnea de accin; o que am bas partculas se muevan hacia la izquierda, en ese caso, la velocidad inicial de la partcula que persigue (m2 en la figura 7.4) a la o ra, debe ser mayor para que l a alcance y se produzca el choque, dando como resul ado una velocidad final del conjun o hacia la izquierda. Es as si uaciones las puede resolver el alumno. 203

 

Cap.7 Momen o lineal y choques Ejemplo 7.4: dos par culas de masas m1 y m2 que inicialmen e se mueven en lnea rec a, en sen idos con rarios, con velocidades vi1 y vi2, chocan fron almen e en fo rma els ica. Calcular la velocidad final vf de cada una, despus del choque. olucin : Como no se conoce ni el valor numrico de las masas ni de las velocidades inicia les, no se puede saber a priori el sen ido de las velocidades finales de las par culas, as que supongamos que despus del choque se mueven en sen idos opues os. Com o el choque es els ico, se conserva an o el momen o como la energa cin ica, aplica ndo es os principios, y considerando que el choque es en una direccin, se ob iene : Figura 7.5 Choque els ico en una dimensin. Conservacin del momen o lineal: pan es del choque = pdespus del choque pi1 + pi 2 = p f 1 + p f 2 m1vi1 m 2 vi 2 = m1v f 1 + m 2 v f 2 Conservacin de la energa cin ica: EC an es del choque = EC despus del choque 1 1 1 1 m1vi2 + m2 vi22 = m1v 2 1 + m2 v 2 2 f f 1 2 2 2 2 Para resolver el sis ema de dos ecuaciones, para las dos incgni as vf1 y vf2, en la ecuacin de la energa cin ica, se puede dividir por , reagrupar los rminos de m1 y m2 en cada miembro d e la ecuacin y escribirla como: 204

 

 

 

Cap.7 Momen o lineal y choques m1 vi2 v 2 1 = m 2 v 2 2 vi22 1 f f ( ) ( ) m1 vi1 + v f 1 vi1 v f 1 = m2 v f 2 + vi 2 v f 2 vi 2 ( )( ) ( )( ) Ahora se pueden separar los rminos en m1 y m2 de la ecuacin del momen o y escribi rla de la siguien e forma:

v f1 = m 2 m1 2m 2 vi1 + vi 2 m1 + m2 m1 + m2 m m2 2m1 vi1 + 1 vi 2 m1 + m 2 m1 + m 2 vf2 = Los resul ados an eriores no deben considerarse como generales, ya que fueron de ducidas para es e caso par icular, con los sen idos de las velocidades iniciales dados, por lo an o no se pueden aplicar como formulas para resolver cualquier problema. Como en el ejemplo 7.3, exis en o ras opciones respec o a la direccin q ue pueden ener las velocidades iniciales de las par culas para que se produzca e l choque els ico fron al, anlisis que se deja de area para el alumno. 7.5 CHOQUE EN DO DIMEN IONE . i una par cula de masa m1 que se mueve con una de erminada velocidad inicial vi1 , choca de cos ado con o ra de masa m2 inicialmen e en reposo (no iene porque e s ar en reposo, pero en es e caso, considermosla en ese es ado), el movimien o fi nal ser bidimensional, por lo que se considera un choque en 205

 

m1 (vi1 + v f 1 ) = m 2 Combinando es as dos l jan como ejercicio para final vf1 y vf2 de cada

(v f 2 + vi 2 ) imas ecuaciones (desarrollos algebraicos in ermedios se de el alumno), se ob ienen las expresiones para la rapidez par cula:

  

 

 

Cap.7 Momen o lineal y choques dos dimensiones. Despus del choque, como se mues ra en la figura 7.6, m1 se mueve en un ngulo sobre el eje x y m2 en un ngulo de ajo del eje x. Figura 7.6 Choque en dos dimensiones. Por la ley de conservacin del momento, desarrollada en sus componentes en cada di reccin x e y: Eje x: Eje y: pix = p fx m1vi1 + 0 = m1v f 1 cos + m2 v f 2 cos piy = p fy 0 = m1v f 1 sen m2 f 2 sen Si adems el choque es elstico, por la conservacin de la energa se tiene: 1 1 1 m1vi2 + 0 = m1v 2 1 + m2 v 2 2 f f 1 2 2 2 Este es un sistema de tres ecuaciones, par a resolverlo se de en dejar slo tres incgnitas, por ejemplo vf1, vf2, y . Se dej c omo t re resolver el sistem , dems que en los problem s de fin l del c ptulo se proponen v rios donde se debe resolver este sistem . 206

C p.7 Momento line l y choques Ejemplo 7.5. Un esfer de bill r bl nc que se mueve con ciert velocid d inici l choc de cost do con otr roj , inici lmente detenid . Si despus del choque l bol bl nc se mueve en 40 respecto su direccin inici l, c lcul r l desvi cin d e l bol roj . Solucin: el esquem se muestr en l figur 7.6; suponiendo que el choque es elsti co, se conserv l energ cintic del sistem y como l vi2 = 0, se tiene: 1 1 1 m1vi2 + 0 = m1v 2 1 + m2 v 2 2 f f 1 2 2 2 T mbin se conserv el momento li ne l, que escrito en form vectori l es: r r r m1vi1 + 0 = m1v f 1 + m2 v f 2 Como l s m s s son igu les, se obtienen l s siguientes dos ecu ciones: vi2 = v 2 1 + v 2 2 1 f f r r r vi1 = v f 1 + v f 2

r r r r r r vi2 = v f 1 + v f 2 v f 1 + v f 2 = v 2 1 + 2v f 1 v f 2 + v 2 2 1 f f r r vi2 = vi2 + 2v f 1 v f 2 1 1 r r vf1 vf 2 = 0 ( )( ) 207

P r resolver este sistem se puede intent r elev r l cu dr do l ltim y luego combin rl con l primer , l h cerlo se obtiene:

ecu cin,

Cap.7 Momen o lineal y choques Por la definicin de produc o escalar, al desarrollar la l ima ecuacin, considerando que el ngulo que forman los vec ores vf1 y vf2 es + 40 , se o tiene: v f 1v f 2 co s( + 40 ) = 0 cos( + 40 ) = 0 Pero el coseno de un ngulo es cero, cuando ese ngulo vale 90 , entonces

Este resultado muestra que siempre en un choque elstico de costado, o no frontal, entre dos masas iguales, con una de ellas inicialmente en reposo, las masas fin almente se movern en un ngulo recto una respecto a la otra. 208

+ 40 = 90

= 50

Cap.7 Momento lineal y choques PROBLEMAS. 7.1. Se acuerdan del pro lema del Chino Ros del captulo 5? (usar esos datos) Supong a que por la fuerza elstica del raquetazo, de 5 ms de duracin, la pelota gana un 5 % de la rapidez con la que golpea a la raqueta. Calcular: a) el impulso so re la pelota, ) la fuerza media. c) Estimar el nmero de raquetazos que pega el Chino en un partido de tenis y calcular la fuerza media en todo un partido. R: a) 5.7 kgm/s, ) 1139 N. 7.2. Una ola de palitroque de 5 kg se mueve en lnea recta a 3 m/s.Qu tan rpido de e moverse una ola de ping pong de 2.5 gr en una lnea recta, de manera que las dos olas tengan el mismo momento? R: 6000 m/s. 7.3. Una pelota c on una masa de 60 gr se deja caer desde una altura de 2 m. Re ota hasta una altu ra de 1.8 m. Cul es el cam io en su momento lineal durante el choque con el piso? 7.4. Una ametralladora dispara alas de 35 g a una velocidad de 750 m/s. Si el a rma puede disparar 200 alas/min, Cul es la fuerza promedio que el tirador de e ej ercer para evitar que la ametralladora se mueva? R: 87.5 N. 7.5. En la figura 7. 7 se muestra la curva fuerza tiempo estimada para una pelota de tenis golpeada p or la raqueta del Chino. A partir de esta curva, calcular, a) el impulso dado a la pelota, ) la fuerza media so re la pelota. R: a) 10 kg m/s, ) 10 kN. Figura 7.7 Pro lema 7.5. 209

Cap.7 Momento lineal y choques 7.6. Salas que otra vez se viene a jugar por la U, le hace un gol de tiro li re al Colo. La fuerza con la cual golpea la pelota de 400 gr, inicialmente detenida , es tal que aumenta linealmente de 0 a 1000 N durante 1 ms, luego se mantiene c onstante durante otro ms y finalmente disminuye linealmente a 0 durante 2 ms. Ca lcular: a) el impulso so re la pelota, ) la rapidez con que sale disparada, c) la fuerza media. Cunto le ganar la U, prximo campen, al Colo? R: a) 2.5 kgm/s, ) 6.2 5 m/s, c) 625 N. 7.7. Zamorano patea un aln de ft ol de 0.5 kg con una rapidez de 15 m/s. Chilavert, solidsimo, atrapa la pelota y la detiene en 0.02 s. a) Cul es e l impulso dado al aln? ) Cul es la fuerza promedio ejercida so re Chilavert? R: a ) 7.5 kg m/s, ) 375 N. 7.8. Un auto se detiene frente a un semforo. Cuando la lu z vuelve al verde, el auto acelera, aumentando su rapidez de cero a 5 m/s en 1 s . Qu momento lineal y fuerza promedio experimenta un pasajero de 70 kg en el auto? 7.9. Una ola de acero de masa M que se mueve en el plano xy, golpea una pared u icada so re el eje y, con una velocidad v a un ngulo con l p red. Rebot con l mism velocid d y ngulo. Si l bol est en cont cto con l p red dur nte un tiem po T, Cul es l fuerz promedio ejercid por l p red sobre l bol ? R: 2Mv sen/T. 7.10. Un meteorito de 2000 kg tiene un velocid d de 120 m/s justo ntes de choc r de frente con l Tierr . Determine l velocid d de retroceso de l Tierr . R: 4x10-20 m/s. 7.11. Un chilen ut de 60 kg c min en el esp cio lej do de l n ve esp ci l cu ndo l cuerd que lo m ntiene unido l n ve se rompe. El puede l nz r su t nque de oxgeno de 10 kg de m ner que ste se leje de l n ve esp ci l con un r pidez de 12 m/s, p r impuls rse s mismo de regreso l n ve. Supon iendo que inici su movimiento desde el reposo (respecto de l n ve), determine l dist nci mxim l cu l puede est r de l n ve esp ci l cu ndo l cuerd se rompe p r regres r en menos de 60 s (es decir, el tiempo que podr est r sin res pir r). R: 120 m. 7.12. Un v gn de ferroc rril de 2.5x104 kg de m s que se mueve con un velocid d de 4 m/s choc p r conect rse con otros tres v gones de fe21 0

 

C p.7 Momento line l y choques rroc rril copl dos, c d uno de l mism m s que el primero y movindose en l m ism direccin con un velocid d de 2 m/s. ) Cul es l velocid d de los cu tro v go nes despus del choque? b) Cunt energ se pierde en el choque? 7.13. Un p tin dor de 80 kg que est p r do sobre un est nque congel do cerc no un muro sostiene un bol de 0.5 kg, que luego l nz contr el muro con un r pidez de 10 m/s respe cto l suelo y l tr p despus que golpe el muro. ) Con que r pidez se mueve el p tin dor despus de tr p r l bol ?, b) cunt s veces puede seguir con este proces o ntes de que su r pidez llegue 1 m/s respecto l suelo. 7.14. Un b l de m s m1, se disp r contr un bloque de m der de m s m2, inici lmente en reposo sobre un superficie horizont l. Despus del imp cto el bloque se desliz un dist nci D ntes de detenerse. Si el coeficiente de roce entre el bloque y l super ficie es . C lcul r l velocid d de l b l justo ntes del imp cto. 7.15. Lucho de 75 kg, est p r do en el extremo de un c rro de 1000 kg y 10 m de l rgo, inici lmente detenido respecto l suelo. Lucho comienz c min r h ci el otro extrem o del c rro r zn de 1 m/s rel tivo l suelo. Supong que no h y roce entre el c rro y el suelo. ) An lice cu lit tiv mente el movimiento de Lucho mientr s c m in sobre el c rro. b) Determine el tiempo que demor en lleg r l otro extremo. c) Qu sucede cu ndo se detiene en el otro extremo del c rro? R: b) 9.3 s 7.16. Un c brochico de 40 kg est p r do 3 m de un muelle, en un extremo de un bote de 7 0 kg, que mide 4 m de l rgo. El c bro observ un recurso loco sobre un roc jus to en el otro extremo del bote y comienz c min r sobre el bote p r lleg r do nde el loco. ) C lcul r l posicin del c bro cu ndo lleg l otro extremo del bo te. b) Suponiendo que el c bro se puede estir r fuer del bote h st 1 m, lc nz r l loco? R: 5.5 m, b) no. 7.17. Un neutrn que se mueve con un velocid d de 3x10 6 m/s choc elstic y front lmente con un ncleo de helio en reposo. Determine: ) l velocid d fin l de c d p rtcul , b) l fr ccin de energ cintic tr nsferid l ncleo de helio. R: ) 1.8x106 m/s, 1.2x106 m/s. 211

C p.7 Momento line l y choques

7.19. Un explosin intern sep r en dos ped zos A y B un m s de 1 kg que se mo v horizont l y libremente en direccin del eje x, con un r pidez de 10 m/s. Despus de l explosin, el trozo A de 250 gr se mov en direccin y 15 m/s. ) H cer un e squem de l situ cin. b) C lcul r el momento ntes de l explosin. c) C lcul r l velocid d de B despus de l explosin. R: b) 10 kgm/s, c) 13.3 - 5 m/s. 7.20. Cu ndo Supertribi, de m s 50 kg, se le c b el efecto de su superm n, c e libremente llev ndo consigo un m cetero de 5 kg. Cu ndo f lt n 10 s p r lleg r l suelo, Supertribi tir horizont lmente el m cetero con un r pidez de 5 m/s. C lcul r d onde c en Supertribi (y no le p s n d porque es super) y el m cetero. R: 5 m, 50 m. 7.21. Un ncleo inest ble de 17x10-27 kg inici lmente en reposo, se desinteg r en tres p rtcul s. Un de ell s de 5x10-27 kg, se mueve lo l rgo del eje y c on un r pidez de 6x106 m/s. Otr p rtcul , de m s 8.4x10-27 kg se mueve lo l rgo del eje x con un r pidez de 4x106 m/s. C lcul r ) l velocid d de l terce r p rtcul , b) l energ tot l emitid en el proceso. R: ) (-9.3 8.3)x106 m/s, b) 4.4x10-13 J. 7.22. Un p rtcul de m s m, que se mueve con velocid d v, choc de cost do con un p rtcul idntic que est en reposo. Demuestre que si el 212

Figur

7.8. Problem 7.18.

7.18. Un esfer de m s 2m que se mueve con r pidez v0 e frente elstic mente con otr esfer de m s m, inici 8). Despus del choque, l esfer 2m retrocede con vo/2 st subir por un pl no inclin do en gr dos, sin roce. e sube m por el pl no. R: 4.5vo2/g sen.

h ci l derech choc d lmente detenid (figur 7. y l de m s m se mueve h C lcul r l dist nci D qu

C p.7 Momento line l y choques choque es elstico l s dos p rtcul s se mueven en 90 un respecto de l otr despus d el choque. 7.23. Considere un pist sin friccin como l mostr d en l figur 7. 9. Un bloque de m s 5 kg que se suelt desde un ltur de 5 m choc front lmen te con otro bloque de m s 10 kg coloc do en l b se de l pist curv , inici lm ente en reposo. C lcul r l ltur mxim l cu l se elev l m s de 5 kg despus del choque. R: 0.56 m. Figur 7.9 Problem 7.23. 7.24. Un p rtcul de m s m1 que se mueve con velocid d inici l vi1 sobre el eje x choc de cost do con otr de m s m2 en reposo. Despus del choque, m1 y m2 se mueven sobre el pl no xy t l que l velocid d fin l vf1 de m1 form un ngulo sobr e el eje x y l velocid d fin l vf2 de m2 form un ngulo ajo el eje x (figura 7. 6). Demuestre que: v f 1 sen t n = vi1 v f 1 cos 7.25. Un bol de bill r que se mueve 5 m/s golpe otr bol est cion ri de l mism m s . Despus del choque, l primer bol se mueve 4.33 m/s en un ngulo de 30 respecto de l lne origin l de movimiento. Suponiendo un choque elstico, c lcul r l velocid d de l bol golpe d . R: 2.5 m/s, -60. 7.26. Un b l de m s m se disp r contr un bloque de m s M inici lmente en reposo en el borde un m es sin friccin de ltur h. L b l se incrust en el bloque y despus del imp cto ste c e un dist nci horizont l D del borde de l mes . Determine l velocid d inici l de l b l . R: D(1+M/m)/(0.2h)1/2. 213

C p.7 Momento line l y choques 7.27. Dos c rritos de igu l m s , 0.25 kg, se coloc n sobre un pist sin friccin que tiene un resorte ligero de const nte fuerz k = 50 N/m unido l extremo der echo de l pist . Al c rrito de l izquierd se le d un velocid d inici l de 3 m/s h ci l derech y el otro c rrito l derech del primero est inici lmente en reposo. Si los c rros choc n elstic mente, encuentre ) l velocid d de c d uno justo despus del primer choque, y b) l comprensin mxim en el resorte. 7.28. D os p rtcul s, de m s s m y 3m, se proxim n un l otr lo l rgo del eje x co n l s mism s velocid des inici les vo. L m s m se mueve h ci l izquierd y l m s 3m h ci l derech . Choc n de frente y c d un rebot lo l rgo de l mism lne en l que se proxim b n. C lcul r l s velocid des fin les de l s p rtc ul s. R: 2vo , 0. 7.29. Dos p rtcul s, de m s s m y 3m se proxim n un l otr lo l rgo del eje x con l s mism s velocid des inici les vo. L m s m se desp l z h ci l izquierd y l m s 3m h ci l derech . Experiment n un choque no front l de modo que m se mueve h ci b jo despus del choque en un ngulo recto re specto su direccin inici l. C lcul r: ) l s velocid des fin les de l s dos m s s, b) el ngulo l cu l se desv 3m. R: )2vo/3cos, 2vot n, b) 35. 7.30. Un cohete co n m s inici l Mi despeg desde l Tierr . Cu ndo su combustible se h consumido complet mente, el cohete se encuentr un ltur peque comp r d con el r dio terrestre. Demostr r que su r pidez fin l es v = -veln(Mi/ Mf)-gt, con t = (MiMf)(dm/dt)-1, donde ve es l r pidez de esc pe de los g ses, Mf l m s fin l d el cohete y dm/dt el consumo const nte de combustible. 214

C p. 8 Dinmic de rot cin. CAPITULO 8. DINAMICA DE ROTACIN. Cu ndo un objeto re l gir lrededor de lgn eje, su movimiento no se puede n liz r como si fuer un p rtcul , porque en cu lqui er inst nte, diferentes p rtes del cuerpo tienen velocid des y celer ciones dis tint s. Por esto es conveniente consider r l objeto re l como un gr n nmero de p rtcul s, c d un con su propi velocid d, celer cin. El nlisis se simplific si se consider l objeto re l como un cuerpo rgido. En este c ptulo se tr t r l rot cin de un cuerpo rgido en torno un eje fijo, conocido como movimiento rot cion l puro.

8.1 ENERGA CINTICA DE ROTACIN. P r un cuerpo rgido form do por un coleccin de p rtc l s que gir lrededor del eje z fijo con velocid d ngul r , c d p rtcul del cu erpo rgido tiene energ cintic de tr sl cin. Si l p rtcul de m s mi, se mueve con velocid d vi, su energ cintic es: 1 mi vi2 2 E ci = C d p rtcul del cuerpo rgido tiene l mism velocid d ngul r , pero distint s ve locid des line les, porque est s dependen de l dist nci r l eje de rot cin, y se rel cion n por vi = ri. Entonces l energ cintic de l p rtcul i es: Ei = 1 1 2 mi (ri ) = mi ri2 2 2 2 L energ cintic tot l del cuerpo rgido en rot cin es l sum de l s energ s cintic de c d p rtcul individu l, esto es: 215

C p. 8 Dinmic de rot cin.

1 1 1 Ei = Ei = mi ri2 2 = mi ri2 2 2 2 2 donde se factoriz 2 porque es todo el cuerpo rgido. A la cantidad entre parntesis en la ecuacin anterior se la de fine como el momento de inercia, I, del cuerpo rgido: I = mi ri2 De la definicin momento de inercia, sus unidades de medida en el SI son kgm2. Con esta definicin, se puede escribir a energa cintica de rotacin de un cuerpo rgido com o: Ec = 1 2 I 2 (8.1) La energa cintica de rotacin no es un nueva forma de energa, sino que es e equiva e nte rotaciona de a energa cintica de tras acin, se dedujo a partir de esa forma d e energa. La ana oga entre ambas energas mv2 y I2 es directa, as cantidades I y movimiento de rotacin son an ogas a m y v de movimiento inea , por o tanto I e s e equiva ente rotaciona de m (a go as como a masa de rotacin), y siempre se c onsidera como una cantidad conocida, igua que m, por o que genera mente se da como un dato. Pero existen tcnicas de ca cu o integra para ca cu ar I, y teorem as asociados, que no se usarn en este curso. E momento de inercia I es una canti dad que depende de eje de rotacin, e tamao y a forma de objeto. En a siguient e tab a 8.1 se dan os momentos de inercia respecto a centro de masa de figuras geomtricas conocidas, de distribucin de masa homognea, cuando giran en torno a ej e que se indica. 216

Cap. 8 Dinmica de rotacin. TABLA 8.1 Objeto (de masa M) Aro o cascarn ci ndrico de radio R, eje de rotacin por su eje de simetra Disco o ci indro s ido de radio R, eje de rotacin por su eje de simetra Ci indro hueco, de radios interno R1 y externo R2, eje de rotacin por su eje de sime tra Esfera s ida de radio R, eje de rotacin por su eje de simetra Cascarn esfrico de ado de radio R, eje de rotacin por su eje de simetra Barra de gada de argo L, con eje de rotacin por e centro Barra de gada de argo L, con eje de rotacin en e e xtremo P aca rectangu ar de ados a y b, eje rotacin en e centro perpendicu ar a a p aca Icm MR 2 1 MR 2 2 1 2 2 M R1 + R2 2 2 MR 2 5 2 MR 2 3 1 ML2 12 1 ML2 3 1 M a 2 + b2 12 ( ) ( ) 8.2 RELACIN ENTRE TORQUE Y ACELERACIN ANGULAR. Para una partcu a de masa m, que gira como se muestra en a figura 8.1, en una ci rcunferencia de radio r con a accin de una fuerza tangencia Ft, adems de a fuer za centrpeta necesaria para mantener a rotacin. La fuerza tangencia se re aciona con a ace eracin tangencia at por Ft = mat. E torque a rededor de centro de crcu o producido por Ft es: =F r = (ma )r Como la a se relaciona con la aceleracin angular por a = r, el torque se puede e scribir como: = (m r) r =(m r2) 217

C p. 8 Dinmic de rot cin.

= El torque que ct sobre un p rtcul es proporcion l su celer cin ngul r , dond e es la constante de proporcionalidad. O servar que = es el nlogo rot cion l de l segund ley de Ne ton F = m . Se puede extender este nlisis un cuerpo rgido rbitr rio que rot en torno un eje fijo que p se por , como se ve en la figura 8.2. El cuerpo rgido se puede considerar formado por elementos de masa dm, que gi ran en torno a en una circunferencia de radio r, por efecto de alguna fuerza tan gencial externa dFt que acta sobre dm. Por la segunda ley de Newton aplicada a dm , se tiene: dFt = (dm) at El torque d producido por la fuerza dF es: d = rdF = ( rdm)a = (rdm)r = (r2 dm) 218

y como mr2 es el momento de inerci de l l tr yectori circul r, entonces:

Figur

8.1 m s m que gir en torno l centro de

C p. 8 Dinmic de rot cin.

= I

(8.2) Observ r que unque l deduccin es complej , el result do fin l es extrem d mente simple, como tod s l s ecu ciones de l Fsic . Ejemplo 8.1. Un b rr uniforme de longitud L y m s M, que gir libremente lre dedor de un bis gr sin friccin, se suelt desde el reposo en su posicin horizont l, como se muestr en l figur 8.3. C lcul r l celer cin ngul r de l b rr y su celer cin line l inici l de su extremo. Solucin. Como el torque de l fuerz en l bis gr es cero, se puede c lcul r el torque en torno l bis gr producido por l otr fuerz extern que ct 219

= d = r 2 dm = r 2 dm Pero l integr l es el momento de inerci I del cuerpo rgido rot cin que p s por , entonces,

El torque neto se obtiene integr ndo est mo v lor en todo el cuerpo rgido,

Figur

8.2 expresin, consider ndo que tiene el mis

lrededor del eje de

C p. 8 Dinmic de rot cin. sobre l b rr , que es su peso, suponiendo que l b rr es homogne y que el peso ct en su centro geomtrico. Entonces: = rP = LMg 2 Figura 8.3 Ejemplo 8.1

Ejemplo torno rededor l rued

8.2. Un rued de r dio R, m s M y momento de inerci I, puede gir r en un eje horizont l sin roce (figur 8.4). Un cuerd ide l se enroll l de l rued y sostiene un bloque de m s m. Cu ndo se suelt en bloque, comienz gir r en torno su eje. C lcul r l ce220

Como = , es I =(1/3) cin line l ndo : t =

y el momento de inerci de l b rr (que se obtiene de l t bl nterior) ML2, se tiene: LMg LMg I = = 22 2 ML 3 3g = 2L P r c lcul r l celer del extremo de l b rr , us mos l ecu cin t = r, con r = L, reempl z L = 3 g 2

C p. 8 Dinmic de rot cin.

Figur 8.4. Ejemplo 8.2 Solucin: el peso de l rued y l fuerz del eje de rot cin no producen torque en torno l eje, por lo que el torque que ct sobre l rued en torno su eje es p roducido por l tensin de l cuerd , su v lor es = RT. Como = , igu l ndo se obtien e I = RT T = I R

Ahor se plic l segund ley de Ne ton l bloque que c e, del DCL se tiene: T mg = ma T = mg ma gualando las tensiones y considerando que a = R = /R, se obti ne 221

ler cin line l del bloque, l d .

tensin de l cuerd

y l

celer cin ngul r de l rue

C p. 8 Dinmic de rot cin. mg ma = I I I = 2 2 + ma = mg a + 1 = g 2 R R R mR

a= g 1 + I mR 2 Con este valor de a se calculan T y , estos clculos d n: T= mg 1 + mR 2 I g R + I mR = 8.3 TRABAJO, ENERGA Y POTENCIA EN EL MOVIMIENTO DE ROTACIN.

Figura 8.5 222

P r un cuerpo rgido la figura 8.5. Si una un distancia r de , ia infinitesimal ds =

que gir en torno un eje fijo que p s por , como se ve en fuerza externa F se aplica en un punto Q del cuerpo rgido a el trabajo realizado por F cuando el objeto gira una distanc rd es:

Cap. 8 Dinmica de rotacin. dW =Fds =( F sen) rd = Ft rd donde F sen = Ft es la componente tangencial de F o la c omponente de la uerza a lo largo del desplazamiento ds, ue es la componente u e realiza trabajo. La componente radial de F no realiza trabajo por ue es perpen dicular al desplazamiento. Como el tor ue es: = r F sen, el trabajo se escribe: d W = d, integrando, se obtiene: W = d i r r El trabajo de rotacin es anlogo el de traslacin W = F dr i

La potencia con la cual se realiza el trabajo es dW d = d = , la potencia instantnea es: P= dW = , dt

d Como dW/d = P y d/dt

expresin anloga el cono del movimiento lineal P =Fv. Tomando ahora la expresin del tor ue rotacional = I, plic ndo l regl de l c den : = I = I d d d d =I = I dt d dt d 223

 

Cap. 8 Dinmica de rotacin.

Al reagrupar esta expresin y considerando ue d = dW dW = Id. Integrando se encuent a el trabajo total realizado durante la rotacin: W = d = Id = i i

Por lo tanto, el trabajo neto realizado por las uerzas externas al hacer girar un cuerpo rgido es igual a la variacin de energa cintica rotacional del objeto. Ejemplo 8.3. Para la barra giratoria del ejemplo 8.1, calcular su rapidez angula r, la rapidez lineal de su centro de masa y del punto mas bajo de la barra cuand o est vertical. Solucin: Usando el principio de conservacin de la energa, considerando ue la energa potencial se calcula respecto al centro de masa y la energa cintica es de rotacin: Ei = E Eci + Egi = Ecf + Egf Cuando la barra esta inicialmente horizontal no t iene Eci y cuando esta vertical tiene solo Ecf, entonces: 1 1 11 MgL = I 2 = ML2 2 2 2 23 = 3g L Para calcular la rapidez del centro de masa, se usa: vcm = r = vcm = L 2 1 3gL 2 224

1 2 1 2 I

I i 2 2

 

Cap. 8 Dinmica de rotacin. En el punto mas bajo la rapidez es v = 2vcm = 3 Lg . Ejemplo 8.4. Para el sistema de la figura 8.6, las masas tienen momento de inerc ia I en torno a su eje de rotacin, la cuerda no resbala en la polea y el sistema se suelta desde el reposo. Calcular la rapidez lineal de las masas despus que una ha descendido H y la rapidez angular de la polea. Figura 8.6 Ejemplo 8.4 Solucin: como no hay roce en la polea, se conserva la energa, que aplicada a cada masa m1 y m2, suponiendo que m2 se encuentra inicialmente en la parte superior d el sistema, es: Ei = Ef Eci1 + Eci2 + Egi1 + Egi2 = Ecf1 + Ecf2 + Egf1 + Egf2 0 + m2 gH = 1 1 1 m1v 2 + m2 v 2 + I 2 + m1 gH 2 2 2

1 I 2 m1 + m2 + 2 v = (m2 m1 )gH 2 R donde se ha usado la relacin v = R, despe se obtiene: v= 2(m2 m1 )gH m1 + m2 + I R 2 225

Cap. 8 Dinmica de rotacin. 8.4 MOVIMIENTO DE RODADURA DE UN CUERPO RGIDO. Se considerar ahora el caso ms general de movimiento de rotacin, donde el eje de ro tacin no est ijo en el espacio, sino ue en movimiento, este se llama movimiento de rodadura. El movimiento general de un cuerpo rgido es muy complejo, pero se pu ede usar un modelo simpli icado limitando el anlisis a un cuerpo rgido homogneo con gran simetra, como un cilindro, una es era o un aro, y suponiendo ue el cuerpo tiene movimiento de rodadura en un plano. Considerar un cilindro uni orme de rad io R ue rueda sin deslizar en una trayectoria recta, como en la igura 8.7. El centro de masa se mueve en lnea recta, pero un punto en el borde se mueve en una trayectoria ms compleja, llamada cicloide. A medida ue el cilindro gira un ngulo , su centro de masa se mueve una distancia s = R, por lo tanto, las magnitudes de la velocidad y la aceleracin del centro de masa para el movimiento de rodadura pu ro son: vcm = ds d =R = R dt dt dvcm d =R = R dt dt cm =

Figur 8.7 L s velocid des line les en los diferentes puntos P, Q, P y Q sobre el cilindro en rot cin se ven en los vectores de l figur 8.7. L velocid d line l de 226

C p. 8 Dinmic de rot cin. cu lquier punto est en direccin perpendicul r l lne de ese punto l punto de co nt cto P, que en cu lquier inst nte est en reposo, porque no h y desliz miento. U n punto gener l del cilindro, como Q tiene un velocid d con componente horizont l y vertic l. Pero los puntos P, CM y P tienen velocid des respectiv mente cero en P porque R =0, vcm= R en el CM y (2R) =2(R) = 2cm e P, ya que todos los pu tos de l cili dro tie e la misma . La e erga ci tica total del cili dro roda te es Ec = 1 I P 2 2 do de IP es el mome to de i ercia alrededor de u eje que pasa por P. Se puede d emostrar que Ip = Icm + MR2 y al reemplazar e Ec se tie e: 1 1 I cm 2 + MR 2 2 , 2 2 Ec = pero vcm= R, e to ces: 1 1 2 Ec = I cm 2 + Mvcm 2 2 (8.3)

Esto sig ifica que la e erga ci tica total de u objeto e movimie to de rodadura est dada por la e erga ci tica de rotaci e tor o al ce tro de masa y la e erga ci t ca de traslaci del ce tro de masa del objeto. El movimie to de rodadura slo es po sible si existe roce e tre el cuerpo rgido que se mueve y la superficie, ya que l a fuerza de roce produce el torque ecesario para hacer rodar el cuerpo rgido e tor o al ce tro de masa. A pesar del roce o hay prdida de e erga mec ica, porque e l pu to de co tacto est e reposo respecto a la superficie e cualquier i sta te. 227

 

  

  

 

 

Cap. 8 Di mica de rotaci . Ejemplo 8.5: Usar la co servaci de la e erga para describir el movimie to de roda dura de u cuerpo rgido de masa M que rueda por u pla o i cli ado y rugoso, que se muestr en l figur 8.8.

E = cte E c + E g = cte E ci + E gi = E cf + E gf Pero Eci = 0 y Egf = 0,entonces Mgh = 1 1 2 I cm 2 + Mvcm 2 2 Como vcm= R = vcm/R, se reemplaza en la ecuacin anterior 2 vcm 1 1 2 I cm 2 + Mvcm = Mgh 2 2 R Despejando vcm se obtiene: vcm = 2 gh 1 + I cm / MR 2 228

Solucin: Se supone que el cuerpo rgido p rte del reposo desde un ued por el pl no sin resb l r. L conserv cin de energ d :

Figur

8.8. Ejemplo 8.5 ltur h y que r

Cap. 8 Dinmica de rotacin. Por ejemplo, para una esfera slida uniforme, de momento de inercia 2 I cm = MR 2 , se puede calcular su vcm en el punto ms bajo del plano y su 5 aceleracin lineal. 2 v cm = 2 gh 2 gh 10 = = gh 2 2 7 (2 / 5) MR 1+ 1+ 2 5 MR 10 gh 7 v cm = La aceleracin lineal se puede calcular con la ecuacin 2 vcm = 2 vicm + 2 acm x = 2 acm x acm 2 vcm = 2x De la geometra de la figura, se tiene: h = x sen, donde x es l longitud del pl no , reempl z ndo en cm: 10 gxsen 5 7 = = gsen 2x 7 cm

8.5 MOMENTO ANGULAR DE UNA PARTCULA. r Un p rtcul de m s m, ubic d en un posicin r desde el origen O, que se r r m ueve con velocid d v , tiene momento line l p . Se define el momento ngur l r L de un p rtcul respecto l origen, como el producto vectori l entre l r r posi cin r y el momento line l p , esto es: 229

C p. 8 Dinmic de rot cin. r r r L=r p (8.4) L unid d de medid de L en el SI es kg m2/s. L direccin de L es perpendicul r e l pl no form do por r y p y su sentido d do por l regl de l m no derech . En l figur 8.9 se muestr los vectores r y p que estn en el pl no xy, por lo t nto L punt en direccin del eje z. L es cero cu ndo r es p r lel p ( = 0 180), est e es el c so cu ndo l p rtcul p s por el origen. Si r es perpendicul r p, =9 0, entonces L=mvr. Como p = m v, l m gnitud de L si es el ngulo entre r y p, es: L = mvrsen Figur 8.9

230

r r r dL d r r dr r r dp = (r

p ) =

mv + r

dt dt dt dt

Si se c lcul l deriv d interes nte, en efecto:

tempor l del momento ngul r, se obtiene un result do

C p. 8 Dinmic de rot cin. r v Como dr dt = v , el primer trmino es cero y que es el producto vectori l vectores p r lelos; en el segundo trmino se us l segund ley de Ne ton en r form F = dp / dt , entonces qued : r r r dL dL r r = r F = d d que es el anlogo ro acional de la segunda Ley de New on. Es a ecuacin indica l orque sobre una par cula es igual a variacin emporal del momen o angular par cula. Para un sis ema de par culas, el momen o angular o al es la suma rial de los momen os angulares de las par culas individuales, es o es: r r r r r L = L1 + L2 + LL + Ln = Li

r i el orque ne o, , es dis in o de cero, en onces puede cambiar el momen o ang ular o al del sis ema de par culas ya que se iene: r r r dL dLi d = d = d Li = d r que significa que la variacin emporal del momen o angular o al del sis ema de p ar culas en orno a algn origen es igual al orque ne o que ac a sobre el sis ema. 8.6 ROTACIN DE UN CUERPO RGIDO EN TORNO A UN EJE FIJO. Considerar un cuerpo rgido que gira alrededor de un eje que iene una direccin fij a y supongamos que es a direccin coincide con el eje z, como se ve en la figura 8 .10. Cada par cula del cuerpo rgido gira en el plano xy en orno al 231

  

 

 

 

 

 

   

de r l que e de la vec o

Cap. 8 Dinmica de ro acin. eje z con rapidez angular . E to ces la mag itud del mome to a gular de la partcul a e tor o al orige es Li = miviri, ya que v es perpe dicular a r. Pero como vi =ri, la mag itud del mome to a gular para u a partcula i se puede escribir como: Li = mi ri2 Figura 8.10 El vector L est e direcci del eje z igual que el vector , por lo que se co sidera como la compo e te z del mome to a gular de la partcula i. Para todo el cuerpo rg ido, la compo e te z del mome to a gular total es la suma de Li de cada partcula del cuerpo rgido: L z = mi ri2 L z = I donde I es el momento de inercia del cuerpo rgido alrededor del eje z. Notar que L = I es el anlogo rotacional del momento lineal p = mv. Se puede derivar Lz respe cto al tiempo considerando que I es constante: dLz d =I = I dt dt 232

 

  

 

   

C p. 8 Dinmic de rot cin.

= I que dice que el torque neto sobre un cuerpo que gir en torno un eje fijo es i gu l l momento de inerci por l celer cin ngul r, ecu cin que y h b sido dedu cid nteriormente.

Figura 8.11 Ejemplo 8.6 Solucin: El momento de inercia por el eje de rotacin del sistema es igual a la sum a de los momentos de inercia de los tres componentes del sistema: m1, barra y m2 ,, con los valores de la tabla 8.1, se obtiene: L2 M 1 L L 2 I = ML + m1 + m2 = m1 + m2 + 12 4 3 2 2 2 2 233

Ejemplo 8.6: Un b rr rgid de m s M y l rgo L gir en un pl no dor de un eje sin friccin que p s por su centro. En los extremos unen dos cuerpos de m s s m1 y m2, como se ve en l figur 8.11. C nitud del momento ngul r del sistem cu ndo su r pidez ngul r es in ngul r cu ndo l b rr form un ngulo con la horizontal.

vertic l lrede de l b rr se lcul r l m g y l celer c

donde es l celer cin tonces se puede escribir

ngul r del cuerpo rgido. Pero dLz/dt es el torque neto, en

Cap. 8 Dinmica de rotacin. Como el sistema gira con rapidez angular , la magnitud del momento angular es:

L2 M L = I = m1 + m2 + 4 3 Para calcular la aceleracin angular usamos la rela = /I, al calcular el orque o al en orno el eje de ro acin, se ob iene: L L 1 c os m2 g cos = (m1 m2 )gL cos 2 2 2 = m1 g

Reemplazando en los v lores de I y de , se ob iene la aceleracin angular: = I = 2(m1 m2 )g cos L(m1 + m2 + M 3) Ejemplo 8.7. En la igura 8.12 las masas m1 y m2 se conectan por una cuerda idea l ue pasa por una polea de radio R y momento de inercia I alrededor de su eje. La mesa no tiene roce, calcular la aceleracin del sistema. Solucin: primero se calcula en momento angular del sistema de las dos masas mas l a polea: L = m1vR + m2 vR + I v R Figura 8.12 Ejemplo 8.7 234

 

Cap. 8 Dinmica de rotacin. Luego se calcula el tor ue externo sobre el sistema, la nica uerza externa ue c ontribuye al tor ue total es m1g, el valor de este tor ue es: = m1gR. En onces s e iene: = dL d v m1 gR = (m1 + m2 )vR + I dt dt R dv I dv + dt R dt m1 gR = (m1 + m2 )R m1 g m1 + m2 + I R 2 a= 8.7 CONSERVACIN DEL MOMENTO ANGULAR. De la ecuacin: r dL = d r si el orque ne o que ac a sobre el sis ema es cero, en onces: r r dL = 0 L = cte dt (8.5) Esta ecuacin dice que el momento angular total de un sistema es constante si el t orque neto que acta sobre el sistema es cero: es el principio de conservacin del m omento angular. 235

Cap. 8 Dinmica de rotacin. Si un cuerpo rgido experimenta una redistribucin de su masa, entonces su momento d e inercia cambia, en este caso la conservacin del momento angular se escribe en l a forma: Li = L f Si el cuerpo gira entorno a un eje fijo, entonces L = I, y se puede escribir I i i = I f f Esta es la tercera Ley de conservacin que hemos deducido. Entonces ahora podemos afirmar que para un sistema aislado, la energa, el momento lineal y el momento an gular permanecen constantes. Son los principios de conservacin en Fsica. Ejemplo 8.8. Un proyectil de masa m y velocidad vo se dispara contra un cilindro slido de masa M y radio R (figura 8.13). El cilindro est inicialmente en reposo m ontado sobre un eje horizontal fijo que pasa por su centro de masa. El proyectil se mueve perpendicular al eje y se encuentra a una distancia D < R sobre el eje . Calcular la rapidez angular del sistema despus que el proyectil golpea al cilin dro y queda adherido a su superficie. Figura 8.13 Ejemplo 8.8 236

Cap. 8 Dinmica de rotacin. Solucin: el momento angular del sistema se conserva, entonces Li = L f : 1 mvo D = I = MR 2 + mR 2 2 = mvo D 1 MR 2 + mR 2 2 Ejemplo 8.9. Un disco de masa M y radio R gira en un plano horizontal en torno a un eje vertical sin roce. Un gato de masa m camina desde el borde del disco hac ia el centro. Si la rapidez angular del sistema es o cuando el gato est en el bord e del disco, calcular: a) la rapidez angular cuando el gato ha llegado a un punt o a R/4 del centro, b) la energa rotacional inicial y final del sistema. Solucin. Llamando Id al momento de inercia del disco e Ig al momento de inercia del gato, el momento de inercia total inicial y final del sistema es:

Ii = Id + Ig = MR2 +m R2 If = MR2 +m r2 = MR2 +m (R/4)2 a) Como no hay torques e xternos sobre el sistema en torno al eje de rotacin, se puede aplicar la conserva cin del momento angular I i i = I f f ( MR2 +m R2)o = ( MR2 +m (R/4)2)f MR 2 2 + mR M 2+m f = o = o M 2 + m 16 MR 2 2 + mR 2 16 237

Cap. 8 Dinmica de rotacin. b) ECi = 1 11 2 2 I i o = MR 2 + mR 2 o 2 22 ECf 2 1 11 2 MR 2 + m R 2 = I f f = f 2 22 4 ECf 11 R = MR 2 + m 22 4 2 M 2 + m 2 M 2 + m 16 o La energa rotacional aumenta. 238

Cap. 8 Dinmica de rotacin. PROBLEMAS.

8.1. El centro de masa de una pelota de radio R, se mueve a una rapidez v. La pe lota gira en torno a un eje que pasa por su centro de masa con una rapidez angul ar . Calcule la razn entre la energa rotacional y la energa cintica de traslacin. Con idere la pelota una esfera uniforme. 8.2. Un volante en la forma de un cilindro slido de radio R = 0.6 m y masa M = 15 kg puede llevarse hasta una velocidad angu lar de 12 rad/s en 0.6 s por medio de un motor que ejerce un torque constante. D espus de que el motor se apaga, el volante efecta 20 rev antes de detenerse por ca usa de la friccin (supuesta constante). Qu porcentaje de la potencia generada por e l motor se emplea para vencer la friccin? R: 2.8%. 8.3. Un bloque de masa m1 y un o de masa m2 se conectan por medio de una cuerda sin masa que pasa por una polea en forma de disco de radio R, momento de inercia I y masa M. Asimismo, se deja que los bloques se muevan sobre una superficie en forma de cua con un ngulo como m uestra la igura 8.14. El coe iciente de riccin cintico es para ambos blo ues. De termine a) la aceleracin de los dos blo ues y b) la tensin en cada cuerda. R: a) ( m2sen )(m1 + m2cos)g/(m1 + m2 + M), b) T1 = m2g + m1a, T2 = T1 + Ma. 8.4. Una masa m1 y una masa m2 estn suspendidas por una polea ue tiene un radio R y una masa m 3 ( igura 8.15). La cuerda tiene un masa despreciable y hace ue la polea gire s in deslizar y sin riccin. Las masas empiezan a moverse desde el reposo cuando es tn separadas por una distancia D. Trate a la polea como un disco uni orme, y dete rmine las velocidades de las dos masas cuando pasan una rente a la otra. 8.5. U n disco slido uni orme de radio R y masa M puede girar libremente sobre un pivote sin riccin ue pasa por un punto sobre su borde ( igura 8.16). Si el disco se l ibera desde el reposo en la posicin mostrada por el crculo. a) Cul es la rapidez de su centro de masa cuando el disco alcanza la posicin indicada en el crculo puntead o? b) Cul es la rapidez del punto ms bajo sobre el disco en la posicin de la circun erencia punteada? c) Repetir para un aro uni orme. R: a) 2(Rg/3), b) 4(Rg/3), c) ( Rg). 239

Cap. 8 Dinmica de rotacin. Figura 8.14 Figura 8.15 Figura 8.16

8.6. Un peso de 50 N se une al extremo libre de una cuerda ligera enrollada alre dedor de una pelota de 0.25 m de radio y 3 kg de masa. La polea puede girar libr emente en un plano vertical en torno al eje horizontal ue pasa por su centro. E l peso se libera 6 m sobre el piso. a) calcular la tensin de la cuerda, la aceler acin de la masa y la velocidad con la cual el peso golpea el piso. b) Calcular la rapidez con el principio de la conservacin de la energa. R: a) 11.4N, 7.6 m/s2, 9 .5 m/s, b) 9.5 m/s. 8.7. Una ligera cuerda de nylon de 4 m est enrollada en un ca rrete cilndrico uni orme de 0.5 m de radio y 1 kg de masa. El carrete est montado sobre un eje sin riccin y se encuentra inicialmente en reposo. La cuerda se tira del carrete con una aceleracin constante de 2.5 m/s2. a) Cunto trabajo se ha e ect uado sobre el carrete cuando ste alcanza una velocidad angular de 8 rad/s? b) Sup oniendo ue no hay la su iciente cuerda sobre el carrete, Cunto tarda ste en alcanz ar esta velocidad angular? c) Hay su iciente cuerda sobre el carrete? R: a) 4 J, 1.6 s, c) s. 8.8. Una barra uni orme de longitud L y masa M gira alrededor de un eje horizontal sin riccin ue pasa por uno de sus extremos. La barra se suelta d esde el reposo en una posicin vertical ( igura 8.17). En el instante en ue est ho rizontal, encuentre a) su rapidez angular, b) la magnitud de su aceleracin angula r, c) las componentes x e y de la aceleracin de su centro de masa, y d) las compo nentes de la uerza de reaccin en el eje. R: a) (3g/L), b) 3g/2L, c) (3/2 + )g, d) (/2 + )Mg. 8.9. Los bloques mostr dos en l figur 8.18 estn unidos entre si por un pole de r dio R y momento de inerci I. El bloque sobre l pendiente sin fricc in se mueve h ci rrib con un celer cin const nte de 240

C p. 8 Dinmic de rot cin. m gnitud . ) Determine l s tensiones en l s dos p rtes de l cuerd , b) encuen tre el momento de inerci de pole . R: ) T1 = m1( + gsen), T2 = m2(g a), b) m2R 2g/a m1R2 m2R2 m1R2(g/a)sen. Figura 8.17 Figura 8.18 8.10. Un carrete cilndrico hueco y uni orme tiene radio interior R/2, radio exter ior R y masa M ( igura 8.19). Est montado de manera ue gira sobre un eje horizon tal ijo. Una masa m se conecta al extremo de una cuerda enrollada alrededor del carrete. La masa m desciende a partir del reposo una distancia y durante un tie mpo t. Demuestre ue el tor ue debido a las uerza de roce entre el carrete y el eje es: = R m g 2 M 2 t 4 t 2y 5 y Figura 8.19 8.11. Un cilindro de 10 kg de masa rueda sin deslizar sobre una superficie horiz ontal. En el instante en que se su centro de masa tiene una rapidez de 10 m/s, d etermine: a) la energa cintica traslacional de su centro de masa, b) la energa rota cional de su centro de masa, y c) su energa total. R: a) 500 J, b) 250 J, c) 750 J. 241

Cap. 8 Dinmica de rotacin. 8.12. Una esfera slida tiene un radio de 0.2 m y una masa de 150 kg. Cunto trabajo se necesita para lograr que la esfera ruede con una rapidez angular de 50 rad/s sobre una superficie horizontal? (Suponga que la esfera parte del reposo y rueda sin deslizar). 8.13. Un disco slido uniforme y un aro uniforme se colocan uno fr ente al otro en la parte superior de una pendiente de altura h. Si se sueltan am bos desde el reposo y ruedan sin deslizar, determine sus rapideces cuando alcanz an el pie de la pendiente Qu objeto llega primero a la parte inferior? 8.14. Una b ola de boliche tiene una masa M, radio R y un momento de inercia de (2/5)MR2. Si rueda por la pista sin deslizar a una rapidez lineal v, Cul es su energa total en funcin de M y v? R: 0.7Mv2. 8.15. Un anillo de 2.4 kg de masa de radio interior d e 6 cm y radio exterior de 8 cm sube rodando (sin deslizar) por un plano inclina do que forma un ngulo de = 37 con la horizontal. En el momento en ue el anillo ha recorrido una distancia de 2 m al ascender por el plano su rapidez es de 2.8 m/ s. El anillo continua ascendiendo por el plano cierta distancia adicional y desp us rueda hacia abajo. Suponiendo ue el plano es lo su icientemente largo de mane ra ue el anillo no ruede uera en la parte superior, u tan arriba puede llegar? 8.16. Una barra rgida ligera de longitud D gira en el plano xy alrededor de un pi vote ue pasa por el centro de la barra. Dos partculas de masas m1 y m2 se conect an a sus extremos. Determine el momento angular del sistema alrededor del centro de la barra en el instante en ue la rapidez de cada partcula es v. R: ( m1 + m2) vD. 8.17. Un pndulo cnico consta de masa M ue se mueve en una trayectoria circula r en un plano horizontal. Durante el movimiento la cuerda de longitud L mantiene un ngulo constante con la vertical. Muestre ue la magnitud del momento angular de la masa respecto del punto de soporte es: 242

Cap. 8 Dinmica de rotacin. L= gM 2 L3 sen 4 cos 8.18. Una partcula de masa m se dispara con una rapidez vo ormando un ngulo con l a horizontal. Determine el momento angular de la partcula respecto del origen cua ndo sta se encuentra en: a) el origen, b) el punto ms alto de su trayectoria, c) j usto antes de chocar con el suelo. R: a) 0, b) mvo3sen2 cos/2g, c) 2mvo3sen2 cos/g . 8.19. Un disco slido uni orme de masa M y radio R gira alrededor de un eje ijo perpendicular su cara. Si la rapidez angular es , calcular el momento angular de l disco cuando el eje de rotacin a) pasa por su centro de masa, y b) pasa por un punto a la mitad entre el centro y el borde. 8.20. Una partcula de 0.4 kg de masa se une a la marca de 100 cm de una regla de 0.1 kg de masa. La regla gira sobre una mesa horizontal sin riccin con una velocidad angular de 4 rad/s. Calcular e l momento angular del sistema cuando la regla se articulan torno de un eje, a) p erpendicular a la mesa y ue pasa por la marca de 50 cm, b) perpendicular a la m esa y ue pasa por la marca de 0 cm. R: a) 0.43 kgm2/s, b) 1.7 kgm2/s. 8.21. Una mujer de 60 kg ue est parada en el borde de una mesa giratoria horizontal ue t iene un momento de inercia de 500 kgm2 y un radio de 2 m. La mesa giratoria al pr incipio est en reposo y tiene libertad de girar alrededor de un eje vertical sin riccin ue pasa por su centro. La mujer empieza a caminar alrededor de la orilla en sentido horario (cuando se observa desde arriba del sistema) a una rapidez c onstante de 1.5 m/s en relacin con la Tierra. a) En u direccin y con u rapidez angu lar gira la mesa giratoria b) Cunto trabajo realiza la mujer para poner en movimie nto la mesa giratoria? R: a) 0.36 rad/s, antihorario. 8.22. Una barra uni orme d e masa M y longitud d gira en un plano horizontal en torno de un eje vertical i jo sin riccin ue pasa por su centro. Dos pe ueas cuentas, cada una de masa m, se montan sobre la barra de manera tal ue pueden deslizar sin riccin a lo largo d e su longitud. Al principio las cuentas se ijan por medio de retenes ubicados e n las posiciones x (donde x < d/2) a cada lado del centro, tiempo durante el cua l el sistema gira una rapidez angular . Repentinamente, los retenes se ui243

Cap. 8 Dinmica de rotacin. tan y las pe ueas cuentas se deslizan saliendo de la barra. Encuentre, a) la rapi dez angular del sistema en el instante en ue las cuentas alcanzan los extremos de la barra, y b) la rapidez angular de la barra despus de ue las cuentan han sa lido de ella. 8.23. Un blo ue de madera de masa M ue descansa sobre una super i cie horizontal sin riccin est unido a una barra rgida de longitud L y masa desprec iable. La barra gira alrededor de un pivote en el otro extremo. Una bala de masa m ue se desplaza paralela a la super icie horizontal y normal a la barra con r apidez v golpea el blo ue y ueda incrustada en l. a) Cul es el momento angular del sistema bala blo ue? b) Qu raccin de la energa cintica original se pierde en la col isin? R: a) mvl, b) M/(M+m). 8.24. Una cuerda se enrolla alrededor de un disco un i orme de radio R y masa M. El disco se suelta desde el reposo con la cuerda ver tical y su extremo superior amarrado a un soporte ijo. A medida ue el disco de sciende, demuestre ue a) la tensin en la cuerda es un tercio del peso del disco. b) La magnitud de la aceleracin del centro de masa es 2g/3, y c) la rapidez del centro de masa es (4gh/3). Veri i ue su respuesta a la pregunta c) utilizando mtod os de energa. 8.25. Una pe uea es era slida de masa m y de radio r rueda sin desliz ar a lo largo de la pista mostrada en la igura 8.20. Si parte del reposo en la parte superior de la pista a una altura h, donde h es grande comparada con r a) Cul es el valor mnimo de h (en uncin de R) de modo ue la es era complete la traye ctoria? b) Cules son las componentes de uerza de la es era en el punto P si h = 3 R? 8.26. Un proyectil de masa m se mueve a la derecha con rapidez vo. El proyect il golpea y ueda ijo en el extremo de una barra estacionaria de masa M y longi tud D ue est articulada alrededor de un eje sin riccin ue pasa por su centro ( igura 8.21). a) Encuentre la rapidez angular del sistema justo despus de la colis in. b) Determine la prdida raccionaria de energa mecnica debida a la colisin. 244

Cap. 8 Dinmica de rotacin. Figura 8.20 Figura 8.21 8.27. A una bola de boliche se le da una rapidez inicial vo en una canal de mane ra tal ue inicialmente se desliza sin rodar. El coe iciente de riccin entre la bola y la canal es . Demuestre ue durante el tiempo en ue ocurre el movimiento de rodamiento puro, a) la rapidez del centro de masa de la bola es 5vo/7, y b) l a distancia ue recorre es 12vo2/49g. (Sugerencia: Cuando ocurre el movimiento de rodamiento puro, vcm = R. Puesto ue la uerza de riccin proporciona la desacele racin, a partir de la segunda ley de Ne ton se concluye ue acm = g.) 8.28. El ala mbre de un carrete de masa M y radio R se desenrolla con una uerza constante F ( igura 8.22). Suponiendo ue el carrete es un cilindro slido uni orme ue no des liza, muestre ue, a) la aceleracin del centro de masa es 4F/3M, y b) la uerza d e riccin es hacia la derecha y su magnitud es igual a F/3. c) Si el cilindro par te del reposo y rueda sin deslizar, Cul es la rapidez de su centro de masa despus ue ha rodado una distancia D? R: c) (8FD/3M). 8.29. Suponga un disco slido de radi o R al cual se le da una rapidez angular o alrededor de un eje ue pasa por su ce ntro y despus se baja hasta una super icie horizontal y se suelta, como en la ( i gura 8.23). Suponga tambin ue el coe iciente de riccin entre el disco y la super icie es . a) Calcular la rapidez angular del disco una vez ue ocurre el rodamie nto puro. b) Calcular la prdida raccionaria de energa cintica desde el momento en ue el disco se suelta hasta ue ocurre el rodamiento puro c) Muestre ue el tie mpo ue tarda en ocurrir el movimiento de rodamiento puro es R0/3g. d) Muestre ue el tiempo ue recorre el disco antes de ue ocurra el rodamiento puro es R202/18g . 245

 

 

Cap. 8 Dinmica de rotacin. 8.30. La igura 8.24 muestra un carrete de alambre ue descansa sobre una super icie horizontal. Cuando se tira, no se desliza en el punto de contacto P. El car rete se tira en las direcciones indicadas por medio de los vectores F1, F2, F3 y F4. Para cada uerza determine la direccin en ue rueda el carrete. Advierta ue la lnea de accin de F2 pasa por P. 8.31. El carrete mostrado en la igura 8.24 ti ene un radio interior r y un radio externo R. El ngulo entre la uerza aplicada y la horizontal puede variar. Demuestre ue el ngulo crtico para el cual el carrete no rueda y permanece estacionario est dado por cos = r/R. (Sugerencia: En el ngulo crtico la lnea de accin de la uerza aplicada pasa por el punto de contacto.) Figura 8.22 Figura 8.23 Figura 8.24 246

Cap. 9. Ley de gravitacin. CAPITULO 9. LEY DE GRAVITACION UNIVERSAL. 9.1 LA LEY Y LA FUERZA GRAVITACIONAL. La Ley de Gravitacin Universal ue descubierta por Ne ton, cuando le cay una manza na en la cabeza mientras hacia una siesta debajo de un manzano. Por este hecho N e ton le pregunto al manzano manzano, si la manzana cae, uiz todos los cuerpos en el Universo se atraen entre s de la misma orma como la manzana ue atrada por la Tierra?. Como el manzano nada le respondi, Ne ton comenz a trabajar sobre eso hasta ue descubri la Ley de Gravitacin Universal, ue public en 1686 en sus Mathematica l Principles o Natural Philosophy. Se puede enunciar de la siguiente orma: Toda partcula material del universo atrae a cual uier otra partcula con una uerza dir ectamente proporcional al producto de sus masas e inversamente proporcional al c uadrado de la distancia ue las separa Si las partculas ue tienen masas m1 y m2 e stn separadas una distancia r medida desde sus centros, como se ve en la igura 9 .1, entonces, de acuerdo a la ley de gravitacin universal, la uerza de atraccin g ravitacional FG ejercida por la masa m1 sobre la masa m2 es: FG m2 r m1 r12 Figura 9.1 247

Cap. 9. Ley de gravitacin. r mm FG = G 1 2 2 r12 r (9.1) Su magnitud es: FG = G m1 m2 r2 La constante de proporcionalidad G se llama Constante de Gravitacin Uni versal, y r12 es un vector unitario radial dirigido desde la masa m1 a la masa m2. El valo r de G, ue se determina experimentalmente, y su unidad de medida en el SI es 6. 672 x 10 11 N m2/kg2. El signo menos en la FG indica ue la uerza es de atraccin , dirigida desde m2 hacia m1, es decir es opuesta a la direccin radial hacia uer a, desde la masa m1 ue ejerce la uerza sobre m2; en los clculos su valor numrico es siempre positivo. En este punto se debe tener presente ue: La constante uni versal G no se debe con undir con el vector g, ue ni es universal ni es constan te. La ley de gravitacin universal no es ecuacin de de inicin de ninguna de las var iables sicas contenidas en ella. La ley de gravitacin universal expresa la uerza entre partculas. Si se uiere determinar la uerza gravitacional entre cuerpos r eales, se los debe considerar ormado por un conjunto de partculas y usar clculo i ntegral. Las uerzas de gravitacin entre partculas son parejas de accin y reaccin. 9.2 FUERZA GRAVITACIONAL Y PESO. La uerza con ue la Tierra atrae a los cuerpos cerca de la super icie terrestre se de ini como el peso del cuerpo, P = mg. Esta es la uerza gravitacional FG entre el cuerpo de masa m y la Tierra de masa MT, separados una distancia entre sus centros r = RT + z, donde RT es el radio de l a Tierra y z es la altura de m sobre el suelo. Igualando las expresiones de las uerzas P y FG se obtiene: 248

 

Cap. 9. Ley de gravitacin. mg = G mM T ( RT + z ) 2 g =G MT ( RT + z ) 2 Esta ecuacin permite calcular el valor de la aceleracin de gravedad g a cual uier altura z sobre la super icie, ya ue se conoce G, la MT y el RT. De esta ecuacin se observa ue g disminuye con la altura. En la tabla 9.1 se muestra la variacin de g con la latitud y con la altura z (en la Universidad de Concepcin, el gravmetr o del Observatorio Geodsico Transportable Integrado, TIGO, ubicado all arriba en l os cerros permite medir las variaciones de g en el noveno decimal, estas variaci ones son principalmente por e ecto de la atraccin gravitacional de la Luna). TABLA 9.1. Variacin de g con la latitud en z = 0 g (m/s2) ( ) 0 9.78036 10 9.78195 20 9.78641 30 9.79329 40 9.80171 45 9.80616 50 9.81071 60 9.81719 70 9.82368 80 9.83016 90 9.83208 Variacin de g con la altura z en = 45 z (km) g (m/s2) 0 9.80616 1 9.803 5 9.791 10 9.775 20 9.745 30 9.708 100 9.598 1000 7.33 5000 3.08 10000 1.49 0 La aceleracin de gravedad g tambin varia con la latitud debido a ue la Tierra no es una es era, es un elipsoide achatado levemente en los polos, de manera ue el radio ecuatorial es 21 km mayor ue el radio polar, valor pe ueo 249

Cap. 9. Ley de gravitacin. comparado con el radio medio de la Tierra de 6367.47 km. La Tierra no es un cuer po rgido, tiene un comportamiento plstico. Por e ecto de la rotacin terrestre, la a celeracin centrpeta disminuye desde el ecuador, donde es mxima, hacia los polos, do nde se anula, produciendo una mayor uerza centrpeta en zonas ecuatoriales, ue es tira a la Tierra hacia a uera ms ue en zonas polares, por eso la Tierra es achata da en los polos. Esto tiene como consecuencia ue la aceleracin de gravedad no ap unte directamente hacia el centro de la Tierra, sino ue est levemente desviada d e la direccin vertical. La desviacin mxima ue tiene g de la vertical es de 1140 a 45 de latitud, y la variacin del valor de g en super icie es menos ue 0.5 %, por lo ue se puede considerar constante. Ejemplo 9.1: Un satlite de 300 kg describe una rbita circular alrededor de la Tier ra a una altura igual al radio terrestre ( igura 9.2). Calcular a) la rapidez or bital del satlite, b) su perodo de revolucin, c) la uerza gravitacional sobre el s atlite, d) comparar su peso en la rbita con su peso en la super icie de la Tierra. v FG 2RT Figura 9.2 Ejemplo 9.1 a) El satlite de masa mS, se mantiene en rbita por la accin de la uerza gravitacio nal, ue acta como uerza centrpeta, es decir FG = FC, entonces se igualan las exp resiones de ambas uerzas: 250

 

Cap. 9. Ley de gravitacin. v2 FC = m S r FG = G Como r = 2RT, reemplazando GM T GMm v2 FG = FC =m v2 = 2 2 RT 2 RT 4 RT Datos: G = 6.7 x10 11 Nm 2 , M T = 6 x10 24 Kg , RT = 6.37 x10 6 m 2 Kg (6.7 x10 11 ) Nm 2 /kg 2 (6 x10 24 )kg m = 5600 s 2 6.37 x10 6 m M T mS r2 v= b) El satlite completa u a vuelta e tor o a la Tierra a la altura de 2RT movi dos e co la rapidez a terior, e to ces: 2 2 (2 RT ) x 2 = t = = t v v t v= t =

c) La fuerza gravitacional en la rbita corresponde al peso del satlite en ese luga r, se calcula como sigue: 251

6,37 x10 6 m = 14294s t = 3.97 horas 5600m/s

Cap. 9. Ley de gravitacin. F= 6.7 10 11 Nm 2 /kg 2 6 10 24 kg 300kg (2 6.37 x10 m) 6 2 F = 740 N d) Para hacer esta comparaci , calculamos su peso e tierra. P = mg = 300 x9.8 = 2940 N Pz =0 2940 = Pz = 2 R = 0,25Pz =0 Pz = 2 R 740 9.3 ENERGIA POTENCIAL DE LA FUERZA GRAVITACIONAL. Una partcula de masa m que se encuentre sobre la superficie terrestre, movindose e ntre dos puntos cualesquiera, esta bajo la influencia de la fuerza gravitacional , cuya magnitud es: FG = GM T m r2 El cambio de energa potencial de la partcula de masa m se define como el trabajo n egativo realizado por la fuerza gravitacional, en este caso: r r E P = E Pf E Pi = W = FG dr r r r ri Reemplazando en esta expresin la uerza gravitacional, para calcular la energa pot encial gravitacional de la partcula de masa m, se obtiene: 252

Cap. 9. Ley de gravitacin. E g E gi =

ri 1 rf dr GM T m 2 = GM T m r ri r 1 1 E gf E gi = GM T m r f ri Como el punto de referencia inicial para la energa potencial es arbitrario, se pu ede elegir en r = , donde la fuerza gravitacional (y la aceleracin de gravedad) es cero. Con esta eleccin se obtiene la energa potencial gravitacional general para una partcula de masa m ubicada a una altura r medida desde el centro de la Tierra : Eg ( r ) = GMT m r (9.2) La e erga pote cial gravitacio al e tre partculas varia e 1/r, y es egativa porq ue la fuerza gravitacio al es de atracci y se ha tomado la e erga pote cial como cero cua do la separaci e tre las partculas es i fi ita. Como la fuerza gravitaci o al es de atracci , u age te exter o debe realizar trabajo positivo para aume t ar la separaci e tre las partculas. El trabajo produce u aume to de la e erga pot e cial cua do las dos partculas est separadas, esto sig ifica que Eg se vuelve me os egativa cua do r aume ta. Esta ecuaci es ge eral y vale para cualquier par de partculas de masas m1 y m2 separadas u a dista cia r, y exte derse a u sistem a que co te ga varias partculas, e ese caso la e erga total del sistema es la sum a sobre todos los pares de partculas, e to ces para dos partculas se tie e:

Eg ( r ) = Gm1m2 r 253

 

 

  

 

 

    

  

 

 

Cap. 9. Ley de gravitaci . Ejemplo 9.2: calcular la e erga total para u satlite de masa m, que se mueve e u a rbita circular co rapidez ta ge cial co sta te v, a u a altura r desde el ce tro de la Tierra (figura 9.3). Soluci : la e erga total del satlite es la suma de l a e erga ci tica ms la pote cial, que es co sta te, reemplaza do los valores corres po die tes de cada e erga, se tie e: E = E c + E P = cte. E= 1 2 GMm mv 2 r v FG r Figura 9.3. Ejemplo 9.2. Pero se debe calcular la v del satlite, como la rbita es circular aplica do la seg u da ley de Ne to al satlite de masa m, co sidera do que la fuerza gravitacio al es la fuerza ce trpeta ecesaria para ma te er al satlite e rbita, 254

 

 

  

  

 

  

 

Cap. 9. Ley de gravitaci . FG = FC GMm v2 = ma c = m r r2 GMm 1 2 = mv 2r 2 reemplazando en la energa total E, queda: E= GMm GMm 2r r GMm 2r E= se observa que la e erga total es egativa e el caso de rbitas circulares. Ge era liza do este resultado al sistema solar, la e erga total del sistema Solpla eta e s u a co sta te del movimie to. 9.3.1 Velocidad de escape Supo ga que u objeto de masa m se la za verticalme te hacia arriba desde la sup erficie terrestre co u a velocidad vi, como se muestra e la figura 9.4. Podemo s utilizar co sideracio es de e erga para e co trar el valor m imo de la velocidad i icial co la cual el objeto escapar del campo gravitacio al de la Tierra. La e cuaci a terior os bri da la e erga total del objeto e cualquier pu to cua do se co oce su velocidad y dista cia desde el ce tro de la Tierra. E la superficie de sta vi = v y ri = RT . Cua do el objeto alca za su altura mxima, vf = 0 y rf = rmx. Debido a que la e erga total del sistema es co sta te, al reemplazar estas c o dicio es se obtie e: E ci + E Pi = E cf + E Pf GM T m 1 2 GM T m mvi =0+ 2 RT r mx 255

 

  

 

    

 

Cap. 9. Ley de gravitaci . Al despejar v2i se obtie e 1 1 v12 = 2GM T R T rmx vf=0 h rmax vi RT Figura 9.4 En consecuencia, si se conoce la velocidad inicial, esta expresin puede usarse pa ra calcular la altura mxima h, puesto que sabemos que h = rmx - RT. Ahora tenemos la posibilidad de calcular la velocidad mnima que el objeto debe tener en la supe rficie terrestre para escapar de la influencia del campo gravitacional del plane ta. Al viajar a esta velocidad mnima, el objeto puede alcanzar justamente el infi nito con una velocidad final igual a cero. Al establecer rmx = en la ecuacin anter ior y tomando vi = vesc, que se llama la velocidad de escape, obtenemos vesc = 2 GM RT Advierta que esta expresin para vesc es independiente de la masa del objeto. En o tras palabras, una nave espacial tiene la misma velocidad de escape que una molcu la. Adems, el resultado es independiente de la direccin de la velocidad, siempre q ue la trayectoria no intersecte la Tierra. Si al objeto se le da una velocidad i nicial igual a vesc, su energa total es igual a cero. Esto puede verse cuando r = , la energa cintica del objeto y su ener256

Cap. 9. Ley de gravitacin. ga potencial son ambas cero. Si vi es ms grande que vesc, la energa total es mayor que cero y el objeto tiene un poco de energa cintica residual en r = . Por ltimo, us ted debe observar que las ecuaciones anteriores pueden aplicarse a objetos lanza dos desde cualquier planeta. Es decir, en general, la velocidad de escape desde cualquier planeta de masa M y radio R es v esc = 2GM R Ejemplo 9.3. Calcular la velocidad de escape de la Tierra para una nave espacial de 5000 kg y determine la energa cintica que debe tener en la superficie terrestr e para escapar del campo gravitacional de la Tierra. Solucin: Utilizando la ecuac in anterior con MT = 5.98x1024 kg y RT = 6.37x106 m, obtenemos vesc = 2GM T RT =

La energa cintica de la nave espacial es EC =

257

1 2 1 mvesc = (5

10 3 kg )(1.12

10 4 m / s ) 2 2 2 EC = 3.14x1011 J

2(6.67 10 11 N m 2 / kg 2 )(5.98

10 24 kg ) 6.37

10 6 m vesc = 11.2x103 m/s

Cap. 9. Ley de gravitacin. Las velocidades de escape para los planetas, la Luna y el Sol las puede calcular como ejercicio. Los valores varan de 1.1 km/s para Plutn a casi 618 km/s para el Sol. Estos resultados, junto con algunas ideas de la teora cintica de los gases, e xplican por u algunos planetas tienen atms eras y otros no. Una molcula de gas tie ne una energa cintica promedio ue depende de su temperatura. Por consiguiente, la s molculas ms ligeras, como el hidrgeno y el helio, tienen una velocidad promedio ms alta ue las partculas ms pesadas a la misma temperatura. Cuando la velocidad de las molculas ms ligeras no es mucho menor ue la velocidad de escape, una raccin s igni icativa de ellas tiene oportunidad de escapar del planeta, dejndolo a este s in atms era. Este mecanismo explica tambin por ue la Tierra retiene muy poco las m olculas de hidrgeno y helio en su atms era, en tanto ue las molculas mas pesadas co mo el oxigeno y nitrgeno no escapan tan cilmente. 9.4 LAS LEYES DE KEPLER. Los movimientos de los planetas, estrellas y otros cuerpos celestes han sido obs ervados por la gente durante miles de aos. En la antigedad, los cient icos consider aban a la Tierra como el centro del universo. As el modelo llamado geocntrico ue elaborado por el astrnomo griego Claudio Ptolomeo (100 170) en el segundo siglo D C y ue aceptado durante los siguientes 1400 aos. En 1543, el astrnomo polaco Nico ls Coprnico (1473 1543) sugiri ue la Tierra y los otros planetas giraban en rbitas circulares alrededor del Sol (el modelo heliocntrico). El astrnomo dans Tycho Brahe (1546 1601) hizo mediciones astronmicas ms precisas por un periodo de 20 aos y pro porcion una prueba rigurosa de los modelos alternativos del sistema solar. Es int eresante observar ue estas precisas observaciones sobre los planetas y de 777 e strellas visibles a simple vista se llevaron a cabo con un gran sextante y un co mps, sin un telescopio, el cual an no se haba inventado. El astrnomo alemn Johannes K epler, uien era ayudante de Brahe, obtuvo los datos astronmicos de este ltimo y e mple casi 16 aos en tratar de desarrollar un modelo matemtico para el movimiento de los planetas. El anlisis completo se resume en tres enunciados, conocidos como l as leyes de Kepler: 258

 

 

Cap. 9. Ley de gravitacin. 1. Todos los planetas se mueven en rbitas elpticas con el Sol en uno de los puntos ocales. 2. El radio vector trazado desde el Sol hasta un planeta barre reas igu ales en intervalos de tiempo iguales 3. El cuadrado del periodo orbital de cual uier planeta es proporcional al cubo del semieje mayor de la rbita elptica. Medio siglo despus, Ne ton demostr ue estas leyes son la consecuencia de una uerza nica ue existe entre cuales uiera dos masas. La ley de la gravedad de Ne ton, junto con su desarrollo de las leyes del movimiento, entrega las bases para la solucin matemtica completa del movimiento de planetas y satlites. 9.4.1 La tercera ley de Kepler. La tercera ley de Kepler puede predecirse a partir de la ley de gravitacin univer sal. Considere un planeta de masa MP ue se mueve alrededor del Sol de masa MS e n una rbita circular, como en la igura 9.5. Puesto ue la uerza gravitacional e jercida sobre el planeta por el Sol es igual a la uerza central necesaria para mantener al planeta movindose en un crculo, GM S M P r2 M Pv2 = r Sin embargo, la velocidad orbital del planeta es simplemente 2 /T donde T es su e iodo; o lo tanto, la ex esin ante io se convie te en GM S M 2 4 2 T = GM S 2 (2 / T )2 = 3 r = K S r 3 (9.3) 259

Cap. 9. Ley de gravitacin.

v FG MP MS

Figu a 9.5. La ecuacin 9.3 es la te ce a ley de Ke le . La ley es vlida tambin a a bitas el tic as si sustituimos o la longitud del semieje mayo , a (figu a 9.6). Advie ta que la constante de o o cionalidad, KS es inde endiente de la masa del laneta . En consecuencia, la ecuacin 9.3 es vlida a a cualquie laneta. Si hubi amos con side ado la bita de un satlite al ededo de la Tie a, como la Luna, entonces la constante tend a un valo dife ente, con la masa del Sol sustituida o la masa d e la Tie a. En este caso, la constante de o o cionalidad es igual a 42/GMT . Ejem lo 9.4. Calcula la masa del Sol a a ti del hecho de que el e iodo de t aslacin de la Tie a en to no al Sol es un ao y la distancia de la Tie a al Sol e s 1.496x1011 m. Solucin: Usando la te ce a ley de Ke le , des ejando MS, se obtiene: 260

donde KS es una constante dada por KS = 4 2 = 2.97

10 19 s 2 /m 3 GM S

Ca . 9. Ley de g avitacin.

( ) 3 ( ) 2 MS = 1.99x1030 kg. Advierta que el Sol tiene 333000 veces ms masa que la Tierra. b a Figura 9.6 9.4.2 La segunda ley de Kepler y la conservacin del momento angular. Considere un planeta de masa MP que se mueve en torno al Sol en una rbita elptica, como se ilustra en la figura 9.7. La fuerza gravitacional que acta sobre el plan eta siempre es a lo largo del radio vector, dirigido hacia el Sol. El torque que acta sobre el planeta debido a esta fuerza es cero puesto que F es paralelo a r. Esto es, r r r = r F = r F (r )r = 0 r 261

4 2 1.496

1011 m MS = Nm 2 3.156 10 7 s 6.67 10 11 2 kg

4 2

3 MS = GT 2 Reem lazando los valo es num icos, con T = 1 ao = 3.156x107s:

Cap. 9. Ley de gravi acin. v r ol FG dA MP Figura 9.7 Pero recordemos que el orque es igual a la asa de cambio en el iempo del mome n o angular o = dL/d . Por lo an o, debido a que = 0, el momen o angular L del plane a es una cons an e del movimien o:

En vir ud de que L es una cons an e del movimien o, vemos que el movimien o del plane a en cualquier ins an e es res ringido al plano formado por r y v. Es e im por an e resul ado significa que: Tan o el momen o angular o al como la energa o al del sis ema ol plane a son cons an es del movimien o.

dA =

262

1 r r 1 r r L r

dr = r

v d = d 2 2 2M P

 

Podemos relacionar es e o vec or r en la figura a la mi ad del rea r o que el desplazamien o

resul ado con la siguien e consideracin geom rica. El radi 9.7 barre un rea dA en un iempo d . Es a rea es r r igual dr del paralelogramo formado por los vec ores r y dr. Pues del plane a en un iempo d es dr = vd , ob enemos

 

    

  

r r r r r L = r

p = M P r

v = cons an e

 

  

 

      

Cap. 9. Ley de gravi acin. dA L = = cons an (9.4) donde L y MP son cons an es del movimien o. As pues, concluimos que el radio vec or desde el ol has a un plane a barre rea iguales en iempos iguales. Es e resul ado es la segunda ley de Kepler. La segunda ley de Kepler no revela la na urale za inversa al cuadrado de la fuerza de gravedad. Aunque no lo demos ramos aqu, la primera ley de Kepler es una consecuencia direc a del hecho de que la fuerza gr avi acional vara como 1/r2. Es o es, bajo una ley de fuerza del inverso al cuadra do, es posible demos rar que las rbi as de los plane as son elipses con el ol en un foco. Ejemplo 9.5. Un sa li e de masa M se mueve en una rbi a elp ica alrededor de la Ti erra. Las dis ancias mnima y mxima al sa li e desde la Tierra reciben el nombre de perihelio (rp en la figura 9.8) y afelio (indicado por ra). i la velocidad del sa li e en rp es vp, cul es su velocidad en ra. olucin. El momen o angular del sa li e en relacin con la Tierra es M r v. En los pun os ra y rp, v es perpendicular a r . En consecuencia la magni ud del momen o angular en es os pun os es La = M vara y Lp = M vprp. Debido a que el momen o angular es cons an e, vemos que: va M vara = M vprp ra va = rp ra vp rp vp Figura 9.8 Ejemplo 9.5. 263 e d 2 M p

 

  

 

 

 

 

 

 

Cap. 9. Ley de gravi acin. 9.5 EL CAMPO GRAVITACIONAL. Cuando New on public por primera vez su eora de la gravi acin, para sus con emporne os fue difcil acep ar la idea de un campo de fuerza que pudiera ac uar a ravs de una dis ancia. e pregun aban cmo era posible que dos masas in erac uaran aun cua ndo no es uvieran en con ac o en re s. Aunque el propio New on no pudo responder a es a pregun a, su eora fue ampliamen e acep ada debido a que explic de manera s a isfac oria el movimien o de los plane as. Un plan eamien o al erna ivo en la d escripcin de la in eraccin gravi acional, por lo an o, es in roducir el concep o de un campo gravi acional que cubre cada pun o en el espacio. Cuando una par cula de masa m se si a en un pun o donde el campo es el vec or g, la par cula experime n a una fuerza Fg = mg. En o ras palabras, el campo ejerce una fuerza sobre la p ar cula. Por lo an o, el campo gravi acional se define por medio de r g= r Fg m Es decir, el campo gravi acional en un pun o en el espacio es igual a la fuerza gravi acional experimen ada por una masa de prueba si uada en el pun o, dividido por la masa de prueba. Por ejemplo, considere un obje o de masa m cerca de la s uperficie erres re. La fuerza gravi acional sobre el obje o es dirigida hacia e l cen ro de la Tierra y iene una magni ud mg. Pues o que la fuerza gravi aciona l sobre el obje o iene una magni ud GMTm/r2 (donde MT es la masa de la Tierra), el campo g a una dis ancia r del cen ro de la Tierra es r g= r Fg m = GM T r2 r donde r es un vec or uni ario que apun a radialmen e hacia fuera de la Tierra, y el signo menos indica que el campo apun a hacia el cen ro erres re, como en 264

 

 

  

 

 

 

  

 

 

  

 

 

 

 

 

Cap. 9. Ley de gravi acin. la figura 9.9. Advier a que los vec ores de campos en diferen es pun os que circ undan la Tierra varan an o en direccin como en magni ud. En una regin pequea cercan a a la superficie de la Tierra, el campo hacia abajo g es aproximadamen e cons a n e y uniforme, como se indica en la figura 9.9. La ecuacin an erior es vlida en odos los pun os fuera de la superficie erres re, suponiendo que la Tierra es es frica. En la superficie erres re, donde r = RT, g iene una magni ud de 9.8 N/kg . Figura 9.9 Represen acin del campo gravi acional erres re. 265

  

Cap. 9. Ley de gravi acin. PROBLEMA . 9.1. Dos obje os se a raen en re s con una fuerza gravi acional de magni ud 1x10 8 N cuando es n separados 20 cm. i la masa o al de los dos obje os es 5 kg, cul e s la masa de cada uno? 9.2. La dis ancia en re los cen ros de dos esferas es 3 m . La fuerza en re ellas es 2.75 x 10 12N. Cul es la masa de cada esfera, si la mas a de una de ellas es el doble de la o ra? 9.3. Toms que iene una masa de 70 kg y ara de 55 kg, se encuen ran en una pis a de bailes separados 10 m. ara levan a la mirada y ve a Toms, ella sien e una a raccin. a) i la a raccin es gravi acion al, calcule su magni ud. b) Pero la Tierra ejerce una a raccin gravi acional sobr e ara Cul es su magni ud? 9.4. La masa de la Luna es 7.34x1022 kg y se encuen ra a 3.8x108 m de la Tierra. a) Calcule la fuerza de a raccin gravi acional en re la s dos. b) Encuen re el valor del campo gravi acional erres re en la Luna. 9.5. Q u pasara con el valor de G y de g, si la ierra uviera el doble de su masa pero e l mismo amao? 9.6. Comparar la masa y el peso de un as ronau a de 75 kg en la Ti erra, con su peso cuando es a en una nave espacial en rbi a circular alrededor de la Tierra, a una al ura de 105 km. 9.7. a) Un sa li e es a una dis ancia de la T ierra igual al radio erres re. Cmo es la aceleracin de la gravedad en ese pun o co mparada con la de la superficie de la Tierra? b) A qu al ura sobre la superficie d e la Tierra iene que elevarse el sa li e para que su peso sea la mi ad del que iene sobre la Tierra? 9.8. La masa de Jpi er es aproximadamen e 300 veces la masa de la Tierra, y su radio es aproximadamen e 10 veces el erres re. Calcule el v alor de g en la superficie de Jpi er. 266

 

 

 



 

 

 

 

 

 

 

Cap. 9. Ley de gravi acin. 9.9. Urano emplea 84 aos en darle la vuel a al ol. Encuen re el radio de la rbi a de Urano como ml iplo del radio de la rbi a de la Tierra 9.10. i la dis ancia de l sol a un plane a fuera 5 veces la dis ancia de la Tierra al ol, En cun os aos el plane a comple a una vuel a alrededor del ol? 9.11. El 19 de Julio de 1969 la r bi a de la nave espacial Apolo 11 alrededor de la Luna fue ajus ada a una rbi a m edia de 111 km. El radio de la Luna es 1785 km. a) Cun os minu os le om comple ar una rbi a? b) Qu velocidad ena alrededor de la Luna? 9.12. Conocidas las dis ancias en re la Luna, la Tierra y el ol respec ivamen e y sus masas, encuen re la razn de las fuerzas gravi acionales ejercidas por la Tierra y el ol sobre la Luna. 9.13. Calcular la energa po encial de un sa li e de 1000 kg que se encuen ra a una al ura de 2000 km sobre la Tierra. 9.14. Un sa li e de 500 kg es en una rbi a cir cular de radio 2RT, calcular la energa requerida para cambiar al sa li e a o ra rbi a de radio 4RT. 9.15. Calcular la energa requerida para enviar una nave de 1000 kg desde la Tierra has a una dis ancia donde la fuerza de gravedad sea desprecia ble. 9.16. Un sa li e me eorolgico de 100 kg describe una rbi a circular alrededor de la Tierra a una al ura de 9630 km. Calcular: a) su rapidez angencial en la rb i a, b) el rabajo necesario para ponerlo en esa rbi a. R: a) 5000 m/s, b) 3.75x1 09 J. 9.17. Un sa li e de 300 kg describe una rbi a circular en orno a la Tierra a una al ura de 3 radios erres res. Calcular: a) su rapidez angencial, b) el rabajo para ponerlo en rbi a, c) la aceleracin de gravedad a la al ura del sa li e. R: a) 3963 m/s, b) 1.4x1010 J, c) 0.61 m/s2. 9.18. Un sa li e geoes acionario es aquel que se mueve en sincronismo con la Tierra, permaneciendo en una posicin fi ja sobre algn pun o del ecuador, comple ando por lo an o una vuel a en orno a l a Tierra en un da. 267

 

 

  

  

 

 

 

 

  

 

 

Cap. 9. Ley de gravi acin.

Calcular: a) su al ura, b) su rapidez angencial. R: a) 35930 km, b) 3075 m/s. 9 .19. Los sa li es de rbi a polar orbi an a una al ura de 850 km de la superficie erres re. Calcular a) la rapidez angencial para un sa li e de 300 kg, b) el iem po en comple ar una vuel a. R: a) 7450 m/s, b) 1.7 horas. 9.20. Demues re que la energa po encial de un sis ema que cons e de cua ro par culas iguales de masa M, colocadas en las esquinas de un cuadrado de lado D, es E = (4+ 2 )(GM2/D). 9.21 . e dispara un cohe e ver icalmen e desde la superficie erres re y alcanza una al ura mxima de res veces el radio de la Tierra. Cul fue la rapidez inicial del c ohe e? 9.22. Buscar los da os necesarios para calcular la energa po encial o al del sis ema Tierra ol Luna. uponga que la Tierra y la Luna es n a la misma dis ancia del ol. 9.23. El sis ema binario de Plaske se compone de dos es rellas que giran en una rbi a circular en orno de un cen ro de gravedad si uado a la mi ad en re ellas. Es o significa que las masas de las dos es rellas son iguales. i la velocidad orbi al de cada es rella es de v y el periodo de cada una es de T, calcule la masa M de cada es rella. R: 2v3T/G. 9.24. Dos lanetas X e Y se mue ven en bitas ci cula es en sentido antiho a io en to no de una est ella. Los ad ios de sus bitas estn en la o o cin 3:1. En cie to momento estn alineados, fo man do una lnea ecta con la est ella. Cinco aos des us el laneta X ha gi ado 90 Dnde es el laneta Y en ese momento? R: a 1.3 ev de su osicin o iginal. 9.25. Des us de que se agote su combustible nuclea , el destino final de nuest o Sol es cola sa se en una enana blanca, es deci , una est ella que tiene a oximadamente la masa del Sol, e o el adio de la Tie a. Calcule a) la densidad omedio de la enan a blanca, b) la acele acin de cada lib e en su su e ficie, c) la ene ga otencial g avitacional de un objeto de 1kg en su su e ficie. R: a) 1.85x109 kg/m3, b) 3.3x 106 m/s2, c) 2.1x1013 J. 268

 

 

 

  

   

 

 

 

Ca . 9. Ley de g avitacin. 9.26. El cometa Halley se ace ca al Sol a una distancia a oximada de 0.57UA, y su e iodo o bital es de 75.6 aos. (UA es la ab eviatu a de unidad ast onmica, don de 1UA = 1.50x106 km es la distancia media Tie a Sol.) Qu tan lejos del Sol viaja el cometa Halley antes de que inicie su viaje de eg eso? 9.27. a) Cul es velocid ad mnima necesa ia a a que una nave es acial esca e del sistema sola , em ezando en la bita de la Tie a? b) El Voyage I alcanz una velocidad mxima de 125000 km. /h en su camino a a fotog afia J ite . Ms all de que distancia desde el Sol esta v elocidad es suficiente a a esca a del Sistema Sola ? R: a) 42 m/s, b) 2.2x1011 m. 9.28. Pa a cualquie que bita al ededo del Sol, la te ce a ley de Ke le u ede esc ibi se como T2 = k 3, donde T es el e iodo o bital y es el semieje ma yo de la bita. a) Cul es valo de la k si T se mide en aos y se mide en UA? b) C on el valo de k encuent e el e iodo o bital de J ite si su adio medio desde e l Sol es 5.2UA. 9.29. T es masas iguales son colocadas en t es esquinas de un cu ad ado de lado D. Encuent e este cam o g avitacional g en la cua ta esquina debi da a estas masas. R: ((2 2 +1)/2)(GM/D2). 9.30. T es objetos untuales que tiene n masas m, 2m y 3m estn fijos en las esquinas de un cuad ado de longitud de lado a de modo tal que el objeto ms lige o se ubica en la esquina su e io izquie da, el objeto ms esado est en la esquina infe io izquie da y el te ce o, en la esqui na su e io de echa. Dete mine la magnitud y di eccin del cam o g avitacional g esultante en el cent o del cuad ado. R: 2 2 Gm/a2 . 9.31. Dos lanetas hi otticos de masa m1 y m2 y adios 1 y 2, es ectivamente, estn en e oso cuando estn se a ados una distancia infinita. Debido a su at accin g avitacional, se mueven uno hacia ot o en el cu so de una colisin. a) Cuando la se a acin ent e sus cent o es d, calcula la a idez de cada laneta y su a idez elativa. b) Calcula la ene ga cintica de cada laneta justo antes de que choquen, si m1 = 2x1024 kg, m2 = 8x 1024 kg, 1 = 3x106 m y 2 = 5x106 m. ( Suge encia: tanto la ene 269

Ca . 9. Ley de g avitacin.

2G , d (m1 + m 2 ) 2G (m1 + m 2 ) , b) E1 = 1.1x1032 J, E2 = d 9.32. El Vangua d I, lanzado el 3 de ma zo de 1958, es el satlite a tificial ms vi ejo an en bita. Su bita inicial tena un a ogeo de 3970 km y un e igeo de 650 km. Su velocidad mxima e a de 8.23 km/s y tena una masa de 1.60 kg. a) Dete mine el e iodo de bita (utilice el semieje mayo ). b) Dete mine las velocidades en el a o geo y en el e igeo. c) Encuent e la ene ga total del satlite. 9.33. Des us de una ex losin su e nova, una est ella uede ex e imenta un cola so g avitacional hast a alcanza un estado ext emadamente denso conocido como una est ella de neut one s, en el cual todos los elect ones y otones se com imen a a fo ma neut ones . Una est ella de neut ones que tiene una masa a oximada o igual a la del Sol t end a un adio de casi 10 km. Encuent e a) la acele acin de cada lib e en su su e f icie, y c) la ene ga eque ida a a lleva un neut n de 1.67 x 10 27 kg de masa d esde su su e ficie hasta el infinito. 270

ga como el momento se conse van). R: v1 = m 2 2G , v J. v 2 = m1

el = d (m1 + m 2 ) 2.7x1031

Ca . 10. Mecnica de fluidos. CAPITULO 10. NOCIONES DE MECANICA DE FLUIDOS. 10.1 ESTRUCTURA DE LA MATERIA. En G iego, tomo significa indivisible, o eso est a alab a fue ado tado o los fsicos a a a lica la a la a tcula ms equea y funda mental. Pe o aho a se sabe que los elementos qumicos estn fo mados o a tculas el ementales ms equeas que son los elect ones, otones y neut ones, que en conjunto constituyen el tomo. Los tomos de la mate ia comn, que tienen un dimet o del o den de 10 10 m, se com onen de un ncleo esado, de dimet o del o den de 10 15 m, que c ontiene otones ca gados ositivamente y neut ones sin ca ga, que esta no malme nte odeado o uno o va ios elect ones livianos ca gados negativamente. La func in de los neut ones es actua como egamento a a mantene unidos los otones en e l ncleo, si los neut ones no estuvie an esente, la fue za e ulsiva ent e las a tculas ca gadas ositivamente desinteg a a al ncleo. La masa del otn, 1.67x10 27 kg, que se define como la unidad de masa atmica u, y su ca ga, 1.6x10 19 Coulomb , se usan como unidad. El tomo mas sim le es el hid geno neut o, su modelo clsico s e muest a en la figu a 10.1a, su ncleo tiene un otn y se dice que tiene nme o de masa 1 y ca ga elct ica +1. Al ededo del ncleo del tomo de hid geno neut o, o bita un elect n, que tiene ca ga igual a 1, una masa de 9.1x10 31 kg, igual a u/1840, c on un adio de la bita de 0.5x10 10 m. Figu a 10.1a Figu a 10.1b 271

Ca . 10. Mecnica de fluidos. La mate ia comn, como el ai e o agua, se com one de molculas que son elct icamente neut as. Una molcula uede tene un solo tomo o bien uede se la unin de dos o ms to mos. Existen molculas com uestas de cientos, miles, incluso millones de tomos. En la figu a 10.1b se muest a un esquema de una molcula de agua. Pe o te mina aqu la d ivisin? Se ha descubie to que existen a tculas ms equeas an, llamadas qua ks, fo ma das o seis va iedades dife entes de ot as a tculas bautizadas con nomb es extic os: a iba, abajo, ext ao, encanto, belleza y su e io . Pe o la mate ia no es con tinua, ya que ent e cada a de a tculas hay un eno me es acio vaco. An as las dista ncias en la f onte a de la Fsica nuclea son so endentemente co tas! En el ot o ext emo, las distancias en el Unive so son s e equetecont ahi e g andes. Los e xt emos de la Fsica los odemos esumi en los t es infinitos, que se ilust an en la figu a 10.2. Figu a 10.2 Los t es infinitos

Adems o qu hab an de existi nicamente a tculas livianas con ca ga negativa y a t las esadas con ca ga ositiva? Cuando se esuelven las ecuaciones de la mecnica cuntica, gene almente se encuent an dos solucio272

Ca . 10. Mecnica de fluidos. nes simt icas, una da los esultados del com o tamiento de la mate ia, e o no ha y ninguna azn a a desca ta la ot a solucin, o lo tanto se o uso que desc iba el com o tamiento de la antimate ia, que no se conoca. De acue do a las leyes de la fsica, en el inci io todo e a simt ico: mate ia y antimate ia estaban esen te en el Unive so en a tes iguales. Se busco la existencia de esta antimate ia hasta que se descub ie on las anti a tculas elementales: antielect n o osit n, ant i otn, antineut n, antiqua ks, etc, que tienen la misma masa que las a tculas ele mentales, e o ca gas o uestas. 10.1.1 Estados de la mate ia. La mate ia gene almente se clasifica de acue do co n algunos de los cuat o estados en que se encuent a, slido, lquido, gaseoso y las ma. Un slido tiene fo ma y volumen definidos. Un lquido tiene un volumen definido e o no una fo ma definida. Un gas no tiene ni volumen ni fo ma definidos. Pa a cualquie sustancia, el estado lquido existe a una tem e atu a mayo que la del e stado slido, tiene mayo agitacin t mica y las fue zas molecula es no son suficient es a a mantene a las molculas en osiciones fijas y se ueden move en el lquido . Lo comn que tienen los lquidos con los slidos es que si actan fue zas exte nas de com esin, su gen g andes fue zas atmicas que se esisten a la com esin del lquido. En el estado gaseoso, las molculas tienen un continuo movimiento al aza y eje c en fue zas muy dbiles unas con ot as; las se a aciones omedios ent e las molcula s de un gas son mucho ms g andes que las dimensiones de las mismas. Un slido se co m ime bajo la accin de fue zas exte nas, e o si estas fue zas dejan de actua , tiende a etoma su fo ma y tamao o iginal, o esto se dice que tiene elasticida d. Segn el tiem o de es uesta del cambio de la fo ma a una fue za exte na o es in, la mate ia uede com o ta se como un slido o como un fluido. En algunos casos, el mate ial se com o ta en un estado inte medio, como o ejem lo lstico, goma, asfalto, g asa, miel, masilla, etc. 10.1.2 Plasma. Cuando se calienta un slido, se t ansfo ma en lquido, si se contina calentando se convie te en gas. Pe o si aumenta an ms la tem e atu a del gas, los 273

Ca . 10. Mecnica de fluidos. choques ent e las a tculas se vuelven tan violentos que son ca aces de va ia la est uctu a de las a tculas. Los elect ones ueden se libe ados de los tomos o duciendo iones ca gados ositivamente. Las molculas de un gas ueden om e se al somete las a la accin de la luz ult avioleta, ayos X, co iente elct ica o a inte nso calo y los elect ones ueden se violentamente se a ados de la molcula. Al esto de la molcula que le falta uno o ms elect ones, queda ca gada ositivamente, se le llama un in y el gas queda ionizado. El gas ionizado fo mado de elect ones con ca ga negativa y de iones con ca ga ositiva se llama lasma, que es ot o es tado fluido de la mate ia, slo existe a altas tem e atu as (mayo que 2000 K). A esa de se oco comn en la vida cotidiana, es el estado edominante de la mate ia en el Unive so. El Sol, las est ellas o el gas de la luz en un tubo fluo esc ente estn en estado de lasma.

10.1.3 Fluido. Un fluido es un conjunto de molculas dist ibuidas al aza que se m antienen unidas o fue zas cohesivas dbiles y o fue zas eje cidas o las a e des de un envase. De ot a fo ma, si definimos un fluido como aquellos mate iales que no lo son, los fluidos son todos aquellos que no son slidos. Po lo tanto, s on fluidos los lquidos y los gases. Una dife encia esencial ent e un fluido y un slido es que un fluido no so o ta esfue zos tangenciales y los slidos s. De acue do con esto, los fluidos son sistemas que estn en continuo movimiento. En este cont exto, la mecnica clsica debe modifica se un oco, o la oca utilidad que tiene a qu el conce to de masa, o lo que esta se eem laza o ot o conce to, llamado d ensidad, que co es onde a la masa o unidad de volumen. En los oblemas que n os inte esan, los fluidos con los que t ata emos inci almente son el ai e y el agua. Cuando estudiamos la atmsfe a y el ocano en sus movimientos de escala lane ta ia, nos efe imos a estos como fluidos geofsicos. Po ejem lo el estudio de lo s ciclones y anticiclones, de la co iente de Humboldt, o en ot os lanetas de l a g an Mancha Roja de J ite . 10.2 DENSIDAD. Una o iedad de cualquie sustancia es su densidad. La densidad de cualquie mate ial se define como la cantidad de masa m contenida en cada uni dad 274

Ca . 10. Mecnica de fluidos. de volumen V. Como la dist ibucin de masa uede va ia si se conside a el volumen com leto de sustancia, se debe defini en fo ma mic osc ica la densidad en cada unto del cue o en fo ma dife encial, esto es: = dm dV (10.1) La densidad es una magnitud fsica escala , su unidad de medida en el SI es kg/m3. La densidad cambia con la tem e atu a ya que el volumen de ende de la tem e atu a, o lo que se dan valo es bajo condiciones de esin y tem e atu as dadas. Si un cue o tiene la misma densidad en todo el volumen, es deci es constante, se dice que es homogneo, en caso cont a io es hete ogneo, en este caso el cue o tie ne una dist ibucin de masa va iable dent o del volumen. La densidad de los lquidos (y slidos) es del o den de 1000 veces la de los gases. En la tabla siguiente se dan los valo es de la densidad de algunas sustancias comunes. MATERIAL DENSIDAD (kg/m3) Hid geno 0.09 Ai e 1.28 Made a de ino 500 Pet leo 800 H ielo 917 Agua 1000 Aluminio 2700 Hie o 7860 Cob e 8900 Plomo 11340 Me cu io 135 00 O o 19300 Platino 21400 La densidad de los fluidos de ende de la tem e atu a y de la esin. La ecuacin qu e ex esa esta de endencia se llama ecuacin de estado, e o este tema 275

Ca . 10. Mecnica de fluidos. es un as ecto de los fluidos que se t ata en fo ma cuantitativa en el cu so de Fs ica de Te modinmica. Baste deci aho a que la densidad de ende del inve so de la tem e atu a. La va iacin de densidad con la tem e atu a en los gases da luga al fenmeno de conveccin, muy im o tante a a el t ans o te de calo en un fluido. Po ejem lo, la conveccin en la atmsfe a oduce el movimiento ve tical ascendente de l ai e, lo que o igina disminucin de esin en su e ficie, ex ansin de la masa de a i e, enf iamiento o la ex ansin y el ascenso, condensacin o efecto del enf iam iento, fo macin de nubes debido a la condensacin y de eci itacin. 10.3 PRESION. Las fue zas que existen sob e un objeto sume gido en un fluido son slo aquellas que tienden a com imi al objeto. La fue za eje cida o un fluido sob e el objeto inme so en l, e esentado o el cubo de la figu a 10.3, es sie m e e endicula a la su e ficie del objeto. La esin del fluido en el nivel donde se encuent a sume gido el cue o se define como la azn de la magnitud de la fue za F no mal a la su e ficie y el ea A. La esin dent o del fluido no es l a misma en todos los untos, o lo que se debe defini la esin en un unto det e minado conside ando una fue za dF no mal a un elemento de su e ficie dA, enton ces la esin en el unto es: ai e dF agua dA Figu a 10.3 =

dF dA (10.2) 276

Ca . 10. Mecnica de fluidos. La unidad de medida de la esin en el sistema SI es N/m2, que se llama Pascal, c on smbolo Pa. Ot as unidades de uso comn a a la esin son atmsfe a (atm), centmet o s de me cu io (cm de Hg) o ba . Algunos facto es de conve sin comunes ent e dife entes unidades son: 1 ba = 105 Pa y 1 miliba (mba ) = 10 3 ba = 100 Pa = 1 hP a 1 atm = 1.013x105 Pa = 1.013 ba = 1013 mba = 1013 hPa = 76 cm de Hg 10.4 LA ECUACIN HIDROSTATICA. Pa a un fluido en e oso dent o de un envase, todos los untos a la misma ofundidad tienen la misma esin, si no fue a as no esta a en e oso. Imagina un volumen de fluido (ai e) elemental en la atmsfe a, de su e ficie dA y alto dz, como se ve en la figu a 10.4. La fue za en la a te infe i o del volumen es ve tical hacia a iba de magnitud F1 = 1dA = (z)dA y en la a te su e io es hacia abajo de valo F2 = 2dA = (z+dz)dA. El eso del volumen es dP = (dm)g. Como el volumen est en equilib io, o la ime a Ley de Ne ton, se tiene:

F2 dA dz ai e F1

Figu a 10.4 277

ma

z z+dz, 2 z,

1 P

Ca . 10. Mecnica de fluidos. F = 0 F1 - F2 - P = 0 p(z)dA - p(z+dz)dA - (dm)g = 0 [p(z) - p(z+dz)]dA - (dm)g = 0 Pero p(z+dz) - p(z )= dp, = dm/dV dm = dV y dV = dAdz, eem lazando se obtiene: d dA dAdz g = 0 dp = g dz (10.3) Esta se llama ecuacin hid osttica, se le da ese nomb e o que fue deducida a a un a o cin de fluido en equilib io esttico. Se obse va que la esin disminuye con la altu a y aumenta con la ofundidad en el fluido. Si o es el valo de la esin en el nivel zo (que uede se el nivel del ma ) y el valo de la esin a una altu a z en la atmsfe a o una ofundidad z en el ocano, y si la densidad es const ante, se uede integ a la ecuacin hid osttica y se obtiene:

+ gh

o = g ( z z o ) Si se cuya su e ficie acta la esin el ma , lago o cualquie envase e : = o + g ( z o z ) = o 278

conside a como volumen de fluido una o cin de ocano, en atmosf ica o, la esin a la ofundidad h = zo z en que contenga algn lquido de densidad constante, s

Ca . 10. Mecnica de fluidos. Esta ecuacin, que es vlida slo cuando la densidad es constante, dice que la esin a la ofundidad h de la su e ficie lib e de un fluido es mayo que la esin atmo sf ica o en gh. De esto tambin se deduce que la esin es la misma en cualquie un to ubicado a la misma ofundidad y no se ve afectada o la fo ma del envase. E l t mino gh se llama esin manomt ica, ya que co es onde a la esin obtenida de la lectu a de un manmet o, es deci , la dife encia ent e la esin total y una esin de efe encia, que con f ecuencia es la esin atmosf ica. La esin del agua aume nta a medida que se baja hacia el fondo del ocano y disminuye en la atmsfe a si no s elevamos sob e el nivel del ma . Como la densidad del ai e es unas 1000 veces meno que la del agua, el aumento de esin al descende un met o en agua es ce c a de mil veces mayo a la disminucin de la esin al ascende un met o de altu a. En la atmsfe a ce ca de su e ficie, la esin disminuye a oximadamente un hPa cad a 10 met os de elevacin en la ve tical y en el ocano la esin aumenta a oximadame nte 100 hPa cada un met o de ofundidad. Ejem lo 10.1. Calcula la fue za esultante eje cida o el agua sob e una e e sa de ofundidad H y de ancho D, que se muest a en la figu a 10.5. o H h o

D dz z Figu a 10.5. Esquema de una e esa. Solucin. La coo denada ve tical z se mide desde el fondo de la e esa hacia a i ba, entonces la ofundidad H de la e esa es igual a zo. La esin a una ofun didad h medida desde la su e ficie del agua hacia abajo, como se ve en la figu a 10.5, se calcula usando la ecuacin hid osttica, teniendo en cuenta 279

Ca . 10. Mecnica de fluidos.

que la esin atmosf ica o acta en todos lados sob e la e esa, o lo que no alt e a el valo de , el clculo da: o = g ( z o z ) p p o = g ( H z ) Pe o dF o)dA = g(H z)Ddz, integ ando se tiene, F = dF = dA = g (H z )Ddz = o H 1 gDH 2 2

10.4.1 El ba met o. Los inst umentos usados a a medi la esin son el ba met o y el manmet o. El ba me t o de me cu io, inventado en 1643 o To icelli (que fue alumno de Galileo) es un tubo ce ado en uno de sus ext emos que se llena con me cu io (Hg) y des us s e da vuelta y se int oduce en ot o envase lleno tambin con me cu io (figu a 10.6a ). En este oceso, el me cu io del tubo desciende o lo que en su ext emo ce ado se oduce un vaco, donde la esin es ce o. Po la esin de la atmsfe a sob e la su e ficie lib e del envase, la columna de me cu io dent o del tubo se eleva; al nivel del ma en condiciones no males, se encuent a que siem e la columna d e me cu io en el tubo es de 76 cm. De la ecuacin hid osttica integ ada se obtiene o = gh, donde es la densidad del me cu io y h su altu a. Con g = 9.8 m/s2 y la d ensidad del me cu io que es 13595 kg/m3, se obtiene que la esin atmosf ica en co ndiciones no males es o = 1.013x105 Pa. El manmet o es un tubo en U abie to a la atmsfe a en uno de sus ext emos, que contiene un lquido y en el ot o ext emo se c onecta a un sistema de esin desconocida (figu a 10.6b). La esin se llama e sin absoluta y la dife encia de esin o = gh se llama esin manomt ica. 280

Como la esin aumenta con la ofundidad, las ando su es eso con la ofundidad.

e esas se deben const ui aument

Ca . 10. Mecnica de fluidos. Figu a 10.6a Ba met o de me cu io. Figu a 10.6b Manmet o 10.5 LEY DE PASCAL. Segn la ecuacin hid osttica, la esin en un fluido slo de ende de la ofundidad, lo tanto cualquie va iacin de esin en su e ficie se t ansmite a cualquie a te del fluido. Entonces si se a lica una fue za F1 sob e un ea A1 como se ve en la figu a 10.7, la misma esin se t ansmite con una fue za F2 sob e un ea A2, y o la definicin de esin: =

F1 F2 = A1 A2 (10.4) F1 A1 F2 A2 Figu a 10.7 Las he amientas hid ulicas tales como f enos, gatas y elevado es de ca ga a ove chan este inci io descubie to o Blas Pascal y se conoce como Ley de Pascal. 281

Ca . 10. Mecnica de fluidos. Ejem lo 10.2. En un elevado de ca ga el ai e com imido eje ce una fue za sob e un equeo mbolo de ea ci cula de 5 cm de adio, que se t ansmite o agua a ot o mbolo de 20 cm de adio. Calcula la fue za que se debe eje ce al ai e com imi do a a levanta un auto de 10000 N y la esin que eje ce a esa fue za. Solucin: o la ley de Pascal, tenemos

= 10 4 Pa = 790hPa 2 A1 (0.05 m )

10.6 PRINCIPIO DE ARQUIMEDES. Una consecuencia de la ecuacin hid osttica es el inci io de A qumedes. Su ongamos que un objeto se sume ge en un fluido como se ve en la figu a 10.4. Antes de su me gi el objeto, el fluido est en equilib io, o lo tanto el esto del fluido e je ce una fue za sob e la o cin de fluido que des us ocu a el objeto, que iguala el eso de la o cin de fluido. Esta fue za tambin actua sob e el objeto sume gido y se conoce como fue za de em uje. El inci io de A qumedes se enuncia como sig ue: cualquie cue o total o a cialmente sume gido en un fluido es em ujado haci a a iba o una fue za que es igual al eso del volumen de fluido des lazado o el cue o. Cualquier cuerpo i merso e u fluido es empujado siempre verticalme te hacia arriba por el fluido, a esa fuerza se le llama fuerza de empuje (o de flotaci ), E. Seg el pri cipio de Arqumedes, la mag itud de la fuerza de empuje es igual al peso del volume de fluido desalojado por el objeto. La fuerza 282

 

F1 625 N = = 7.9

A1 52 F1 = F2 = 10000 A2 20 2 F1 = 625 N Nota que el valo de la fue za necesa ia, equivalente a la eje cida o una masa de 62.5 kg, es equea com a ada con la ca ga a levanta .

Cap. 10. Mec ica de fluidos. de empuje acta verticalme te hacia arriba y su l ea de acci pasa por el pu to do d e se e co traba el ce tro de gravedad del fluido desplazado. Se puede demostrar que la fuerza de empuje es igual al peso. E efecto, la presi e el fo do de u cubo de fluido imagi ario i merso e el fluido, como se ve e la figura 10.4, es mayor que e la parte superior por la ca tidad gz, donde z es la altura del cubo d e fluido imaginario. Esta diferencia de presin por unidad de rea A, es decir la di ferencia entre las fuerzas aplicadas en la cara inferior y superior del volumen hipottico, es igual a la fuerza de empuje E, entonces: F1 - F2 = E Pero F1 = p1A y F2 = p2A Por la ecuacin hidrosttica: (p1 - p2)A = E p1 - p 2 = g z g z A = E E=P E = gV = mg (10.5) Para un objeto que flota sobre un fluido, la fuerza de empuje equilibra al peso del objeto. Si V es el volumen de fluido desplazado al sumergir el cuerpo en el fluido de densidad , y Vo es el volumen del cue o de densidad o, la fue za de em uje del fluido, segn la ecuacin ante io , es E = Vg, que es de igual magnitud al e so del cue o P = mg = oVo g, entonces: E = P Vg = oVo g V = o Vo (10.6) Esta ecuacin e mite dete mina la f accin de volumen de un objeto sume gido en un fluido de mayo densidad que la del objeto. 283

 

 

Ca . 10. Mecnica de fluidos. Ejem lo 10.3. Calcula la f accin del volumen de un cubo de hielo que sob esale d el nivel de agua, cuando flota en un vaso con agua. Solucin: el hielo flota sob e el agua o que tiene una densidad meno que el agua , hielo = 917 kg/m3. El eso del cubo de hielo es Ph = mhg = hVh g. La fue za de e m uje igual al eso del agua des lazada es E = aVg, donde V es el volumen de la a te del cubo de hielo bajo el agua. Como Ph = E, entonces la f accin de hielo su me gido es: Vg = hVh g = aVg V 917 = = 0.917 Vh 1000 V = h Vh a Po lo tanto, lo que sob esale del agua es: 1 V/Vh = 1 0.917 = 0.083 u 8.3%. Una co ona de o o. He n II, ey de Si acusa, idi un da a su a iente A qumedes, que com oba a si una co ona que haba enca gado a un o feb e local e a ealmente de o o u o. El ey le idi tambin de fo ma ex esa que no daase la co ona. A qumedes d io vueltas y vueltas al oblema sin sabe cmo ataca lo, hasta que un da, al mete se en la bae a a a da se un bao, se le ocu i la solucin. Pens que el agua que se de sbo daba tena que se igual al volumen de su cue o que estaba sume gido. Si meda el agua que ebosaba al mete la co ona, conoce a el volumen de la misma y a cont inuacin od a com a a lo con el volumen de un objeto de o o del mismo eso que la co ona. Si los volmenes no fuesen iguales, se a una ueba de que la co ona no e a de o o u o. A consecuencia de la excitacin que le odujo su descub imiento, A qumedes sali del bao y fue co iendo desnudo como estaba hacia el alacio g itando : Lo encont ! Lo encont !, (Eureka, Eureka). La palabra griega "Eureka!" utilizada por Arqumedes, ha quedado desde e to ces com o u a expresi que i dica la realizaci de u descubrimie to. Al llevar a la prctic a lo descubierto, se comprob que la coro a te a u volume mayor que u objeto de oro de su mismo peso. Co te a plata que es u metal me os de so que el oro. 284

 

 

  

 

 

Cap. 10. Mec ica de fluidos. 10.7 N CI NES ELEMENTALES DE DINAMICA DE FLUID S. Ahora a alizaremos e forma muy eleme tal el comportamie to de los fluidos e mo vimie to. Cua do u fluido est e movimie to, el flujo se puede clasificar e dos tipos: a) Flujo estacio ario o lami ar si cada partcula de fluido sigue u a tray ectoria u iforme y estas o se cruza , es u flujo ideal. Por ejemplo el humo de cigarrillo justo despus de salir del cigarro es lami ar. E el flujo estacio ari o la velocidad del fluido perma ece co sta te e el tiempo. Sobre u a velocidad crtica, el flujo se hace turbule to. b) Flujo turbule to es u flujo irregular co regio es do de se produce torbelli os. Por ejemplo el humo de cigarrillo e l a parte superior alejada del cigarro es turbule to. El flujo lami ar se vuelve t urbule to por efecto de la fricci que tambi est prese te e los fluidos y surge c ua do u objeto o capa del fluido que se mueve a travs de l desplaza a otra porci de fluido; lo otas por ejemplo cua do corres e el agua. La fricci i ter a e u fluido es la resiste cia que prese ta cada capa de fluido a moverse respecto a otra capa. La fricci i ter a o roce de u fluido e movimie to se mide por u c oeficie te de viscosidad. Por efecto de la viscosidad parte de la e erga ci tica d el fluido se tra sforma e e erga trmica, similar al caso de los slidos. Debido a q ue el movimie to de u fluido real es muy complejo, co sideraremos u modelo de fluido ideal co las siguie tes restriccio es: fluido i compresible, es decir de de sidad co sta te, o viscoso, flujo estacio ario e irrotacio al, e este ltimo caso se refiere a la rotaci de cada partcula de fluido y o del fluido como u t odo, que puede te er u a trayectoria curva o girar. 10.8 ECUACI N DE C NTINUIDAD.

La trayectoria seguida por u a partcula de fluido estacio ario se llama l ea de co rrie te, as que por defi ici la velocidad es siempre ta ge te a la l ea de corrie te e cualquier pu to. Por lo ta to las l eas de corrie te o se puede cruzar, s i o e el pu to de cruce, la partcula de fluido podra irse por 285

 

    

  

 

 

 

 

  

 

   

       

  

 

Cap. 10. Mec ica de fluidos. cualquiera de las l eas y el flujo o sera estacio ario. U co ju to de l eas de co rrie te forma u tubo de corrie te o de flujo (figura 10.8), las partculas de flu ido se puede mover slo a lo largo del tubo, ya que las l eas de corrie te o se c ruza . Figura 10.8 Figura 10.9 Co siderar u fluido que se mueve a lo largo de u tubo de corrie te, cuya secci tra sversal aume ta e direcci del flujo, como e la figura 10.9. E u i terva lo t en la seccin ms angosta del tubo de rea A1, el fluido se mueve una distancia x1 = v1t. La masa contenida en el volumen A1x1 es m1 = 1A1x1. De manera similar, en la s eccin ancha del tubo de rea A2, se obtienen expresiones equivalentes en el mismo t, cambiando el subndice 1 por 2. Pero la masa se conserva en el flujo estacionario , esto es la masa que cruza por A1 es igual a la masa que pasa por A2 en el inte rvalo de tiempo t, entonces: m1 = m2 1 A1 x1 = 2 A2 x 2 1 A1v1 t = 2 A2 v 2 t 1 A1v1 = 2 A2 v2 (10.7) 286

  

  

  

Ca . 10. Mecnica de fluidos. Esta se llama ecuacin de continuidad, e esenta la conse vacin de la masa: signif ica que la masa no uede se c eada ni dest uida, slo se uede t ansfo ma , simil a a la conse vacin de la ene ga. Pa a un fluido incom esible, es deci de densid ad constante, la ecuacin de continuidad se educe a: A1v1 = A2 v 2 = cte , esto es, el oducto del ea o la a idez no mal a la su e ficie en todos los untos a lo la go del tubo de co iente es constante. La a idez es mayo (meno ) donde el tubo es ms angosto (ancho) y como la masa se conse va, la misma cantida d de fluido que ent a o un lado del tubo es la que sale o el ot o lado, en e l mismo inte valo de tiem o. La cantidad Av, que en el SI tiene unidades de m3/s , se llama flujo de volumen o caudal Q = Av. Ejem lo 10.4. Un ja dine o est egando el astito con una mangue a de 2 cm de dime t o, o la que uede flui 30 lt de agua en un minuto. Calcula la a idez con la cual el agua sale de la mangue a. Solucin: De los datos, el caudal de agua es Q = 30 lt/min, t ansfo mando las unid ades, se obtiene: 30 10 3 cm 3 cm 3 lt Q = 30 = = 500 min 60 s s el ea de la sec cin t ansve sal de la mangue a es: A = 2 = (1cm)2 = cm2

287

Po lo tanto, la a idez de salida del agua o s = 160 = 1.6 Q = Av v = = 2 s s A cm

la mangue a se : cm m Q 500 cm 3

Ca . 10. Mecnica de fluidos. 10.9 ECUACION DE BERNOULLI. Cuando fluye el fluido o un tubo de seccin t ansve sal no unifo me y de un nive l a ot o, o la ecuacin hid osttica, la esin cambia a lo la go del tubo (figu a 10.10). La fue za de la esin 1 en el ext emo infe io del tubo de ea A1 es F1 = 1 A1. El t abajo ealizado o esta fue za sob e el fluido es W1 = F1x1 = p1A1x 1 = p1V, donde V es el volumen de fluido considerado. De manera equivalente en el nivel superior, si se considera un mismo intervalo de tiempo, el volumen V de flu ido que cruza la seccin superior de rea A2 es el mismo, entonces el trabajo es W2 = - p2A2x1 = - p2V. El trabajo neto realizado por las fuerzas en el intervalo de t iempo t es: W = W1 + W2 = ( p1 p 2 )V Figura 10.10 Parte de este trabajo se usa en cambiar tanto la energa cintica como la energa pote ncial gravitacional del fluido. Si m es la masa que pasa por el tubo de corriente en el tiempo t, entonces la variacin de energa cintica es: Ec = 1 1 2 mv 2 mv12 2 2 y la variacin de energa potencial gravitacional es: 288

Cap. 10. Mecnica de fluidos. E g = mgy 2 mgy1 Por el teorema del trabajo y energa se tiene: W = E c + E g ( p1 p 2 )V = 1 1 2 mv 2 mv12 + mgy 2 mgy1 2 2 Dividiendo por V y como = m/V, se obtiene la ecuacin de Bernoulli para un fluido no viscoso, incompresible, estacionario e irrotacional. p1 p 2 = p1 + 1 2 1 2 v 2 v1 + gy 2 gy1 2 2 1 2 1 2 v1 + gy1 =

La ecuacin de Be noulli, que es un esultado de la conse vacin de la ene ga a licad a a un fluido ideal, gene almente se ex esa como: 1 + v 2 + gy = cte. 2

(10.8) Ejem lo 10.5: Demost a que a a un fluido en e oso se obtiene la ecuacin hid os ttica integ ada. Solucin: si el fluido est en e oso, v1 = v2 = 0 y de la ecuacin de Be noulli se obtiene: 289

2 + v 2 + gy 2 2 2

Ca . 10. Mecnica de fluidos. 1 + gz1 = 2 + gz 2 p1 p 2 = gz 2 gz1 = gh p 2 = p1 gh Ejem lo 10.6: Tubo de Ventu i. Una tube a ho izontal con una est echez, como se m uest a en la figu a 10.11, que se usa a a medi la velocidad del flujo en fluid os incom esibles, se llama tubo de Ventu i. Si con un manmet o se mide la esin en los untos 1 y 2, se uede calcula la a idez del flujo que sale (o ent a) o el tubo. Figu a 10.11 Tubo de Ventu i. Solucin. A licando la ecuacin de Be noulli, como la tube a es ho izontal, y1 = y2, se tiene:

1 1 2 1 + v12 + gy1 = 2 + v 2 + gy1 2 2 1 1 2 p1 + v12 = 2 + v 2 2 2 Con la ec in de continuidad: A1v1 = A2 v 2 v1 = Combinando las ecuaciones, queda: A2 v2 A1 290

Cap. 10. Mecnica de fluidos.

1 A 1 2 p1 + 2 v 2 = p 2 + v 2 2 A1 2 v2 = A1 2( p1 p 2 ) 2 A12 A 2 ( )

1 1 2 1 + v12 + gy1 =

2 + v 2 + gy 2 2 2 po + 1 2 v1 =

+ g ( y 2 y1 ) 2

A2 2= h y2 y1 1= o A1 v1 Como h = y2 y1, se tiene: Figu a 10.12 Ejem lo 10.7 291

Obse va que debido a que A1 > A2, 1 > 2, la esin en 1 es mayo que en 2, es deci la esin disminuye en la a te est echa de la tube a. La disminucin de la esin en la a te angosta del tubo tiene va ias a licaciones, o ejem lo, conecta ndo un tubo de Ventu i al ca bu ado de un automvil, se hace ent a el va o de g asolina a la cma a de combustin. Ejem lo 10.7: Ley de To icelli. Un estanque que contiene un lquido de densidad t iene un o ificio equeo en un lado a una altu a y1 del fondo (figu a 10.12. El ai e o encima del lquido se mantiene a una esin . Dete mina la a idez con la cual sale el lquido o el o ificio cuando el nivel del lquido est a una altu a h s ob e el aguje o. Solucin: si se su one que el estanque tiene una su e ficie mucho mayo que la del aguje o (A2 >> A1), entonces la a idez de descenso del fluido es mucho meno que la a idez de salida del agua o el hoyo (v2 << v1). A lica ndo la ecuacin de Be noulli en los untos 1 y 2, con 1 = esin atmosf ica = o y 2 = , se tiene:

Ca . 10. Mecnica de fluidos. v1 = 2( + 2 gh Esta ecuacin se llama Ley de To icelli. Casos a ticula es: a) Si ces 2gh ~ 0 y v1 = es solo funcin de la esin. 2 , esto significa que la a idez >> o, enton

b) Si = o, entonces v1 = 2 gh , en este caso la a idez es idntica a la adqui ida o un cue o en cada lib e. Ejem lo 10.8: Tubo de Pitot. Es uno de los medido es ms exactos a a medi la a idez de un gas dent o de una tube a. El equi o, que se muest a en la figu a 10.13 , consta de un tubo en U con un lquido manomt ico, donde la ama a e la figura 10.1 3, se co ecta a la tubera y la otra rama b, cuya abertura est dirigida corrie te arr iba, se deja e el i terior por do de circula el gas co rapidez v, de modo que el fluido i grese de tro de sta y suba hasta que la presi aume te lo suficie te d e tro del mismo y equilibre el impacto producido por la velocidad. Figura 10.13.Tubo de Pitot. 292

 

o )

Cap. 10. Mec ica de fluidos. La presi pa e la rama a izquierda del tubo, cuya abertura es paralela al movimi e to del gas, es igual a la presi del gas. La presi pb de la otra rama b puede c alcularse aplica do la ecuaci de Ber oulli a los pu tos e a y b, que se co side ra ubicados a u a misma altura de tro de la tubera. Como la rapidez e el pu to b es ula: pb = pa + 1 2 v 2 donde es la densidad del gas. Po ot a a te, la b > a o lo que el lquido man omt ico dent o del tubo en U se des laza o iginando una dife encia de altu a h. S ea o la densidad del lquido manomt ico, o lo que: b = a + o gh combinando ambas ecuaciones, se obtiene: o gh = v 2 2 o gh 1 2 v= Los aviones usan sistemas basados en este equi o a a dete mina su velocidad e s ecto al ai e. 293

 

 

 

 

Ca . 10. Mecnica de fluidos. PROBLEMAS. 10.1 Ma ia se enca icho o tene un a de a os de o o esf icos, g andes y slidos de 1.25 cm de adio y le idi a su novio que se los com a a. Cuntos g amos de o o ten d n que so o ta sus o ejas o ca ichosa? R: 158 g c/u. En el cent o de un cic ln (esto es un ea de bajas esiones donde se cie a una isoba a, que es una lnea que une untos de igual esin), se midi un descenso de esin de 10 mm de Hg es e cto a la no mal, cunto fue la esin atmosf ica? Po el cont a io, en un anticicln (c ent o de altas esiones) el aumento de esin fue de 12 hPa, a cunto ascendi la col umna de me cu io? R: 1006 hPa, 76.9 cm de Hg. Hace las su osiciones necesa ias a a calcula la densidad media y la masa de la atmsfe a conside ando que el 90% de la masa de ai e est debajo de la t o o ausa. R: a ox. 5.3x1018 kg. Calcula l a densidad del ncleo de un tomo. La masa de un otn es de 1.6x10 27 kg y su adio es del o den de 10 15 m. Qu sugie e este esultado en elacin con la est uctu a de la mate ia? R: 4x1017 kg/m3. Paula de 50 kg se balancea sob e uno de los altos t acones de sus za atos. Si el tacn es ci cula de adio de 0.5 cm, qu esin eje ce P aula sob e el iso? R: 6.36x106 Pa. Una iscina tiene una su e ficie de 30 m x 1 0 m y un fondo lano. Cuando la iscina est llena a una ofundidad de 2 m con ag ua, cul es la fue za total eje cida o el agua sob e el fondo? Sob e cada lado? An aliza la utilidad ctica de usa un ba met o de agua. El tubo ve tical abie to d e la figu a 10.14 contiene dos fluidos de densidades 1 y 2, que no se mezclan. Dem uest e que la esin en el fondo del tubo est dada o la ex esin = o +g(1h1 + 2h 2). 10.2 10.3 10.4 10.5 10.6 10.7 10.8 294

Ca . 10. Mecnica de fluidos. 10.9 Un tubo en U abie to en ambos ext emos se llena a cialmente con agua. Des us se hecha aceite de densidad ac en el b azo de echo del tubo, fo mando una columna de altu a h1, (figu a 10.15). Calcula la dife encia h en las altu as de las dos s u e ficies de lquido. R: h1(1 ac/ag). 10.10 Un tubo en U de ea de seccin t ansve sal constante, abie to a la atmsfe a, s e llena a cialmente con me cu io. Des us se hecha agua en ambos b azos. Si la co nfigu acin de equilib io del tubo es como la most ada en la figu a 10.16, con h2 conocido, dete mine el valo de h1. R: h2(M/a 1). h1 h2 Figu a 10.14. Figu a 10.15 Figu a 10.16 10.11 Demost a que el 11% del volumen de un tm ano de hielo sob esale de la su e ficie del ma . 10.12 Calcula la densidad de una boya de lstico de adio R, si flota en agua de ma , con 2/3 de su volumen sob e el agua. 10.13 a) Calcula la altu a sob e el nivel del agua de un cubo de made a de 10 cm de lado y densidad 650 kg/m3 que flota en el agua. b) Calcula la cantidad de masa se debe one so b e el cubo a a que se hunda justo hasta el nivel de agua. R: a) 4 cm, b) 400 g . 10.14 Una elota esf ica de lstico flota en el agua con 50% de su volumen sume gido. Esta misma elota flota en aceite con 40% de su volumen 295

Ca . 10. Mecnica de fluidos. sume gido. Dete mine las densidades del aceite y de la elota. R: ac=1250 kg/m3, e sf=500 kg/m3. 10.15 Heidi buscando bichos a a Sanidad, encont una ana y la us o en un tazn lstico semiesf ico de adio 5 cm, que luego hizo flota en la laguna Los Patos, quedando el tazn flotando justo a as del agua (figu a 10.17). Su onga que el agua tiene una densidad de 1050 kg/m3 y que la masa de la ana es 100 g , calcula la densidad del tazn. R: 667 kg/m3. Figu a 10.17. 10.16 Un bloque de metal de 10 kg de dimensiones 12cmx10cmx10cm, se sus ende de una balanza y se sume ge en agua. El lado de 12cm est ve tical y la a te su e io del bloque sob esale 5 cm de la su e ficie del agua. Calcula : a) la fue za de tensin de la balanza, b) la fue za de em uje sob e el bloque. R: a) 93 N, b) 7 N . 10.17 Calcula el ea de una tabla de fib a de vid io de es eso H y densidad , cuando flota en el ma con un nadado de masa M sob e la tabla. R: M/H(ag ). 10. 18 Calcula la fue za a a mantene com letamente sume gida en agua de densidad a a una elota de ing ong de adio R y densidad a/12.5. R: 3.85ga R3. 10.19 El es tanque a alele i oidal de la figu a 10.18 se llena con agua hasta 2 m de ofun didad. En la a te infe io de una a ed del estanque hay una escotilla ectangu la de 1 m de alto y 2 m de ancho, a ticulada en su a te su e io . Calcula : a) la fue za neta sob e la escotilla, b) el to que eje cido al ededo de las bisag as. 296

Ca . 10. Mecnica de fluidos. 10.20 a) Demost a que la fue za esultante sob e una a ed ve tical de un estan que cbico de ancho D lleno con agua de H m de ofundidad es gDH2. b) Demost a que el to que total eje cido o el agua sob e la a ed del estanque, en un eje que asa o la base de la a ed, es (1/6)gDH3 y que la lnea de accin efectiva de la f ue za total eje cida o el agua est a una distancia de 1/3 H sob e el eje. 10.21 Un mosquito que choc con el estanque del oblema 10.20, le hizo un aguje o en u n unto a 1.25 m debajo del nivel su e io de agua, o el cual se ha medido un flujo de agua de 60 lt/min. Calcula : a) la a idez de salida del agua, b) el a dio del aguje o. R: a) 5 m/s, b) 0.8 cm 10.22 El suminist o de agua llega al niv el del suelo o una cae a de 5 cm de dimet o. Una llave de 2 cm de dimet o ubicada a 15m de altu a llena un envase de 20lt en un minuto. Calcula : a) la a idez co n la que sale el agua de la llave, b) la esin en la cae a inci al. R: a) 1m/s, b) 2.5 o. 10.23 Un estanque de agua tiene un equeo aguje o en su costado a una a ltu a h debajo del nivel de agua, o donde sale agua con un flujo de Q lt/min ( figu a 10.19). Calcula : a) la a idez con la que sale el agua o el aguje o, b ) el dimet o del aguje o. R: a) 2 gh , b) 2.2x10 3 Q gh . h Figu a 10.18. Figu a 10.19. 10.24 En un estanque con agua de 2 m de ofundidad, se hace un aguje o de 5 cm2 a una altu a h desde la su e ficie de agua (figu a 10.19). Po la a te su e io del estanque se le hecha agua con un flujo continuo de 297

Ca . 10. Mecnica de fluidos. 1000 cm3/s de mane a que el nivel de agua e manece constante en 2 m. Calcula a ) la altu a h, b) la a idez de salida del agua. R: a) 0.2 m, b) 2 m/s. 10.25 Po una mangue a de incendios de 6 cm de dimet o, fluye agua a azn de 600 lt/min. C alcula la a idez de salida del agua de la mangue a si el dimet o o donde sale es 2 cm. 10.26 Po una tube a ho izontal fluye agua con una a idez de 5 m/s. Si la esin es de 1.5x105 Pa en un unto donde la seccin t ansve sal del tubo es A, dete mine en un unto donde el ea es A/3: a) la a idez y b) la esin de salida del agua. R: a) 15 m/s, b) 0.5x105 Pa. 10.27 Un globo esf ico de adio 0.4 m, in flado con helio se ama a a una cue da unifo me de 2 m de la go y de kg de masa. Calcula la altu a de equilib io que se eleva el globo des us que se suelta. R: 1.9 m. 10.28 Un globo inflado con helio, de densidad 0.18 kg/m3, se eleva hasta una altu a de 8 km en la atmsfe a cuya densidad disminuye con la altu a en la fo ma a = o e z/8, donde z est en km y o = 1.29 kg/m3 en su e ficie. Calcula el volume n de helio con el que se debe infla el globo a a que ueda lleva una ca ga de 400 kg hasta los 8 km de altu a, su oniendo que su volumen se mantiene constant e. R: 1430 m3. 10.29 Su oniendo la atmsfe a y el ocano homogneos, esto es de densid ad constante, calcula la esin en dife entes niveles de altu a en la atmsfe a ha sta un km de altu a y en el ocano hasta 100 m de ofundidad. Hace el g fico ve sus z. 10.30 Dete mina la va iacin de esin con la altu a en la atmsfe a sob e C once cin, conside ando que la tem e atu a disminuye linealmente con la altu a has ta el nivel de la t o o ausa, en la fo ma T = To z, donde To es la temperatura en zo = 0 y = 9.8 C/km, se llama gradiente adiabtico seco de temperatura. Sugeren cia: considerar la atmsfera formada por aire seco como un gas ideal. z R: p ( z ) = po 1 T o g Rd , con Rd la constante especfica del aire seco. 298

Cap. 11. Movimiento oscilatorio. CAP TUL 11. M V M ENT SC LAT R . Los principales objetivos de los captulos an teriores estaban orientados a describir el movimiento de un cuerpo que se puede predecir si se conocen las condiciones iniciales del movimiento y las fuerzas ex ternas que actan sobre el cuerpo. Si una fuerza cambia en el tiempo, la velocidad y la aceleracin del cuerpo tambin cambiarn en el tiempo. Un tipo de movimiento par ticular ocurre cuando sobre el cuerpo acta una fuerza que es directamente proporc ional al desplazamiento del cuerpo desde su posicin de equilibrio. Si dicha fuerz a siempre acta en la direccin de la posicin de equilibrio del cuerpo, se producir un movimiento de ida y de vuelta respecto de esa posicin, por eso a estas fuerzas s e les da el nombre de fuerzas de restitucin, porque tratan siempre de restituir o llevar al cuerpo a su posicin original de equilibrio. El movimiento que se produ ce es un ejemplo de lo que se llama movimiento peridico u oscilatorio. Ejemplos d e movimientos peridicos son la oscilacin de una masa acoplada a un resorte, el mov imiento de un pndulo, las vibraciones de las cuerdas de un instrumento musical, l a rotacin de la Tierra, las ondas electromagnticas tales como ondas de luz y de ra dio, la corriente elctrica en los circuitos de corriente alterna y muchsimos otros ms. Un tipo particular es el movimiento armnico simple. En este tipo de movimient o, un cuerpo oscila indefinidamente entre dos posiciones espaciales sin perder e nerga mecnica. Pero en los sistemas mecnicos reales, siempre se encuentran presente fuerzas de rozamiento, que disminuyen la energa mecnica a medida que transcurre e l tiempo, en este caso las oscilaciones se llaman amortiguadas. Si se agrega una fuerza externa impulsora de tal manera que la prdida de energa se equilibre con l a energa de entrada, el movimiento se llama oscilacin forzada. 11.1 M V M ENT ARM N C S MPLE. Una partcula que se mueve a lo largo del eje x, tiene un movimiento armnico simple cuando su desplazamiento x desde la posicin de equilibrio, vara en el tiempo de acuerdo con la relacin 299

  

  

 

 

 

Cap. 11. Movimiento oscilatorio. x = A cos(t + ) (11.1) on e A, , y son constantes el movimiento. Esta es una ecuacin peri ica y se repit e cuan o t se incrementa en 2 adianes. Pa a da un significado fsico a estas const antes, es conveniente g afica x en funcin de t, como se muest a en la figu a 11. 1. La constante A se llama am litud del movimiento, es sim lemente el mximo des l azamiento de la a tcula, ya sea en la di eccin ositiva o negativa de x. La const ante se llama f ecuencia angula , el ngulo se llama ngulo o constante e fase, y j unto con la amplitu que an etermina os por el esplazamiento y veloci a inici al e la partcula. Las constantes A y nos icen cual era el esplazamiento en el instante t = 0. La canti a (t + ) se llama la fase el movimiento y es e utili a en la comparacin el movimiento e os sistemas e partculas. Figura 11.1 Esquema el grafico posicin tiempo e la ecuacin 11.1.

El perio o T es el tiempo que emora la partcula en completar un ciclo e su movi miento, esto es, es el valor e x en el instante t + T. Se pue e emostrar que e l perio o el movimiento esta a o por T = 2/, sabiendo que la fase aumenta 2 adia nes en un tiem o T: t + + 2 =(t+T) + Comparan o, se concluye que T = 2, o T= 2 300

Ca . 11. Movimiento oscilato io. Al inve so del e iodo se le llama f ecuencia f del movimiento. La f ecuencia e esenta el nme o de oscilaciones que hace la a tcula en un e iodo de tiem o, se esc ibe como: f = 1 = T 2

Las unidades de medida de f en el SI son 1/s o ciclos/s, llamados He tz, Hz. Rea comodando la ecuacin de la f ecuencia, se obtiene la f ecuencia angula , que se m ide en ad/s, de valo : = 2f = 2 T

(11.2) La aceleracin (11.3) Como x = A cos(t + ) , se pue e expresar la aceleracin en la forma: a = 2x De las ecuaciones e veloci a y e aceleracin, tenien o en cuenta que los valores extre mos e las funciones seno o coseno son 1, sus valores extremos mximos o mnimos son : v = A 301 e la partcula est a a por v/ t: a= v = 2 A cos(t + ) t

La velocidad de una a tcula que tiene un movimiento a mnico sim le se obtiene de ivando es ecto al tiem o la ecuacin 11.1: v= dx = A sin(t + ) t

Cap. 11. Movimiento oscilatorio. a = 2A Las curvas e posicin, veloci a y aceleracin con el tiempo se muestran en la figu ra 11.2. En estas curvas se ve, (figura 11.2 b), como la fase e la veloci a i fiere en /2 ad o 90 con la fase del des lazamiento. Esto es, cuando x es un mximo o un mnimo, la velocidad es ce o. De igual fo ma, cuando x es ce o, la a idez es un mximo o un mnimo. Del mismo modo, como la fase de la acele acin difie e en ad o 180 con la fase del des lazamiento, (figu a 11.2 c), cuando x es un mximo o un mn imo, la acele acin es un mnimo o un mximo. Figu a 11.2. G ficos de osicin, velocidad y acele acin en funcin del tiem o. La ecuacin x = A cos(t + ) es una solucin general e la ecuacin iferencial que esc ribe el movimiento armnico simple, on e la constante e fase y la amplitu A se eben elegir para satisfacer las con iciones iniciales el movimiento. La consta nte e fase es importante cuan o se quiere comparar el movimiento e os o ms par tculas oscilantes. Suponien o que se conocen la posicin inicial y la veloci a ini cial e un oscila or, esto es, en t = 0, x = xo y v = vo. Con esas con iciones, las ecuaciones 11.1 y 11.2 se re ucen a: xo = A cos y 302

vo =

A sen

Cap. 11. Movimiento oscilatorio. que son os ecuaciones e on e se pue en calcular los valores e la constante e fase y la amplitu A. Divi in olas, se obtiene: vo = tan xo tan = vo xo Si ahora las ecuaciones para xo y vo se elevan al cua ra o y se suman, se obtien e: 2 xo v + o = A2 (cos 2 + sen 2 ) v + o 2 2 A= 2 xo Entonces se observa que y A se pue en conocer si se especifican las con iciones iniciales xo, y vo. Para concluir esta escripcin, po emos resumir algunas propie a es e una partcula que se mueve con un movimiento armnico simple: 1. El esplaz amiento, la veloci a y la aceleracin varan senoi almente con el tiempo, pero no s e encuentran en fase. 2. La aceleracin e la partcula es proporcional al esplazam iento, pero en ireccin opuesta. 3. El perio o y la frecuencia son in epen ientes e la amplitu . Ejemplo 11.1. Una partcula oscila con un movimiento armnico simple a lo largo el eje x. Su esplazamiento vara con el tiempo e acuer o con la ecuacin: 303

Cap. 11. Movimiento oscilatorio. x = 4 cos t + 4 donde x se mide en m, t en s y los ngulos en radianes. Calcular: a) la amplitud, frecuencia y periodo del movimiento, b) la velocidad y la aceleracin de la partcul a en cualquier instante t, c) la posicin, la velocidad y la aceleracin en el insta nte t = 1s, d) la velocidad y la aceleracin mximas de la partcula, e) el desplazami ento entre t = 0 y t = 1s, f) la fase del movimiento en t = 2s. Solucin: a) compa rando la ecuacin dada con la ecuacin general del movimiento, se encuentra que A = 4m, = ad, la f ecuencia y el e iodo son: f = b) v= 1 = = = 0.5 Hz T 2 2 , T= 1 = = 2s f 0.5 dx = 4sen t + 4 dt a= c) para t = 1 s, dv = 4 2 cos t + 4 dt

5 x = 4 cos + = 4 cos = 4(0.707) = 2.83m 4 4 5 v = 4sen = 4 (0.707) = 8.9 m s 4 5 a = 4 2 cos 4 2 = 4 (0 2 d) los valores mximos se obtienen cuando seno y coseno valen uno: vmax = 4 =12.6 m /s 304

Ca . 11. Movimiento oscilato io. amax = 42 = 39.5 m/s2 e) a a t = 0 y t = 1s, xo = 4 cos 0 + = 4 cos = 4(0.707) = 2.83m 4 4 x1 = -2.83 m x = x1 xo = -2.83 - 2.83 = -5.66 m f) la fase se define como: fase = t + , para t = 2s y =/4: fase =2 + /4 =9/4 ad. 11.2 MASA SUJETA A UN RESORTE. Una masa sujeta al ext emo de un eso te, con la masa movindose lib emente sob e una su e ficie ho izontal sin f iccin o ve ticalmente en el ai e, oscila si se la a a ta de su osicin de equilib io x = 0 donde el eso te se encuent a sin defo ma , con un movimiento a mnico sim le. En la figu a 11.3 se obse va un esquema a a una masa que oscila sob e una su e ficie ho izontal sin f iccin. Cuando la mas a se des laza una equea distancia x desde su osicin de equilib io, el eso te ej e ce una fue za dada o la Ley de Hooke, F = kx Sabemos que esta fue za siem e es o uesta al movimiento. A licando la segunda ley de Ne ton, su oniendo que e sta es la nica fue za que acta sob e la masa m, se obtiene: F = kx = ma 305

Cap. 11. Movimiento oscilatorio. a= k x m esto es, la aceleracin e m es proporcional al esplazamiento es e su posicin e equilibrio y en ireccin opuesta. Como la aceleracin es la segun a eriva a e la posicin, y efinien o el cuociente k/m = 2, se pue e escribir: 2x = 2 x 2 t (11.4) Figura 11.3 Masa uni a a un resorte oscilan o sobre una superficie horizontal si n roce. La solucin e la ecuacin iferencial 11.4 es la que escribe el movimiento armnico simple, tiene la misma forma que la ecuacin 11.1: x = A cos(t + ) Esto se pue e ge neralizar para afirmar que cualquier fuerza que acte sobre una partcula, que sea l inealmente proporcional al esplazamiento y e ireccin opuesta, le pro ucir a la partcula un movimiento armnico simple. Para la masa sujeta al resorte, el pero o y la frecuencia el sistema es: 306

Cap. 11. Movimiento oscilatorio. T= 2 = 2 m k k m f = 1 1 = T 2

= k 5N /m = = 5 ad s m 0.2 kg T= b) 2 = 2 = 1.26 s 5

c) a a t = 2 s:

a = 2A cost = (5)2 (0.05) cos(10) = 1.25 m/s2 307

x = A cost = 0.05 cos(10) = 0.05 m v =

A sent = (5)(0.05) sen(10) = 0

vmax = A = (5

ad/s)(0.05 m) = 0.25 m/s amax = 2A=(5 ad/s)2 (0.05 m) = 1.25 m/s2

Ejem lo 11.2. Una masa de 200 es lib e de oscila sob e una laza 5 cm del equilib io y se lcula : a) la f ecuencia y el cin mxima de la masa, c) el Solucin: los datos son: m =

g se conecta a un eso te de constante 5 N/m, que su e ficie ho izontal sin oce. Si la masa se des suelta desde el e oso, como en la figu a 11.3, ca e iodo del movimiento, b) la a idez y la acele a des lazamiento, la a idez y la acele acin a a t = 2 s. 0.2 kg, k = 5 N/m, A = 0.05 m, vo = 0, a)

Ca . 11. Movimiento oscilato io. 11.3 ENERGIA EN EL MOVIMIENTO ARMONICO SIMPLE. De la definicin de ene ga cintica, eem lazando la ecuacin de la a idez de una a tc ula con movimiento a mnico sim le, se obtiene: 1 1 Ec = mv 2 = m 2 A2 sen 2 (t + ) 2 2 (11.5) La energa potencial elstica almacena a en un resorte, para cualquier eformacin x e s: 1 1 E E = kx 2 = kA2 cos 2 (t + ) 2 2 (11.6) La energa mecnica total en el movimiento armnico simple, consi eran o que 2 = k/m o bien m2 = k, se pue e escribir como: E = Ec + E E = 1 2 kA sen 2 (t + ) + cos 2 (t + ) 2 1 2 kA 2 [ ] (11.7) E= De on e se e uce que la energa mecnica total en el movimiento armnico simple es u na constante el movimiento, proporcional al cua ra o e la amplitu . Este valor es igual a la mxima energa potencial elstica almacena a en un resorte cuan o x = A , ya que en esos puntos v = 0 y no hay energa cintica. Por otro la o, en la posicin e equilibrio, x = 0 y por lo tanto EE= 0, a ems en este punto la rapi ez es la mxima, e tal manera que to a la energa es energa cintica, es ecir en x = 0: 1 2 1 E Ec = mvmax = 2 A2 2 2 308

Cap. 11. Movimiento oscilatorio. Como la superficie sobre la cual oscila el resorte es sin friccin, la energa se co nserva, usan o la ecuacin e conservacin e la energa, se pue e escribir: E = Ec + EE = cte 1 2 1 2 1 2 mv + kx = kA 2 2 2 (11.8) De esta expresin se pue e calcular la rapi ez para un : v= k 2 A x 2 = A2 x 2 m ( ) (11.9) encontrn ose nuevamente que la rapi ez es mxima en x = 0 y es cero en los puntos e regreso el oscila or x = A. Ejemplo 11.3. Una masa e 0.5 kg conecta a a un resorte e constante 20 N/m, osc ila sobre una superficie horizontal sin roce, con una amplitu e 3 cm. Calcular a) la energa total el sistema y la rapi ez mxima e la masa, b) la rapi ez e la masa cuan o el esplazamiento es 2 cm, c) la energa cintica y potencial el siste ma cuan o el esplazamiento es 2 cm, ) el valor e x cuan o la rapi ez es 0.1 m /s. Solucin: los atos son m = 0.5 kg, k = 20 N/m, A = 3 cm = 0.03 m, a) la energa total es: 1 1 E = kA2 = (20)(0.03) 2 = 9 10 3 J 2 2 y la rapi ez mxima se pue e calcular con la ecuacin: E= 1 2 mvmax = 9 10 3 J 2 309 esplazamiento arbitrario x

Cap. 11. Movimiento oscilatorio. vmax 2 9 103 = = 0.19 m s 0.5

b) Ahora se pue e usar la ecuacin: v= k 2 20 A x2 = (0.032 0.02 2 ) = 0.14 m s m 0 5 ( ) los signos positivo y negativo in ican que la masa po ra estarse movien o hacia l a erecha o hacia la izquier a en ese instante. c) la energa cintica es: Ec = y la energa potencial: EE = 1 2 1 kx = (20)(0.02) 2 = 4 10 3 J 2 2 1 2 1 mv = (0.5)(0.14) 2 = 5 10 3 J 2 2 ) e la ecuacin: v= k 2 m A x 2 x = ( ) A2 v 2 m k

11.4 EL PENDULO. 11.4.1. Pndulo simple. El pndulo simple es otro sistema mecnico que tiene un movimiento peridico oscilator io, si se mueve en un medio sin friccin. Un pndulo es un sistema 310

0.5 x =

0.032 (0.1) 2 = 0.025m = 2.5cm 20

Cap. 11. Movimiento oscilatorio. formado por una masa puntual m suspendida en el aire por una cuerda de longitud L, de masa muy pequea comparada con la masa m, por lo que se desprecia; la parte superior de la cuerda se encuentra fija, como se muestra en la figura 11.4. El m ovimiento del pndulo producido por la fuerza de gravedad se realiza en un plano v ertical, y es un movimiento armnico simple si el ngulo ue orma la cuerda del pndu lo con la vertical es pe ueo, como se puede demostrar a continuacin. Figura 11.4 Pndulo simple. Las uerzas ue actan sobre la masa m son la tensin T de la cuerda y el peso mg de la masa, se muestran en la igura 11.4. La componente tangencial del peso, mg s en, siempre apunta hacia = 0, en direccin opuesta la desplazamiento. Esta es la u erza de restitucin, entonces puede escribirse la ecuacin de movimiento en la direc cin tangencial de la orma: d 2s Ft = mgsen m 2 = mgsen dt donde s es el desplazamiento medido a lo largo del arco de trayectoria y el sign o menos indica ue Ft acta opuesta al movimiento. Como s = L y L es constante, la ecuacin se trans orma en: 311

Cap. 11. Movimiento oscilatorio. g d 2 = sen 2 L dt

Como el lado derecho es proporcional a sen, y no solo a , se concluye ue el movim iento no es armnico simple. Esa es una ecuacin di erencial di cil de resolver, por lo ue se supone ue el pndulo se mueve en pe ueos desplazamientos, tal ue es pe ueo, en este caso se puede usar la aproximacin sen y la ecuacin di erencial del mov miento se reduce a: d 2 g = 2 L dt (11.10) ue tiene la misma orma ue la ecuacin ue describe al movimiento armnico simple, por lo ue solo en esas condiciones el movimiento del pndulo es un movimiento ar mnico simple. Su solucin es entonces: = cos(t + ) (11.11) on e es la amplitu que correspon e al mximo esplazamiento angular y es la frec uencia angular, e valor: = El perio o el movimiento es: T= 2 g L = 2 L g El e iodo y la f ecuencia de un ndulo sim le de enden solo de la longitud de la cue da y la acele acin de g avedad, y son inde endiente de la masa m del ndulo. Esto significa que todos los ndulos sim les de igual longitud en el mismo luga , oscila n con el mismo e iodo. 312

Ca . 11. Movimiento oscilato io. Comnmente se usa el ndulo sim le como un medido de tiem o. Tambin es un dis ositi vo adecuado a a hace mediciones ecisas de la acele acin de g avedad, que son im o tantes o ejem lo cuando las va iaciones locales de g ueden da info macin sob e las fuentes subte neas de et leo u ot os ecu sos mine ales. Ejem lo 11.4. Una e sona que anda t ayendo un c onmet o, e o no una huincha a a medi la altu a de un edificio, quie e sabe su altu a. Entonces instala un nd ulo que se extiende desde el techo hasta el iso y mide que tiene un e iodo de 15 s. a) Calcula la altu a de ese edificio. b) Si el mismo ndulo estuvie a en l a Luna, donde g =1.7 m/s2, calcula el e iodo. Solucin: se conoce T = 15 s, ento nces: a) gT 2 (10)(15) 2 L L= = = 57 m T = 2 g 4 2 4 2 b) en la Luna g =1.7 m/s2, T = 2 L 57 = 2 = 36.4s g 1.7 11.4.2. Pndulo fsico. Un ndulo fsico consta de cualquie cue o gido sus endido de un eje fijo que no asa o su cent o de masa. El cue o gido oscila cuando se des laza de su osicin de equilib io. Si el cue o gido se sujeta en un eje que asa o un unto O a una distancia d del cent o de masa, como se muest a en la figu a 11.5, la fue za debido a la g avedad oduce un to que es ecto de O, de magnitud mgd sen. Como el tor ue se escribe = I, donde I es el momento de inerci respecto l eje que p s por O y es l segund deriv d de l r pidez ngul r, se obtiene: 313

C p. 11. Movimiento oscil torio. d 2 I 2 = mgd sen dt Figura 11.5 Pndulo sico.

El signo menos indica ue la uerza de gravedad es una uerza de restitucin ue p roduce un tor ue ue hace disminuir el ngulo . Para resolver esta ecuacin, nuevamen te se supone ue el pndulo sico se mueve en pe ueos desplazamientos, tal ue es pe ueo, en este caso se puede usar la aproximacin sen y la ecuacin di erencial del mo imiento se reduce a: d 2 mgd = = 2 2 I dt (11.12) ue tiene la misma orma ue la ecuacin ue describe al movimiento armnico simple, por lo ue en esas condiciones as es el movimiento del pndulo. Su solucin es enton ces:

= cos(t + ) on e es la amplitu que correspon e al mximo esplazamiento angular y es la frec uencia angular, e valor: = mg I 314

Cap. 11. Movimiento oscilatorio. El perio o el movimiento es: T= 2 = 2 I mgd Se ueden usa estos esultados a a medi el momento de ine cia de cue os gido s lanos. Si se ubica el cent o de masa y se mide d, se uede obtene el momento de ine cia midiendo el e iodo del ndulo fsico. El e iodo del ndulo fsico se uce al del ndulo sim le, cuando toda la masa del cue o gido se concent a en su cent o de masa, ya que en este caso I = md2. Ejem lo 11.5. Una ba a unifo me de masa M y la go L tiene un ivote en uno de s us ext emos, como se muest a en la figu a 11.6, y oscila en un lano ve tical co n una equea am litud. Calcula el e iodo de la oscilacin. Figu a 11.6 Ejem lo 11.5 Solucin: la ba a oscila como un ndulo fsico, su e iodo es: I T = 2 = 2 mgd 1 3 ML2 2L = 2 Mg L 3g 2 315

ed

Ca . 11. Movimiento oscilato io. 11.4.3. Pndulo de to sin. Un ndulo de to sin, es un sistema const uido de la siguiente fo ma. Consta de una va illa ve tical que o el ext emo su e io se fija a un so o te y en el ext e mo o uesto se encuent a unida a un cue o gido, como se muest a en la figu a 11. 7. Cuando a la va illa se la etue ce y luego se la deja gi a , eje ce un to que de estitucin sob e el cue o gido, o o cional al des lazamiento angula , ue se puede escribir como: Figura 11.7 Pndulo de torsin. = donde es la constante de torsin de la varilla de soporte. Por la segunda ley de N e ton, se obtiene: d 2 I 2 = dt d 2 = I dt 2 (11.13)

ue es la ecuacin de un movimiento armnico simple, de frecuencia y periodo, respec tivamente: 316

Cap. 11. Movimiento oscilatorio. = T= 2 I = 2 I Para este pndulo no se tiene la restriccin de un ngulo pe ueo, slo se debe tener cuid ado ue no se exceda el lmite elstico de la varilla. Este pndulo es comn en los relo jes mecnicos, ue produce el tictac, tictac, tictac, tictac... 11.5. OSCILACIONES AMORTIGUADAS. Los movimientos oscilatorios hasta a u considerados se refieren a sistemas ideale s, ue oscilan indefinidamente por la accin de una fuerza lineal de restitucin, de la forma F = x. Pero en los sistemas reales estn presentes fuerzas disipativas , como la friccin, las cuales retardan el movimiento del sistema. Por lo tanto la energa mecnica del sistema se va perdiendo conforme transcurre el tiempo, lo ue hace ue la amplitud del sistema disminuya con el tiempo, y se dice ue el movim iento es amortiguado. Un tipo comn de fuerza de friccin es proporcional a la rapid ez y acta en direccin opuesta al movimiento. Estas fuerzas se producen frecuenteme nte en los fluidos, principalmente en l uidos y gases, a u se llaman fuerzas de vi scosidad, donde actan cuando un cuerpo se mueve, por ejemplo en el agua o en el a ire. Se expresan en la forma F = bv, donde b es una constante ue mide el grad o de viscosidad del fluido. Aplicando la segunda ley de Ne ton a un sistema amor tiguado, donde sobre el cuerpo en movimiento oscilatorio actan las fuerzas de res titucin y de amortiguamiento o de viscosidad, se obtiene: x bv = ma dx d 2x =m 2 dt dt kx b (11.14) 317

Cap. 11. Movimiento oscilatorio. La solucin e esta ecuacin esta fuera el alcance e este libro, por lo que se a sin emostracin. Cuan o la fuerza e viscosi a es pequea compara a con kx, es ec ir, cuan o b es pequea, la solucin es: x = Ae b t 2m cos(t + ) (11.15) on e la frecuencia el movimiento es: k b = m 2m 2 (11.16) En la figura 11.8 se grafica la posicin x en funcin del tiempo t para este movimie nto amortiguado. Se observa que cuando la fuerza disipativa es pequea comparada c on la fuerza de restitucin, el carcter oscilatorio del movimiento se mantiene, per o la amplitud de la oscilacin disminuye con el tiempo, hasta que finalmente el mo vimiento se amortigua y detiene. La lnea de trazos en la figura 11.8 que es la en volvente de la curva de oscilacin, representa el factor exponencial en la ecuacin 11.15, corresponde a la amplitud decreciente en el tiempo. Figura 11.8 Grfico posicin tiempo en las oscilaciones amortiguadas. De la ecuacin de la frecuencia se observa que si b = 0, se tiene la frecuencia na tural de vibracin del oscilador no amortiguado, o2 = k/m. Cuando la mag318

Cap. 11. Movimiento oscilatorio. nitud de la fuerza de friccin se aproxima ms a la magnitud de la fuerza de restitu cin, las oscilaciones se amortiguan ms rpidamente. Cuando b alcanza un valor crtico tal que b/2m = o, el sistema no oscila y se dice que est crticamente amortiguado, p or lo que el sistema regresa al equilibrio en forma exponencial con el tiempo. S i el medio es tan viscoso que la fuerza de friccin es mayor que la de restitucin, con lo cual b/2m > o, el sistema est sobreamortiguado. En este caso tampoco oscila , sino que simplemente regresa a su posicin de equilibrio. En todos los casos, cu ando hay friccin presente, la energa del oscilador disminuye hasta cero; la energa mecnica que se pierde se transforma en el medio en energa trmica. 11.6. OSCILACIONES FORZADAS. Para el caso de un oscilador amortiguado, la energa disminuye en el tiempo por ef ecto de la fuerza disipativa. Se puede compensar esta prdida y entregar energa al sistema aplicando una fuerza externa que en cualquier instante acte en la direccin del movimiento del oscilador, que debe hacer un trabajo positivo sobre el siste ma. La amplitud del movimiento permanecer constante si la energa de entrada al sis tema en cada ciclo del movimiento es igual a la energa que se pierde por la fricc in. Un oscilador forzado se puede obtener cuando un oscilador amortiguado es impu lsado por una fuerza externa que varia armnicamente en el tiempo, de la forma F = Fo cos t, donde es la frecuencia angular de la fuerza y Fo es una constante. Agr egando esta fuerza a la ecuacin diferencial del oscilador amortiguado, se obtiene : dx d 2x Fo cos t kx b = m 2 t t (11.17) Ya sabemos que la solucin e esta ecuacin es complica a, por lo que amos su resul ta o sin emostracin; en un curso mas avanza o e Mecnica Clsica uste es van a tene r amplias posibili a es e entretenerse resolvien o ecuaciones como esta. Despus e un tiempo suficientemente largo, cuan o la energa e entra a en ca a ciclo es igual a la energa per i a en ca a ciclo, se alcanza la con icin e esta o estacion ario, on e las oscilaciones se pro ucen con 319

Cap. 11. Movimiento oscilatorio. amplitu constante. En esas con iciones, la solucin 7 es: x = A cos(t + ) on e la amplitu es: A= 2 Fo m ( 2 2 o) b + m 2 (11.18) con o2 = k/m, la frecuencia del oscilador no amortiguado (b = 0). Estas solucione s se justifican, pues fsicamente en estado estacionario el oscilador debe tener l a misma frecuencia de la fuerza externa aplicada. Se puede comprobar que x es so lucin si se reemplaza en la ecuacin diferencial 11.17, esta se satisface cuando la amplitud es la ecuacin 11.18. En la ecuacin 11.18 se observa que el movimiento de l oscilador forzado no es amortiguado, ya que se est impulsando por una fuerza ex terna, pues la condicin es que el agente externo entregue la energa necesaria para compensar la energa que se pierde por friccin. Observar que la masa oscila con la frecuencia de la fuerza impulsora. Para un amortiguamiento pequeo, la amplitud a umenta cuando la frecuencia de la fuerza impulsora se aproxima a la frecuencia n atural de la oscilacin, o cuando o. El aumento tan significativo de la amplitud ce rca de la frecuencia natural se conoce como resonancia, y la frecuencia o se llam a frecuencia de resonancia del sistema. En la figura 11.9 se muestra una grfica d e la amplitud como funcin de la frecuencia para un oscilador forzado con y sin fu erza de friccin. Notar que la amplitud aumenta cuando disminuye el amortiguamient o (b 0). Adems la curva de resonancia se ensancha al aumentar el amortiguamiento. En condiciones de estado estacionario, y a cualquier frecuencia de impulso, la energa transferida es igual a la energa que se pierde por la fuerza de amortiguami ento, por eso la energa total promedio del oscilador permanece constante. En ause ncia de fuerzas de amortiguamiento (b = 0), de la ecuacin 11.18 se observa que, e n estado estacionario, A aumenta hasta el infinito cuando o. Es decir, si no hay prdidas en el sistema, y se continua impulsando un oscila320 e la ecuacin iferencial 11.1

Cap. 11. Movimiento oscilatorio. dor que se encontraba inicialmente en reposo, con una fuerza senoidal que se enc uentra en fase con la velocidad, la amplitud crecer sin lmite. Esto no se produce en la realidad ya que siempre estn presentes las fuerzas de friccin, aunque sean p equeas, por lo tanto, en la resonancia la amplitud ser grande, pero finita para pe queos amortiguamientos. Figura 11.9 Grfico de la amplitud en funcin de la frecuencia para un oscilador for zado con y sin fuerza de friccin. 321

Cap. 11. Movimiento oscilatorio. PROBLEMAS. 11.1 El desplazamiento de una partcula est dado por la ecuacin x = 4 cos(3t + ) , donde x esta en m y t en s. Calcula : a) la f ecuencia y el e iodo del movimiento, b) la am litud del movimiento, c) la constante de fase, d) la osicin de la a tcula en t = 0 y 5s, e) la a idez y acele acin en cualquie instante, f) la a idez y acele acin mximas, g) la a idez y acele acin en t = 0 y 5 s. R: a) 1.5 Hz, 0.667 s , b) 4 m, c) ad, d) 4 m. Una a tcula oscila con un movimiento a mnico sim le de tal fo ma que su des lazamiento va ia de acue do con la ex esin x = 5 cos(2t + / 6) , donde x esta en cm y t en s. Calcula : a) la f ecuencia y el e iodo del movimiento, b) la am litud del movimiento, c) la osicin de la a tcula en t = 0, d) la a idez y acele acin en t = 0. R: a) s, b) 5 cm, c) 4.33 cm, d) 5 cm/s, 1 7.3 cm/s2. Una a tcula que se mueve con movimiento a mnico sim le eco e una dis tancia total de 20 cm en cada ciclo, y su mxima acele acin es de 50 m/s2. Calcula : a) la f ecuencia angula , b) su mxima a idez. El des lazamiento de una a tcula est dado o la ecuacin x = 8 cos(2t + / 3) , donde x esta en cm y t en s. Calcul a : a) la f ecuencia y el e iodo del movimiento, b) la am litud del movimiento, c) la constante de fase, d) la osicin, la a idez y acele acin de la a tcula en t = /2 s, e) la mxima a idez y el tiem o mas co to en alcanza la, f) la mxima acel e acin y el tiem o mas co to en alcanza la. R: d) 13.9 cm/s, 16 cm/s2, e) 16 cm/s , 0.263 s, f) 32 cm/s2, 1.05 s. Una a tcula que se mueve con movimiento a mnico s im le, en t = 0 se encuent a en xo = 2 cm, con a idez vo = 24 cm/s. Si el e i odo del movimiento es 0.5 s, calcula : a) la constante de fase, b) la am litud, c) la osicin, la a idez y acele acin en funcin del tiem o, d) la a idez y acele acin mximas. Una a tcula que se mueve con movimiento a mnico sim le a lo la go del eje x, em ieza desde el o igen en t = 0 y se mueve hacia la de322 11.2 11.3 11.4 11.5 11.6

Ca . 11. Movimiento oscilato io. echa. Si la am litud de su movimiento es de 2 cm y la f ecuencia 1.5 Hz, calcul a : a) la osicin en funcin del tiem o, b) la mxima a idez y el tiem o mas co to e n alcanza la, c) la mxima acele acin y el tiem o mas co to en alcanza la, d) la di stancia total eco ida ent e t=0 y t=1s. R: a) 2 sen(3t), b) 6cm/s, 0.33s, c) 182c m/s2, 0.5s, d) 12cm. 11.7 Un istn, de masa 2kg, en un moto de automvil tiene un movimiento a mnico sim le, con una am litud de 5 cm. Calcula la a idez y la ace le acin mximas del istn cuando se mueve a 3600 ev/min. Un eso de 0.2 N se sus en de de un eso te de constante 6 N/m. Calcula el des lazamiento del eso te. R: 3.33 cm. Un eso te se ala ga 4 cm cuando se le cuelga una masa de 10 g amos. Si se le cuelga una masa de 25 g amos, oscila con un movimiento a mnico sim le. Cal cula el e iodo del movimiento. La f ecuencia de vib acin de un sistema masa es o te es de 5 Hz cuando se le cuelga una masa de 4 g amos. Calcula la constante del eso te. R: 3.95 N/m. 11.8 11.9 11.10 11.11 Una masa de 1 kg sujeta a un eso te de constante 25 N/m, oscila en una su e ficie ho izontal sin f iccin. La masa se suelta desde el e oso en el instante t = 0, donde el eso te se encuent a com imido en la osicin en x = 3cm. Calcu la : a) el e iodo, b) la a idez y acele acin mxima, c) la osicin, la a idez y a cele acin en funcin del tiem o. 11.12 A un oscilado a mnico sim le le toma 12 s co m leta 5 vib aciones. Calcula a): el e iodo, b) la f ecuencia, c) la f ecuenc ia angula . R: a) 2.4 s, b) 0.417 Hz, c) 2.62 ad/s. 11.13 Un sistema masa eso te oscila de tal fo ma que su osicin est dada o x = 0.25cos(2t), donde x esta en m y t en s. Calcula : a) la a idez y acele acin de l a masa cuando x = 0.1 m, b) la a idez y acele acin mximas. 11.14 Una masa de 0.5 kg sujeta a un eso te de constante 8 N/m, oscila con movimiento a mnico sim le, con una am litud de 10 cm. Calcula a) 323

Ca . 11. Movimiento oscilato io. la a idez y acele acin mximas, b) la a idez y acele acin cuando la masa se encuen t a en x = 6 cm de la osicin de equilib io, c) el tiem o que demo a la masa en m ove se ent e x = 0 y x = 8 cm. R: a) 0.4 m/s, 1.6 m/s2, b) 0.32 m/s, 9.6 m/s2, c ) 0.232 s. 11.15 Una a tcula sujeta a un eso te ve tical, se esti a hacia abajo una distancia de 4 cm desde su osicin de equilib io y se suelta desde el e oso . La acele acin inicial hacia a iba de la a tcula es 0.3 m/s2. Calcula a) el e iodo de las oscilaciones siguientes, b) la a idez cuando asa o la osicin de equilib io. c) Esc ibi la ecuacin de movimiento de la a tcula. 11.16 Una masa d e 0.2 kg sujeta a un eso te oscila con movimiento a mnico sim le, con un e iodo de 0.25s. Si la ene ga total del sistema es 2 J, calcula a) la constante del e so te, b) la am litud del movimiento. 11.17 Un sistema masa eso te oscila con u na am litud de 3.5 cm. Si la constante del eso te es 250 N/m, y la masa de 0.5 kg, calcula a) la ene ga mecnica del sistema, b) la a idez mxima de la masa, c) l a acele acin mxima. R: a)0.153 J, b) 0.783 m/s, c) 17.5 m/s2. 11.18 La am litud de un sistema movindose con un movimiento a mnico sim le se du lica. Calcula la va iacin en: a) la ene ga total, b) la a idez mxima, c) la acele acin mxima, d) el e i odo. R: a) cuad u lica, b) du lica, c) du lica, d) no cambia. 11.19 Un sistema m asa eso te tiene un movimiento a mnico sim le en una su e ficie ho izontal sin f iccin, con una am litud de 12 cm. Si la constante del eso te es 50 N/m, calcula : a) la ene ga total del sistema, b) la ene ga cintica del sistema cuando la masa est a 9 cm de la osicin de equilib io, c) la ene ga otencial cuando la osicin es x=9 cm. 11.20 Una a tcula tiene un movimiento a mnico sim le con una am litud de 3 cm. Calcula la osicin es ecto al unto medio de su movimiento donde la a id ez se igual a la mitad de la a idez mxima. R: 2.6cm. 324

Ca . 11. Movimiento oscilato io. 11.21 Un bloque de 50 g se sujeta al ext emo lib e de un eso te ideal que tiene una fue za de estitucin de 40 N o cada met o de extensin. El bloque se uede d esliza lib e sob e una su e ficie ho izontal sin f iccin, se one en movimiento dndole una ene ga otencial inicial de 2 J y una ene ga cintica inicial de 1.5 J. a) Dibuja la g fica de la ene ga otencial del sistema a a valo es en el ango 0.5m x +5m. b) Calcula la am litud de la oscilacin del g fico y en fo ma analtica. c) Calcula la a idez del bloque cuando asa o la osicin de equilib io. d) Calcu la la osicin donde la ene ga cintica es igual a la ene ga otencial. e) Calcula l a f ecuencia angula y el e iodo. f) Si el des lazamiento inicial fue x > 0 y l a a idez inicial fue v < 0, calcula el ngulo de fase. g) Esc ibi la ecuacin de movimiento x(t). 11.22 Un ndulo sim le tiene un e iodo de 2.5 s. Calcula : a) s u longitud, su e iodo si estuvie a en la Luna, donde g = 1.67 m/s2. R: a) 1.55 m, b) 6.1 s. 11.23 Calcula la f ecuencia y el e iodo de un ndulo sim le de 10 m de longitud. 11.24 Si la longitud de un ndulo sim le se cuad u lica, que sucede con la f ecuencia y el e iodo? R: se divide en a tes iguales, se du lica. 11. 25 Un ndulo sim le de longitud 2 m oscila de ac a a all y de all a a ac. Calcula el nme o de oscilaciones que ha en 5 minutos. 11.26 Un ndulo sim le que tiene una masa de 0.25 kg y una longitud 1 m, se desva un ngulo de 15 y se suelta. Calcula : a) la a idez mxima, b) la acele acin angula mxima, c) la mxima fue za de estituc in. R: a) 0.82 m/s, b) 2.57 ad/s2, c) 0.64 N. 11.27 Una ba a unifo me se encuen t a ivoteada en un ext emo como se muest a en la figu a 11.6. Si la ba a oscil a con un movimiento a mnico sim le, calcula su longitud a a que su e iodo sea igual al de un ndulo sim le de 1 m de longitud. 11.28 Un a o ci cula de adio R oscila sob e el filo de un cuchillo. Demuest e que su e iodo de oscilacin es el mismo que el de un ndulo sim le de longitud 2R. 325

Ca . 11. Movimiento oscilato io. 11.29 Un ndulo fsico en fo ma de cue o lano tiene un movimiento a mnico sim le c on una f ecuencia de 1.5 Hz. Si tiene una masa de 2.2 kg y el ivote se encuent a a 0.35 m del cent o de masa, calcula el momento de ine cia del ndulo. R: 0.08 5 kgm2. 11.30 Una va illa delgada tiene una masa M y una longitud de 1.6 m. Uno de los ext emos de la va illa se sujeta en un ivote fijo, en to no al cual osci la la va illa. a) Calcula la f ecuencia de estas oscilaciones. Si se ag ega una a tcula de masa M al ext emo final de la va illa, b) calcula el facto en el q ue cambia el e iodo. 11.31 Un volante de un eloj tiene un e iodo de oscilacin de 0.25 s. El volante se const uyo de tal mane a que 20 g de masa estn concent ad os al ededo de un a o de 0.5cm de adio. Calcula : a) el momento de ine cia del volante, b) la constante de to sin del eso te sujeto al volante. R: a) 5x10 7 k gm2, b) 3.16x10 4 Nm/ ad. 11.32 Un ndulo de 1 m de longitud se suelta desde un ng ulo inicial de 15. Des us de 1000 s su am litud se educe o la f iccin a 5. Calcul a el valo de b/2m. R: 1x10 3 s 11.33 Demuest e que la constante de amo tiguami ento b tiene unidades kg/s. 11.34 Demuest e que la ecuacin 11.15 es una solucin de la ecuacin 11.14, siem e y cuando b2 < 4mk. 11.35 Demuest e que la a idez de c ambio de la ene ga mecnica a a un oscilado amo tiguado, no im ulsado, esta dada o dE/dt = bv2 y o lo tanto siem e es negativa. 11.36 Una masa de 2 kg suje ta a un eso te de constante 20 N/m, se im ulsa o una fue za exte na de la fo ma F = 3cos(2t), donde F esta en N y t en s. Calcula : a) el e iodo del movimien to, b) la am litud. Su onga que no hay amo tiguamiento, es deci que b = 0. 11.3 7 Calcula la f ecuencia de esonancia de los siguientes sistemas: a) una masa d e 3 kg sujeta a un eso te de constante 240 N/m, b) un ndulo sim le de 1.5 m de longitud. R: a) 1.42 Hz, b) 0.41 Hz. 326

Ca . 11. Movimiento oscilato io. 11.38 Conside e un oscilado fo zado no amo tiguado (b = 0), y demuest e que la ecuacin 11.1 es solucin de la ecuacin 11.17 con una am litud dada o la ecuacin 11. 18. 11.39 Un eso de 40 N se sus ende de un eso te de constante 200 N/m. El sis tema no est amo tiguado y se im ulsa o una fue za a mnica de f ecuencia 10 Hz, d ando o esultado un movimiento a mnico de am litud 2 cm. Calcula el valo mximo de la fue za a licada. R: 318 N. 11.40 Un ndulo de longitud L y masa M, tiene c onectado un eso te de constante k a una distancia h o debajo del unto de sus ensin, como se muest a en la figu a 11.10. Calcula la f ecuencia de vib acin del sistema a a valo es equeos de la am litud. Su onga que tanto el so o te ve tic al como el eso te son gidos de masa des eciable. R:

1 f = 2 MgL + kh 2 ML2 . Figu a 11.10. P oblema 11.40. 327

Ca . 12. Tem e atu a, dilatacin t mica y gases. CAP 12. TEMPERATURA, DILATACION TERMICA Y GASES. La mecnica ne toniana ex lica un a dive sidad muy am lia de fenmenos en la escala mac osc ica, tales como el movimi ento de los cue os, de oyectiles y de lanetas. Aho a inicia emos el estudio en la te modinmica, ea de la fsica elacionada con los conce tos de calo y tem e atu a. Con la te modinmica se hace la desc i cin de las o iedades volumt icas de la mate ia y la co elacin ent e esas o iedades y la mecnica de los tomos y molcul as. Se ha eguntado alguna vez qu le asa a la ene ga cintica de un objeto cuando c ae al suelo y queda en e oso?, o como uede enf ia un ef ige ado ? Las leyes d e la te modinmica y los conce tos de calo y de tem e atu a nos e miti n contesta estas eguntas. En gene al, la te modinmica t ata con las t ansfo maciones fsic as y qumicas de la mate ia en todos sus estados: slido, lquido, gas o lasma. El ob jeto de estudio de la fsica t mica t ata con los fenmenos que com enden t ansfe en cias de ene ga ent e cue os que estn a dife entes tem e atu as. En el estudio de la mecnica, se definie on cla amente conce tos como masa, fue za y ene ga, a a ha ce la desc i cin en fo ma cuantitativa. De la misma fo ma, una desc i cin cuantit ativa de los fenmenos t micos equie e una definicin cla a de los conce tos de tem e atu a, calo y ene ga inte na. La ciencia de la te modinmica est elacionada con el estudio del flujo de calo desde un unto de vista mac osc ico. 12.1 TEMPERATURA Y LEY CERO DE LA TERMODINAMICA. La tem e atu a es una magnitud fsica que se efie e a la sensacin de f o o caliente al toca alguna sustancia. As, nuest os sentidos nos ent egan una indicacin cualitativa de la tem e atu a, e o no odemos confia siem e en nuest os sentidos, ya que ueden engaa nos. Po eje m lo, si se saca del ef ige ado un eci iente metlico con cubos de hielo y un e nvase de ca tn con ve du as congeladas, se siente ms f o el metal que el ca tn, aunq ue ambos estn a la misma tem e atu a; la misma sensacin se nota cuando se isa la baldosa del iso solo y la alfomb a. Esto se debe a que el metal o la ce mica es mejo conducto del calo que el ca tn o la tela. Po lo tanto se necesita un mtod o confiable a a medi la sensacin de f o o caliente de los cue os. 329

Ca . 12. Tem e atu a, dilatacin t mica y gases. Ex e imentalmente se encuent a que si dos objetos que inicialmente estn a dife en te tem e atu a, se onen en contacto t mico ent e s, llega el momento en que alcan za an una cie ta tem e atu a inte media. Pa a com ende el conce to de tem e at u a definamos contacto t mico: es cuando ent e dos cue os uede ocu i un inte cambio de ene ga ent e ellos sin que se ealice t abajo mac osc ico; y equilib io t mico: es una situacin en la que dos cue os en contacto t mico ent e s, dejan de t ene todo inte cambio neto de ene ga. El tiem o que ta dan los cue os en alcanza el equilib io t mico de ende de las o iedades de los mismos. Se uede ensa en la tem e atu a como una o iedad que dete mina cuando se encuent a o no un c ue o en equilib io t mico con ot os cue os. Conside emos dos cue os A y B que no estn en contacto t mico y un te ce cue o C que usa emos como medido de tem e atu a. Se quie e dete mina cuando estn A y B estn en equilib io t mico ent e s. P ime o se coloca el cue o C en contacto t mico con A hasta que se alcanza el equi lib io t mico. Luego se coloca el cue o C en contacto t mico con B hasta que alca nzan el nuevo equilib io t mico. Si la tem e atu a de C des us de one se en conta cto t mico con A y con B es la misma en ambos casos, entonces A y B estn en equili b io t mico ent e s. Este esultado se esume en un enunciado llamado inci io ce o de la te modinmica: Si dos cue os A y B estn o se a ado en equilib io t mico con un te ce cue o C, entonces A y B estn en equilib io t mico ent e s. Este enun ciado, aunque a ezca obvio, es lo ms fundamental en el cam o de la te modinmica, ya que se uede usa a a defini la tem e atu a. 12.2 TERMOMETROS Y ESCALAS DE TEMPERATURA. Los te mmet os son dis ositivos a a d efini y medi la tem e atu a de un sistema. Todos los te mmet os se basan en el cambio de alguna o iedad fsica con la tem e atu a, como el cambio de volumen de un lquido, el cambio en la longitud de un slido, el cambio en la esin de un gas a volumen constante, el cambio en el volumen de un gas a esin constante, el cam bio en la esistencia de un conducto o el cambio en el colo de objetos a muy a lta tem e atu a. Los cambios de tem e atu a se miden a a ti de los cambios en las ot as o iedades de una sustancia, con un inst umento llamado te mmet o, de los 330

Ca . 12. Tem e atu a, dilatacin t mica y gases. cuales existen va ios ti os. El te mmet o mecnico se basa en la o iedad de dilat acin con el calo o cont accin con el f o de alguna sustancia. Po ejem lo, el te mm et o de me cu io convencional mide la dilatacin de una columna de me cu io en un ca ila de vid io, ya que el cambio de longitud de la columna est elacionado con el cambio de tem e atu a. El te mmet o se uede calib a colocndolo en contacto t mico con algn sistema natu al cuya tem e atu a e manezca constante, conocida com o tem e atu a de unto fijo. Una de las tem e atu as de unto fijo que se elige no malmente es la de una mezcla de agua y hielo a la esin atmosf ica, que se def ine como ce o g ado Celsius denotado o 0 C. Ot o unto fijo conveniente es la t em e atu a de una mezcla de agua y va o de agua en equilib io a la esin atmosf ica, al que se le asigna el valo de 100 C. Una vez que se han fijado los niveles de la columna de me cu io en estos untos, se divide en 100 a tes iguales, don de cada una de estas e esenta un cambio de tem e atu a equivalente a un g ado Celsius (Ande s Celsius, sueco, 1701 1744). As se define una escala de tem e atu a llamada escala centg ada o escala Celsius. Un oblema ctico de cualquie te mm et o es su ango limitado de tem e atu as. Po ejem lo, como el me cu io se cong ela a 39 C, a a asegu a se de medi tem e atu as meno es que estas, se usan los te mmet os de alcohol, que se congela a 130 C. Los te mmet os a a medi la mnima di a ia en meteo ologa son de alcohol, se muest a en la figu a 12.1 infe io , el te mmet o su e io en esta figu a, de me cu io, se usa a a medi la mxima dia ia. Figu a 12.1 Te mmet os de mxima y de mnima usados en meteo ologa. 331

Ca . 12. Tem e atu a, dilatacin t mica y gases. 12.3 TERMOMETRO DE GAS Y ESCALA KELVIN. Se equie e un te mmet o unive sal cuyas lectu as sean inde endientes de la sustancia que se use. En un te mmet o de gas, las lectu as de las tem e atu as son casi inde endientes de la sustancia que se utilice en el te mmet o. En este caso, la o iedad fsica es la va iacin de la es in del gas con la tem e atu a. Al calenta se (enf ia se) el gas, la esin aumenta (disminuye) y la altu a de la columna de me cu io aumenta (disminuye). El cambi o de tem e atu a uede dete mina se a a ti del cambio en la esin. Si se su on e que la va iacin de tem e atu a T con la esin P es lineal, se uede esc ibi : T = aP + b (12.1) donde a y b son constantes, que se ueden dete mina a a ti de dos untos fijo s, tales como los untos de hielo y va o . Los ex e imentos demuest an que cuand o se miden las tem e atu as con dife entes te mmet os y con dife entes gases, las lectu as son inde endientes del ti o de gas que se use, siem e que la esin no sea muy alta. La conco dancia mejo a al educi se la esin. Esta conco dancia s ignifica que la inte seccin b de la ecuacin 12.1 es la misma a a todos los gases. Esto se muest a en la figu a 12.2, en un g fico de esin tem e atu a, a a un ga s 1, gas 2 y gas 3, cualesquie a. Cuando se ext a ola la ecta de esin a tem e atu as muy bajas (lneas de untos), se encuent a que la esin es ce o cuando la t em e atu a alcanza el valo 273.15 C. Esta tem e atu a co es onde a la constant e b en la ecuacin 12.1. La ext a olacin es necesa ia, ya que todos los gases se li cuan antes de llega a esa tem e atu a. Punto t i le del agua. Co es onde a la tem e atu a y esin nicas en las que el hielo, el agua y el va o de agua ueden coexisti en equilib io. Estos valo es son a oximadamente T = 0.01 C y P = 610 P a. La tem e atu a en el unto t i le del agua en la nueva escala, dada o la ec uacin 12.1, se tom como 273.16 kelvin, ab eviado 273.16 K. Esta eleccin se hizo a a que la vieja escala centg ada de tem e atu a basada en los untos del hielo y d el va o coincidie a ce canamente con esta nueva escala basada en el unto t i l e del agua. Esta nueva escala se llama escala de tem 332

Ca . 12. Tem e atu a, dilatacin t mica y gases. e atu a te modinmica y la unidad SI de la tem e atu a te modinmica, el Kelvin K, se define como la f accin 1/273.16 de la tem e atu a del unto t i le del agua (W illiams Thom son, ingls, 1824 1907, P ime Ba n de Kelvin, Lo d Kelvin).

Si se toma b = 0 en la ecuacin 12.1 y llamando P3 a la esin en la tem e atu a de l unto t i le del agua, T3 = 273.16K, se encuent a que a =273.16K/P3. Po lo ta nto, la tem e atu a como una medida de la esin P de un gas a a un te mmet o de gas a volumen constante se define como: T = 273.16 P P3 (12.2) Como se mencion antes, ex e imentalmente se encuent a que si la esin P3 disminuy e, el valo medido de la tem e atu a se a oxima al mismo valo a a todos los g ases. En la figu a 12.3 se da un ejem lo de tales medidas, la cual muest a el u nto del va o medido con un te mmet o de gas a volumen constante usando dife ente s gases. A medida que la esin P3 se a oxima a ce o, todas las medidas se ace c an a un valo comn de 373.15 K. en fo ma anloga, se encuent a que la tem e atu a d el unto de hielo es 273.15 K. 333

Figu a 12.2 G fico de

esin ve sus tem e atu a a a dife entes gases.

Ca . 12. Tem e atu a, dilatacin t mica y gases. En el lmite de las esiones bajas (valo es ce canos a la esin atmosf ica) y las tem e atu as altas del gas (del o den de la tem e atu a ambiente o mayo ), los g ases eales se com o tan como lo que se conoce como un gas ideal, que se analiza con detalle mas adelante. La escala de tem e atu as definida en este lmite de esiones bajas del gas, ecibe el nomb e de tem e atu a del gas ideal, T, dada o : T = 273.16 lim P3 0 P P3 (12.3) Figu a 12.3 G fico de tem e atu a medida con un te mmet o de gas a volumen constan te ve sus esin P3 del unto de va o de agua, a a dife entes gases. Po lo tanto, el te mmet o de gas a volumen constante define una escala de tem e atu as que uede e oduci se en todos los labo ato ios del mundo. An cuando la e scala de ende de las o iedades de un gas, es inde endiente del gas que se use. Es conveniente tene una escala de tem e atu as inde endiente de las o iedade s de cualquie sustancia. Una escala de este ti o se llama escala de tem e atu a s absoluta o escala Kelvin. Se usa el smbolo T a a indica las tem e atu as abso lutas. 334

Ca . 12. Tem e atu a, dilatacin t mica y gases. En la tabla 12.1 se muest an las tem e atu as a a untos fijos a ticula es de dife entes mate iales. Tabla 12.1 Tem e atu a de algunos fenmenos fsicos y biolgicos. P oceso T (C) Ce o absoluto 273.15 Punto t i le del hid geno 259.34 Punto ebulli cin del hid geno 252.87 Punto fusin del nit geno 206 Solidificacin de la gasolina 150 Solidificacin del alcohol 130 Solidificacin CO2 (hielo seco) 78 Solidificacin del me cu io 39 T (K) 0 13.81 20.28 67.15 123.15 143.15 195.15 234.15 P oceso No mal de e sona Solidificacin agua Ebullicin del agua Fusin del o o Llama de gas Su e ficie del Sol Cent o de la Tie a Cent o del Sol T (C) T (K) 37 310.15 0 273 .15 100 373.15 1063 1336 1627 1900 5700 6000 16000 16273 1.4x107 1.4x107 Ejem lo 12.1 Un te mmet o de gas a volumen constante se calib a en los untos de tem e atu a de 58 C a 1 atm de esin y en el unto de tem e atu a 157 C a 2 atm. a) Deduci la escala de tem e atu a. Calcula : b) el valo de ce o absoluto que oduce esta calib acin, c) la esin en unto de congelamiento del agua, d) la es in en el unto de ebullicin del agua. Figu a 12.4. Relacin lineal ent e T y P del ejem lo 12.1

Solucin: a) los untos de calib acin son T1 = atm. Su oniendo que existe una elacin lineal omo se muest a en la figu a 12.4, se tend a: T el unto de inte seccin a a P = 0 y b es la la: 335

58 C, P1 = 1 atm y T2 = 157 C, P2 = 2 ent e la esin y la tem e atu a, c = a + bP con T en C, P en atm, a es endiente de la ecta, que se calcu

Ca . 12. Tem e atu a, dilatacin t mica y gases. b=

Reemplazan o en la ecuacin lineal e T, se obtiene: T = a + 215 P Para calcular a , se reemplaza los valores T1, P1 (o T2, P2) en la ecuacin anterior: 58 C = a + 21 5 C 1atm a = 273 C atm Por lo tanto, la relacin lineal entre T y P, que a: T = 273 + 215 P b) En el cero absoluto, P = 0, por lo tanto T = 273 +215x0 = 273 C. c) En el punto e congela cin el agua, T = 0 C, 273 C = 1.27 atm 215 C atm 0 = 273 + 215 P P = d) En el punto de ebullicin del agua, T =100 C, 373 C = 1.73atm 215 C atm 100 = 273 + 215 P P = 336

T T2 T1 157 (58)

C = = = 215 P P2 P 2 1 atm 1

Cap. 12. Temperatura, dilatacin trmica y gases. 12.4 ESCALAS DE TEMPERATURA CELSIUS Y FAHRENHEIT. La temperatura Celsius, TC, est desplazada respecto a la escala absoluta (o Kelvi n) T en 273.15 , ya que por definicin el punto triple del agua (273.16 K) vale 0.01 C. La relacin entre estas escalas es: TC = T 273.15 (12.4) Se observa que el valor de un grado en la escala Kelvin es igual al de la escala Celsius. Por ejemplo, una diferencia o variacin de temperatura de 5 C es igual a una diferencia de temperatura de 5 K. Las dos escalas solo se diferencian en la eleccin del punto cero. Otra escala usada en pases anglosajones es la escala Fahre nheit (Daniel Fahrenheit, alemn, 1686-1736). La temperatura Fahrenheit, TF, se re laciona con la temperatura Celsius por la expresin: 9 TF = TC + 32 F 5 (12.5) Ejemplo 12.2 La temperatura promedio mensual mnima en Concepcin es aproximadamente 6 C y se produce en agosto y la mxima es 24 C y se produce en enero. Un gringo qui ere conocer estos valores en grados Fahrenheit. Hacer la conversin. Solucin: Se us a la ecuacin 12.5, para TCmin = 6 C: 9 TF min = TC min + 32 F = (1.8)(6) + 32 = 42. 8 F 5 para TCmax = 24 C: 9 TF max = TC max + 32 F = (1.8)(24) + 32 = 75.2 F 5 337

Cap. 12. Temperatura, dilatacin trmica y gases. 12.5 DILATACION TERMICA DE SOLIDOS Y LIQUIDOS. La mayora de los objetos se dilatan (contraen) cuando se aumenta (disminuye) su t emperatura. En escala microscpica, la dilatacin trmica de un cuerpo es consecuencia del cambio en la separacin media entre sus tomos o molculas. Para comprender esto, se considerar un slido que consta de un arreglo regular de tomos mantenidos unidos por fuerzas elctricas. Un modelo mecnico de estas fuerzas es imaginar que los tomo s estn unidos por resorte rgidos, como se muestra en la figura 12.4. Por su natura leza, las fuerzas interatmicas se consideran elsticas. Para temperaturas en los ra ngos comunes de la naturaleza, los tomos vibran respecto a sus posiciones de equi librio con una amplitud aproximada de 10-11 m y una frecuencia de 1013 Hz. La se paracin promedio entre los tomos es del orden de 10-10 m. Al aumentar la temperatu ra del slido, los tomos vibran con amplitudes ms grandes y la separacin promedio ent re ellos aumenta, dando por resultado que el slido como un todo se dilate cuando aumente su temperatura. Si la dilatacin de cualquier objeto es lo suficientemente pequea en comparacin con sus dimensiones, el cambio de cualquier parte, largo, an cho o alto, dentro de una buena aproximacin, es una funcin lineal de la temperatur a. Figura 12.5 Modelo mecnico de un slido, donde los tomos se imaginan unidos unos con otros por resortes. Supongamos que la dimensin lineal de un cuerpo a una cierta temperatura, a lo lar go de alguna direccin es l. La longitud aumentara en una cantidad l para un cambio de temperatura T. Experimentalmente se demuestra que el cambio en la longitud es proporcional al cambio de temperatura y a la longitud inicial siempre que T sea pequea. Por lo tanto, la ecuacin bsica para la dilatacin de un slido es: 338

Cap. 12. Temperatura, dilatacin trmica y gases. l = l T (12.6) donde la constante de proporcionalidad se ll m coeficiente promedio de dil t cin line l p r un m teri l d do, se mide en C-1. P r ilustr r, por ejemplo un v lo r de de 5x10-6 C-1 signific que l longitud del objeto c mbi 5 millonsim s su v lor origin l por c d gr do Celsius de v ri cin de temper tur . El orden de m gni tud de p r los slidos es de 1mm por m, c d 100 C. El coeficiente se consider pr omedio, porque en gener l v ri con l temper tur , pero comnmente est v ri cin e s despreci ble l esc l en que se re liz n l m yor de l s mediciones. Como l s dimensiones line les de los cuerpos c mbi n con l temper tur , se deduce que el re y el volumen del cuerpo t mbin c mbi n con l temper tur . El c mbio en el volumen presin const nte es proporcion l l volumen origin l V y l c mbio de temper tur , lo que se puede escribir como: V = V T (12.7) donde es el coeficiente promedio de dilatacin volumtrica. Para un slido isotrpico (a quel en el cual el coeficiente de dilatacin lineal es el mismo en todas las direc ciones), el coeficiente de dilatacin volumtrica es el triple del coeficiente de di latacin lineal, o sea, = 3. Por lo t nto l ecu cin 12.7 se puede escribir como V = 3 V T (12.8) De la misma forma, para una hoja o placa delgada, el cambio en el rea de una plac a isotrpica es: A = 2 A T (12.9) 339

Cap. 12. Temperatura, dilatacin trmica y gases. En la tabla 12.2 se presenta una lista de coeficientes de dilatacin lineal para d iferentes materiales, estos datos son valores medios en el intervalo de 0 a 100 C (excepto *). Para los gases, la presin es constante y baja (presin atmosfrica o in ferior). Observar que es positivo p r esos m teri les, pero pueden existir v lo res neg tivos de o de , lo que significa que los materiales se contraen en alguna direccin cuando aumenta la temperatura, por ejemplo la goma. Ta la 12.2 Coeficientes de dilatacin, cerca de la temperatura am iente. Material nvar (aleacin Ni Fe) Vidrio (pyrex) Vidrio (comn) Acero Concreto Co re L atn y ronce Aluminio Zinc Plomo Hielo (10 C a 0 C) * (C)-1 x 10-6 0,9 3,2 9 12 12 17 9 25 26 29 51 M teri l Agu (0 C) Agu (100 C) Alcohol etlico Benceno Aceton Mercu rio Glicerin Petrleo G solin Helio Aire ( C) 1 x 10 4 0.68 7.5 1,1 1,2 1,5 1,8 4 ,8 9,0 9,6 36,66 36,67 Por lo general, los lquidos aumentan su volumen al aumentar la temperatura y tien en coeficientes de dilatacin volumtrica aproximadamente 10 veces ms grandes que el de los slidos (ta la 12.2). El agua es la excepcin a esta regla. Expansin del agua. El agua es una sustancia compuesta por un tomo de oxgeno y dos d e hidrgeno. A temperatura am iente es lquida, inodora, inspida e incolora (aunque a dquiere una leve tonalidad azul en grandes volmenes). Se considera fundamental pa ra la existencia de la vida. No se conoce ninguna forma de vida que tenga lugar en ausencia completa de esta molcula, cuyo esquema se muestra en la figura 12.6. Casi todos los lquidos se expanden al calentarse, pero el agua fra hace todo lo con trario!, este comportamiento del agua es muy extrao, pero que se le va a hacer, s uponemos que la creacin es perfecta y se de e aceptar tal como es. 340

 

Cap. 12. Temperatura, dilatacin trmica y gases. El agua a 0 C se contrae al aumentar su temperatura hasta alcanzar los 4 C, valor a partir del cual comienza a expandirse, expansin que continua hasta el punto de e ullicin. Una cantidad dada de agua alcanza su volumen mnimo, y por lo tanto su d ensidad mxima a 4 C. La misma cantidad de agua tiene su volumen mximo, y por lo tan to su densidad mnima en forma de hielo, por eso el hielo flota so re el agua. Una vez convertida en hielo, el agua se contrae si seguimos reduciendo su temperatu ra. La explicacin de este comportamiento tiene que ver con la extraa estructura cr istalina del hielo. Los cristales de la mayor parte de los slidos estn estructurad os de tal manera que el estado slido ocupa un volumen menor que el estado lquido. Pero el hielo tiene cristales de estructura a ierta, consecuencia de la forma an gular de las molculas de agua (figura 12.6) y del hecho de que las fuerzas que un en las molculas de agua son ms intensas a ciertos ngulos. Las molculas de agua en es ta estructura a ierta ocupan un volumen mayor que en el estado lquido. Por eso el hielo es menos denso que el agua. Figura 12.6 Estructura de la molcula de agua. Ejemplo 12.3. La va de acero de un ferrocarril tiene una longitud de 30 m cuando la temperatura es 0 C. Calcular la variacin de su longitud en un da caluroso de ver ano en Concepcin. Solucin: suponiendo que en da de verano la temperatura ascienda a 30 C y usando los valores de de l t bl 12.2, se obtiene: 341

Se describirn las propiedades de un gas de masa m, contenido en un envase de volu men V a una presin P y a una temperatura T y se buscar una relacin entre estas vari ables, llamadas variables termodinmicas. En general, la ecuacin que relaciona las variables termodinmicas, que se llama ecuacin de estado, es ecuacin muy complicada, pero si el gas se mantiene a baja presin (baja densidad), se puede establecer en forma experimental una ecuacin muy simple. Comnmente, a un gas a baja densidad se le llama un gas ideal. La mayora de los gases a la temperatura ambiente y a la p resin atmosfrica, se comportan aproximadamente como gases ideales. La cantidad de gas en un volumen dado se expresa en trminos del nmero de moles, n. Se define un m ol de cualquier sustancia a la masa que contiene un nmero NA de molculas, llamado nmero de Avogadro (Amadeo Avogadro, italiano, 1776-1856). Este nmero NA (tambin sim bolizado con N o No) tiene un valor aproximado de 6,0220943 x 1023 6.3 x 1017 mo lculas/mol. El nmero de moles n de una sustancia se relaciona con su masa m por la expresin: n= m (PM ) (12.10) donde la cantidad (PM) se llama peso (o masa) molecular de la sustancia, comnment e medido en g/mol. Por ejemplo, el peso molecular del oxgeno, O2 es 32 g/mol, ent onces la masa de un mol de oxgeno es: m = n(PM) m = (1 mol)(32 g/mol) = 32 g 342

l = l T = (12 10 6 C 1 )(30m)(30 CROSCPICA DE UN GAS IDEAL.

C 0 C ) l = 0.0108 m = 10.8 mm 11 cm 12.6

C p. 12. Temper tur , dil t cin trmic

y g ses.

Cap. 12. Temperatura, dilatacin trmica y gases. Para establecer la ecuacin de estado de un gas ideal, considerar un gas encerrado en un envase cilndrico, cuyo volumen puede variar por medio de un mbolo mvil, como se muestra en la figura 12.7. Se supone que el cilindro no tiene fugas, por lo tanto la masa del gas permanece constante. Para este sistema, experimentalmente se encuentra que: a) Si la temperatura T se mantiene constante, su presin P es in versamente proporcional a su volumen V, es decir PV = cte; esta es la ley de Boy le de los gases (Robert Boyle, ingls, 1627-1691). b) Si la presin P del gas se man tiene constante, su volumen V es directamente proporcional a la temperatura T, e s decir V/T = cte; esta es la ley de Charles de los gases (Jacques Charles, fran cs, 1746-1823). c) Si el volumen V se mantiene constante, la presin P del gas es d irectamente proporcional a su temperatura T, es decir P/T = cte; esta es la ley de Gay-Lussac de los gases (Louis Gay-Lussac, francs, 1778-1850). Gas Figura 12.7 Gas ideal contenido en un cilindro con un mbolo movible que permite l a variacin del volumen. Estos resultados pueden resumirse en la siguiente ecuacin de estado de un gas ide al, de la forma: PV = nRT (12.11) 343

Cap. 12. Temperatura, dilatacin trmica y gases. donde R es una constante para un gas especfico, que se puede obtener en forma exp erimental, y T es la temperatura absoluta. Los experimentos con diferentes gases demuestran que cuando la presin se aproxima a cero, la cantidad PV/nT tiende al mismo valor de R para todos los gases. Por esta razn es que R se llama constante universal de los gases. En el SI donde la presin se mide en Pa y el volumen en m3 , el valor y la unidad de medida de R es: R = 8.31 J/(mol K) Si la presin se mide en atmsfera y el volumen en litros, lt, entonces el valor de la constante univer sal de los gases es: R = 0.0821 (atm lt)/(mol K). Como el nmero total de molculas N es igual al producto del nmero de moles n y el nmero de Avogadro NA, es decir, N = n NA, se puede escribir la ecuacin de estado, ya que n = N/NA, como: PV = nRT = R NT NA Se define la constante de Boltzmann k (Ludwing Boltzmann, austriaco, 18441906) c omo: k= J R 8.31 J molK k = k = 1.38 10 23 23 K NA 6.022 10 molec mol Con estos resulta os, la ecuacin e esta o e un gas i eal se pue e escribir ento nces como: 344

Cap. 12. Temperatura, ilatacin trmica y gases. PV = N k T (12.12) Se observa e la ecuacin 12.12 que la presin pro uci a por un volumen fijo e gas solo epen e e la temperatura y el nmero e molculas entro e ese volumen. Se h a efini o un gas i eal como aquel que obe ece la ecuacin e esta o. En la natura leza no existe un gas i eal; pero este concepto e gas i eal es muy til, ya que l os gases reales a bajas presiones se comportan aproxima amente como gases i eale s. Ejemplo 12.4. Calcular el nmero e tomos e una mone a e cobre (Cu) e 3 gramo s. Solucin: hay que calcular N; para el cobre el PM = 64 gr/mol, el nmero e moles e la mone a es: n= m 3g n= n = 0.047 moles (PM )Cu 64 g mol El nmero de tomos, N, en la moneda de cobre, se calcula de: N = n NA = 0.047 moles x 6.022 x1023 tomos/mol N = 2.8 x1023 tomos Ejemplo 12.5. Calcular el nmero de mol es de un gas ideal que se encuentra en 100 cm3, a 20 C y 105 Pa de presin. Solucin: Los datos son: V = 100cm3 = 10-4m3, T = 20 C = 293K, P = 105Pa. De la ecuacin de e stado 345

Cap. 12. Temperatura, dilatacin trmica y gases. PV = nRT n = PV RT m3 ) n= = 4.1 10 3 moles 8.31( J molK )293K 5 4 (10 Pa )(10 Ejemplo 12.6. Un envase e eso orante cerra o hermticamente, contiene un volumen e 200 cm3 e un gas i eal comprimi o a una presin e 2 atmsferas, en un ambiente a 23 C. En esas con iciones se tira al fuego, on e alcanza una temperatura e 1 27 C. Calcular la presin entro el envase. Despreciar cualquier cambio e volumen el envase. Solucin: Los atos son: Vi = 200cm3 = Vf, Ti = 23 C = 300K, Pi = 2 at m = 202kPa, Tf = 227 C = 400K, Pf = ? Suponien o que no hay pr i as e gas, el nmer o e moles permanece constante, entonces e la ecuacin e esta o se tiene: PV = nRT nR = PV P V PV = cte. i i = f f Ti Tf T Como Vi = Vf, se reduce a: T 101kPa 500 K Pi Pf = Pf = f Pi = 2atm = 3.33atm Ti 300 K 1atm Ti T f Pf = 336. 7kPa Este alto valor de presin puede hacer explotar el envase. 12.7 TEORIA CINETICA DE LOS GASES. Diferente al caso de la descripcin macroscpica en trminos de las variables presin, v olumen y temperatura, ahora se demostrar que esas propiedades se 346

Cap. 12. Temperatura, dilatacin trmica y gases. pueden describir en escala microscpica, donde la materia se trata como una colecc in de molculas. Las leyes de Ne ton aplicadas en forma estadstica a una coleccin de partculas proporcionan una descripcin razonable de los procesos termodinmicos. En e ste enfoque, llamado teora cintica, las molculas del gas se mueven en todas las dir ecciones de manera aleatoria, chocando contra las paredes del envase y entre s. L a importancia de esta teora es que proporciona las bases fsicas con las cuales de puede entender el concepto de temperatura. Nuestra descripcin terica la restringir emos a gases monoatmicos (un solo tomo) donde toda la energa es cintica de traslacin, no se consideran rotaciones ni vibraciones. En un modelo microscpico de un gas i deal, se considera que la presin que ejerce un gas en las paredes de un envase qu e lo contiene es por efecto de los choques de las molculas con las paredes. En es te modelo se hacen las siguientes suposiciones: 1. El nmero de molculas es grande y la separacin media entre ellas es grande comparada con sus dimensiones. Por lo tanto ocupan un volumen despreciable en comparacin con el volumen del envase y se consideran masas puntuales. 2. Las molculas obedecen las leyes de Ne ton, pero i ndividualmente se mueven en forma aleatoria, con diferentes velocidades cada una , pero con una velocidad promedio que no cambia con el tiempo. 3. Las molculas re alizan choques elsticos entre s, por lo tanto se conserva tanto el momento lineal como la energa cintica de las molculas. 4. Las fuerzas entre molculas son despreciab les, excepto durante el choque. Se considera que las fuerzas elctricas o nucleare s entre las molculas son de corto alcance, por lo tanto solo se consideran las fu erzas impulsivas que surgen durante el choque. 5. El gas es considerado puro, es decir todas las molculas son idnticas. 6. El gas se encuentra en equilibrio trmico con las paredes del envase. Se deducir una expresin para la presin de un gas ideal que consta de N molculas en un envase de volumen V. Se supone que el envase tien e forma de un cubo de lado d, como se ve en la figura 12.8. Considerar el choque de una molcula con velocidad v movindose contra la cara derecha de la caja. Las c omponentes de la velocidad de la molcula son vx, vy y vz. Al chocar elsticamente c on la pared, la componente x de la velocidad se invierte, pero las componentes y y z no se modifican, estas componentes (excepto la z) se ilustran en la 347

Cap. 12. Temperatura, dilatacin trmica y gases. figura 12.9. Como la componente x del momento de la molcula antes del choque es m vx y despus del choque se invierte siendo ahora -mvx, y la componente y del momen to lineal no cambia, la variacin del momento lineal de la molcula est dado por: y v -vx vy d m v d z x vy v vx d Figura 12.8 Caja cbica de lado d que contiene un gas ideal, mostrando una molcula que se mueve con rapidez v. Figura 12.9 Esquema de una molcula que realiza un choque elstico con una pared del envase que la contiene. p = mv x mv x = 2mv x La canti a e movimiento entrega a a la pare en ca a choque es 2mvx, ya que se conserva el momento el sistema molcula + envase. Para que una molcula realice o s choques sucesivos con la misma pare ebe recorrer una istancia 2 a lo largo el eje x en un tiempo t. Pero en el tiempo t, la molcula se mueve una distancia d = vxt en la direccin x, por lo tanto, el tiempo entre dos choques sucesivos es t = 2d/vx. Si F es la magnitud de la fuerza promedio ejercida por una molcula sobre la pared en el tiempo t, de la definicin del impulso, se obtiene: Ft = p = 2mv x 2mv x 2mv x mv x2 F= = = t 2d / v x d (12.13) 348

Cap. 12. Temperatura, dilatacin trmica y gases. La fuerza total sobre la pared es la suma de todos los trminos de este tipo corre spondientes a todas las partculas. Para obtener la presin total sobre la pared, se divide la fuerza total con el rea, de valor d2: P= F m 2 = 3 (v x1 + v x22 +) A d donde vx1, vx2, se refiere a as componentes x de a ve ocidad para as partcu as 1 , 2, etc. Como e va or medio de vx2 es: v x21 + v x22 + v = N 2 x

P= (12.14) E cuadrado de a rapidez de cua quier partcu a es: 2 v 2 = v x2 + v y + v z2 Ya que no existe direccin preferida por as mo cu as, os va ores medios 2 2 v x , v y y v z2 son igua es entre s. Entonces se deduce que: 2 v x2 = v y = v z2 = 1 v 2 3

P= (12.15) 349

con V

o cua

a presin dada en a ecuacin 12.14 se puede escribir como: 1 Nm 2 v 3

y e vo umen est dado por V = d3, se puede expresar V

a presin en a forma: Nm 2 vx

Cap. 12. Temperatura, di atacin trmica y gases. La cantidad Nm es a masa tota de as mo cu as, a cua es igua a n(PM), donde n es e nmero de mo es de gas y (PM) es e peso mo ecu ar. Por o tanto, a pres in tambin se puede expresar en a forma: 1 n( PM ) 2 v 3 V P= (12.16)

P= 2 N 1 2 mv 3V 2 (12.17) Esta ecuacin nos dice que la presin es proporcional al nmero de molculas por unidad de volumen y a la energa cintica media de traslacin por molcula. Con este modelo sim ple de un gas ideal se ha llegado a un resultado que relaciona las variables mac roscpicas de presin y volumen con una variable microscpica, la velocidad molecular media. As se ha encontrado una relacin entre el mundo microscpico de las molculas de un gas y el mundo macroscpico. An cuando este resultado se dedujo para un envase cbico, se puede generalizar y validar para un envase de cualquier forma que conte nga un gas ideal. 12.8 INTERPRETACION MOLECULAR DE LA TEMPERATURA. Es posible comprender mejor el significado de la temperatura escribiendo la ecua cin 12.17 en una forma ms conocida: PV = 2 1 2 N mv 3 2 350

Modificando a expresin 12.16, tambin se puede expresar

a presin como:

Cap. 12. Temperatura, dilatacin trmica y gases. Comparndola con la ecuacin de estado de un gas ideal: PV = NkT Igualando los segundos trminos de estas ecuaciones, se obtiene: T= 2 1 2 mv 3k 2 (12.18) Como 1 mv 2 es la energa cintica media de traslacin por molcula, se en2 cuentra que la temperatura es una medida directa de la energa cintica molecular media y se pue de escribir de la forma: T= 2 EC 3k Reacomodando la ecuacin 12.18, se puede relacionar la energa cintica media de trasl acin por molcula con la temperatura: 1 3 EC = mv 2 = kT 2 2 (12.19)

Es decir, la energa cintica media de traslacin por molcula es 3 kT . La 2 energa cint ca total de traslacin EC de N molculas de un gas es simplemente N veces la energa m edia por cada molcula, dada por la ecuacin 12.19, y usando las definiciones de k = R/NA y n =N/NA, se puede reescribir esta ecuacin para las N molculas, de la forma : 1 3 3 EC = N mv 2 = NkT = nRT 2 2 2 (12.20) 351

Cap. 12. Temperatura, dilatacin trmica y gases. Este resultado, junto con la ecuacin 12.17 indica que la presin ejercida por un ga s ideal slo depende del nmero de molculas por unidad de volumen y de la temperatura . La raz cuadrada de v 2 se llama raz cuadrtica media de la velocidad (rms) de las molculas. De la ecuacin 12.19 se puede despejar la rms: vrms = v 2 = 3kT 3RT = m ( PM ) (12.21) Esta expresin de la rms muestra que para una temperatura dada, las molculas ms livi anas, en promedio se mueven ms rpido que las ms pesadas, que tienen un PM mayor. Po r ejemplo el hidrgeno, con un peso molecular de 2 g/mol, se mueve cuatro veces ms rpido que el oxgeno, que tiene un peso molecular de 32 g/mol. La rapidez rms no es la rapidez con la que se mueve una molcula de gas, ya que tal molcula experimenta varios miles de millones de choques por segundo con otras molculas. En la tabla 12.3 se da valores de rms para algunas molculas conocidas, a 20 C. Tabla 12.3 Velocidad rms a 20 C para gases conocidos. Gas H2 He H2O Ne N2 CO NO C O2 SO2 PM (g/mol) 2.02 4 18 20.1 28 28 30 44 48 vrms a 20 C (m/s) 1902 1352 637 6 03 511 511 494 408 390 EJEMPLO 12.7 Un envase con un volumen de 0.3 m3 contiene 2 moles de helio a 20 C. Suponiendo que el helio se comporta como un gas ideal, calcular a) la energa cint ica total del sistema, b) la energa cintica promedio por molcula, c) la rms del hel io. 352

Cap. 12. Temperatura, dilatacin trmica y gases. Solucin: a) usando la ecuacin 12.20 con n = 2 y T =20 C = 293 K, se obtiene: 3 EC = nRT = 1.5(2mol )(8.31J / molK )(293K ) = 7300 J 2 b) de la ecuacin 12.19 se tien e: 1 3 EC = mv 2 = kT = 1.5(1.38 10 23 J / K )(293K ) = 6.1 10 21 J 2 2 c) usan o la ecuacin 12.21 vrms = 3RT 3(8.31J / molK )(293K ) = = 1350 m s ( PM ) 4 g / mol 353

Cap. 12. Temperatura, ilatacin trmica y gases. PROBLEMAS 12.1

Un termmetro e gas a volumen constante se calibra en hielo seco, que es bixi o e carbono, CO2, en esta o sli o, que tiene una temperatura e 80 C a 0.9 atm de pres i , y e el pu to de ebullici del alcohol etlico, que se produce a 78 C y a 1.64 at m. a) Qu valor de cero absoluto produce esta calibraci ? b) Cul es la presi e pu to de co gelamie to del agua? c) Cul es la presi e el pu to de ebullici del agua? R: a) 272.15 C, b) 1.27 atm, c) 1.74 atm. U termmetro de gas da u a lectura de 40 mm Hg para la presi e el pu to triple del agua. Calcular la presi e el pu to a ) de ebullici del agua, b) de fu dici del oro (1064.4 C). R: a) 54.6 mm Hg, b) 19 5.86 mm Hg. U termmetro de gas a volume co sta te registra u a presi de 50 mm H g cua do est a u a temperatura de 450 K. Calcular: a) la presi e el pu to triple del agua, b) la temperatura cua do la presi es 2 mm Hg. R: a) 82.3 mm Hg, b) 10 .93 K. La presi e u termmetro de gas a volume co sta te es de 0.7 atm a 100 C y de 0.512 atm a 0 C. Calcular: a) la temperatura cua do la presi es 0.04 atm, b) la presi a 450 C. R: a) 251 C, b) 1.36 atm. U termmetro de gas a volume co sta t e se lle a co helio. Cua do se sumerge e itrge o lquido hirvie do a u a tempera tura de 77.34 K, la presi absoluta es 25 kPa. Calcular a) la temperatura e grad os Celsius y Kelvi cua do la presi es 45 kPa, b) la presi cua do el termmetro se sumerge e hidrge o lquido hirvie do ( 252.9 C). R: a) 134 C, 139 K, b) 6.56 kPa. U a de las temperaturas ms altas registradas sobre la tierra fue de 136 F (Libia 1 922) y u a de las ms baja fue de 127 F (A trtica 1960). Exprese estas temperaturas e grados Celsius. R: a) 57.8 C, b) 88.3 C. La temperatura ormal del cuerpo hum a o es 98.6 F. U io co fiebre puede registrar 102 F. Expresar esos valores e gr ados Celsius. 12.2 12.3 12.4 12.5 12.6 12.7 354

   

  

 

 

  

 

    

  

 

 

 

 

  

 

 

 

 

 

 

 

 

 

Cap. 12. Temperatura, dilataci 12.8

12.9 12.10 E u a escala de temperatura desco ocida (D), el pu to de co gelaci del ag ua es 15 D y el pu to de ebullici es 60 D. bte er la ecuaci li eal e tre la escala D y la Celsius. R: TD = 0.75TC 15. 12.11 E u a escala de temperatura i cg ita (X), el pu to de co gelaci del agua es 30 X y el pu to de ebullici es 80 X. bte ga la ecuaci li eal e tre la escala X y la Fahre heit. 12.12 La temperatura i ic ial de u objeto tie e el mismo valor umrico e grados C y grados F. Ms tarde, la temperatura cambia, para esta ueva temperatura hay u a relaci de 1:3 ( de 3:1) e tre sus valores e C y F. Calcular el cambio de temperatura, e grados K. 12.13 Medidas precisas de temperatura se puede hacer usa do el cambio de resiste cia elctrica de u metal co la temperatura. Si la resiste cia varia aproximadame te seg la relaci R = Ro(1 + ATC ), do de Ro y A so co sta tes y TC la temperatura e grados Celsius. Cierto material tie e u a resiste cia de 50 ( hms) a 0 C y de 71.5 e el pu to de co gelamie to del estao (232 C). Calcular: a) las co sta tes R o y A, b) la temperatura cua do la resiste cia es 89 . R: a) Ro = 50 , A = 1.85x10 3 C 1, b) 421 C. 12.14 Para i terpolar temperaturas e la escala prctica i ter aci o al, se usa u termmetro de resiste cia de plati o co especificacio es defi ida s e el i tervalo e tre 0 C y 960 C. La temperatura TC, e grados Celsius, est dete rmi ada por la frmula R = Ro(1 + ATC + BTC2), que se aplica a la variaci de la re siste cia R co la temperatura. Las co sta tes Ro, A y B se determi a por medid as e pu tos fijos. Si R = 1000 e el pu to de fusi del hielo, R = 1050 e el pu to de ebullici del agua y R = 1500 e el pu to de fusi de la plata 355

 

 

 

 

 

 

 

 

 

 

 

El pu to de fusi omo es de 327.3 C s valores a grados s de u termmetro

del oro es de 1064 C, el de ebullici de 2660 C, el de fusi del pl y el de ebullici del itrge o lquido es 195.8 C. Tra sformar esto Kelvi y Fahre heit. A que temperatura so iguales las lectura Fahre heit y u termmetro Celsius? R: 40 C.

trmica y gases.

 

 

 

 

 

Cap. 12. Temperatura, dilataci

(950 C), a) calcular las co sta tes. b) Trazar la curva de calibraci del termmetro . Sugere cia: puede usar alg soft are (por ejemplo pla illa Excel) para ajustar la ley cuadrtica a los pu tos y trazar la curva e el ra go de operaci . R: a) Ro = 1000 , A = 5x10 4 C 1 , B= 3.1x10 8 C 2. 12.15 U a barra de vidrio com tie e 30 c m de largo y 1.5 cm de dimetro. Calcular la expa si de su a) lo gitud, b) dimetro y c) volume , cua do la temperatura aume ta e 65 C. R: a) 0.176 mm, b) 8.8 m, c) 93 mm3. 12.16 U cable de cobre se cuelga e tre dos pu tos separados 35 m (la cu rva que forma el cable suspe dido se llama cate aria). Calcular su variaci de lo gitud e u da de vera o co 35 C respecto a u da de i vier o co 20 C. R: 3.27 c m. 12.17 U a viga estructural de acero tie e 15 m de largo cua do se i stala a 2 0 C. Calcular el cambio de su lo gitud para variacio es extremas de temperatura d e 30 C a 50 C. R: 1.32 cm. 12.18 U riel de acero tie e 20 m de largo cua do se i stala e u a va a la temperatura ambie te de 20 C. Calcular el cambio e su lo gi tud si las variacio es esperadas de temperatura e tre i vier o y vera o fuera d e 20 C a 40 C. R: 0.0144 m. 12.19 U a illo de lat que tie e 10 cm de dimetro cua do est a 20 C se calie ta para hacerlo deslizar sobre u a barra de alumi io de 10.01 cm de dimetro a 20 C. Supo ie do co sta tes los coeficie tes medios de expa si li eal. a) qu temperatura debe alca zar al me os el a illo? b) a qu temperatura debe e friarse esta combi aci para separarla? Esto, puede lograrse? c) Qu ocurre si la ba rra de alumi io tuviera 10.02cm de dimetro? R: a) 72.6 C, b) 146 C, o co facilid ad. 12.20 Demostrar que el coeficie te promedio de dilataci superficial para u slido isotrpico es = 2 y el de dil t cin volumtric es = 3. 12.21 Un nillo de lu tiene un dimetro interior de 5 cm y un v rill de ronce tiene un dimetro de 5.0 5 cm cu ndo estn 20 C. ) C lcul r l temper tur l que se de e c lent r el nillo de luminio p r 356

 

  

 

 

  

 

trmica y gases.

 

  

 

   

C p. 12. Temper tur , dil t cin trmic

que se juste so re l v rill de ronce. ) C lcul r l temper tur l que se de en c lent r m os p r que el nillo de luminio se juste so re l v rill de ronce; coment r. R: ) 420 C, ) 1741 C. 12.22 L s secciones de concreto de ci ert utopist se dise n p r tener un lon itud de 25 m. L s secciones se v c n y fr u n 10 C. Qu esp ci miento mnimo entre l s secciones de er dise r el in enier p r elimin r el p ndeo, si el concreto lc nz r un temper tur de 50 C? R: 1. 2 cm. 12.23 Un riel de cero tiene 20 m de l r o. Se junt n los rieles por los e xtremos con un esp cio de om entre ellos. El coeficiente de dil t cin line l de l om -22x10-5 C-1. C lcul r el espesor que de e tener l om p r que se con tr i lo mismo que se exp nden los rieles cu ndo l temper tur umente en 30 C. R: 1.1mm 12.24 Un cilindro hueco de luminio tiene 20 C un c p cid d intern de 2 litros y 15 cm de fondo. El conjunto se llen complet mente con petrleo y lu e o se c lient h st 80 C. Posteriormente se enfr de nuevo h st 20 C. ) Qu c ntid d de petrleo se derr m l c lent r el conjunto? ) A qu dist nci jo el orde d el cilindro est r l superficie de petrleo? c) Comente l posi ilid d de despreci r l dil t cin del depsito. R: ) 0.099 lt, ) 0.75 cm. 12.25 El est nque de enci n de un utomvil se llen h st el orde con 45 litros de l mism 10 C, justo ntes de est cion rlo l sol 35 C. C lcul r l c ntid d de encin que se derr m r por efecto de l exp nsin. R: 1.08 lt. 12.26 Un rr de co re y otr de cer o sufren los mismos c m ios de temper tur . A 0 C l rr de co re tiene un lon itud Lc y l de cero un lon itud LA. Cu ndo l s rr s se c lient n o se enfr n, se m ntiene un diferenci de 5 cm entre sus lon itudes. Determine los v lore s de LC y LA. R: LC = 17 cm, LA = 12 cm. 12.27 Un fluido tiene un densid d . a) Demuest e que el cambio f acciona io en la densidad a a un cambio T en la temper atura est dado por la expresin / = T. Qu significa el signo negativo? El agua

357

"

     "

 

"   

"

 "
y

ses.

"

" 

"

"

" "

 

"

"

Cap. 12. Temperatura, dilatacin trmica y gases. tiene una densidad mxima de 1 gr/cm3 a 4 C. A 10 C su densidad es 0.9997 gr/cm3, ca lcular p r el u en ese interv lo de temper tur . 12.28 Demuestre que un mol de cu lquier s presin tmosfric estnd r y temper tur estnd r ocup un volumen de 22.4 litros. 12.29 Un uditorio tiene dimensiones de 10m x 20m x 30m. Cunt s mo lcul s de ire se necesit n p r llen r el uditorio 20 C y 1 tm? R: 1.5 x 1029 molcul s. 12.30 Un cilindro con un m olo mvil contiene un s un temper tur de 125 C, un presin de 30 kP y un volumen de 4 m3. C lcul r su temper tur fin l s i el s se comprime 2.5 m3 y l presin ument 90 kP . R: 739.1 K. 12.31 Un s encerr do en un est nque, est un presin de 30 tm y un temper tur de 15 C. Si se s c l mit d del s y se ument l temper tur 65 C, c lcul r l nu ev presin en el est nque. R: 1.76 tm. 12.32 Un mol de ox eno est un presin de 6 tm y 25 C de temper tur . ) Si el s se c lient volumen const nte h st que l presin se triplic , c lcul r l temper tur fin l. ) Si el s se c lient de m ner que t nto l presin como el volumen se duplic n, c lcul r l temper t ur fin l. R: ) 894 K, ) 1192 K. 12.33 Un ln que tiene un volumen de 0.1 m3 c ontiene s helio 50 tm. Cuntos lo os se pueden infl r si c d uno es un esfe r de 0.3 m de dimetro un presin solut de 1.2 tm? Supon que l temper tur del s perm nece const nte dur nte el llen do de los lo os. R: prox. 300 l o os. 12.34 Un ur uj de s en un l o su e desde un profundid d de 4.2 m, d onde h y un temper tur de 5 C h st l superficie donde l temper tur es de 12 C. C lcul r l rel cin entre los r dios de l ur uj en los dos puntos. R: r dio fin l = 1.12 r dio inici l. 12.35 A 25 m de jo de l superficie del m r (densi d d 1025 k /m3), donde l temper tur es de 5 C, un uzo exh l un ur uj de i re que tiene un volumen de 1 cm3. Si l temper tur de l superficie del m r es 358

"       " "     " "  " "  "  "   "  " " " "

"

" "

"

C p. 12. Temper tur , dil t cin trmic

i u l 20 C, c lcul r el volumen de l ur uj ntes que se romp l lle r l superficie. R: 3.7 cm3. 12.36 Un ln lleno contiene 12 k de ox eno, O2, jo un presin m nomtric de 40 tm. Determine l m s de ox eno que se h extr do del ln cu ndo l presin solut lc nz el v lor de 25 tm. Supon que l temper tur del ln perm nece const nte. R: 7.3 k . 12.37 Un lo o poroso tiene un volumen de 2 m3 un temper tur de 10 C y un presin de 1.1 tm. Cu ndo se c lient h st 150 C el volumen ument 2.3 m3 y se h esc p do por los poros el 5% del s. C lcul r ) l c ntid d de s, en moles, que h en el lo o 10 C, ) l pr esin en el lo o 150 C. R: ) 94.8 moles, ) 0.8 tm. 12.38 ) L ll nt de un utomvil se infl us ndo ire ori in lmente 10 C y presin tmosfric norm l. Dur nt e el proceso, el ire se comprime h st 28% de su volumen ori in l y l temper t ur ument 40 C. C lcul r l presin de l ll nt . ) Despus que l ll nt se uti liz lt velocid d, l temper tur del ire dentro de l mism se elev 85 C y su volumen interior ument 2%. C lcul r l nuev presin ( solut ) de l ll n t . R: ) 3.98 x 105 P , ) 15 x 105 P . 12.39 L t p de un env se cilndrico est conect d por un resorte de const nte elstic 2x103 N/m un p rte superior fij , como muestr l fi ur 12.10. El cilindro est lleno con 5 litros de s 1 tm y 20 C, con el resorte sin estir r. L t p tiene un re de seccin tr nsvers l de 100 cm2 y m s despreci le. C lcul r: ) l ltur l que su e l t p cu ndo l temper tur ument h st 250 C, ) l presin del s 250 C. R: ) 16.9 cm. Fi ur 12.10 Pro lem 359 12.39

"

 " 

 "  "  "

"

"

"

"  "   "   " " 

"

"

  "
y

ses.

"

"

" 

12.40 ) Demuestre que l densid d de un s que ocup un volumen V est d d po r = P(PM)/RT, donde PM es el eso (o masa) molecula del gas. b) Calcula la den sidad del oxgeno y la del nit geno a la esin atmosf ica y a 20 C. 12.41 La tabla 12 .4 da la com osicin del ai e debajo de los 80 km de altu a. Calcula : a) las es iones a ciales de los gases ms abundantes a la esin atmosf ica no mal, b) el vol umen ocu ado o 100 g de ai e a 15 C y 1 atm, c) la densidad del ai e en esas co ndiciones, d) el eso molecula , PM, del ai e seco. R: c) 1.28 kg/m3, d) 28.964 g /mol. Tabla 12.4 Com osicin de la atmsfe a. Gas Nit geno Oxgeno A gn Nen Helio Metano K i tn hid geno xenn ozono yodo adn ot os... o de ca bono va o de agua F mula N2 O2 A Ne He CH4 K H2 X O3 I Rn CO2 H2O PM 28.0 32.0 39.94 20.2 4.0 16.0 83.8 2.0 131.3 48.0 126.9 222.0 44 18 Volumen en % 78.09 20.95 0.93 1.8 x 10 3 5.3 x 10 4 1.5 x 10 4 1.1 x 10 4 5.0 x 10 5 8.0 x 10 6 1.0 x 10 8 3.5 x 10 9 6.0 x 10 18 menos de 10 10 0.02 0.04 0.0 4.0 12.42 Dos moles de oxgeno dent o de un envase de 5 lit os estn a una esin de 8 at m. Calcula la ene ga cintica media de una molcula de oxgeno, de masa 5.31x10 26 kg. 12.43 Du ante un e iodo de 1 s, 5x1023 molculas gol ean una a ed sob e un ea d e 8 cm2. Si las molculas se mueven con una a idez de 300 m/s y chocan en un ngulo de 45 es ecto a la no mal de la a ed, calcula la esin eje cida sob e la a e d. (La masa de una molcula de nit geno es 4.65x10 26 kg). 360

"

C p. 12. Temper tur , dil t cin trmic

"

ses.

Ca . 12. Tem e atu a, dilatacin t mica y gases. 12.44 Un globo esf ico de volumen 4000 cm3 contiene helio a una esin de 1.2 atm. Si cada tomo de helio tiene una ene ga cintica media de 3.6x10 22 J, calcula el nm e o de moles de helio dent o del globo. R: 3.3 mol. 12.45 En un inte valo de 30 s, 500 g anizos que caen en un ngulo de 45 es ecto a una ventana, chocan cont a e l vid io de la ventana que tiene un ea de 0.6 m2. Cada g anizo tiene una masa de 5 g y una a idez de 8 m/s. Si se su one que los choques son elsticos, calcula la fue za y la esin media sob e el vid io. 12.46 Un cilind o contiene una mezcl a de helio y a gn en equilib io a una tem e atu a de 150 C. Calcula la ene ga cinti ca media de cada molcula de gas en el envase. R: 8.8x10 21 J. 12.47 Calcula : a) la tem e atu a a a la cual la ms de un tomo de helio es 500 m/s, b) la ms del helio sob e la su e ficie del Sol, con una tem e atu a de 6100K. R: a) 40.1 K, b ) 6.2 km/s. 12.48 Un envase de 5 lit os contiene nit geno a una tem e atu a de 27 C y a una esin de 3 atm. Calcula : a) la ene ga cintica total de las molculas, b) la ene ga cintica media o molcula. 12.49 Calcula la a idez cuad tica media de la s molculas de nit geno, N2, y de bixido de ca bono, CO2, en condiciones no males. L a masa (o eso) molecula del N2 es 28 g/mol y la del CO2 es 44 g/mol. R: 506.4 m/s, 403.9 m/s. 12.50 Calcula la tem e atu a a a la cual la a idez cuad tica m edia de las molculas de oxgeno, O2, es igual a la de las molculas de hid geno, H2, a 27 C. La masa molecula del H2 es 2,02 g/mol y la del O2 es 32 g/mol. Conside e que los esultados de la Teo a cintica de los gases se ueden a lica a estos gase s. R: 4752.5 K. 12.51 Calcula : a) el nme o de tomos de helio necesa ios a a llen a un globo hasta un dimet o de 30 cm, a 20 C y 1 atm, b) la ene ga cintica media de cada tomo, c) la a idez media de cada tomo de helio. R: a) 3.5x1023 tomos, b) 6.0 7x10 2 J, c) 1341.6 m/s. 361

Ca . 12. Tem e atu a, dilatacin t mica y gases. 12.52 El Helio tiene una masa atmica de 4 g/mol. Calcula : a) la ene ga cintica de t aslacin media de una molcula de He a 300K, b) la a idez cuad tica media, c) el m omento lineal de una molcula de He si viaja con esa a idez. Su onga que un cie t o nme o de tomos de He ocu an un eci iente cbico de 0.1 m de lado y se encuent an a 1 atm y 300 K. Calcula : d) la fue za media que eje ce un tomo de He sob e una de las a edes del eci iente cuando su velocidad es e endicula a los lados o uestos que gol ea al ebota , e) el nme o de tomos viajando a esa a idez, en una misma di eccin, que se necesitan a a oduci una esin media de 1 atm, f) El nm e o de tomos contenidas ealmente un eci iente de ese tamao y en esas condiciones , g) su es uesta de f) debe se 3 veces mayo que la obtenida en e), qu o igen ti ene esa disc e ancia? R: a) 6.2x10 21 J, b) 1367.5 m/s, 5.5 kgm/s, d) 1.5x105 N, e) 2.5x1017 tomos.

362

Ca . 13. Calo y la P ime a Ley de la Te modinmica

CAPITULO 13. CALOR Y LA PRIMERA LEY DE LA TERMODINAMICA. La te modinmica es la a ma de la fsica que estudia los ocesos donde hay t ansfe encia de ene ga en fo ma de calo y de t abajo. Cuando dos cue os a dife entes tem e atu as se onen en contacto t mico ent e s, la tem e atu a del cue o ms clido disminuye y la del ms f aumenta. Si e manecen en contacto t mico du ante cie to tiem o, finalmente alca nzan una tem e atu a comn de equilib io, de valo com endido ent e las tem e atu as iniciales. En este oceso se odujo una t ansfe encia de calo del cue o ms clido al ms f o. La egunta que su ge es cules son las ca acte sticas de esa t a e encia de calo ? En el ximo ca tulo intenta emos da una es uesta a esa egun ta, ya que en este debemos a ende a conoce la ca acidad de abso be o libe a calo de los cue os, las dife entes fo mas de calo , el t abajo te modinmico, l a ene ga inte na de los cue os y como se elacionan ent e s esas va iables a t avs de la ime a ley de la te modinmica. 13.1 DEFINICIONES. Sistema: cualquie g u o de tomos, molculas, a tculas u objetos en estudio te modinmico. Po ejem lo el agua dent o de un envase, el cue o de u n se vivo o la atmsfe a. Un esquema se muest a en la figu a 13.1. Ambiente: todo lo que no e tenece al sistema, es lo que odea al sistema, sus al ededo es. Po ejem lo el exte io al envase donde est el agua, o el es acio que odea a la at msfe a ( uede se todo el Unive so). Ent e el sistema y el ambiente uede habe i nte cambio de calo y de ene ga y se uede ealiza t abajo (figu a 13.1). Sistem a ce ado: sistema en el cual no ent a ni sale masa, e o que uede inte cambia calo y ene ga con el ambiente. Sistema abie to: sistema que uede tene va iacin de masa, como o ejem lo inte cambio de gases o lquidos, o de alimentos en los se es vivos. Sistema ce ado aislado: sistema en el cual no se oduce ningn inte cambio de calo o ene ga con el ambiente a t avs de sus f onte as. 363

Ca . 13. Calo y la P ime a Ley de la Te modinmica Ambiente Sistema T abajo Calo Figu a 13.1. Esquema donde se e esenta un siste ma te modinmico odeado o su ambiente. 13.2 CALOR. Se debe distingui desde un inci io cla amente ent e los conce tos de calo y ene ga inte na de un objeto. El calo , (smbolo Q), se define como la e ne ga cintica total de todos los tomos o molculas de una sustancia. El conce to de c alo , se usa a a desc ibi la ene ga que se t ansfie e de un luga a ot o, es de ci flujo de calo es una t ansfe encia de ene ga que se oduce nicamente como co nsecuencia de las dife encias de tem e atu a. La ene ga inte na, estudia emos ms e n detalle en la seccin 13.6, es la ene ga que tiene una sustancia debido a su tem e atu a. La ene ga inte na de un gas es esencialmente su ene ga cintica en escala m ic osc ica: mient as mayo sea la tem e atu a del gas, mayo se su ene ga inte na. Pe o tambin uede habe t ansfe encia de ene ga ent e dos sistemas, an cuando no h aya flujo de calo . Po ejem lo, cuando un objeto esbala sob e una su e ficie h asta detene se o efecto de la f iccin, su ene ga cintica se t ansfo ma en ene ga i nte na que se e a te ent e la su e ficie y el objeto (y aumentan su tem e atu a ) debido al t abajo mecnico ealizado, que le ag ega ene ga al sistema. Estos camb ios de ene ga inte na se miden o los cambios de tem e atu a. Cuando la ciencia te modinmica e a bebe, digamos a inci ios del 1800, y no se com enda bien el co nce to de calo , los cientficos definie on el calo en t minos de los cambios en l a tem e atu a que el calo oduce en los cue os. Po lo que se defini una unida d de medida del calo , llamada calo a, smbolo cal, como la cantidad de calo neces a ia a a eleva la tem e atu a de un g amo de agua en un g ado Celsius desde 14 .5 C a 15.5 C. La unidad de calo en el sistema ingles se llama Unidad t mica b itni ca, (Btu), definida como la cantidad de calo necesa ia a a eleva la tem e atu a de una 364

Ca . 13. Calo y la P ime a Ley de la Te modinmica lib a de agua en un g ado Celsius de 63 F a 64 F. Se elige ese ango de tem e atu a, o que la cantidad de calo eque ida de ende levemente de la tem e atu a; se equie e ms calo a a eleva la tem e atu a del agua f a que la del agua a unto de he vi . Cuando se desc ibi el conce to de ene ga en el ca tulo 5, se afi mo que en cualquie sistema mecnico siem e esta esente la f iccin, o lo que siem e se ie de ene ga mecnica y a a entemente no se conse va. Los ex e imentos demuest an cla amente que o efecto de la f iccin, la ene ga no desa a ece, sino que se t ansfo ma en ene ga t mica. James Joule (ingls, 1818 1889) fue el ime o en estab lece la equivalencia ent e estas dos fo mas de ene ga. Joule encont que la ene ga mecnica que se t ansfo ma en calo , es o o cional al aumento de tem e atu a. L a constante de o o cionalidad, llamada calo es ecfico, es igual a 4.186 J/(g C) . Se demuest a que una calo a, que se conoce como el equivalente mecnico del calo , es exactamente igual a 4.186 J, sin im o ta quien oduce el aumento de tem e atu a: 1 cal = 4.186 J Como en la actualidad se econoce al calo como una fo m a de ene ga, la unidad de medida de calo en el SI es el Joule, J. Algunas de las conve siones ms comunes ent es las unidades de calo y ene ga son las siguientes: 1 cal = 4.186 J = 3.97x10 3 Btu 1 J = 0.239 cal = 9.48x10 4 Btu 1 Btu = 1055 J = 252 cal En nut icin se llama Calo a, Cal con mayscula, a las calo as alimenticias o dietticas, usada en la desc i cin del contenido de ene ga de los alimentos y equi vale a 1000 calo as o 1kilocalo a, es deci 1 Cal = 1kcal = 1000 cal. Ejem lo 13.1 Una lola se si ve 1000 Cal en alimentos, los que luego quie e e de levantando esas de 25 kg hasta una altu a de 1.8 m. Calcula el nme o de veces que debe le vanta las esas a a e de la misma cantidad de ene ga que adqui i en alimentos y el tiem o que debe esta haciendo el eje cicio. Su onga que du ante el eje cic io no se ie de ene ga o f iccin. 365

Ca . 13. Calo y la P ime a Ley de la Te modinmica Solucin: a a e de las 1000 Cal, la lola debe ealiza la misma cantidad de t a bajo mecnico, es deci W = 1000 Cal. T ansfo mando este valo al SI:

Esta es la cantidad de t abajo que debe se ealizado levantando esas de 25 kg. El t abajo en un solo levantamiento hasta 1.8 m es: W1 = mgy = (25kg)x(10m/s2)x (1.8m) = 450J Como el t abajo W1 debe se ealizado n veces hasta com leta la c antidad W, entonces W = n W1, des ejando n, W 4.186 106 J n= = = 9300 veces W1 450 J Su ongamos que la lola es muy ida a a levanta esas, tal que oduce un levantamiento cada 5 segundos, entonces el t iem o total del eje cicio es: t = 9300 5s 1h = 12.9 horas 3600s Por lo que es obvio que es ms fcil bajar de peso hacie do dieta. 13.3 CAPACIDAD CAL RICA Y CAL R ESPECIFIC . La ca tidad de e erga e forma de calor que se requiere para cambiar la temperatu ra de u a masa dada de materia, o es la misma para todos los materiales. Por ej emplo, el calor ecesario para elevar la temperatura e u grado Celsius de u k ilogramo de agua es 4186 J, pero el calor ecesario para elevar la temperatura e 1 C de 1 kg de cobre es solo 387 J. La capacidad calrica, C, de cualquier susta cia se defi e como la ca tidad de cal or, Q, que se requiere para elevar la temperatura de u a susta cia e u grado C elsius. 366

 

1000cal 4.186 J

W = 1000Cal

= 4.186

106 J 1Cal 1cal

Cap. 13. Calor y la Primera Ley de la Termodi mica A partir de esta defi ici , se observa que si al agregar Q u idades de calor a u a susta cia le produce u cambio de temperatura T, se puede escribir: Q = C T (13 .1) La capacidad calrica de cualquier sustancia es proporcional a su masa. Por esta r azn es conveniente definir la capacidad calrica por unidad de masa, es decir que n o dependa de la masa, a la que se llama calor especfico, c: c= C m (13.2) La unidad de medida de C en el SI es J/K (que es lo mismo que J/C) y la de c es J /kgK (o J/(kg C)). En la tabla 13.1 se da el calor especfico de varias sustancias medidas a presin atmosfrica y a temperatura ambiente. Los calores especficos en gen eral varan con la temperatura. Si la variacin de temperatura no es muy grande, se puede despreciar esa variacin de c y considerarla como una constante. Tambin se pu ede definir el calor especfico molar de una sustancia como la capacidad calrica po r unidad de moles, entonces una sustancia que contiene n moles, tiene un calor e specfico molar igual a c = C/n, que se mide en el SI en J/(mol K) o J/(mol C). Val ores se listan en la ltima columna de la tabla 13.1. Tabla 13.1 Calores especficos de algunos materiales. Sustancia Agua (15 C) Alcohol Hielo (-5 C) Berilio Madera (aprox) Aluminio Mrmol (C aCO3) Vidrio Hierro Cobre Latn Plata Cadmio Mercurio Oro Plomo c (J/kg K) 4186 24 00 2090 1830 1700 900 860 837 448 387 380 234 230 140 129 128 PM (kg/mol) 0,0180 0,0180 0,00901 0,0270 c molar (J/mol K) 75,4 36,5 16,5 24,3 0,0559 25,0 24,5 25,4 25,9 27,7 25,4 26,4 0,108 0,210 0,207 367

  

Cap. 13. Calor y la Primera Ley de la Termodinmica Se puede observar de la tabla 13.1 que de los materiales comunes, el agua es la que tiene el mayor calor especfico. Este gran valor de c del agua, que es casi tr es veces mayor que para las tierras (cagua = 3ctierra), es un importante factor climtico sobre la superficie de la Tierra, ya que es en parte responsable de la m oderacin de temperatura en las zonas costeras. Se requiere mucho ms calor para ele var la temperatura del agua, que de una misma cantidad de tierra, es decir una m isma cantidad de radiacin solar eleva ms la temperatura de los suelos que de las a guas. En latitudes medias, las grandes masas ocenicas tienen una menor variacin di urna o anual de temperatura que los continentes y en general, las temperaturas d e los ocanos son menores (mayores) en verano (invierno) que en los continentes. C omo en latitudes medias, el viento de gran escala predominante es desde el oeste , las masas de aire que se acercan a los continentes transportan aire mas fresco en verano o mas clido en invierno, por lo que las zonas costeras occidentales de los continentes son mas frescas en verano y mas clidas en invierno que las zonas interiores de los continentes. Esto no siempre es as en los bordes orientales de los continentes, ya que en latitudes medias los vientos del oeste transportan e l aire desde el continente hacia el ocano, por lo que no puede haber efecto regul ador de los ocanos. De la definicin del calor especfico de la ecuacin 13.2, se puede determinar la energa calrica Q transferida entre una sustancia de masa m y los al rededores para un cambio de temperatura, como: Q = mc T (13.3) Observar que cuando se le agrega calor a una sustancia, Q y T son ambos positivos y la temperatura aumenta. Cuando se le quita calor a una sustancia, Q y T son am bos negativos y la temperatura disminuye. Una forma de medir el calor especfico d e slidos o lquidos consiste en calentar el material hasta una cierta temperatura, ponerla en un envase con una masa de agua y temperatura conocidas y medir la tem peratura del agua una vez que se ha alcanzado el equilibrio trmico. La ley de con servacin de la energa requiere que el calor que entrega el material mas caliente, de calor especfico 368

Cap. 13. Calor y la Primera Ley de la Termodinmica desconocido, sea igual al calor que absorbe el agua. Los aparatos en los cuales se produce esa transferencia de calor, se llaman calormetros. Ejemplo 13.2 Un tro zo de material de masa m que tiene una temperatura inicial Tim, se sumerge en un envase que contiene una masa M de agua a la temperatura inicial TiA < Tim. Si l a temperatura de equilibrio de la mezcla es T, calcular el calor especfico del ma terial. Despreciar la transferencia de calor al envase y al ambiente. Solucin: co mo la temperatura inicial del agua es menor que la del material, este le entrega calor al agua. Cuando se alcanza el estado de equilibrio, por la conservacin de la energa, el calor Qm entregado por el material debe ser igual al calor QA absor bido por el agua, entonces: Calor perdido por el material: Qm =-mcT = -mc(T - Tim ) Calor ganado por el agua: QA =McAT = McA(T - TiA) QA = Qm McA(T - TiA) = mc(Tim - T) Despejando el calor especfico c del material, se obtiene: c= Mc A (T TiA ) m(Tim T ) Ejemplo 13.3 Un trozo e metal e 50 g que se encuentra a 200 C se sumerge en un envase que contiene 0.4 kg e agua inicialmente a 20 C. Si la temperatura final e equilibrio el sistema mezcla o es 22.4 C, calcular: a) el calor especfico el aterial, b) el calor gana o por el agua. Despreciar la transferencia e calor al envase y al me io ambiente. Solucin: los atos son cA=4186 J/kg C, mm = 50g, Tim 200 C, mA = 400g, TiA = 20 C, Tfm=22.4 C =TfA. a) Al intro ucir el metal caliente el agua mas fra, el metal se enfra y el agua se calienta, alcanzan o ambos 22.4 es ecir, el metal pier e calor y el agua gana calor. 369

m = en C,

Cap. 13. Calor y la Primera Ley e la Termo inmica Calor ce i o por el metal: Calor gana o por el agua:

QA = Qm mA cA(TfA - TiA) = -mm cm (Tfm - Tim) cm = mAc A (T fA TiA ) mm (T fm Tim ) Reemplazan o los valores numricos, se obtiene: cm = 0.4 4186 (22.4 20) = 452.5 J kg C 0.05(22.4 200) b) El calor gana o por el agua es A = mA cA(TfA TiA), con los valores: A = 0.4kg 4186 J (22.4 C 20 C ) = 4018.6 J kg C

Ejemplo 13.4. Una bala e plomo e 2 g e masa ispara a con una rapi ez e 300 m/s, se incrusta en un poste e ma era. Suponien o que to a la energa trmica gener a a urante el impacto permanece en la bala, calcular su cambio e temperatura. Solucin: los atos son m = 2 g, v = 300 m/s. La energa cintica e la bala es: EC = 1 mv 2 = 1 (0.002kg )(300m / s ) 2 = 90 J 2 2 To a esta energa cintica se transforma en calor en la bala, como para el plomo c = 128 J/kg C, entonces: = mcT T = Q 90 J = = 351.6 C mc (0.002kg )(128 J / kg C ) 370

# #

m = mm cm T = - mm cm (Tfm - Tim) QA = mA cAT = mA cA(TfA - TiA)

Cap. 13. Calor y la Primera Ley de la Termodinmica 13.4 CALOR LATENTE Y CAMBIOS DE ESTADO. Normalmente en un material se produce un cambio de su temperatura cuando se tran sfiere calor entre el material y los alrededores. Cuando se le agrega o quita ca lor a una sustancia, se producen variaciones de temperatura (aumento o disminucin ), es el calor Q llamado calor sensible, porque el objeto siente el calor agrega do o perdido al cambiar su temperatura. Pero en ciertas condiciones se le agrega calor a una sustancia sin que cambie su temperatura, por ejemplo cuando se evap ora el agua, en ese caso se produce un cambio en las caractersticas fsicas y en la forma del material, llamado cambio de estado o de fase y al calor necesario par a producir el cambio de fase se le llama calor latente, porque este calor est pre sente y a punto para ser usado cuando termina el proceso de cambio de estado. Po r ejemplo, si se hierve agua en un recipiente abierto a la presin atmosfrica norma l, la temperatura no aumenta por encima de los 100 C por mucho calor que se sumin istre. El calor que se absorbe sin cambiar la temperatura del agua es el calor l atente; no se pierde, sino que se emplea en transformar el agua en vapor y se al macena como energa en el vapor. Cuando el vapor se condensa para formar agua, est a energa vuelve a liberarse, recuperndose el calor latente como calor sensible. De l mismo modo, si se calienta una mezcla de hielo y agua, su temperatura no cambi a hasta que se funde todo el hielo. El calor latente absorbido se emplea para ve ncer las fuerzas que mantienen unidas las partculas de hielo, y se almacena como energa en el agua. Los diferentes cambios de fase son de slido a lquido o fusin (fun dicin o derretimiento en el caso del agua), de lquido a gas o evaporacin (vaporizac in), de slido a gas o sublimacin y los procesos en sentido opuesto llamados solidif icacin (o congelamiento en el caso del agua), condensacin y deposicin, respectivame nte. Los diferentes procesos de cambio de estado, tomado como sustancia el agua, para los cuales se da una breve explicacin cualitativa, se ilustran en la figura 13.2. 13.4.1 Vaporizacin o evaporacin. Es la transformacin de lquido a gas. La evaporacin e s la conversin gradual de un lquido en gas sin que haya ebullicin, que se realiza e n la superficie del lquido. Las molculas de cualquier lquido se encuentran en const ante movimiento. La velocidad media de las molculas slo depende de la temperatura, pero puede haber molculas individuales que se muevan a una velocidad mu371

Cap. 13. Calor y la Primera Ley de la Termodinmica cho mayor o mucho menor que la media. A temperaturas por debajo del punto de ebu llicin, es posible que molculas individuales que se aproximen a la superficie con una velocidad superior a la media tengan suficiente energa para escapar de la sup erficie y pasar al espacio situado por encima como molculas de gas. Como slo se es capan las molculas ms rpidas, la velocidad media de las dems molculas disminuye; dado que la temperatura, a su vez, slo depende de la velocidad media de las molculas, la temperatura del lquido que queda tambin disminuye. Es decir, la evaporacin es un proceso de enfriamiento; si se pone una gota de agua sobre la piel, se siente f ro cuando se evapora. En el caso de una gota de alcohol, que se evapora con ms rap idez que el agua, la sensacin de fro es todava mayor. Por ejemplo la transpiracin hu mana es un mecanismo de defensa del cuerpo hacia el exceso de calor; los perros no transpiran pero cuando sienten calor jadean produciendo evaporacin, reduciendo de esa manera su temperatura corporal; los cerdos que tampoco transpiran, se re frescan en el barro. Figura 13.2 esquema de los procesos de cambio de fase en el caso del agua. Si un lquido se evapora en un recipiente cerrado, el espacio situado sobre el lqui do se llena rpidamente de vapor, y la evaporacin se ve pronto compensada por el pr oceso opuesto, la condensacin. Para que la evaporacin contine producindose con rapid ez hay que eliminar el vapor tan rpido como se forma. Por este motivo, un lquido s e evapora con la mxima rapidez cuando se crea una corriente de aire sobre su supe rficie. Cuando despus de que ha llovido la energa del Sol comienza a secar el suel o, el calor se consume en evaporar la humedad de la tierra, lo que hace disminui r la temperatura del aire, haciendo que los das sean ms frescos que si no hubiese llovido. 372

Cap. 13. Calor y la Primera Ley de la Termodinmica 13.4.2 Condensacin. Es la transformacin de un gas a lquido. Las molculas de gas que se condensan entregan energa cintica a la superficie sobre la que condensan, por l o que este es un proceso de calentamiento. Cuando el vapor de agua en la atmsfera se transforma en gotitas para formar las nubes, se libera calor a la atmsfera, p roduciendo un aumento de temperatura. 13.4.3 Fusin o derretimiento. Es la transfo rmacin de slido a lquido. 13.4.4 Solidificacin o congelacin. Es el cambio de estado d e lquido a slido. 13.4.5 Sublimacin. Es la transformacin directa de slido a gas, sin pasar por la fase lquida. 13.4.6 Deposicin. Es la transformacin directa de gas a sli do. En la atmsfera este proceso es frecuente en poca de bajas temperaturas, cuando el vapor de agua al entrar en contacto con las superficies que se encuentran a temperatura bajo 0 C, se congela formando la escarcha. 13.4.7 Ebullicin. Es un pro ceso en el cual el lquido pasa al estado de gas en el interior del lquido, donde e l gas se concentra para forma burbujas que flotan hasta la superficie y desde ah escapan al aire adyacente. La presin dentro de las burbujas debe ser grande para vencer la presin del agua que las rodea. Si la presin atmosfrica aumenta, la temperatura de ebullicin se eleva y viceversa. Cuando ascendemos a mayor altura sobre el nivel del mar, el agua hierve con temp eraturas menores porque la presin disminuye. Pero los alimentos se cuecen cuando la temperatura del agua es elevada y no por la temperatura de ebullicin, por lo t anto a mayor altura se debe esperar ms tiempo para cocer los alimentos, por ejemp lo un huevo duro en Concepcin se cuece en pocos minutos y en Visviri (4070 m de a ltura snm, en el extremo norte de Chile) en varias horas. La ebullicin es un proc eso de enfriamiento, en condiciones normales el agua que hierve a 100 C, se enfra con la misma rapidez 373

Cap. 13. Calor y la Primera Ley de la Termodinmica con la cual la calienta la fuente de calor, sino la temperatura del agua aumenta ra siempre con la aplicacin del calor. El calor necesario para que una sustancia d e masa m cambie de fase, se puede calcular de la siguiente forma: Q = mL (13.4) donde L es el calor latente del material, depende de la forma del cambio de fase y de las propiedades del material. El calor latente es la energa trmica necesaria para que un kilogramo de una sustancia cambie de un estado a otro, en el SI se mide en J/kg, tambin se usa la unidad cal/gr. Existen calores latentes de fusin, L F, de vaporizacin, LV, y de sublimacin, LS, para los respectivos cambios de fase. Por ejemplo, para el agua a la presin atmosfrica normal LF = 3.33x105 J/kg y LV = 22.6x105 J/kg. Los calores latentes de diferentes sustancias varan significativam ente, como se muestra en la tabla 13.2. Puesto que en la fase gaseosa, la distan cia media entre los tomos es mucho mayor que en la fase lquida o slida, se requiere mayor trabajo (y energa) para evaporar una masa de sustancia que para fundirla, por eso el calor de vaporizacin es mayor que el calor de fusin, como se observa en la tabla 13.2. Tabla 13.2 Constantes de cambios de fase (a 1 atm). Calor latente de Punto de ebullicin Calor latente de fusin (y solidi- (y condensac in) (C) vaporizacin (y ficacin) (J/kg) condensacin) (J/kg) 4 Helio * * -268,93 2,09 x 10 4 4 Hidrgeno -269.5 5,86 x 10 -252,89 45,2 x 10 4 4 Nitrgeno -209.97 2,55 x 10 -195,81 20,1 x 10 4 4 Oxgeno -218.79 1,38 x 10 -182,97 21,3 x 10 4 4 Alcohol etli co -114.0 10,4 x 10 78.0 85,4 x 10 4 4 Mercurio -38.8 1,18 x 10 356,9 27,2 x 10 4 4 Agua 0.0 33,3 x 10 100,0 225,6 x 10 4 4 Azufre 119.0 3,81 x 10 444,69 32,6 x 10 4 4 Plomo 327.3 2,45 x 10 1750 87,0 x 10 4 6 Aluminio 660.0 9,00 x 10 2450 1 1,4 x 10 4 6 Plata 960.8 8,82 x 10 2193 2,33 x 10 4 6 Oro 1063.0 6,44 x 10 2660 1,58 x 10 4 6 Cobre 1083.0 13,4 x 10 1187 5,06 x 10 4 6 Hierro 1808.0 28,9 x 10 3023 6,34 x 10 *Para solidificar el Helio se requiere una presin mayor que 25 atm . A 1atm, sigue siendo lquido hasta cero absoluto. Sustancia Punto de fusin (y sol idificacin) (C) 374

Cap. 13. Calor y la Primera Ley de la Termodinmica Ejemplo 13.5. Calcular la cantidad de calor necesario para transformar un gramo de hielo a -30 C en vapor de agua hasta 120 C. Solucin: es conveniente analizar cad a proceso fsico en forma separada. El subndice H se refiere la hielo, el A al agua y el V al vapor. 1) clculo del calor que se le debe agregar al hielo para elevar su temperatura desde -30 C hasta 0 C; en este proceso hay cambio de temperatura, s e calcula el calor sensible Q1: Q1 = mH cH T, con cH = 2090 J/kg C J Q1 = (10 3 kg ) 2090 [0 (30)] C = 62.7 J kg C 2 ) calor agregado para fundir el hielo (en 0 C), no hay cambio de temperatura, per o hay cambio de fase, se calcula el calor latente Q2: Q2 = mLFH, con LFH = 3.33x105 J/kg J Q2 = (10 3 kg ) 3.33 105 = 333 J kg 3 ) clculo del calor que se le debe agregar al agua para aumentar su temperatura de sde 0 C hasta 100 C; en este proceso hay cambio de temperatura, se calcula el calo r sensible Q3: Q3 = mA cA T, con cA = 4186 J/kg C J Q3 = (10 3 kg ) 4186 (100 0) C = 418.6 J kg C 4 ) calor agregado para evaporar el agua (en 100 C), no hay cambio de temperatura, pero hay cambio de fase, se calcula el calor latente Q4: Q4 = mLVA, con LVA = 22.6x105 J/kg 375

Cap. 13. Calor y la Primera Ley de la Termodinmica J Q4 = (10 3 kg ) 22.6 105 = 2260 J kg 5 ) clculo del calor que se le debe agregar al vapor de agua para aumentar su tempe ratura desde 100 C hasta 120 C; en este proceso hay cambio de temperatura, se calc ula el calor sensible Q5: Q5 = mV cV T, con cV = 2000 J/kg C J (120 100) C = 40 J 5 = (10 3 kg ) 2000 Por lo tanto, la cantidad total de calor necesario para transformar un gramo de hielo a -30 C en vapor de agua hasta 120 C es la suma del calor de cada proceso: QT = Q1 + Q2 + Q3 + Q4 + Q5 QT =62.7+333+418.6+2260+40=3114.3 J En forma grfica este proceso se puede ilustrar con la figura 13.3. Figura 13.3 Grfico de temperatura versus calor agregado para transformar 1g de hi elo a -30 C en vapor de agua a 120 C. 376

Cap. 13. Calor y la Primera Ley de la Termodinmica

Ejemplo 13.6. Calcular la cantidad de vapor de agua inicialmente a 130 C, que se requiere para calentar 200g de agua en un envase de vidrio de 100 g, desde 20 C h asta 50 C. Solucin: es un problema de intercambio de calor donde se debe igualar e l calor perdido por el vapor de agua al enfriarse hasta 50 C, con el calor ganado por el envase y el agua al calentarse hasta 50 C. Sea mX la masa de vapor descon ocida. 1 ) enfriamiento del vapor de agua desde 130 C hasta 100 C, hay cambio de tem peratura y el calor sensible Q1 liberado en este proceso es: Q1 = mX cV T, con cV = 2010 J/kg C J J Q1 = mX 2010 (100 130) C = mX 6 2 ) condensacin del vapor de agua en agua lquida a 100 C, no hay cambio de temperatur a, pero hay cambio de fase y se libera calor latente Q2: Q2 = mX LV, con LV = 22.6x105 J/kg J Q2 = m X 22.6 105 kg 3 ) enfriamiento del agua desde 100 C hasta 50 C, hay cambio de temperatura y el cal or sensible Q3 liberado en este proceso es: Q3 = mX cA T, con cA = 4186 J/kg C J J (50 100) C = mX 2.1 105 Q3 = mX 4186 kg C kg 377

Cap. 13. Calor y la Primera Ley de la Termodinmica

4 ) calor que gana el agua del envase para aumentar su temperatura desde 20 C hasta 50 C, hay cambio de temperatura y el calor sensible Q4 ganado en este proceso es : Q4 = mA cA T, con cA = 4186 J/kg C J Q4 = (0.2kg ) 4186 (50 20) C = 2.5 10 4 J

5 ) calor que gana el envase de vidrio para aumentar su temperatura desde 20 C hast a 50 C, hay cambio de temperatura y el calor sensible Q5 ganado en este proceso e s (el subndice Vi es para el vidrio): Q5 = mVi cVi T, con cVi = 837 J/kg C J 3 Q5 = (0.1kg ) 837 (50 20) C = 2.5 Se realiza el balance entre el calor perdido y el calor ganado: Q1 + Q2 + Q3 = Q4 + Q5 m X 6 10 4 J J J + m X 22.6 10 5 + m X 2.1 10 5 = 2.5 10 4 J + 2.5 10 3 J kg kg kg Despejando el valor de mX: mX =0.0109 kg = 10.9 g 13.5 TRABAJO EN LOS PROCESOS TERMODINAMICOS. Para un gas contenido en un envase cilndrico ajustado con un mbolo mvil, como estra en la figura 13.4, si el gas est en equilibrio trmico ocupa un volumen V roduce una presin constante P sobre las paredes del cilindro y sobre el mbolo, ea A. La fuerza ejercida por la presin del gas sobre el mbolo es F = PA. Si el se expande desde el volumen V hasta el volumen V+dV lo suficientemente lento, e l sistema permanecer en equilibrio termodi378 se mu y p de r gas

Cap. 13. Calor y la Primera Ley de la Termodinmica nmico. Por efecto de la expansin, el mbolo de desplazar verticalmente hacia arriba u na distancia dy, y el trabajo realizado por el gas sobre el mbolo, ser: dW = F dy = PA dy A V+dV dy V P Figura 13.4 El gas contenido en un envase a una presin P, cuando se expande reali za trabajo sobre el mbolo. Como Ady es el aumento de volumen dV del gas, se puede escribir el trabajo reali zado como: dW = PdV Si el gas se expande, entonces dV es positivo y el trabajo realizado por el gas es positivo, por el contrario, si el gas se comprime, dV es negativo y el trabaj o realizado por el gas es negativo, en este caso se interpreta como el trabajo r ealizado sobre el sistema. Si no cambia el volumen, no se realiza trabajo. Para obtener el trabajo total realizado por el gas cuando la variacin de presin hace ca mbiar el volumen desde un valor Vi hasta un valor Vf, se debe integrar la ecuacin anterior, de la forma: W = PdV Vi Vf (13.5) 379

Cap. 13. Calor y la Primera Ley de la Termodinmica Para evaluar esta integral, se debe saber como vara la presin durante el proceso. En general la presin no es constante, depende del volumen y de la temperatura. Si se conoce la presin y el volumen durante el proceso, los estados del gas se pued en representar por una curva en un diagrama PV, como la que se muestra en la fig ura 13.5. De este grfico, se obtiene que el trabajo realizado por un gas al expan dirse o comprimirse desde un estado inicial Vi hasta un estado final Vf es igual al rea bajo la curva de un diagrama PV. Figura 13.5. Curva presin volumen para un gas que se expande desde Vi hasta Vf. Este trabajo depende de la trayectoria seguida para realizar el proceso entre lo s estados inicial y final, como se ilustra con la figura 13.6. Si el proceso que se realiza es a volumen constante Vi disminuyendo la presin desde Pi hasta Pf, s eguida de un proceso a presin constante Pf aumentando el volumen desde Vi hasta V f (figura 13.6a), el valor del trabajo es diferente al que se obtiene en un proc eso donde primero se produce una expansin desde Vi hasta Vf a presin constante Pi y despus se disminuye la presin desde Pi hasta Pf, manteniendo constante el volume n final Vf (figura 13.6b). Las reas bajo las curvas en cada caso, tienen un valor diferente, es mayor en la figura 13.6b. Por lo tanto, el trabajo realizado por un sistema depende del proceso por el cual el sistema cambia desde un estado ini cial a otro final. De manera similar se encuentra que el calor transferido hacia adentro o hacia fuera del sistema, depende del proceso. Tanto el calor como el trabajo dependen de los estados inicial, final e intermedios del sistema. Como e stas dos 380

Cap. 13. Calor y la Primera Ley de la Termodinmica cantidades dependen de la trayectoria, ninguna de las dos se conserva en los pro cesos termodinmicos. Figura 13.6. a) Izquierda, b) derecha. Ejemplo 13.7 Un gas se expande desde i hasta f por tres trayectorias posibles, c omo se indica en la figura 13.7. Calcular el trabajo realizado por el gas a lo l argo de las trayectorias iAf, if y iBf. Considerar los valores dados en la figur a. Figura 13.7. Diagrama presin volumen del ejemplo 13.7. Solucin: se calcula el rea bajo la curva en cada proceso. De la figura 13.7, se ti enen los datos: Pi = 4atm = 4.05x105 Pa, Pf = 1atm = 1.013x105 Pa, Vi = 2lt = 0. 002m3 = VB, VA = 4lt = 0.004m3 = Vf. 381

Cap. 13. Calor y la Primera Ley de la Termodinmica trayectoria iAf: W1 = WiA + WAf = rea + 0 = Pi(VA - Vi) W1 = (4.05x105 Pa)(4-2)0.001m3 = 810 J trayectoria if:

W2 = Wif = rea =(Pi Pf)(VB Vf) + Pf (VB Vf) W2 = (4-1)(1.013x105 Pa)(4-2)0.001m3+( .013x105 Pa)(4-2)0.001m3= W2 = 515J trayectoria iBf: W3 = WiB + WBf = 0 + rea = Pf(Vf - VB) W3 = (1.01x105 Pa)(4-2)0.001m3 = 202 J Ejemplo 13.8 Una muestra de gas ideal se expande al doble de su volumen original de 1m3 en un proceso para el cual P = V2, con = 5 tm/m6, como se muestr en l i ur 13.8. C lcul r el tr jo re liz do por el s dur nte l exp nsin. Fi ur 13.8. Di r m presin volumen del ejemplo 13.8. 382

"

"

"

"

W = PdV = V 2 dV = V 2 dV Vi Vi Vi Vf Vf Vf W= (V 3 3 f Vi 3 ) = 5 atm m (2m3 )3 (1m3 )3 = 11.7(atm)(m3 ) 3 6 [ ] W = 11.7(atm)(m3 ) 1.01 105 Pa = 1.2 106 J 1atm 13.6 PRIMERA LEY DE LA TERMODINAMICA. En mecnica la energa se conserva si las fuerzas son conservativas y no actan fuerza s como la friccin. En ese mo elo no se incluyeron los cambios e energa interna e l sistema. La primera ley e la termo inmica es una generalizacin e la ley e con servacin e la energa que incluye los posibles cambios en la energa interna. Es una ley vli a en to o el Universo y se pue e aplicar a to os los tipos e procesos, permite la conexin entre el mun o macroscpico con el microscpico. La energa se pue e intercambiar entre un sistema y sus alre e ores e os formas. Una es realizan o trabajo por o sobre el sistema, consi eran o la me icin e las variables macros cpicas tales como presin, volumen y temperatura. La otra forma es por transferenci a e calor, la que se realiza a escala microscpica. Consi erar un sistema termo i nmico on e se pro uce un cambio es e un esta o inicial i a otro final f, en el cual se absorbe o libera una canti a e calor y se realiza trabajo W por o so bre el sistema. Si se mi e experimentalmente la canti a W p r diferentes pro cesos que se re licen p r ir desde el est do inici l l est do fin l, se encuen tr que su v lor no c m i , est diferenci de Q W se le ll m c m io de ener intern del sistem . Aunque por sep r dos Q y W dependen de l tr yectori , l c ntid d Q W, esto es, el c m io de ener intern es independiente de l tr yect ori o del proceso que se re lice p r ir desde el est do inici l l est do fin l. Por est r zn se consider l ener intern como un funcin de est do, que se mide en J 383

"

"

"

Solucin: us ndo l

definicin del tr

jo termodinmico:

C p. 13. C lor y l Primer

Ley de l

Termodinmic

y se sim oliz por U, el c m io de ener intern es U = Uf Ui, entonces se puede escribir la primera ley de la termodinmica: U = Uf Ui = Q W (13.6) En la ecuacin 13.6, Q es positivo (negativo) si se le agrega (quita) calor al sis tema y W es positivo cuando el sistema realiza trabajo y negativo cuando se real iza trabajo sobre el sistema. La forma correcta de escribir la ecuacin 13.6 es co nsiderando diferenciales, ya que si se le agrega o quita una pequea cantidad de c alor dQ al sistema y se realiza una cantidad de trabajo diferencial dW por o sob re el sistema, la energa interna cambia en una cantidad dU: dU = dQ dW (13.7) En escala microscpica, la energa interna de un sistema incluye la energa cintica y p otencial de las molculas que constituyen el sistema. Para un gas, el aumento de e nerga interna se asocia con el aumento de energa cintica de las molculas, es decir c on su temperatura. Al igual que en el caso de la mecnica, en termodinmica no inter esa conocer la forma particular de la energa interna, sino que interesan solo sus variaciones U. Por lo tanto, se puede elegir cualquier estado de referencia para la energa interna, ya que se han definido solo sus diferencias, no sus valores a bsolutos. En la ecuacin 13.7, tanto dQ como dW son diferenciales inexactas, pero dU es una diferencial exacta. 13.6.1 Casos particulares. Sistema aislado. Para un sistema aislado, que no inte racta con los alrededores, no hay transferencia de calor, Q = 0, el trabajo reali zado tambin es cero y por lo tanto no hay cambio de energa interna, esto es, la en erga interna de un sistema aislado permanece constante: Q = W = 0, U = 0 y Uf = Ui 384

"

C p. 13. C lor y l Primer

Ley de l

Termodinmic

Cap. 13. Calor y la Primera Ley de la Termodinmica Proceso cclico. Es un proceso que empieza y termina en el mismo estado. En este c aso el cambio de energa interna es cero y el calor agregado al sistema debe ser i gual al trabajo realizado durante el ciclo, entonces: U = 0 y Q=W Proceso con W = 0. Si se produce un proceso donde el trabajo que se realiza es c ero, el cambio en la energa interna es igual al calor agregado o liberado por el sistema. En este caso, si se le agrega (quita) calor al sistema, Q es positivo ( negativo) y la energa interna aumenta (disminuye). Esto es: W = 0, U = Q Proceso con Q = 0. Si ahora se realiza un proceso donde la transferencia de calo r es cero y el sistema realiza trabajo, entonces el cambio de la energa interna e s igual al valor negativo del trabajo realizado por el sistema, por lo tanto la energa interna disminuye; lo contrario ocurre si se realiza trabajo sobre el sist ema. Al cambiar la energa interna, cambia la energa cintica de las molculas en el si stema, lo que a su vez produce cambios en la temperatura del sistema. Q = 0, U = -W El calor y el trabajo son variables macroscpicas que pueden producir un cambio en la energa interna de un sistema, que es una variable microscpica. Aunque Q y W no son propiedades del sistema, se pueden relacionar con U por la primera ley de l a termodinmica. Como U determina el estado de un sistema, se considera una funcin de estado. 13.7 za a alor ley 385 PROCESOS TERMODINAMICOS. 13.7.1 Proceso isobrico. Es un proceso que se reali presin constante. En un proceso isobrico, se realiza tanto transferencia de c como trabajo. El valor del trabajo es simplemente P (Vf - Vi), y la primera de la termodinmica se escribe:

Cap. 13. Calor y la Primera Ley de la Termodinmica U = Q P (Vf - Vi) 13.7.2 Proceso isovolumtrico. Un proceso que se realiza a volumen constante se ll ama isovolumtrico. En estos procesos evidentemente el trabajo es cero y la primer a ley de la termodinmica se escribe: U = Q Esto significa que si se agrega (quita) calor a un sistema manteniendo el v olumen constante, todo el calor se usa para aumentar (disminuir) la energa intern a del sistema. 13.7.3 Proceso adiabtico. Un proceso adiabtico es aquel que se realiza sin interca mbio de calor entre el sistema y el medioambiente, es decir, Q = 0. Al aplicar l a primera ley de la termodinmica, se obtiene: U = -W En un proceso adiabtico, si un gas se expande (comprime), la presin disminuy e (aumenta), el volumen aumenta (disminuye), el trabajo es positivo (negativo), la variacin de energa interna U es negativa (positiva), es decir la Uf < Ui (Uf > U i) y el gas se enfra (calienta). Los procesos adiabticos son comunes en la atmsfera : cada vez que el aire se eleva, llega a capas de menor presin, como resultado se expande y se enfra adiabticamente. Inversamente, si el aire desciende llega a niv eles de mayor presin, se comprime y se calienta. La variacin de temperatura en los movimientos verticales de aire no saturado se llama gradiente adiabtico seco, y las mediciones indican que su valor es aproximadamente -9.8 C/km. Si el aire se e leva lo suficiente, se enfra hasta alcanzar el punto de roco, y se produce la cond ensacin. En este proceso, el calor que fue absorbido como calor sensible durante la evaporacin se libera como calor latente, y aunque la masa de aire continua enf rindose, lo hace en una proporcin menor, porque la entrega de calor latente al amb iente produce aumento de temperatura. En otras palabras, la masa de aire puede a scender con un gradiente adiabtico seco hasta 386

Cap. 13. Calor y la Primera Ley de la Termodinmica una altura llamada nivel de condensacin, que es la altura donde comienza la conde nsacin y eventualmente la formacin de nubes y de precipitacin. Sobre ese nivel la t asa de enfriamiento con la altura se reduce por la liberacin de calor latente y a hora se llama gradiente adiabtico hmedo, su valor vara desde -5 C/km a -9 C/km de dis minucin con la altura, dependiendo de si el aire tiene un alto o bajo contenido d e humedad. 13.7.4 Proceso isotrmico. Un proceso isotrmico es aquel que se realiza a temperatu ra constante. La grfica de P versus V para un gas ideal, manteniendo la temperatu ra constante es una curva hiperblica llamada isoterma (figura 13.9). Como la ener ga interna de un gas ideal es solo funcin de la temperatura, entonces en un proces o isotrmico para un gas ideal U = 0 y Q = W. Figura 13.9. Grfico presin volumen en un proceso isotrmico. Se calcular el trabajo para un gas ideal que se expande isotrmicamente desde el es tado inicial i al estado final f, como se muestra en el grfico PV de la figura 13 .9. La isoterma es una curva hiperblica de ecuacin PV = cte. El trabajo realizado por el gas se puede calcular con la ecuacin 13.5, usando la ecuacin de estado de g as ideal, PV = nRT, para reemplazar P: W = PdV = Vi Vf Vf Vi nRT dV V 387

Cap. 13. Calor y la Primera Ley de la Termodinmica Como la temperatura es constante, se puede sacar fuera de la integral: W = nRT E l resultado final es: Vf Vi dV V = nRT lnV V V Vf Vi f i W = nRT ln (13.8) Este trabajo es numricamente igual al rea bajo la curva PV de la figura 13.9. Si e l gas se expande (comprime) isotrmicamente, Vf > Vi, (Vf < Vi) y el trabajo es po sitivo (negativo). Ejemplo 13.9 Calcular el trabajo realizado por un mol de un g as ideal que se mantiene a 0 C, en una expansin de 3 litros a 10 litros. Solucin: c omo la expansin es isotrmica, el clculo es directo reemplazando los datos en la ecu acin 13.8: W = nRT ln Vf J 10 = (1mol ) 8.31 (273K ) ln = 2730 J Vi molK 3 Ejemplo 13.10 Una barra de cobre de 1 kg se calienta desde 10 C hasta 100 C, a la presin atmosfrica. Calcular la variacin de energa interna del cobre. Solucin: por la primera ley de la termodinmica U = Q W, donde se deben calcular Q y W. Clculo de Q: Q = mcT, con c = 387 J/(kg K) para el cobre. Q = 1kg 387 J (373 283) K = 3.5 10 4 J kgK Clculo e W, como P es constante: 388

Cap. 13. Calor y la Primera Ley e la Termo inmica

V = V0 T , con = 3 = 3(1.7x10-5 C-1) = 5.1x10-5 C-1

V =5.1x10-5 C-1 Vo(100-10)C =4.6x10-3 Vo m m 1kg Vo = = = 1.1 10 4 m 3 kg Vo 890 3 Vo se calcula de Cu = W = (1.01x105Pa)( 4.6x10 3)( 1.1x10 4m3) = 0.05 J Finalmente, el cambio de ene ga inte na es: U = 3.5x104 J 0.05 J = 3.49x104 J Se observa que casi todo el calor se usa para cambiar la energa interna de la barra de cobre. Por esta razn, normalm ente en la dilatacin trmica de un slido o un lquido se desprecia la pequea cantidad d e trabajo realizado contra la atmsfera. 13.8. CAPACIDAD CALORICA DE UN GAS IDEAL. Se ha encontrado que la temperatura de un gas es una medida de la energa cintica p romedio de traslacin del centro de masa de las molculas del gas, sin considerar la energa asociada al movimiento de rotacin o de vibracin de la molcula respecto al ce ntro de masa. Esto es as, porque en el modelo simple de la teora cintica se supone que la molcula es sin estructura. De acuerdo a esto, se analizar el caso simple de un gas ideal monoatmico, es decir, de un gas que tiene un tomo por molcula, como e l helio, nen o argn. Cuando se agrega energa a un gas monoatmico contenido en un env ase de volumen fijo (por ejemplo calentando el envase), toda la energa agregada 389

W = P(Vf

Vi) V se calcula del proceso de dilatacin cbica

Cap. 13. Calor y la Primera Ley de la Termodinmica se ocupa en aumentar la energa cintica de traslacin de los tomos. No existe otra for ma de almacenar la energa en un gas monoatmico. De la ecuacin 12.20, se tiene que l a energa interna total U de N molculas (o n moles) de un gas ideal monoatmico se pu ede calcular de: 3 3 NkT = nRT 2 2 U= (13.9) De esta ecuacin se deduce que para un gas ideal, U es funcin slo de la temperatura T. Si se transfiere calor al sistema a volumen constante, el trabajo realizado p or el sistema es cero, esto es, W = 0. Por la primera ley de la termodinmica, se obtiene: 3 nRT 2 Q = U = (13.10) Esto significa que todo el calor transferido se ocupa en aumentar la energa inter na (y la temperatura) del sistema. En la figura 13.10 se describe el proceso a v olumen constante desde i hasta f1, donde T es la diferencia de temperatura entre las dos isotermas. Figura 13.10. Procesos a volumen constante if1 y a presin constante if2. 390

Cap. 13. Calor y la Primera Ley de la Termodinmica Reemplazando el valor de Q dado por la ecuacin 13.1, con C = nc, se obtiene: ncV T = cV = 3 nRT 2 (13.11) 3 R 2 donde cV es la capacidad calrica molar del gas a volumen constante, vlida para tod os los gases monoatmicos. El cambio de energa interna para un gas ideal, en un pro ceso a volumen constante, se puede expresar como: U = ncV T (13.12) Suponga ahora que el gas se lleva por un proceso termodinmico isobrico, desde i ha sta f2, como se muestra en la figura 13.10. En esta trayectoria, la temperatura aument en la cantidad T. El calor que se debe transferir al gas en este proceso es t dado por Q = n cP T, donde cP es la capacidad calrica molar a presin constante. Co mo el volumen aumenta en este proceso, se tiene que el trabajo realizado por el gas es W = P V, y aplicando la primera ley de la termodinmica, se obtiene: U = nc P T PV (13.13) En este caso el calor agregado al gas se usa en dos formas: una parte para reali zar trabajo externo, por ejemplo para mover el mbolo del envase y otra parte para aumentar la energa interna del gas. Pero el cambio de energa interna para el proc eso de i hasta f2 es igual en el proceso de i hasta f1, ya que U para un gas ide al dependen slo de la temperatura y T es la misma el cada proceso. Adems como PV = nRT, para un proceso de presin constante se tiene 391

Cap. 13. Calor y la Primera Ley de la Termodinmica que PV = nRT. Reemplazando en la ecuacin 14.5, con U = n cV T, se obtiene: ncV T = nc P T nRT o cP - cV = R Esta expresin que se aplica a cualquier gas ideal, indica que la capacidad calrica molar a presin constante es mayor que la capacidad calrica molar a volumen consta nte por una cantidad R, la constante universal de los gases. Como para un gas mo noatmico cV = (3/2)R = 12.5 J/molK, el valor de cP es cP = (5/2)R = 20.8 J/molK. La razn de estas capacidades calricas es una cantidad adimensional llamada gamma, , de v lor: cP 5 R 5 = = 2 = = 1.67 cV 3 R 3 2 Los v lores de cP y concuerd n st nte ien con los v lores experiment les p r los ses mono tmicos, pero pueden ser muy diferentes p r ses ms complejos, co mo se puede o serv r en l t l 13.3, donde se list n v lores de l c p cid d c lric mol r p r l unos ses. Esto no de e sorprender y que el v lor de cV fu e determin do p r ses ide les mono tmicos y se esper un contri ucin dicion l l c lor especfico de ido l estructur intern de l s molcul s ms complej s. L diferenci de v lor entre cV y cP es consecuenci del hecho de que en los proce sos volumen const nte, no se re liz tr jo y todo el c lor se us p r umen t r l ener intern (y l temper tur ) del s, mientr s que en un proceso pr esin const nte, p rte de l ener c lric se tr nsform en tr jo re liz do por e l s. En el c so de slidos y lquidos c lent dos presin const nte, se re liz muy poco tr jo de ido que l dil t cin trmic es peque (ver ejemplo 13.10). En co nsecuenci , cV y cP son proxim d mente i u les p r slidos y lquidos. 392

"

"

"

"

  " " " 

"

"

"

C p. 13. C lor y l Primer Ley de l Termodinmic T l 13.3. C lores especficos de ses. Tipo de s Mono tmico G s He Ar Ne Kr H2 N2 O2 CO Cl2 CO2 SO2 H2O CH4 H2S NH3 C2 H6 Aire Cp 4.97 4.97 (c l/mol K) Cv 2.98 2.98 Cp - Cv 1.99 1.99 = Cp/Cv 1.67 1.6 7 1.64 1.69 1.41 1.40 1.40 1,40 1.35 1.30 1.29 1.30 1.31 1,33 1.31 1.20 1.40 CP 20,78 20,78 20,78 20,78 28,74 29,07 29,17 29,16 34.70 36,94 40,37 35.40 35.50 34 ,60 (J/mol K) CV 12,47 12,47 12.70 12.30 20.42 20,76 20,85 20,85 25.70 28,46 31, 39 27.00 27.10 25,95 Cp - CV 8,31 8,31 8.08 8.48 8.32 8,31 8,31 8,31 8.96 8.48 8 ,98 8.37 8.41 8,65 Di tmico 6.87 6.95 7.03 8.29 8.83 9.65 4.88 4.96 5.03 6.15 6.80 7.50 1.99 1.99 2.00 2.14 2.03 2.15 Poli tmico 8.80 12.35 0.2404 6.65 10.30 0.1715 2.15 2.05 Ejemplo 13.11. Un cilindro contiene 3 moles de helio temper tur m iente (sup oner 27 C). C lcul r: ) el c lor que se de e tr nsferir l s p r ument r s u temper tur h st 500 K si se c lient volumen const nte, ) el c lor que se de e tr nsferir l s presin const nte p r ument r su temper tur h st 500 K, c) el tr jo re liz do por el s. Solucin: ) en un proceso re liz do vol umen const nte, el tr jo es cero. De l ecu cin 13.10 se o tiene: 3 Q = nRT = nc V T 2 Como cV= 12.5 J/molK para el helio y T = 200K, reemplazando los valores: J Q1 = 3moles12.5 (200 K ) = 7500 J molK b) Usando la ecuacin Q2 = n cP T y reemplazando los valores: Q2 =(3 moles)(20.8 J/molK)(200K) = 12500 J. 393

 "

"

  " 

"

"

Cap. 13. Calor y la Primera Ley de la Termodinmica c) De la primera ley de la termodinmica: volumen constante: U = Q1 W = Q1 presin co nstante: U = Q2 W Esta variacin de energa interna es la misma en ambos procesos, ig ualando se tiene: Q1 = Q2 W y W = Q2 Q1 = 12500J 7500 J =5000 J. Ejemplo 13.12 Para un gas ideal monoatmico, se realiza el ciclo ABCDA que se ilus tra en la figura 13.11. Considerando que nRT = 1000 J en A, calcular Q, W y U en cada proceso del ciclo (suponer 1atm = 105 Pa). Figura 13.11. Diagrama presin volumen del ejemplo 13.12. Solucin: En A: se tiene PA VA = nRTA = 1000 J En B: PB VB = nRTB nRTB = (4x105 Pa )(0.01m3) = 4000 J En C: PC VC = nRTC nRTC = (4x105 Pa)(0.04m3) = 16000 J En D: PD VD = nRTD nRTD = (1x105 Pa)(0.04m3) = 4000 J 394

Cap. 13. Calor y la Primera Ley de la Termodinmica 1 ) el proceso AB es a volumen constante: 3 Q AB = ncV T = n R(TB TA ) = 1.5(nRTB n RTA ) 2 AB = 1.5(4000 1000) J = 4500 J WAB = 0 UAB = QAB WAB = 4500 J 2) el proceso BC e s isobrico, a presin constante: 5 QBC = nc P T = n R(TC TB ) = 2.5(nRTC nRTB ) 2 BC = 2.5(16000 4000) J = 30000 J WBC =P(VC VB) = (4x105 P )(0.04 0.01)m3 = 1200 0J UBC = QBC - WBC = 30000 12000 = 18000 J 3) el proceso CD es a volumen constante : 3 QCD = ncV T = n R(TD TC ) = 1.5(nRTD nRTC ) 2

CD = 1.5(4000 16000) J = 18000 J WCD = 0 UCD = QCD WCD = - 18000 J 4) el proceso DA es isobrico, a presin constante: 5 QDA = nc P T = n R(TA TD ) = 2.5(nRTA nRTD ) 2 395

# # #

Cap. 13. Calor y la Primera Ley e la Termo inmica

En la realidad los verdaderos procesos adiabticos no se producen, ya que no exist e un aislante perfecto entre el sistema y el ambiente. Pero existen procesos rea les que son casi o cuasiadiabticos. Por ejemplo, si se comprime (o expande) un ga s rpidamente, fluye muy poco calor entre el (o hacia fuera del) sistema y el proc eso es cuasiadiabtico. Estos procesos son comunes en la atmsfera, donde una masa d e aire cerca del suelo, ms clida y menos densa que los alrededores, asciende verti calmente, arriba se encuentra con regiones de menor presin y se expande adiabticam ente (o cuasi), esa expansin produce enfriamiento, si la masa de aire tiene sufic iente humedad, se produce la condensacin, que origina la formacin de nubes y proba ble precipitacin. Suponga que un gas ideal experimenta una expansin cuasiadiabtica. En cualquier instante durante el proceso, se supone que el gas est en equilibrio , de tal manera que la ecuacin de estado es vlida. La presin y el volumen en cualqu ier instante durante el proceso adiabtico estn relacionados por la ecuacin PV = cte . (13.14)

396

donde = cP /cV se consider const nte dur nte el proceso. Por lo t nto tod v ri les termodinmic s c m i n dur nte un proceso di tico. En l fi ur se muestr un di r m PV p r un exp nsin di tic . Como > 1, l curv exp nsin di tic es de m yor pendiente que l de l exp nsin isotrmic irse di tic mente el s, no se

"

"  " 

DA = 2.5(1000 4000) J = 7500 J WDA =P(VA VD) = (1x105 P )(0.01 0.04)m3 = -3000J U DA = QDA WDA = -7500 (-3000) = -4500 J 13.9. PROCESO ADIABATICO DE UN GAS IDEAL. Un proceso adiabtico es aquel que se re aliza sin transferencia de calor entre el sistema y los alrededores.

s l s 13.12 PV de l . Al exp nd

tr nsfiere c lor h ci el sistem o fuer de l, por lo t nto, de l primer ley d e l termodinmic , se ve que U es negativo, por lo tanto T tambin es negativo. Enton ces el gas se enfra (Tf < Ti) durante una expansin adiabtica. De manera equivalente , si el gas se comprime adiabticamente, su temperatura aumenta. Figura 13.12. Diagrama presin volumen donde se muestra la curva adiabtica. Usando la ecuacin PV = nRT, se demuestra que la ecuacin 13.14 se puede escribir ta mbin en la forma: TV 1 = cte. (13.15) Ejemplo 13.13. El aire en un cilin ro a 20 C se comprime es e una presin inicial e 1 atm y un volumen e 800 cm3 hasta un volumen e 60 cm3. Suponien o que el a ire se comporta como un gas i eal ( = 1.4) y que l compresin es di tic , c lcul r l presin y temper tur fin l. Solucin: us ndo l ecu cin 13.14 V PV = cte. PiVi = Pf V f Pf = Pi i V f 397

C p. 13. C lor y l Primer

Ley de l

Termodinmic

800cm 3 Pf = 1atm 60cm 3 y de la ecuacin 13.15 1 1.4 = 37.6 atm TV = cte. TiVi 1 = TfVf 1.4 1 1 V T f = Ti i V f 1 800cm 3 T f = 293K 60cm 3

398

= 825.7 K = 552.7

C p. 13. C lor y l Primer

Ley de l

Termodinmic

Cap. 13. Calor y la Primera Ley de la Termodinmica PROBLEMAS. 13.1 a) Una persona de 80 kg intenta bajar de peso subiendo una montaa para quemar el equivalente a una gran rebanada de un rico pastel de chocolate (700 Cal alimenti cias). A qu altura debe subir? b) Otra persona consume energa a razn de 150 W durant e su trabajo, qu cantidad de pan debe ingerir para poder trabajar durante una hora ? (Calor de combustin del pan es 8000 cal/g). Considere que el 25% de la energa li berada del alimento se aprovecha como trabajo til. R: a) 934 m, b) 64.5 g. Se acu erdan del problema del Salto del Laja; suponga ahora que el agua en su parte sup erior tiene una temperatura de 15 C. Si toda su energa potencial se emplea en cale ntar el agua que cae, calcule la temperatura del agua en la base del salto. R: s i altura del salto se estima en 25m, 15.06 C. Se utilizan 2 kcal para calentar 60 0 gr de una sustancia desconocida de 15 C a 40 C. Calcular el calor especfico de la sustancia. R: 0.13 cal/gr C. Un trozo de cadmio de 50 gr tiene una temperatura de 20 C. Si se agregan 400 cal al cadmio, calcular su temperatura final. A un vaso aislante del calor (de plumavit) que contiene 200 cm3 de caf a la temperatura de 95 C, se le agregan 40 cm3 de leche que se encuentra a temperatura ambiente. Calc ular la temperatura de equilibrio que alcanza la mezcla. (Suponer calor especfico de los lquidos igual al del agua y considere un da de primavera). Al desayunar, u sted vierte 50 cm3 de leche refrigerada en su taza que contiene 150 cm3 de caf re cin preparado con agua hirviendo. Calcular la temperatura de equilibrio alcanza e sta apetitosa mezcla. (Desprecie la capacidad calrica de la taza). R: 75 C. Se enf ra un bloque de 40 gr de hielo hasta -50 C. Luego se agrega a 500 gr de agua en un calormetro de 75 gr de cobre a una temperatura 13.2 13.3 13.4 13.5 13.6 13.7 399

Cap. 13. Calor y la Primera Ley de la Termodinmica de 25 C. Calcular la temperatura final de la mezcla. Si no se funde todo el hielo , calcular cuanto hielo queda. 13.8 En un recipiente aislado se mezclan 150 g de hielo a 0 C y 600 g de agua a 18 C. Calcular: a) la temperatura final del sistema , b) la cantidad de hielo queda cuando el sistema alcanza el equilibrio. R: a) 0 C, b) 14.4 g. Un recipiente de aluminio de 300g contiene 200g de agua a 10 C. Si se vierten 100 g ms de agua, pero a 100 C, calcular la temperatura final de equili brio del sistema. R: 34.6 C. 13.9 13.10 Un calormetro de 50 g de cobre contiene 250 g de agua a 20 C. Calcular la ca ntidad de vapor que se debe condensar en el agua para que la temperatura del agu a llegue a 50 C. R: 12.9 g 13.11 Un calormetro de aluminio con una masa 100 gr con tiene 250 gr de agua. Estn en equilibrio trmico a 10 C. Se colocan dos bloques de m etal en el agua. Uno es 50 gr de cobre a 80 C. El otro una muestra de masa de 70 gr a una temperatura de 100 C. Todo el sistema se estabiliza a una temperatura fi nal de 20 C. Deducir de que material se trata la muestra. 13.12 Un envase plumavi t contiene 200 g de mercurio a 0 C. Se le agregan 50 g de alcohol etlico a 50 C y 1 00 g de agua a 100 C. a) Calcular la temperatura final de la mezcla. b) calcular el calor ganado o perdido por el mercurio, alcohol y agua. Desprecie la capacida d trmica del plumavit. R: a) 84.4 C, b) 557 cal, 998 cal, 1560 cal. 13.13 Un cubo de hielo de 20 g a 0 C se calienta hasta que 15 g se han convertido en agua a 100 C y el resto en vapor. Calcular el calor necesario para este proceso. R: 21739 J . 13.14 Un trozo de cobre de 1 kg y a 20 C se sumerge en un recipiente con nitrgen o lquido hirviendo a 77K. Calcular la cantidad de nitrgeno que se evapora hasta el momento en que el cobre alcanza los 77K. Suponga que el recipiente est aislado tr micamente. R: 941 kg. 400

Cap. 13. Calor y la Primera Ley de la Termodinmica 13.15 La temperatura en reas costeras se ve influenciada considerablemente por el gran calor especfico del agua. Una razn es que el calor liberado cuando un metro cbico de agua se enfra en 1 C aumentar la temperatura de un volumen enormemente ms gr ande de aire en 1 C. Calcule este volumen de aire. El calor especfico del aire es aproximadamente 1 kJ/(kg C). Considere la densidad del aire igual a 1,25 kg/m3. R : Vaire = 3433 Vagua. 13.16 Un estudiante inhala aire a 22 C y lo exhala a 37 C. E l volumen promedio del aire en una respiracin es de 200 cm3. Ignore la evaporacin del agua en el aire y estime la cantidad de calor absorbido en un da por el aire respirado por el estudiante. R: 3.75 J por respiracin. 13.17 Un calentador de agu a funciona por medio de potencia solar. Si el colector solar tiene un rea de 6 m2 y la potencia entregada por la luz solar es de 550 W/m2, cul es el tiempo mnimo en aumentar la temperatura de 1 m3 de agua de 20 C a 60 C? Indique la(s) suposicin(es ) hecha(s). R: aprox. 14 horas. 13.18 Cuando un conductor frena su auto, la fric cin entre los tambores y las balatas de los frenos convierten la energa cintica del auto en calor. Si un auto de 1500 kg que viaja a 30 m/s y se detiene, cunto aumen ta la temperatura en cada uno de los cuatro tambores de hierro de 8 kg de los fr enos? (desprecie la prdida de energa hacia los alrededores). R: 47.1 C. 13.19 Dos b alas de plomo- c/u de 5,0 g, a 20 C y movindose a 400 m/schocan de frente. Suponien do una colisin perfectamente inelstica y que no hay ninguna perdida de calor hacia la atmsfera, describa el estado final del sistema (las dos balas). R: 645 C. 13.2 0 Un lago contiene cerca de 5x1011 m3 de agua. a) Cunto calor se necesita para ele var la temperatura de ese volumen de agua de 14.5 a 15.5 C? b) Calcule el tiempo que se requerira para calentar el lago, entre esas temperaturas, si el calor lo s uministra una central elctrica de 1000MW. R: b) aprox. 66 aos. 13.21 Un mol de gas ideal se calienta lentamente de modo que pasa del estado (Po, Vo) al estado (3P o, 3Vo). Este cambio ocurre de tal manera que 401

Cap. 13. Calor y la Primera Ley de la Termodinmica la presin del gas es directamente proporcional al volumen. a) Dibujar el diagrama presin versus volumen. b) Calcular el trabajo que realiza el gas en este proceso . Indicar si es positivo o negativo y comente el significado asociado al signo. c) Analizar cmo se relaciona la temperatura del gas con su volumen durante este p roceso. R: b) 4PoVo, c) PoV2/n(PM)Vo, donde (PM) es el peso molecular del gas. 1 3.22 La figura 13.13 representa la variacin del volumen y la presin de un gas cuan do se expande desde 1 m3 a 6 m3. a) Calcular el trabajo realizado por el gas dur ante el proceso, b) analizar lo qu le ocurre a la temperatura durante el proceso. R: 1418.2 J. 13.23 Un gas est a una presin de 1.5 atm y a un volumen de 4 m3. Cal cular el trabajo realizado por el gas cuando: a) se expande a una presin constant e hasta el doble de su volumen inicial y b) se comprime a presin constante hasta un cuarto de su volumen inicial. R: a) 6.1x105 J, b) -4.6x105 J. 13.24 Un gas id eal est encerrado en un cilindro que tiene un mbolo mvil en su parte superior, de m asa 8 kg y rea de 5 cm2, libre de moverse, manteniendo la presin del gas constante . Calcular el trabajo si la temperatura de 2 moles de gas aumenta de 20 a 300 C. R : 4698 J. Figura 13.13. Problema 13.22 Figura 13.14 Problemas 13.25 y 13.37. 13.25 Una muestra de gas ideal de un mol co cclico, como se muestra en la figura expansin isotrmica ab, una compresin nstante ca. Si T = 300 K, Pa = 5 atm, Pb zado por el gas durante el ciclo. 402 se lleva a travs de un proceso termodinmi 13.14 El ciclo consta de tres partes: una isobrica bc y un aumento de presin a volumen co = Pc = 1 atm, calcular el trabajo reali

Cap. 13. Calor y la Primera Ley de la Termodinmica 13.26 Una muestra de gas ideal se expande al doble de su volumen inicial de 0.1 m3 en un proceso para el cual P = aV2, con a = 2 atm/m6. a) Bosquejar un grfico e n un diagrama PV. b) Calcular el trabajo realizado por el gas durante la expansin . 13.27 Un gas ideal que se encuentra a 1 atm y 0 C, se expande desde un volumen de 25 lt hasta 80 lt, en un proceso en el cual P = 0.5a/V2. a) Bosquejar un grfic o en un diagrama PV. Calcular: b) la constante a en el SI, c) la temperatura y p resin final, d) el trabajo realizado en la expansin. R: b) 126.6 Pa/m3, c) presin: 63.3 Pa/V2, d) 3437.5J. 13.28 Un mol de vapor de agua a 373K se enfra a 283 K. El calor entregado por el vapor del agua que se enfra lo absorben 10 moles de un ga s ideal, haciendo que el gas se expanda a una temperatura constante de 273K. Si el volumen final del gas ideal es de 20 lt, determine su volumen inicial. R: 2.9 litros. 13.29 Un mol de gas ideal realiza 3000 J de trabajo mientras se expande isotrmicamente hasta una presin final de 1 atm y un volumen de 25 litros. Calcula r: a) su volumen inicial, b) la temperatura del gas, c) el cambio de energa inter na que experimenta, indicando si aumenta o disminuye, d) el calor absorbido o ce dido. R: a) 7.6 lt, b) 304 K, c) 0 J, d) 3000 J. 13.30 Un gas ideal inicialmente a 300 K se expande en forma isobrica a una presin de 2.5 kPa. Si el volumen aumen ta de 1 m3 a 3 m3 y se agregan 12500 J de calor al sistema, calcular: a) el camb io de energa interna, b) su temperatura final. R: a) 7500 J, b) 900 K. 13.31 Se c omprime un gas a presin constante de 0.8 atm de un volumen de 9 lt a un volumen d e 2 lt. En el proceso se escapan del gas 400 J de energa calrica. Calcular: a) el trabajo realizado por el gas, b) el cambio de energa interna del gas. R: a) -567 J, b) 167 J. 13.32 Un mol gas ideal monoatmico se somete al ciclo ABC mostrado en la figura 13.15. Si PA = 1.5 atm y TB = TC = 800 K, a) identificar las caracters ticas de cada proceso que compone el ciclo y dibujar el diagrama PV que le corre sponde, indicando los valores de las variables de 403

Cap. 13. Calor y la Primera Ley de la Termodinmica estado al comienzo y final de cada proceso. b) Calcular para cada proceso el tra bajo hecho por el gas, el calor absorbido o cedido, el cambio de energa interna d el gas. c) Averiguar para el ciclo completo qu ocurre con U, W y Q. R: a) VA = 16 .4 lt = VB, VC = 43.7 lt, b) WAB = 0, WBC = 6504 J, WCA = 4148.5 J, WTOTAL = 235 5.7 J. 13.33 Un gas se lleva a travs de un proceso cclico como el de la figura 13. 16. a) Calcular el calor neto transferido al sistema durante el ciclo completo. b) Si el ciclo se invierte, cul es el calor neto transferido por ciclo? R: a) 6000 J, b) -6000 J. Figura 13.15. Problema 13.32 Figura 13.16. Problema 13.33 13.34 Un bloque de 1 kg de aluminio se calienta a presin atmosfrica de modo que su temperatura aumenta desde 22 C hasta 40 C. Calcular: a) el trabajo realizado por el aluminio, b) la energa trmica que se le entrega, y c) el cambio de su energa int erna. R: a) 0.051 J, b) 16200 J, c) 16199.9 J. 13.35 Un mol de gas ideal monoatmi co se somete al proceso cclico ABCA mostrado en la figura 13.17. a) Calcular la e nerga trmica transferida al sistema, el trabajo hecho por el sistema y el cambio d e energa interna durante cada etapa del ciclo y para el ciclo completo. b) Si el ciclo se invierte, calcular cada una de dichas variables. 13.36 Un mol de gas in icialmente a una presin de 2 atm y a un volumen de 0.3 lt, tiene una energa intern a de 91 J. En su estado final, la presin es de 1.5 atm, el volumen de 0.8 lt y la energa interna de 182 J. Para los tres caminos IAF, IBF e IF de la figura 13.18, calcular a) el trabajo realizado por el gas, b) el calor neto transferido en el proceso. R: a) 76 J, 101 J, 88.6 J, b) 167 J, 192 J, 180 J. 404

Cap. 13. Calor y la Primera Ley de la Termodinmica Figura 13.17. Problema 13.35 Figura 13.18. Problema 13.36 13.37 Un mol de gas ideal monoatmico se lleva por un proceso isotrmico, de 400 K, al doble de su volumen original (figura 13.14). Calcular: a) el trabajo que real iz el gas, b) el calor que se le entreg al gas, c) la variacin de energa interna del gas, d) la razn de presiones Pb/Pa. Suponga que ahora se realiza un proceso (de a a c) a volumen constante para reducir la presin inicial Pa a Pb, calcular los n uevos valores de trabajo, calor y variacin de energa interna. 13.38 Calcular el ca mbio de energa interna de 3 moles de helio cuando su temperatura aumenta en 2 K. R: 75 J. 13.39 La capacidad calrica de un gas monoatmico a presin constante es 62.3 J/molK. Calcular: a) el nmero de moles del gas, b) la capacidad calrica a volumen constante, c) la energa interna del gas a 350K. R: a) 3 moles, b) 37.4 J/K, c) 1 3100 J. 13.40 Un mol de hidrgeno se calienta a presin constante de 0 a 100 C. Calcu lar: a) el calor transferido al gas, b) el aumento de energa interna, c) el traba jo realizado por el gas. 13.41 En un proceso a volumen constante, se transfieren 210J de calor a un mol de un gas ideal monoatmico inicialmente a 27 C. Calcular: a) el aumento de energa interna, b) el trabajo realizado por el gas, c) la temper atura final del gas. 13.42 Dos moles de un gas ideal (=1.4) se exp nden di tic m ente desde un presin de 5 tm y un volumen de 12 lt un volumen fin l de 30 405

lt. C lcul r: ) l presin fin l del s, ) l s temper tur s inici l y fin l del s. R: ) 1.4 tm, ) 365.8 K, 256.1 K. 13.43 Un s ide l (=1.4) se exp nde d i tic mente. Si l temper tur fin l es un tercio de l inici l, determin r ) e n que f ctor c m i el volumen, ) en que f ctor c m i l presin. R: ) 15.6, ) 0.0214. 13.44 Un mol de s ide l mono tmico (=1.4) inici lmente 27 C y l pre sin de 1 tm se comprime di tic mente un cu rto de su volumen inici l. C lcul r l presin y l temper tur fin l del s. 13.45 Dur nte el tiempo de compresin d e un motor de encin , l presin ument de 1 20 tm. Suponiendo que el proceso es di tico y el s ide l, con =1.4. C lcul r el volumen y l temper tur fin l del s. R: 8.5Vi, 2.3Ti. 13.46 Un lo o sond meteorol ico exp nde su volumen l scender en l tmsfer . Si l temper tur inici l del s dentro del lo o er de 27 C, c lcul r su temper tur cu ndo el volumen se h duplic do. R: 229 K. 1 3.47 C lcul r el tr jo que se requiere p r comprimir 1/10 de su volumen ini ci l, 5 moles de ire 20 C y 1 tm de presin por un proceso: ) isotrmico, ) di tico. c) C lcul r l s presiones fin les en los dos c sos. 13.48 3 moles de s r n inici lmente l temper tur de 20 C, ocup n un volumen de 10 lt. El s exp eriment un exp nsin presin const nte h st un volumen de 20 lt; lue o se exp n de en form di tic h st que re res su temper tur inici l. ) r fic r el proceso en un di r m PV. C lcul r: ) l c ntid d de c lor que se le entre o l s dur nte todo el proceso, c) el c m io tot l en l ener intern del s, d ) el tr jo tot l re liz do dur nte el proceso, e) el volumen fin l del s. 406

  " " " "

 "

"

" "

 "  "  "   " " "  "  " 

"

"

"  "

C p. 13. C lor y l Primer

Ley de l

Termodinmic

"

 "

" 

C p. 14. Mec nismos de tr nsferenci de c lor CAPITULO 14. MECANISMOS DE TRANSFERENCIA DE CALOR. 14.1 CALOR Y TEMPERATURA. C lor y temper tur son conceptos que en el len u je c otidi no se confunden, pero son diferentes. Por ejemplo l fr se uuuufff, que h c e c lor es u a expresi com para referir os al co cepto de temperatura, a pesar de que me cio amos la palabra calor. La temperatura es u a mag itud fsica que se re fiere a la se saci de fro o calie te al tocar algu a susta cia. E cambio el calo r es u a tra sfere cia de e erga de u a parte a otra de u cuerpo, o e tre difere tes cuerpos, producida por u a difere cia de temperatura. El calor es e erga e tr sito; siempre fluye de u a zo a de mayor temperatura a otra de me or temperatu ra, co lo que eleva la temperatura de la zo a mas fra y reduce la de la zo a ms cl ida, siempre que el volume de los cuerpos se ma te ga co sta te. La e erga o fl uye desde u objeto de temperatura baja a otro de temperatura alta si o se real iza trabajo. La materia esta formada por tomos o molculas que est e co sta te mov imie to, por lo ta to tie e e erga de posici o pote cial y e erga de movimie to o ci tica. Los co ti uos choques e tre los tomos o molculas tra sforma parte de la e erga ci tica e calor, cambia do la temperatura del cuerpo. Calor. El calor se defi e como la e erga ci tica total de todos los tomos o molculas de u a susta cia. Temperatura. La temperatura es u a medida de la e erga ci tica promedio de los tomo s y molculas i dividuales de u a susta cia. Cua do se agrega calor a u a susta ci a, sus tomos o molculas se mueve ms rpido y su temperatura se eleva, o viceversa. C ua do dos cuerpos que tie e disti tas temperaturas se po e e co tacto e tre s, se produce u a tra sfere cia de calor desde el cuerpo de mayor tem 407

"  

 

 

 

 

  

  

 

 

 

 

 

  

  

   

 

Cap. 14. Meca ismos de tra sfere cia de calor peratura al de me or temperatura. La tra sfere cia de calor se puede realizar po r tres meca ismos fsicos: co ducci , co vecci y radiaci , que se ilustra e la fig ura 14.1. Figura 14.1 Esquema de los meca ismos de tra sfere cia de calor. 14.2 C NDUCCI N DE CAL R. La co ducci es el meca ismo de tra sfere cia de calor e escala atmica a travs de la materia por actividad molecular, por el choque de u as molculas co otras, do de las partculas ms e ergticas le e trega e erga a las me os e ergticas, produci dose u flujo de calor desde las temperaturas ms altas a la s ms bajas. Los mejores co ductores de calor so los metales. El aire es u mal c o ductor del calor. Los objetos malos co ductores como el aire o plsticos se llam a aisla tes. La co ducci de calor slo ocurre si hay difere cias de temperatura e tre dos partes del medio co ductor. Para u volume de espesor x, con rea de secc in transversal A y cuyas caras opuestas se encuentran a diferentes T1 y T2, con T 2 > T1, como se muestra en al figura 14.2, se encuentra que el calor Q transferid o en un tiempo t fluye del extremo caliente al fro. Si se llama H (en Watts) al ca lor transferido por unidad de tiempo, la rapidez de transferencia de calor H = Q/t , est dada por la ley de la conduccin de calor de Fourier. 408

 

 

 

  

 

 

 

     

Cap. 14. Mecanismos de transferencia de calor H= dQ dT = kA t x (14.1) on e k (en W/mK) se llama con uctivi a trmica el material, magnitu que repres enta la capaci a con la cual la sustancia con uce calor y pro uce la consiguien te variacin e temperatura; y T/ x es el gra iente e temperatura. El signo meno s in ica que la con uccin e calor es en la ireccin ecreciente e la temperatura . En la tabla 14.1 se listan valores e con uctivi a es trmicas para algunos mate riales, los altos valores e con uctivi a e los metales in ican que son los me jores con uctores el calor. T2 H A T1 x Figura 14.2 Tabla 14.1 Algunos valores de conductividades trmicas. Metales, a 25C Sustancia k (W/mK) Aluminio 238 Cobre 397 Oro 314 Hierro 79.5 Plom o 34.7 Plata 427 Latn 110 Gases, a 20C Sustancia k (W/mK) Aire 0.0234 Helio 0.138 Hidrgeno 0.172 Nitrgeno 0.0234 Oxgeno 0.0238 Otros materiales Sustancia k (W/mK) As besto 0.08 Concreto 0.8 Diamante 2300 Vidrio 0.84 Hule 0.2 Madera 0.08 a 0.16 Co rcho, 0.42 Tejido humano 0.2 Agua 0.56 Hielo 2 Si un material en forma de barra uniforme de largo L, protegida en todo su largo por un material aislante, como se muestra en la figura 14.3, cuyos extremos de r ea A estn en contacto trmico con fuentes de calor a temperaturas T1 y T2 > T1, cua ndo se alcanza el estado de equilibrio trmico, la temperatura 409

Cap. 14. Mecanismos de transferencia de calor a lo largo de la barra es constante. En ese caso el gradiente de temperatura es el mismo en cualquier lugar a lo largo de la barra, y la ley de conduccin de calo r de Fourier se puede escribir en la forma: L T2 Flujo de calor k T1 Aislante Figura 14.3 H = kA (T2 T1 ) L (14.2) Ejemplo 14.1. Dos placas e espesores L1 y L2 y con uctivi a es trmicas k1 y k2 e stn en contacto trmico, como en la figura 14.4. Las temperaturas e las superficie s exteriores son T1 y T2, con T2 > T1. Calcular la temperatura en la interfase y la rapi ez e transferencia e calor a travs e las placas cuan o se ha alcanza o el esta o estacionario. L2 T2 k2 L1 k1 H T T1 Figura 14.4 Ejemplo 14.1 Solucin: si T es la temperatura en la interfase, entonces la rapi ez ncia e calor en ca a placa es: H 1 = k1 A (T T1 ) L1 y H 2 = k2 A (T2 T ) L2 410 e transfere

Cap. 14. Mecanismos e transferencia e calor Cuan o se alcanza el esta o estacionario, estos os valores son iguales: H 1 = H 2 k1 A Despejando la temperatura T: T= k1T1 L2 + k 2T2 L1 k1 L2 + k 2 L1 (T T1 ) (T T ) = k2 A 2 L1 L2 Y la transferencia e calor H1 o H2 es: H1 = A(T2 T1 ) L1 k1 + L2 k 2 con una barra e plata, una a rea transversal (figura 14.5 80 C y el extremo opuesto o el flujo e calor alcanza e

Figura 14.5. Ejemplo 14.2 Solucin: similar al ejemplo anterior, con L1 = L2 = L: H oro = k1 A (T1 T ) L y H plata = k 2 A (T T2 ) L Cuan o se alcanza el esta o estacionario, estos os valores son iguales: 411

Ejemplo 14.2 Una barra e oro est en contacto trmico continuacin e la otra, ambas e la misma longitu y ). Un extremo e la barra compuesta se mantiene a T1 = a T2 = 30 C. Calcular la temperatura e la unin cuan l esta o estacionario. T T1 k1 H k2 T2

Cap. 14. Mecanismos e transferencia e calor H oro = H plata k1 A (T1 T ) (T T2 ) = k2 A L L k1 (T1 T ) = k 2 (T T2 ) Despejan o la temperatura T, con k1 el oro y k2 e la plata, valores obteni os e la tabla 14.1: k1T1 + k 2T2 314 353 + 427 303 = = 324.2 K = 51.2 C k1 + k 2 31 4 + 427 T= 14.3 CONVECCION. La conveccin es el mecanismo e transferencia e calor por movimiento e masa o c irculacin entro e la sustancia. Pue e ser natural pro uci a solo por las ifere ncias e ensi a es e la materia; o forza a, cuan o la materia es obliga a a mo verse e un lugar a otro, por ejemplo el aire con un ventila or o el agua con un a bomba. Slo se pro uce en lqui os y gases on e los tomos y molculas son libres e moverse en el me io. En la naturaleza, la mayor parte el calor gana o por la at msfera por con uccin y ra iacin cerca e la superficie, es transporta o a otras cap as o niveles e la atmsfera por conveccin. Un mo elo e transferencia e calor H p or conveccin, llama o ley e enfriamiento e Ne ton, es el siguiente: H = h A (TA T) (14.3) donde h se ll m coeficiente de conveccin, en W/(m2K), A es l superficie que ent re c lor con un temper tur TA l fluido dy cente, que se encuentr un te mper tur T, como se muestr en el esquem de l fi ur 14.6. L t l 14.2 list l unos v lores proxim dos de coeficiente de conveccin h. 412

"

" "

C p. 14. Mec nismos de tr nsferenci de c lor u

T H TA A Fi ur 14.6. Proceso de conveccin. El flujo de c lor por conveccin es positivo (H > 0) si el c lor se tr nsfiere des de l superficie de re A l fluido (TA > T) y ne tivo si el c lor se tr nsfiere desde el fluido h ci l superficie (TA < T). T l 14.2. V lores tpicos de coeficiente de conveccin. Proceso h (W/m2K) Conveccin li re G ses 2 - 25 Lquidos 50 - 1000 Conveccin forz d G ses 25 - 250 Lquidos 50 - 20000

Ejemplo 14.3. El vidrio de un vent n se encuentr 10 C y su re es 1.2 m2. Si l temper tur del ire exterior es 0 C, c lcul r l ener que se pierde por conv eccin c d se undo. Consider r h = 4 W/(m2K). Solucin: Los d tos son: TA = 10 C = 2 83K, T = 0 C = 273K, A = 1.2 m2. Us ndo l ley de enfri miento de Newton: H = h A (TA T) W 1.2m 2 (283 273) K = 48W 2 m K H =4 413

"

"

"

" "

Cap. 14. Mecanismos e transferencia e calor 14.4 RADIACION. La ra iacin trmica es energa emiti a por la materia que se encuentra a una temperat ura a a, se pro uce irectamente es e la fuente hacia afuera en to as las ire cciones. Esta energa es pro uci a por los cambios en las configuraciones electrnic as e los tomos o molculas constitutivos y transporta a por on as electromagnticas o fotones, por lo recibe el nombre e ra iacin electromagntica. La masa en reposo e un fotn (que significa luz) es i nticamente nula. Por lo tanto, aten ien o a re lativi a especial, un fotn viaja a la veloci a e la luz y no se pue e mantener en reposo. (La trayectoria escrita por un fotn se llama rayo). La ra iacin elect romagntica es una combinacin e campos elctricos y magnticos oscilantes y perpen icu lares entre s, que se propagan a travs el espacio transportan o energa e un lugar a otro. A iferencia e la con uccin y la conveccin, o e otros tipos e on a, co mo el soni o, que necesitan un me io material para propagarse, la ra iacin electr omagntica es in epen iente e la materia para su propagacin, e hecho, la transfer encia e energa por ra iacin es ms efectiva en el vaco. Sin embargo, la veloci a , i ntensi a y ireccin e su flujo e energa se ven influi os por la presencia e ma teria. As, estas on as pue en atravesar el espacio interplanetario e interestelar y llegar a la Tierra es e el Sol y las estrellas. La longitu e on a () y a f recuencia () de las o das electromag ticas, relacio adas media te la expresi = c, s o importa tes para determi ar su e erga, su visibilidad, su poder de pe etraci y otras caractersticas. I depe die teme te de su frecue cia y lo gitud de o da, to das las o das electromag ticas se desplaza e el vaco co u a rapidez co sta te c = 299792 km/s, llamada velocidad de la luz. Los foto es so emitidos o absorbid os por la materia. La lo gitud de o da de la radiaci est relacio ada co la e erga de los foto es, por u a ecuaci desarrollada por Pla ck: hc E= (14.4)

414

donde h se

ama constante de P anck, su va or es h = 6,63 x 10 34 Js.

 

 

 

  

  

 

  

  

 

Cap. 14. Mecanismos de transferencia de ca or 14.4.1 Espectro de radiacin. Atendiendo a su ongitud de onda, a radiacin e ectromagntica recibe diferentes no mbres, y vara desde os energticos rayos gamma, con una ongitud de onda muy corta de orden de picmetros (frecuencias muy a tas) hasta as ondas de radio con ong itudes de onda muy argas de orden de ki metros (frecuencias muy bajas), pasando por a uz visib e, cuya ongitud de onda est en e rango de as dcimas de micrmet ro. E rango comp eto de ongitudes de onda es o que se denomina e espectro e ectromagntico, que se muestra en a figura 14.7. Esta variacin es porque as fuent es que producen as ondas son comp etamente diferentes. E espectro e ectromagnti co no tiene definidos mites superior ni inferior. La uz, amada tambin uz visi b e o uz b anca, es uno de os componentes de espectro e ectromagntico, y se de fine como aque a parte de espectro de radiacin que puede percibir a sensibi id ad de ojo humano. La uz visib e es un minscu o interva o que va desde a ongit ud de onda correspondiente a co or vio eta (aproximadamente 400 nm) hasta a o ngitud de onda correspondiente a co or rojo (aproximadamente 700 nm). Figura 14.7. Espectro e ectromagntico y regin visib e (inferior). 415

Cap. 14. Mecanismos de transferencia de ca or Por orden creciente de ongitudes de onda (o decreciente de frecuencias), e esp ectro e ectromagntico est compuesto por rayos gamma, rayos X duros y b andos, radi acin u travio eta, uz visib e, rayos infrarrojos, microondas y ondas de radio. L os rayos gamma y os rayos X duros tienen una ongitud de onda de entre 5x10 6 y 5x10 4 micrmetros (un micrmetro, smbo o m, es una mi onsima de metro). Los rayos X b andos se superponen con a radiacin u travio eta en ongitudes de onda prximas a os 5x10 2 m. La regin u travio eta, a su vez, da paso a a uz visib e, que va a proximadamente desde 0.4 hasta 0.8 m. Los rayos infrarrojos se mezc an con as fr ecuencias de microondas, entre os 100 y 400 m. Desde esta ongitud de onda hasta unos 15.000 m, e espectro est ocupado por as diferentes ondas de radio; ms a d e a zona de radio, e espectro entra en as bajas frecuencias, cuyas ongitudes de onda egan a medirse en decenas de mi es de ki metros.La tab a 14.3 muestra e espectro e ectromagntico, con sus ongitudes de onda, frecuencias y energas de fotn. Tab a 14.3. Espectro e ectromagntico. Longitud de onda Rayos gamma Rayos X U travio eta Extremo U travio eta Cercano L uz Visib e Infrarrojo Cercano Infrarrojo Medio Infrarrojo Lejano Microondas U tr a A ta Frecuencia Radio Muy A ta Frecuencia Radio Onda Corta Radio Onda Media (A M) Radio Onda Larga Radio Muy Baja Frecuencia Radio < 10 pm < 10 nm < 200 nm < 3 80 nm < 780 nm < 2.5 m < 50 m < 1 mm < 30 cm <1m < 10 m < 180 m < 650 m < 10 km > 10 km Frecuencia >30.0 EHz >30.0 PHz >1.5 PHz >789 THz >384 THz >120 THz >6.00 T Hz >300 GHz >1.0 GHz >300 MHz >30 MHz >1.7 MHz >650 kHz >30 kHz <30 kHz Energa (J ) >19.9 x10 15 >19.9 x10 18 >993 x10 21 >523 x10 21 >255 x10 21 >79.5 x10 21 >3. 98 x10 21 >199 x10 24 >1.99 x10 24 >1.99 x10 25 >2.05 x10 26 >1.13 x10 27 >4.31 x10 28 >1.98 x10 29 <1.99 x10 29 416

Cap. 14. Mecanismos de transferencia de ca or La radiacin de So es emitida en todas as ongitudes de onda, pero tiene un mxim o en a regin de uz visib e. La uz visib e est compuesta por varios co ores, que cuando se mezc an forman a uz b anca. Cada uno de os co ores tiene una ongi tud de onda especfica, con mites entre 0.4 y 0.7 m. Considerando desde as ongitu des de onda ms cortas a as ms argas, os diferentes co ores tienen os va ores c entra es de ongitudes de onda que se indican en a tab a 14.4. Estos co ores es tn dentro de un rango de ongitudes de onda, por ejemp o e vio eta esta en e ra ngo entre 0.4 y 0.45 m. Son os co ores que forman e arcoiris. En sus extremos s e tienen e u travio eta y e infrarrojo. La mayor cantidad de energa radiante de So se concentra en e rango de ongitudes de onda de visib e y visib e cerca no de espectro, con as siguientes proporciones: uz visib e 43%, infrarrojo ce rcano 49%, u travio eta 7%, y e 1% restante en otros rangos. Tab a 14.4 Co ores de espectro visib e y sus extremos. Co or U travio eta Vio eta Azu Verde Amari o Naranjo Rojo Infrarrojo (m) < 0.35 0.4 0.45 0.5 0.55 0.6 0.7 > 0.75 14.4.2. Penetracin de a radiacin e ectromagntica. Cuando a frecuencia es inferior a a frecuencia de a radiacin u travio eta, os fotones no tienen suficiente energa para romper en aces atmicos. Se dice entonces que a radiacin es radiacin no ionizante. A partir de os rayos u travio eta, vie nen os Rayos X y os Rayos gamma, muy energticos y capaces de romper mo cu as, di cha radiacin se denomina radiacin ionizante. La radiacin e ectromagntica reacciona d e manera desigua en funcin de su frecuencia y de materia con e que entra en c ontacto. E nive de penetracin de a radiacin e ectromagntica es inversamente prop orciona a su frecuencia. Cuando a radiacin e ectromagntica es de baja frecuencia , atraviesa impia 417

Cap. 14. Mecanismos de transferencia de ca or mente as barreras a su paso. Cuando a radiacin e ectromagntica es de a ta frecue ncia reacciona ms con os materia es que tiene a su paso. En funcin de a frecuenc ia, as ondas e ectromagnticas pueden no atravesar medios conductores. Esta es a razn por a cua as transmisiones de radio no funcionan bajo e mar y os te fon os mvi es se queden sin cobertura dentro de una caja de meta . Sin embargo, como a energa ni se crea ni se destruye, sino que se transforma, cuando una onda e ec tromagntica choca con un conductor pueden suceder dos cosas. La primera es que se transformen en ca or: este efecto tiene ap icacin en os hornos de microondas. L a segunda es que se ref ejen en a superficie de conductor (como en un espejo).

La radiacin de partcu as tambin puede ser ionizante si tiene suficiente energa. A gu nos ejemp os de radiacin de partcu as son os rayos csmicos, os rayos a fa o os r ayos beta. Los rayos csmicos son chorros de nc eos cargados positivamente, en su m ayora nc eos de hidrgeno (protones). Los rayos csmicos tambin pueden estar formados p or e ectrones, rayos gamma, piones y muones. Los rayos a fa son chorros de nc eos de he io positivamente cargados, genera mente procedentes de materia es radiact ivos. Los rayos beta son corrientes de e ectrones, tambin procedentes de fuentes radiactivas. La radiacin ionizante tiene propiedades penetrantes, importantes en e estudio y uti izacin de materia es radiactivos. Los rayos a fa de origen natur a son frenados por un par de hojas de pape o unos guantes de goma. Los rayos b eta son detenidos por unos pocos centmetros de madera. Los rayos gamma y os 418

Figura 14.8. Poder de penetracin de

a radiacin.

Cap. 14. Mecanismos de transferencia de ca or rayos X, segn sus energas, exigen un b indaje grueso de materia pesado como hierr o, p omo u hormign, como se muestra en a figura 14.8. Tambin existe a radiacin me cnica, que corresponde a ondas que s o se transmiten a travs de a materia, como a s ondas de sonido. 14.4.3 Leyes de radiacin. Ley de Stefan. Todos os objetos emiten energa radiante, cua quiera sea su temperatura, por ejemp o e So , a Tierra, a atmsfera, os P o os, as personas, etc. La energa radiada por e So a diario afecta nuestra exi stencia en diferentes formas. Esta inf uye en a temperatura promedio de a tier ra, as corrientes ocenicas, a agricu tura, e comportamiento de a uvia, etc. Considerar a transferencia de radiacin por una superficie de rea A, que se encuen tra a una temperatura T. La radiacin que emite a superficie, se produce a partir de a energa trmica de a materia imitada por a superficie. La rapidez a a cua se ibera energa se ama potencia de radiacin H, su va or es proporciona a a cuarta potencia de a temperatura abso uta. Esto se conoce como a ey de Stefan (Joseph Stefan, austriaco, 1835 1893), que se escribe como: H = AT4 (14.5) dond = 5.67x10-8 W/(m2K4) llama con tant d St fan-Boltzmann (Ludwing Boltz mann, au triaco, 1844-1906) y una propi dad radiativa d la up rfici llamad a mi ividad, u valor varan n l rango 0 < < 1, una m dida d la fici nc ia con qu la up rfici mit n rga radiant , d p nd d l mat rial. Un cu rpo mit n rga radiant con una rapid z dada por la cuacin 14.5, p ro al mi mo ti mp o ab orb radiacin; i to no ocurri ra, l cu rpo n algn mom nto irradiara toda u n rga y u t mp ratura ll gara al c ro ab oluto. La n rga qu un cu rpo ab orb provi n d u alr d dor , lo cual tambin mit n n rga radiant . Si un cu rpo ncu ntra a t mp ratura T y 419

$ %

$ $ $

% $ $

$ %

$ $

$ $ $ $ $

$ $ %$

$ $ $

$ $

% $

%$ $

$ $ $ $ $ $ % $ $%

%$

%$ %

$ $ %$

$ $ $ $ $% % % $ $ $ $ $ $ $ $ % $ $ % $ % $ $ $

$ % % %

Cap. 14. M cani mo d tran f r ncia d calor l ambi nt a una t mp ratura To, la n rga n ta ganada o p rdida por o r ultado d la radiacin : Hn ta = A(T4 - To4) (14.6)

Cuando l cu rpo t n quilibrio con lo alr d dor , irradia y ab orb la mi m a cantidad d n rga, por lo tanto u t mp ratura p rman c con tant . Cuando l cu rpo t m cali nt qu l ambi nt , irradia m n rga d la qu ab orb , y por l o tanto nfra. Un ab orb dor p rf cto llama cu rpo n gro (no ignifica qu a d color n gro), qu d fin como un obj to id al qu ab orb toda la radi acin qu ll ga a u up rfici y u mi ividad igual a uno. No conoc ningn obj to a , aunqu una up rfici d n gro d carbono pu d ll gar a ab orb r apro ximadam nt un 97% d la radiacin incid nt . El Sol, la Ti rra, la ni v , tc. ba jo ci rta condicion comportan como un cu rpo n gro. En t ora, un cu rpo n g ro ra tambin un mi or p rf cto d radiacin, y mitira a cualqui r t mp ratura la mxima cantidad d n rga di ponibl . A una t mp ratura dada, mitira una cantidad d finida d n rga n cada longitud d onda. En contra t , un cu rpo cuya mi ivid ad a igual a c ro, no ab orb la n rga incid nt obr l, ino qu la r fl ja toda, un r fl ctor p rf cto. Lo cu rpo con mi ividad ntr 0 y 1 lla man cu rpo gri , on lo obj to r al . A raz d l fraca o d lo int nto d calcular la radiacin d un cu rpo n gro id al gn la f ica cl ica, d arrollaron por prim ra v z lo conc pto b ico d la t ora cuntica. Una bu na aproximacin d un cu rpo n gro l int rior d un obj to hu co, como mu tra n la figura 14.9. La natural za d la radiacin mitida por un cu rpo hu co a trav d un p qu o aguj ro lo d p nd d la t mp ratura d la par d d la cavidad. Figura 14.9. R pr 420 ntacin d un cu rpo n gro.

$ $

$ $ $ $ $ $ $ $ $ % $ $%

% $ $% $ % $

$% $ $

$ %

$%

% $ $ %

$ $ %

% % $ %$ $ $ $ $ $ $ $ $ $ $ $ $ $ $ % $ $% $ $ $ $ $ %$ $ $ % % % $ $ % $ $ $ % $% $ $ $ $ % % % % % $ % $ $ $ % $ $% $ % $ % %$ $ %$ $ % $ $ $ $ % $ $ % $ $ $ $ $ % $ $% $ $ % $ $ $ $ $ $ % $ $ $ $ $ $ $ % $ $ $ $ $ $% % $ $ $ $% % $ $ $ $ $ $ $ $ % $ $ $ $ $ $ $ $ % $ $ % % $ $ % $ $ % % $ $ % $ $ $ $ $ $% $ $ $ $% $ $ $% $ $ $ % $ $% % %$ $ $ $ % $ $ $ $ $ $ $ % $ $ $ $ $ % $ $ $ %$ % $ $ $ $ %$ $ $

$ $

$ $ % %$ $

$ $

$ % %

$% $ $

$ $

%$

gundo com

Cap. 14. M cani mo d tran f r ncia d calor

Ej mplo 14.4. Una carr t ra d up rfici nn gr cida a una t mp ratura d 320 K r cib n rga radiant d l Sol por un valor d 700 W/m2. Calcular la radiacin n t a ganada por cada m2 d la up rfici d la carr t ra. Solucin: la n rga qu mit la up rfici d la carr t ra : H = AT4 H = 1 5.67 10 8 H W W A(320 K ) 4 = 594 5 2 A m m2 K 4 Como del Sol recibe 700 W/m2, la radiacin neta es: W H neta = 700 594.5 = 105.5 2 m A Ley e Wien. La figura 14.10 muestra la curva tpica e la intensi a e ra iacin e un cuerpo n egro en funcin e la longitu e on a e la ra iacin emiti a, para iferentes valo res e temperatura in ica os como fro, templa o y cli o. De acuer o a la teora cunti ca, se encuentra que los cuerpos a una temperatura etermina a, emiten ra iacin c on un valor mximo para una longitu e on a dada. A aumentar a temperatura de u n cuerpo negro, a cantidad de energa que emite se incrementa. Tambin, a subir a temperatura, e mximo de a distribucin de energa se desp aza hacia as ongitudes de onda ms cortas. Se encontr que este corrimiento obedece a a siguiente re acin, amada ey de desp azamiento de Wien (Wi he m Wien, a emn, 1864 1928): maxT = 2 897 (14.7) donde max es a ongitud de onda que corresponde a mximo de a curva de radiacin ( figura 14.10), en m, y T es a temperatura abso uta de objeto que emite a radia cin. La ey de Wien afirma que para a radiacin de un cuerpo 421

$ $

$ $

$ $

$ $ $ $ $ $ $ $

% $ $ $ $ $ % $ $ $ $ $

$ % $

$ $ %

$ % %

$ $

$ $ $

$ %

$ $

Cap. 14. Mecanismos de transferencia de ca or negro a ongitud de onda de mxima emisin es inversamente proporciona a a temper atura abso uta. Con esta ey se demuestra que a emisin de radiacin de a superfic ie terrestre tiene un mximo en cerca de 9.9 m, que corresponde a a regin infrarroj a de espectro. Tambin muestra que a temperatura de So , si e mximo de emisin de radiacin so ar ocurre en 0.474 m, es de orden de 6110 K. Figura 14.10. Intensidad de radiacin de un cuerpo negro. Ley de P anck. Los objetos con mayor temperatura radian ms energa tota por unidad de rea que os objetos ms fros. Por ejemp o e So con una temperatura media de 6000 K en su supe rficie, emite 1.6x105 (6000/300)4 veces ms energa que a Tierra con una temperatur a media en superficie de 289 K = 16 C. Por definicin, un cuerpo negro es un absorb edor perfecto. Este tambin emite a mxima cantidad de energa a una temperatura dada . La cantidad de energa emitida por un cuerpo negro est nicamente determinada por s u temperatura y su va or o da a Ley de P anck. En 1900, Max P anck (a emn, 1858 1947), descubri una frmu a para a radiacin de cuerpo negro en todas as ongitude s de onda. La funcin emprica propuesta por P anck afirma que a intensidad de radi acin I(,T), esto es, a energa por unidad de tiempo por unidad de rea emitida en un interva o de ongitud de onda, por un cuerpo negro a a temperatura abso uta T, est dada por: 422

Cap. 14. Mecanismos de transferencia de ca or 2hc 2 5 I ( , T ) = ch kT e 1 (14.8) donde I(,T) es a densidad de f ujo de energa por unidad de ongitud de onda, en W /(m2m), h es a constante de P anck, y k es a constante de Bo tzmann, de va or k = 1.38 x 10 23 J/K. E grfico de a funcin I(,T) para diferentes va ores de temper atura abso uta, se muestra en a figura 14.11. Figura 14.11. Grfico de a funcin I(,T) de a ey de P anck. 423

Cap. 14. Mecanismos de transferencia de ca or PROBLEMAS. 14.1 E gradiente trmico de a Tierra, medido en a superficie es 30 C/km. Suponga que este va or no cambia en todo e trayecto hasta e centro de a Tierra. Si a tem peratura en a superficie terrestre es 17 C, ca cu ar a temperatura en e centro de a Tierra. Considera que es una respuesta razonab e? Considerar e radio terr estre de 6370 km. R: 191117 C Una barra de hierro de 60 cm de ongitud y rea trans versa de 2 cm2, tiene un extremo a 80 C y e otro a 20 C. Ca cu ar: a) e gradien te de temperatura, b) a rapidez de transferencia de ca or, c) su temperatura a 20 cm de extremo ca iente. R: a) 100 C/m, b) 1.6x10 4 W, c) 60 C. Dos barras de a misma ongitud, de diferentes materia es y reas transversa es se co ocan para e as entre s. Encuentre a expresin de a tasa de f ujo de ca or en trminos de as conductividades trmicas y as reas de as barras. Genera ice e resu tado a caso de ms de dos barras. R: (T/x)(k1A1+k2A2) Un carpintero construye una pared. Hacia el exterior coloca una lmina de madera (k = 0.08 W/mK) de 2 cm de espesor y haci a el interior una capa de espuma aislante (k = 0.01 W/mK) de 3,5 cm de espesor. La temperatura de la superficie interior es de 19 C, y la exterior es 10 C. Calcula r: a) la temperatura en la unin entre la madera y la espuma, b) la razn de flujo d e calor por m2 a travs de esta pared. R: a) -15.3 C, b) -53.2 W/m2. Una tabla de re a de 2 m2 y 2 de cm de espesor se usa como una barrera entre un cuarto a 20 C y u na regin a 50 C. Calcular el nmero de clavos de acero de 2 cm de longitud y 4 mm de dimetro que se deben clavar sobre la tabla para que el flujo de calor a travs de la tabla se duplique. R: aprox. 160 clavos. Un extremo de una varilla metlica ais lada se mantiene a 100 C, y el otro se mantiene a 0 C en contacto con una mezcla d e hielo-agua. La varilla tiene 40 cm de longitud y un rea transversal de 0,75 cm2 . El calor conducido por la varilla funde 3 g de hielo en 5 minutos. Calcu14.2 14.3 14.4 14.5 14.6 424

Cap. 14. Mecanismos de transferencia de calor lar: a) el gradiente trmico a lo largo de la varilla, considerando que este es un iforme, b) la cantidad de calor conducida por la varilla, c) la conductividad trm ica del metal. d) Si el extremo que est a 100 C esta en contacto con vapor qu cantid ad de vapor condensa en los 5 minutos sealados? R: a) 250 C/m, b) 3.3 W, c) 173.7 W/mK, d) 0.44 g. 14.7 Una barra de hierro de 20 cm de largo con un dimetro de 1 c m tiene un extremo sumergido en una mezcla de hielo a 0 C, mientras que el otro e xtremo est en un tanque de vapor a 100 C. Suponga que a lo largo de la barra se ha establecido un gradiente de temperatura uniforme. Calcular: a) la rapidez del f lujo de calor a lo largo de la barra, b) la rapidez con la que se funde el hielo en el extremo fro, c) la rapidez con la que se condensa el vapor en el extremo c aliente para mantener el gradiente de temperatura uniforme, d) el gradiente de t emperatura a lo largo de la barra. Una heladera cbica de plumavit, de 30 cm de la do y 2 cm de espesor, tiene una temperatura interna de 5 C y externa de 25 C. Si 5 kg de hielo se funden en 8 horas, calcular la conductividad trmica del material. R: 0.143 W/mC. Un tubo de vapor se cubre con material aislante de 0.5 cm de espes or y 0.2 cal/(s cm C) de conductividad trmica. Inicialmente Cunto calor se pierde po r segundo si el tubo est a 120 C y el aire circundante a 20 C? El tubo tiene un perm etro de 20 cm y una longitud de 50 cm. Ignore las prdidas por los extremos del tu bo. Analice la conveniencia o no de usar la relacin dada para superficies planas. Estrictamente, debera usar la ecuacin diferencial para la tasa conduccin de calor e integrar para un conjunto de capitas superpuestas, cada una de forma cilndrica y muy delgadita. R: 5.3x104 W 14.8 14.9 14.10 Una ventana trmica de 6 m2 se construye con dos capas de e 4 mm de espesor, separadas por un espacio de aire de 5 mm. Si est a 25 C y la externa a 0 C, calcular la prdida de calor .11 A cierta familia le agrada tener la casa a 23 C durante el fuera hay 0 C. Qu temperatura interna deberan elegir si 425 vidrio, cada una d la parte interna a travs de la ventana. 14 invierno, cuando a

Cap. 14. Mecanismos de transferencia de calor quisieran bajar sus gastos en combustibles en 10%? Explique claramente las hiptes is que hizo. R: 20.7 C. 14.12 a) Si la temperatura promedio de la piel de algn alu mno es 30 C, suponiendo una emisividad = 0.97, calcular la radiacin qu mit . b) Si la t mp ratura prom dio d la par d d la ala dond ncu ntra 15 C, calcular la radiacin qu mit , con id rada como cu rpo n gro. c) Calcular la rad iacin n ta para l alumno. 14.13 Una dama ncu ntra n bikini n un auna cuya par d tn a 85 C y ti n n una mi ividad igual a 1. Su pi l ncu ntra a 40 C y u mi ividad 0.8. a) Cunto calor ab orb la dama por radiacin d la par d ? b) Cul la ta a a la cual la dama irradia n rga al m dio xt rior? c) Cunto ud or d b ra vaporar por hora para qu u t mp ratura mant nga normal y tabl ? (Suponga qu t l nico m cani mo d prdida n rga y qu no t produci ndo n r ga por m taboli mo). Con id r qu l calor lat nt d l udor, a 37 C, 2427 kJ/ kg (compar con l d l agua, t nga pr nt qu t ltimo t dado a 100 C). 14.14 A v riguar n algn t xto d ptica, cual la longitud d onda y la fr cu ncia d la radiacin dond l ojo humano ti n la mxima n ibilidad, y l ojo d un gato?, un murcilago?, un bho? 14.15 Calcular la fr cu ncia n H rtz y la n rga n J ( cuacin 1 4.4), para la longitud d onda d cada tipo d radiacin d la tabla 14.4. 14.1 6 La t mp ratura d la up rfici d l Sol d uno 6000 K. Tomando l radio d l Sol igual 7x108 m, calcular la n rga total irradiada por l Sol cada da. R: 3.7 x1031 J. 14.17 La t mp ratura d la up rfici d l Sol d uno 6000 K. a) Cul la longitud d onda n qu produc la mxima radiacin? b) A qu tipo d radiacin d l p ctro l ctromagntico corr pond ? c) Con id rando la longitud d onda d la luz vi ibl d la tabla 14.4, opin i d d l punto d vi ta f ico una bu na o mala norma pintar lo bu colar d amarillo y lo carro d bomb r o d rojo. R: a) 0.474 m, b) vi ibl . 426

$ $

$ $

$ $

$ $ $

$ $ % % % $ $ %$

%$ $ $ %$ %$ $ $ $ %

$ $ $

%$

$ $% $

$ $

$ $

$ $

$ $ $

$ $ $ $% $ %$ $ $ $ $ % $ $

$ % % $ % $% % $ %$ $ $ %$ % $ % $ % $ $ $ $% $ $ %$ $ $ $ % $ $ $ $ % $ %$ $ $ $ % $ $ $ %$ $ $ $ $ % $% $ $ % $ $ $ $ % $ $ $% $ $ % $ $ $ $ $ $ $ % % $ $ $ $ $ $% $ $ % $ $ $ $ % % $ $ $ $ $ $% $ % $ $ $ % $ %$ $ $ $ $ %

$ $ $ $ $ $ $ % $ % $ $ % $ $ $ $ % % $ %$ % $ % % $ %$ $ $ $ $ $ $ % $ $ $ % $ $

$ $

$ % $ $ $ %$ $ %

Cap. 14. M cani mo d tran f r ncia d calor

14.18 La up rfici d la Ti rra ncu ntra a una t mp ratura c rcana a 300 K. a) Cul la longitud d onda n qu produc la mxima radiacin t rr tr ? b) A qu tipo d radiacin d l p ctro l ctromagntico corr pond ? R: a) 9.9 m, b) infrarro ja. 14.19 El t cho d una ca a, con truido para ab orb r la radiacin olar incid nt obr l, ti n un r a d 7 m x 10 m y una inclinacin d 30 r p cto al u lo hor izontal. En l lugar, a niv l d l u lo, la radiacin olar d 340 W/m2 n prom dio anual. a) Si l 20% d la n rga incid nt pu d conv rtir a n rga lctric a til, cunto kWh d n rga til proporciona diariam nt ta fu nt ? Suponga qu l S ol brilla un prom dio d 8 h diaria . b) cul l ahorro m n ual ( n $) n Chil ? c) ta n rga ra ufici nt para aba t c r una ca a familiar tpica? R: a) 4.76 k Wh. 14.20 Una olla d aluminio ti n una ccin tran v r al circular d 8 cm d r adio y 3 mm d p or. S coloca obr una placa cali nt y ll na con 1 kg d agua. Si l fondo d la olla manti n a 101C y la part int rna a 100 C, calc ular: a) la rapid z d tran f r ncia d calor hacia l agua, b) l ti mpo qu d mora l agua n h rvir. D pr ciar l calor p rdido por lo lado d la olla. 14 .21 U ando la l y d Planck calcular la int n idad d radiacin mxima mitida por l Sol y por la Ti rra. 14.22 Suponi ndo qu l ol un radiador d cu rpo n gr o ( = 1), con una t mp ratura d la up rfici radiant d 6000 K, calcular u mi in d radiacin, n W/m2. 14.23 Un cu rpo radiant con = 0.98 y d t mp ratura = 3 0 C, aum nta a 60 C u t mp ratura. En qu porc ntaj aum nta u mi in d radiacin, W/m2? 14.24 Si la t mp ratura d un cu rpo d = 0.8 aum nta l tripl , n qu porc ntaj aum ntar u mi in d radiacin, n W/m2? 427

$ $

$ % $

$ %

$ %$ $ $ $ $ $ $ $ $ $ $ $

% $ % $ $ $ $ $ $ $ $ $ $ $ % $ % $ $ $ $ $ % $ $ $ $ $ $ $ $ $ $ % $ $ $ %$ % $ $ $ $ $ $ % $ $ $ $ $ % $ % $ $ $ %$ $ $ $ $ $ $ $ $ % $ $ $ $ $ $ $% $ $ %$ %$ $ $ $% $ $ $ % $ % $ $ % $% $ $ $ % % $ $ % $ $ $% $ $ % $ % $ $ %$ $ $ % % $ $ $ $ $ $ $ $ $ $ $ $ $ $% $ $ $ $ $ $ % $ %$ $ % $ $ $ $ %$ $ $ $ $ $ $ % % % $ % $ $ % $ $ $ $ $ %$ $ %$ $% $ $ $ $ $ $% $ $ $ $ % $

$ $

$ $ %

$ % %

$ $

%$

% $

% $

$ $ $

Cap. 15. S gunda l y d la t rmodinmica

CAPITULO 15. SEGUNDA LEY DE LA TERMODINAMICA Y ENTROPIA. La prim ra l y d la t rmodinmica la l y d con rvacin d la n rga g n ralizada para incluir l calor como una forma d tran f r ncia d n rga. E ta l y lo afirma qu un aum nto n alguna d la forma d n rga d b tar acompaado por una di minucin n alguna o tra forma d la mi ma. La prim ra l y no produc ninguna r triccin obr lo tip o d conv r ion d n rga qu pu d n ocurrir. Ad m no hac di tincin ntr l t rabajo y l calor. D acu rdo con la prim ra l y, la n rga int rna d un i t ma pu d incr m ntar ya a agr gando calor o r alizando un trabajo obr l i t ma. P ro xi t una dif r ncia muy important ntr l trabajo y l calor qu no vid ncia d la prim ra l y. Por j mplo, po ibl conv rtir compl tam nt l trabajo n calor, p ro n la practica, impo ibl conv rtir compl tam n t l calor n trabajo in modificar lo alr d dor . La gunda l y d la t rmo dinmica tabl c cual proc o d la natural za pu d n ocurrir o no. D todo lo proc o p rmitido por la prim ra l y, olo ci rto tipo d conv r in d n rga pu d n ocurrir. Lo igui nt on alguno proc o compatibl con la prim ra l y d la t rmodinmica, p ro qu cumpl n n un ord n gob rnado por la gun da l y. 1) Cuando do obj to qu tn a dif r nt t mp ratura pon n n contac to trmico ntr , l calor fluy d l obj to m clido al m fro, p ro nunca d l m f l m clido. 2) La al di u lv pontn am nt n l agua, p ro la xtraccin d la al d l agua r qui r alguna influ ncia xt rna. 3) Cuando d ja ca r una p l ota d goma al pi o, r bota ha ta d t n r , p ro l proc o inv r o nunca ocurr . Todo to on j mplo d proc o irr v r ibl , d cir proc o qu ocu rr n naturalm nt n una ola dir ccin. Ninguno d to proc o ocurr n l or d n t mporal opu to. Si lo hici ran, violaran la gunda l y d la t rmodinmica. La natural za unidir ccional d lo proc o t rmodinmico tabl c una dir ccin d l ti mpo. La gunda l y d la t rmodinmica, qu pu d nunciar d dif r nt forma quival nt , ti n mucha aplicacion prctica . D d l punto d vi t a d la ing ni ra, tal v z la m important n r lacin con la fici ncia limitad a d la mquina trmica . Expr ada n forma impl , la gunda l y afirma qu no po ibl con truir una mquina capaz d conv rtir por compl to, d man ra contin ua, la n rga trmica n otra forma d n rga. 429

% $ $ $ % $ $ $ % %$ $ $ $ $ $ $ % %$ % % % % $ $ $ $ $ $% % $ $ $ $ %$ $ $ $ $ $ % % % % % $ $ $ %$ % $ $ $ $ $ $ % $ $ $ $ $ $ $ $% $ $ $ $ $% $ $ % % $ $ $ $ % $ % %$ $ $ $ %$ $ $ $ $ $ $ $% % % % % % $ $ $ $ $ $ $ $ $ $ $ % %$ $ % %$ $ %$ %$ % $ $ $ % $ $ % % %$ % $ %$ % $ % % $ $ $% $ $ $ $ $ % $ $ $ $% $ $ $ $ $ $ $ %$ $ $ % % % $ $ $ $ $ $ $ $ $% % % $ $ $ $ $ $ % $ $ %$ $ % $ % $ $ $% $ $ $ $ $ $ $% $ $ $ $ $% $ $ $ $ $ %$ % %$ % % %$ $ % % $ $ $ $ % $ % % % % $ $ $ % $ $ $ $ % %$ % % % $ $ $ $ $ % %$ $ $ $ $ % % % $ $ $ $% $ $ $ $ $ $ %$ $ $ $ $ % $ $ $ $ $ $ $ $ $ % %$ $ $ $ $ $ $ $ $% $ $ $ $ $ $ $ $ $ % $ $ $ % % $ $ % $ $ $% $ $ $ $ $% $ % % $ $ $ $ $ $ $ $ $ % % $ $ $ $ $ $ $ $ $ $ $ %$ % $ $ % % % % $ % $ $ $ $ $ % %$ $ $ % $ $ $ % $ % $ $ $ $ $ % $ %$ $% $ $ $ $ $ $

% $ $ $

% $ $

Cap. 15. S gunda l y d la t rmodinmica

15.1. MAQUINAS TERMICAS. 15.1.1 Mquina trmica. Una mquina trmica un di po itivo q u convi rt n rga trmica n otra forma til d n rga, como la n rga lctrica y o m cnica. D man ra xplcita, una mquina trmica un di po itivo qu hac qu una u tancia d trabajo r corra un proc o cclico durant l cual 1) ab orb calo r d una fu nt a alta t mp ratura, 2) la mquina r aliza un trabajo y 3) lib ra c alor a una fu nt a t mp ratura m baja. Por j mplo, n un motor d ga olina, 1) l combu tibl qu qu ma n la cmara d combu tin l d p ito d alta t mp ra tura, 2) r aliza trabajo m cnico obr l pi tn y 3) la n rga d d cho al p or l tubo d cap . O n un proc o caract r tico para producir l ctricidad n una planta d pot ncia, l carbn o algn otro tipo d combu tibl qu ma y l ca lor g n rado u a para producir vapor d agua. El vapor dirig hacia la a pa d una turbina, ponindola a girar. Po t riorm nt , la n rga a ociada a dicha rotacin u a para mov r un g n rador lctrico. Como m nciono ant , una mquina trmica tran porta alguna u tancia d trabajo a trav d un proc o cclico, d fini do como aqu l n l qu la u tancia r gr a a u tado inicial. Como j mplo d un proc o cclico, con idr la op racin d una mquina d vapor n la cual la u tancia d trabajo l agua. El agua ll va a trav d un ciclo n l qu prim ro convi rt a vapor n una cald ra y d pu d xpand contra un pi tn. D pu qu l vapor cond n a con agua fra, r gr a a la cald ra y l proc o r pit . En la op racin d cualqui r mquina trmica, xtra una ci rta cantidad d calor d una fu nt a alta t mp ratura, hac algn trabajo m cnico y lib ra o tra cantidad d calor a una fu nt a t mp ratura m baja. R ulta til r pr ntar n forma qu mtica una mquina trmica como mu tra n la figura 15.1. La mquina, r pr ntada por l crculo n l c ntro d l diagrama, ab orb ci rta cantidad d calor QC ( l ubndic C r fi r a cali nt ) tomado d la fu nt a t mp ratura m alta. Hac un trabajo W y lib ra calor QF ( l ubndic F r fi r a fro) a la fu nt d t mp ratura m baja. D bido a qu la u tancia d trabajo ll va a tr av d un ciclo, u n rga int rna inicial y final la mi ma, por lo qu la vari acin d n rga int rna c ro, 430

% $ $ %$ $ $ $ $ %$ % $ $ $ $ % $ % % % % $ $% $ $ $ $ $ $ $ $ % % $ $ $ $% $ $ $ $ % $ $ $ $ $ $ $ $ $ $ $ $% $ $ $ $ % $ $ $ $ $ $% $ $ % $ $ $% $ %$ $ %$ % $% $ $ $ $ $ $ $ $ $% $ $ $% $ $ $ $ $ $ $ $ $ $% $ $ $ $ $ % % $% % $ $ $ $ $ $% $ $% $ $ % %$ % %$ $ $ $ % $ $ $ $ $% $ $ $ $ $ % $ $% $ %$ $ % % % %$ $ $ $ $ $% $ $ %$ % %$ $ % % $ $ $ $ $ $ %$ % % % $ $ % $ % % $ $ $% $ $ $ $ $% % $ $ $ $ $ % $ % % % % $% $ $% $ $ $ % $ $ $% $ $ $ $ $ % $ %$ $ $ $ $ $ %$ $ $ % $ % $% $ $ $ $ $ $ % $ $ $% % % % $ $ $ $ $ %$ $ $ $ $% $ $ $ % % $ $ $ $ $ $ $ $ $ $ $ $ $ $ $ %$ % % $% $ $ $ $ % % % $ $ $ $ $ % % $ $ $ %$ $ $ $ $ $ $

$ $ $ % %

$ % $ $ $

%$

Cap. 15. S gunda l y d la t rmodinmica d cir U = 0. Entonces, de la primera ley de la termodinmica se tiene que el trabajo neto W realizado por la mquina es igual al calor neto que fluye hacia la misma. D e la figura 15.1, el calor eto es Q eto = QC QF, por lo ta to el trabajo es: W = QC QF (15.1) do de QC y QF se toma como ca tidades positivas. Si la susta cia de trabajo es u gas, el trabajo eto realizado e u proceso cclico es igual al rea e cerrada p or la curva que represe ta a tal proceso e el diagrama PV. 15.1.2 Eficie cia trmica. La eficie cia trmica, e (o simpleme te eficie cia), de u a mqui a trmica se defi e como la raz e tre el trabajo eto realizado y el calor absorbido dura te u ciclo, se escribe de la forma:

e= W QC QF Q = 1 F = QC QC QC (15.2) Se puede pe sar e la eficie cia como la raz de lo que se obtie e (trabajo mec ic o) a lo que se paga por (e erga). Este resultado muestra que u a mqui a trmica tie e u a eficie cia de 100% (e = 1) slo si QF = 0, es decir, si o se libera calor a la fue te fra. E otras palabras, u a mqui a trmica co u a eficie cia perfecta de ber co vertir toda la e erga calrica absorbida QC e trabajo mec ico. La segu da ley de la termodi mica, que e seguida a alizamos, establece que esto es imposible. 15.2. SEGUNDA LEY DE LA TERM DINAMICA.

Existe difere tes formas de e u ciar la segu da ley de la termodi mica, pero e su versi ms simple, establece que el calor jams fluye espo t eame te de u objeto fr a u objeto calie te. 431

 

  

  

 

 

 

$ 

$  

 

$ 

Cap. 15. Segu da ley de la termodi mica 15.2.1 Forma de Kelvi Pla ck de la segu da ley de la termodi mica. E la prctica, se e cue tra que todas las mqui as trmicas slo co vierte u a pequea f racci del calor absorbido e trabajo mec ico. Por ejemplo u bue motor de u aut omvil tie e u a eficie cia aproximada de 20% y los motores diesel tie e u a efic ie cia e el ra go de 35% a 40%. E base a este hecho, el e u ciado de Kelvi Pl a ck de la segu da ley de la termodi mica es el siguie te: es imposible co struir u a mqui a trmica que, opera do e u ciclo, o te ga otro e fecto que absorber la e erga trmica de u a fue te y realizar la misma ca tidad de trabajo. Figura 15.1 Represe taci esquemtica de u a mqui a trmica. Figura 15.2 Represe taci esquemtica de u a mqui a trmica imposible de co struir. Figura 15.3 Represe taci esquemtica de u

refrigerador.

Esto es equivale te a afirmar que es imposible co struir u a mqui a de movimie to perpetuo (mvil perpetuo) de segu da clase, es decir, u a mqui a que pudiera violar la segu da ley de la termodi mica. (U a mqui a de movimie to perpetuo de primera c lase es aquella que puede violar la primera ley de la termodi mica (co servaci de la e erga), tambi es imposible co struir u a mqui a de este tipo). La figura 15.2 es u diagrama esquemtico de u a mqui a trmica perfecta imposible de co struir. 432

 

   

 

 

 

 

 

 

  

 

 

Cap. 15. Segu da ley de la termodi mica U refrigerador es u a mqui a trmica que opera e se tido i verso, como se muestra de ma era esquemtica e la figura 15.3. La mqui a absorbe calor QF de la fue te f ra y e trega calor QC a la fue te clida. Esto slo puede ser posible si se hace trab ajo sobre el refrigerador. De la primera ley, se ve que el calor cedido a la fue te calie te debe ser igual a la suma del trabajo realizado y el calor absorbido de la fue te fra. Por lo ta to, se ve que el refrigerador tra sfiere calor del c uerpo ms fro a u cuerpo ms clido (la coci a). 15.2.2 E u ciado de Clausius de la segu da ley de la termodi mica. Resulta deseable co struir u refrigerador que pueda realizar su proceso co el m imo de trabajo. Si se pudiera co struir u o do de el proceso de refrigeraci se realice si i g trabajo, se te dra u refrigerador perfecto. Esto es imposible, porque se violara la segu da ley de la termodi mica, que es el e u ciado de Clausi us de la segu da ley (Rudolf Clausius, alem , 18221888): es imposible co struir u a mqui a cclica, que o te ga otro efecto que tra sferir c alor co ti uame te de u cuerpo hacia otro, que se e cue tre a u a temperatura ms elevada. E trmi os se cillos, el calor o puede fluir espo t eame te de u objeto fro a otr o clido. Este e u ciado de la segu da ley establece la direcci del flujo de calor e tre dos objetos a difere tes temperaturas. El calor slo fluir del cuerpo ms fro a l ms clido si se hace trabajo sobre el sistema. Apare teme te los e u ciados de Ke lvi Pla ck y de Clausius de la segu da ley o est relacio ados, pero so equiva le tes e todos sus aspectos. Se puede demostrar (pero aqu o lo hacemos) que si u e u ciado es falso, el otro tambi lo es.

433

Ejemplo 15.1 a) Calcular la eficie cia de u a mqui a que usa 2000 J de calor dura te la fase de combusti y pierde 1500 J por escape y por fricci . b) Si otra mqui a tie e u a eficie cia de 20% y pierde 3000 J de calor por fricci , calcular el t rabajo que realiza.

 

 

 

 

 

 

 

 

 

 

 

   

 

 

 

 

    

 

 

Cap. 15. Segu da ley de la termodi mica Soluci : a) la eficie cia de u a mqui a esta dada por la ecuaci =1 = 0.25 25%. 2000 QC W Q = 1 F se calcula QC y desQC QC b) usa do la ecuaci 15.2 e la forma e = pus se despeja W.

QF Q 3000 J = 1 e QC = F = = 3750 J QC 1 e 1 0.2 e= W W = eQC = 0.2 3750 J = 75 J QC 15.3 PROCESOS REVERSIBLES E IRREVERSIBLES.

Los procesos reales se producen en una direccin preferente. Es as como el calor fl uye en forma espontnea de un cuerpo ms clido a otro ms fro, pero el proceso inverso s o se puede lograr con alguna influencia externa. Cuando un bloque desliza sobre una superficie, finalmente se detendr. La energa mecnica del bloque se transforma e n energa interna del bloque y de la superficie. Estos procesos unidireccionales s e llaman procesos irreversibles. En general, un proceso es irreversible si el si stema y sus alrededores no pueden regresar a su estado inicial. Por el contrario , un proceso es reversible si su direccin puede invertirse en cualquier punto med iante un cambio infinitesimal en las condiciones externas. Una transformacin reve rsible se realiza mediante una sucesin de estados de equilibrio del sistema con s u entorno y es posible devolver al sistema y su entorno al estado inicial por el mismo camino. Reversibilidad y equilibrio son, por tanto, equivalentes. Si un p roceso real se produce en forma cuasiesttica, es decir lo suficientemente lento c omo para que cada estado se desvi en forma infinitesimal del equilibrio, se puede considerar reversible. En los procesos reversibles, el sistema nunca se desplaz a ms que diferencialmente de su equi434

15.2. e =1 1500 QF

Cap. 15. Segunda ley de la termodinmica librio interno o de su equilibrio con su entorno. Si una transformacin no cumple estas condiciones es irreversible. En la realidad, las transformaciones reversib les no existen, ya que no es posible eliminar por completo efectos disipativos, como la friccin, que produzcan calor o efectos que tiendan a perturbar el equilib rio, como la conduccin de calor por diferencias de temperatura. Por lo tanto no d ebe sorprender que los procesos en la naturaleza sean irreversibles. El concepto de proceso reversible es de especial importancia para establecer el lmite terico de la eficiencia de las mquinas trmicas. 15.4 MAQUINA DE CARNOT. El ciclo de Carnot (Sadi Carnot, francs, 1796 1832), es de gran importancia desde el punto de vista prctico como terico. Carnot demostr que una mquina trmica que oper ara en un ciclo ideal reversible entre dos fuentes de calor, sera la mquina ms efic iente posible. Una mquina ideal de este tipo, llamada mquina de Carnot, establece un lmite superior en la eficiencia de todas las mquinas. Esto significa que el tra bajo neto realizado por una sustancia de trabajo llevada a travs de un ciclo de C arnot, es el mximo posible para una cantidad dada de calor suministrado a la sust ancia de trabajo. El teorema de Carnot se enuncia de la siguiente forma: ninguna mquina trmica real que opera entre dos fuentes de calor, puede ser ms eficie nte que una mquina de Carnot, operando entre las dos mismas fuentes. Para describir el ciclo de Carnot, se debe suponer que la sustancia que trabaja entre las temperaturas TC y TF es un gas ideal contenido en un cilindro con un mb olo mvil en un extremo. Las paredes del cilindro y del mbolo no son conductores trm icos, por lo que no hay prdida de calor al ambiente. El ciclo de Carnot es un pro ceso cclico reversible que utiliza un gas ideal, que consta de dos procesos isotrm icos y de dos procesos adiabticos, como se muestra en la figura 15.4, donde se in dican los cuatro pasos del ciclo. 435

Cap. 15. Segunda ley de la termodinmica Figura 15.4 Diagrama esquemtico del ciclo de Carnot. La representacin grfica del ciclo de Carnot en un diagrama PV se muestra en la fig ura 15.5, donde: 1. El proceso A-B es una expansin isotrmica a la temperatura TC, donde el gas se pone en contacto trmico con una fuente de calor a esa TC. Durante el proceso, el gas absorbe calor QC de la fuente desde la base del cilindro y r ealiza trabajo WAB al subir el mbolo. 2. En el proceso B-C, la base del cilindro se reemplaza por una pared trmicamente no conductora y el gas se expande adiabtica mente. Durante el proceso la temperatura baja de TC a TF y el gas realiza trabaj o WBC al elevar el mbolo. 436

Cap. 15. Segunda ley de la termodinmica 3. En el proceso C-D el gas se coloca en contacto trmico con una fuente de calor a temperatura TF y se comprime isotrmicamente a una temperatura TF. Durante el pr oceso, el gas libera calor QF a la fuente y el trabajo realizado sobre el gas po r un agente externo es WCD. 4. En el proceso final D-A, la base del cilindro se reemplaza por una pared trmicamente no conductora y el gas se comprime adiabticame nte. La temperatura del gas aumenta de TF a TC y el trabajo realizado sobre el g as por un agente externo es WDA. Figura 15.5 Diagrama PV del ciclo de Carnot. Ejemplo 15.2. Calcular la eficiencia de una mquina trmica que opera usando un gas ideal en un ciclo de Carnot. Solucin: Durante la expansin isotrmica A-B en la figur a 15.5, como la temperatura no cambia, la energa interna permanece constante. Por la primera ley, en este proceso el trabajo WAB realizado por el gas es igual al calor absorbido QCal (en este ejemplo QCal y TCal representan el calor y la tem peratura de la fuente clida). Calculando el trabajo, se obtiene: WAB = nRTCal ln VB = QCal VA 437

Cap. 15. Segunda ley de la termodinmica De la misma forma, el calor QF liberado a la fuente fra durante el proceso de com presin isotrmica C-D es igual al valor absoluto del trabajo WCD: WCD = nRTF ln VC = QF VD Dividiendo estas dos expresiones, se obtiene: QF T ln(VC VD ) = F QCal TCal ln(V B VA ) Para cualquier proceso adiabtico cuasiesttico reversible, la temperatura y el volumen se relacionan por la ecuacin TV 1 = constante . Aplican o este resulta o a los procesos a iabticos B C y D A, se obtiene: TCalVB TCalV A 1 = TFVC = TFVD 1 1 1 Divi ien o estas ecuaciones, se obtiene: VB V A 1 V = C V D 1 VB VC = V A VD Este resultado se reemplaza en la ecuacin de QF/QCal, al hacerlo se simplifican l os trminos logartmicos, resultado: QF T = F QCal TCal Ahora se puede calcular la eficiencia de la mquina trmica de Carnot: e =1 F T =1 F Cal TCal 438

Cap. 15. Segun a ley e la termo inmica 15.4.1 Eficiencia e una mquina e Carnot.

El trabajo neto realiza o en el proceso cclico reversible e Carnot es igual al re a encerra a por la trayectoria ABCDA en el iagrama PV e la figura 15.5. Este t rabajo neto es igual al calor neto transferi o al sistema, C QF, ya qu l camb io d n rga int rna c ro. Ad m la fici ncia trmica d una mquina t dada por l a cuacin 15.2: =

W Q = 1 F

C C e los calores

En el ejemplo 15.2 se emostr que para un ciclo e Carnot, la razn F/ C es: F TF = C TC

(15.3) Por lo tanto, la eficiencia trmica eC e una mquina e Carnot, est a a por la expr esin: eC = 1 TF TC (15.4) De acuer o con este resulta o, to as las mquinas e Carnot que operen entre las os mismas temperaturas e manera reversible tienen la misma eficiencia. A partir el ciclo e Carnot, se tiene que la eficiencia e cualquier mquina reversible q ue opere en un ciclo entre os temperaturas, es mayor que la eficiencia e cualq uier mquina irreversible (real) que opere entre las os mismas temperaturas. De a cuer o a este resulta o, la eficiencia es cero si TC = TF. La eficiencia aumenta a me i a que TF isminuye y aumenta TC. La eficiencia slo pue e ser igual a 100% si TF = 0. No es posible tener una fuente fra con esa tempe439

%$ $ $

$ %$

# #

$ $ $ #

# #

$ #

Cap. 15. Segun a ley e la termo inmica ratura, por lo que la eficiencia es siempre menor que la uni a . En la mayora e los casos prcticos, la fuente fra se encuentra a temperatura ambiente. Por lo tant o, se intenta aumentar la eficiencia elevan o la temperatura e la fuente cli a. To as las mquinas reales son menos eficientes que una mquina e Carnot, ya que sie mpre estn presentes ificulta es prcticas como la friccin y las pr i as e calor por con uccin. Ejemplo 15.3. Una mquina e vapor tiene una cal era que opera a 500 K. El calor t ransforma el agua en vapor, el cual mueve un pistn. La temperatura e escape es l a el aire exterior, e unos 300 K. Calcular la eficiencia trmica e esta mquina e vapor. Solucin: la eficiencia trmica mxima e una mquina que opere entre esas temperaturas, es la e Carnot: eC = 1 300 TF =1 = 0.4 40% 500 TC Ejemplo 15.4. La mxima eficiencia terica e un motor e gasolina basa a en un cicl o e Carnot, es e 30%. Si el motor libera sus gases a la atmsfera, a 300 K, calc ular la temperatura el cilin ro inme iatamente espus e la combustin. Si la mquin a absorbe 850 J e calor e la fuente e calor en ca a ciclo, calcular el trabaj o que pue e realizar en ca a ciclo. Solucin: usan o la eficiencia e Carnot para encontrar TC: eC = 1 TC = TF T TC = F TC 1 eC 300 = 429 K 1 0. 3 Para calcular el trabajo se pue e usar la ecuacin: eC = W W = eC QC = 0.3 850 = 2 55 J QC 440

os

Cap. 15. Segunda ley de la termodinmica 15.5 ESCALA DE TEMPERATURA ABSOLUTA. El ciclo de Carnot proporciona una forma de definir una escala de temperaturas q ue sea independiente de las propiedades del material. La ecuacin 15.3 dice que la razn de los calores depende slo de la temperatura de las dos fuentes. La razn de l as dos temperaturas, TF/TC, se puede obtener operando una mquina trmica reversible en un ciclo de Carnot entre esas dos temperaturas y midiendo los calores QC y Q F. Es posible determinar la escala de temperatura con referencia a la temperatur a de algn punto fijo. La escala de temperatura absoluta o Kelvin, se define escog iendo 273.16 K como la temperatura absoluta del punto triple del agua. La temper atura de cualquier sustancia, se puede obtener de la siguiente manera: 1) se hac e que la sustancia recorra un ciclo de Carnot, 2) se mide el calor Q absorbido o liberado por el sistema a cierta temperatura T, 3) se mide el calor Q3 absorbid o o liberado por el sistema cuando se encuentra a la temperatura del punto tripl e del agua. De la ecuacin 15.3 y con este procedimiento, se encuentra que la temp eratura desconocida est dada por: Q Q3 T = 273.16 K (15.5) La escala de temperatura absoluta es idntica a la escala de temperatura de un gas ideal y es independiente de las propiedades del material de trabajo. Por lo tan to puede aplicarse a temperaturas muy bajas. Con esta escala, se define el cero absoluto como la temperatura de una fuente en la cual una mquina de Carnot no lib erar calor alguno. 15.6 BOMBAS DE CALOR Y REFRIGERADORES. Una bomba de calor es un dispositivo mecnico usado en la calefaccin y refrigeracin de casas y edificios. En el modo de calentamiento, un fluido en circulacin absorb e calor del exterior y lo libera en el interior de la estructura. Por lo general , el fluido en circulacin se encuentra en la forma de vapor a baja presin en el em bobinado de la unidad exterior de la estructura, donde ab441

Cap. 15. Segunda ley de la termodinmica sorbe calor, ya sea del aire o del suelo. El gas se comprime y entra hacia la es tructura como vapor caliente a alta presin. En la unidad interior, el gas se cond ensa en lquido y libera su energa interna almacenada. Cuando la bomba de calor se usa como aire acondicionado, el ciclo anterior se opera en forma inversa. La fig ura 15.3 representa tambin un diagrama esquemtico de una bomba de calor, funcionan do en su modo de calefaccin. La temperatura externa es TF, temperatura interna es TC y el calor absorbido por el aire en circulacin es QF. El compresor realiza tr abajo W sobre el fluido y el calor transferido de la bomba de calor (donde dice mquina trmica en la figura 15.3) hacia el interior de la construccin es QC. La efic iencia de una bomba de calor se describe en trminos de un nmero llamado coeficient e de rendimiento, CR, que se define como la razn del calor transferido hacia la f uente de calor y el trabajo realizado para transferir ese calor, en la forma: CR = calor transferido QC = trabajo realizado W (15.6) Normalmente el CR de una bomba de calor es del orden de 4, es decir, el calor tr ansferido hacia la casa es aproximadamente cuatro veces mayor que el trabajo que hace el motor en la bomba de calor. Pero a medida que disminuye la temperatura exterior, se le hace ms difcil a la bomba extraer suficiente calor del aire y el C R disminuye hasta valores menores que uno, y es ms pequeo mientras menor es la tem peratura exterior. Un refrigerador trabaja en forma parecida a una bomba de calo r, donde ste enfra su interior bombeando el calor de los compartimentos para los p roductos hacia el aire exterior ms caliente (figura 15.3). Es un dispositivo cuya finalidad es extraer calor de una fuente fra y cederlo a una fuente caliente. Es to se consigue si se hace trabajo para hacer circular la sustancia refrigerante. En un sistema de refrigeracin tpico, el motor del compresor (ubicado en su parte inferior) introduce la sustancia refrigerante, en estado gaseoso a alta presin, a travs de tubos externos ubicados en la zona posterior (condensador). El gas cede una cantidad de calor QC al ambiente, que es la fuente de alta temperatura 442

Cap. 15. Segunda ley de la termodinmica y se enfra hasta licuarse. Al llegar a la parte superior, el fluido caliente an y a alta presin pasa a los tubos de baja presin, a travs de una vlvula. Estos tubos es tn en el interior. Ah el lquido se evapora, absorbiendo del interior, la fuente fra, una cantidad de calor QF. Luego el fluido regresa al compresor y el ciclo se re inicia. Se extrae calor para enfriar los alimentos y compensar el calor absorbid o por las paredes o la entrada de aire ambiental cada vez que se abre la puerta. Para especificar la calidad del refrigerador se define el coeficiente de rendim iento, CR, como la razn entre el calor absorbido desde la fuente fra y el trabajo hecho por la mquina trmica, en la forma: calor absorbido QF = trabajo realizado W CR = (15.7) Un refrigerador eficiente es aquel que remueve la mayor cantidad de calor de la fuente fra con la menor cantidad de trabajo. Por lo tanto, un buen refrigerador d ebe tener un coeficiente de rendimiento alto, normalmente de 5 o 6. Un refrigera dor imposible tendra un coeficiente de rendimiento infinito. 15.7 ENTROPIA. El concepto de temperatura est comprendido en la ley cero de la termodinmica y el de energa interna en la primera ley. Tanto la temperatura como la energa interna s on funciones de estado. Es decir se pueden utilizar para describir el estado de un sistema. Otra funcin de estado, relacionada con la segunda ley de la termodinmi ca, es la funcin entropa. Para un proceso reversible cuasiesttico entre dos estados de equilibrio, si dQ es el calor absorbido o liberado por el sistema durante al gn intervalo pequeo de la trayectoria, el cambio de entropa, dS, entre dos estados de equilibrio est dado por el calor transferido, dQ, dividido entre la temperatur a absoluta T del sistema, en ese intervalo. Es decir: dQ T dS = (15.8) 443

Cap. 15. Segunda ley de la termodinmica La unidad de medida de la entropa en el SI es J/K. Cuando el sistema absorbe calo r, dQ es positivo y la entropa aumenta. Cuando el sistema libera calor, dQ es neg ativo y la entropa disminuye. La ecuacin 15.8 no define la entropa, sino el cambio de entropa. Al mirar las bellezas de la naturaleza, es fcil reconocer que los even tos de los procesos naturales tienen entre s un gran factor comn. Por ejemplo, el espacio entre los rboles en un bosque es al azar. Si se encuentra un bosque donde todos lo rboles estn igualmente espaciados, es muy probable que se concluya que e l bosque fue plantado por la mano del hombre. De manera similar, las hojas caen al suelo en forma aleatoria. Es muy poco probable encontrar hojas que hayan cado en lneas perfectamente derechas o en montones perfectos. Se pueden expresar estos resultados diciendo que un arreglo desordenado es ms probable que uno ordenado, si se dejan actuar las leyes de la naturaleza sin interferencia. En mecnica estads tica, el comportamiento de una sustancia se describe en trminos del comportamient o estadstico de los tomos y molculas de una sustancia. Uno de los principales resul tados de este tratamiento es que: los sistema aislados tienden al desorden y la e ntropa es una medida de ese desorden Por ejemplo, si todas las molculas de gas en e l aire de una habitacin se movieran juntas en filas, este seria un estado muy ord enado, pero el ms improbable. Si se pudieran ver las molculas, se observara que se mueven azarosamente en todas las direcciones, encontrndose unas con otras, cambia ndo sus velocidades despus de chocar, movindose unas ms rpidas que otras. Este es un estado muy desordenado y el ms probable. Todos los estados fsicos tienden al esta do ms probable y ese siempre es el que tiende a aumentar el desorden. Debido a qu e la entropa es una medida del desorden, una forma alternativa de expresar esto, y otra forma de establecer la segunda ley de la termodinmica es: la entropa del Universo crece en todos los proceso naturales. 444

Cap. 15. Segunda ley de la termodinmica Para calcular el cambio de entropa en un proceso finito, se debe reconocer que en el caso general T no es constante. Si dQ es el calor transferido cuando el sist ema se encuentra a una temperatura T, entonces el cambio de entropa en un proceso reversible cualquiera entre un estado inicial y un estado final es: dQ T S = dS = i f f i (15.9) El cambio de entropa de un sistema para ir de un estado inicial a otro final tien e el mismo valor para todas las trayectorias que conectan a los estados. Es deci r: el cambio en la entropa de un sistema slo depende de las propiedades de los estados de equilibrio inicial y final. E el caso de u proceso reversible y adiabtico, o se tra sfiere calor e tre el sistema y sus alrededores, y por lo ta to, e este caso S = 0. Como no hay cambio en la entropa, un proceso adiabtico tambin se conoce como un proceso isentrpico (de igual entropa). En un ciclo de Carnot, la mquina absorbe calor QC de la fuente a alta temperatura TC y libera calor QF la fuente de baja temperatura TF. Entonces , el cambio total de la entropa en un ciclo es: S = QC QF TC TF El signo negativo en el segun o trmino representa el hecho e que el calor F es libera o por el sistema. Como en el ejemplo 15.2 se emostr que para un ciclo e Carnot se cumple la relacin: F TF = C TC

combinan o estas os ltimas ecuaciones, se encuentra que el cambio total e entro pa para una mquina que opera en un ciclo e Carnot es cero, es ecir: 445

Cap. 15. Segun a ley e la termo inmica S = 0 En general, como la entropa es una funcin de estado, y slo depende de las propiedad es de cierto estado de equilibrio, se concluye que para cualquier ciclo reversib le, S = 0. Otra propiedad importante de la entropa es: la entropa del Universo permanece constante en los procesos reversibles. 15.7.1 E tropa e u proceso reversible de u gas ideal.

Como para u gas ideal, dU = CVdT y p = ferido como: dQ = CV dT + RT dV V

RT/V, se puede expresar el calor tra s

Ahora, dividie do cada trmi o e tre T, se puede escribir: dQ dT dV = CV + R T T V Supo ie do que CV es co sta te, se puede i tegrar la ecuaci a terior desde estad o i icial Ti, Vi hasta el estado fi al Tf, Vf, se obtie e: T V dQ = CV l f + R l f T Ti Vi S = f i (15.10) 446

 

  

 

 

     

Como l e . E o fi dQ =

caso especial, se describir como calcular el cambio de e tropa de u gas idea u proceso reversible cuasiesttico e el cual se absorbe calor de u a fue te este proceso, se lleva u gas desde u estado i icial Ti, Vi hasta u estad al Tf, Vf. De acuerdo co la primera ley: dU + dW = dU + pdV

 

Cap. 15. Segunda ley de la termodinmica Esta expresin muestra que S depende slo de los estados inicial y final y que es ind ependiente de la trayectoria reversible. Adems S puede ser negativo o positivo dep endiendo de cuando el gas absorbe o libera calor durante el proceso. Por ltimo, p ara un proceso cclico (donde Ti = Tf y Vi,= Vf) se tiene que S = 0. Ejemplo 15.5 U na sustancia slida con un calor latente de fusin Lf se funde a una temperatura Tm. Calcular el cambio de entropa cuando se funden m gramos de la sustancia. Hacer e l clculo si se funden 0.3 kg de plomo a 327 C, de calor de fusin 24.5 kJ/kg. Solucin : suponer que el proceso de fusin se produce lentamente en forma reversible. En e ste caso es posible considerar constante la temperatura Tm. Como el calor latent e de fusin es Q = mLf, reemplazando en la ecuacin 15.7, se obtiene: S = dQ 1 Q mL f = dQ = = T Tm Tm Tm Con los valores numricos: 0.3kg 24.5 103 J / kg S = = 12.25 J K 600 K 15.7.2 Entropa en la conduccin de calor. Considerar la transferencia de calor Q, desde una fuente caliente a la temperatu ra TC hacia una fuente fra que est a la temperatura TF. Como la fuente fra absorbe el calor Q, su entropa aumenta en Q/TF. Al mismo tiempo, la fuente caliente pierd e el calor Q, y su entropa disminuye en Q/TC. El aumento en la entropa de la fuent e fra es mayor que la disminucin de la entropa en la fuente caliente, ya que TF es menor que TC. Por lo tanto, el cambio total en la entropa del sistema es mayor qu e cero y su valor es: 447

Cap. 15. Segunda ley de la termodinmica S = Q Q >0 TF TC (15.11) Ejemplo 15.6 Una fuente fra est a 2 C y una fuente caliente a 127 C. Demostrar que e s imposible que una pequea canti a e energa calrica, por ejemplo e 10 J, pue a s er transferi a es e la fuente fra a la fuente caliente sin isminuir la entropa y en consecuencia violar la segun a ley e la termo inmica. Solucin: se supone que urante la transferencia e calor, las os fuentes no cambian su temperatura. El cambio en la entropa e la fuente caliente es: SC = 10 J Q = = 0.025 J / K TC 400 K La fuente fra pierde calor y su cambio de entropa es: S F = Q 10 J = = 0.036 J / K TF 275 K El cambio total e entropa es: S = SC + SF = 0.025 - 0.036 = - 0.011 J/K Esto es una contradiccin al concepto de que la entropa del universo siempre aument a en los procesos naturales. Es decir, la transferencia espontnea de calor de un objeto fro a un objeto caliente no puede ocurrir nunca jams. 15.7.3 Entropa en una expansin libre. Considerar un gas ideal en un envase aislado que ocupa inicialmente un volumen V i a la temperatura inicial Ti, en un espacio separado por una divisin (membrana) de otra parte del mismo envase, donde hay otro espacio vaco, como se muestra en l a figura 15.6. En forma repentina se rompe la membrana, de modo que el gas se ex pande irreversiblemente hacia la regin vaca, hasta ocupar un volumen fina Vf. Se c alcular el cambio de entropa del gas. 448

Cap. 15. Segunda ley de la termodinmica Es evidente que el proceso no es reversible ni cuasiesttico. El trabajo realizado por el gas contra el vaco es cero y como el envase est aislado, no hay transferen cia de calor durante la expansin, es decir W = 0 y Q = 0. De la primera ley, se o bserva que el cambio en la energa interna es cero, por lo tanto Ui = Uf. Como el gas es ideal, U depende slo de la temperatura, por lo que se puede concluir que T i = Tf. Figura 15.6. Expansin libre de un gas dentro de un envase aislado. Como el proceso es irreversible, no se puede usar directamente la ecuacin 15.8 pa ra calcular el cambio de entropa. Para hacer su clculo, hay que imaginar un proces o reversible entre los mismos estados inicial y final. Uno simple que se puede e legir, es una expansin isotrmica reversible en la cual el gas empuja lentamente a un mbolo. Ya que T es constante en ese proceso, de la ecuacin 15.9 se obtiene: S = f i dQ 1 = T T f i dQ Pero la integral de dQ es simplemente el trabajo realizado por el gas durante la expansin isotrmica desde Vi hasta Vf, que est dado por la ecuacin 13.8. Usando ese resultado, se encuentra que: S = nR ln Vf Vi (15.12) 449

Cap. 15. Segunda ley de la termodinmica Como Vf > Vi, se concluye que S es positivo y tanto la entropa como el desorden de l gas aumentan por efecto de la expansin adiabtica. Estos resultados tambin se pued en obtener de la ecuacin 15.10, observando que Ti = Tf, por lo tanto lnTf /Ti = l n 1 = 0. Ejemplo 15.7. Calcular el cambio de entropa de 2 moles de un gas ideal q ue realiza una expansin libre al triple de su volumen original. Solucin: aplicando la ecuacin 15.12, con n = 2 moles y Vf =3Vi, S = nR ln Vf Vi ln 3 = 18.3 J S = 2mol 8.31 J molK K 15.7.4 Entropa en la transferencia de calor irreversible. Una sustancia de masa m1, calor especfico c1 y temperatura inicial T1 se pone en contacto trmico con una segunda sustancia de masa m2, calor especfico c2 y tempera tura inicial T2, con T2 > T1. Las dos sustancias estn contenidas en una caja aisl ante de tal manera que no se pierde calor hacia el ambiente. Se permite que el s istema alcance el equilibrio trmico y se quiere calcular el cambio de entropa del sistema. Por la conservacin de la energa, la cantidad de calor Q1 que pierde una s ustancia debe ser igual al calor Q2 que gana la otra sustancia, entonces: S = Tf T dQ dQ1 2 + T T T f 2 T1 donde Tf es la temperatura final de equilibrio del sistema, que se debe calcular . Esta Tf se calcula sabiendo que Q1 = -Q2 y como por definicin Q = mcT para cada sustancia, se obtiene: m1c1 (T f T1 ) = m2 c2 (T f T2 ) 450

Cap. 15. Segun a ley e la termo inmica Despejan o Tf se tiene: Tf = m1c1T1 + m2 c2T2 m1c1 + m2 c2 Ahora, integran o la expresin e S, se obtiene: Tf T1 Tf T2 S = m1c1 ln + m2 c2 ln (15.13) En esta ecuacin, uno de los trminos es positivo y el otro negativo, pero el trmino positivo siempre es mayor que el trmino negativo, dando por resultado un valor po sitivo de S. Entonces la entropa siempre aumenta en los procesos irreversibles. La ecuacin 15.13 es vlida cuando las dos sustancias que se ponen en contacto trmico e ntre s, no se mezclan. Si las sustancias son lquidos y se mezclan, el resultado slo se aplica si los lquidos son idnticos, como en el siguiente ejemplo. Ejemplo 15.8. Un kilo de agua a 0 C se mezcla con una cantidad igual de agua a 10 0 C. Despus de que se alcanza el equilibrio, la mezcla tiene una temperatura unifo rme de 50 C. Calcular el cambio de entropa del sistema. Solucin: el cambio de entro pa se puede calcular con la ecuacin 15.13, usando los valores m1 = m2 = 1kg, c1 = c2 = 4186 J/(kgK), T1 = 273K, T2 = 373K, Tf =323K. S = m1c1 ln Tf T1 + m2 c2 ln Tf T2 S = 1kg 4186 J kgK ln 323 323 ln + 1kg 4186 J kgK 373 273 451

Cap. 15. Segunda ley de la termodinmica S = 704 J K 602 J K = 102 J K El resulta o e este proceso irreversible es que el aumento en la entropa el agu a fra es mayor que la isminucin e la entropa el agua caliente. Los casos escritos muestran que el cambio en la entropa e un sistema siempre es positivo para un proceso irreversible. En general, la entropa total y el esor en siempre aumentan en los procesos irreversibles. De estas consi eraciones, se pue e enun ciar la segun a ley e la termo inmica como sigue: la entropa total e un sistema aisla o que efecta un cambio no pue e isminuir Adems, si el proceso es irreversible, la e tropa total de u sistema aislado siemp re aume ta. Por otra parte, e u proceso reversible, la e tropa total de u sist ema aislado perma ece co sta te. Cua do se trabaja co cuerpos i teractua do que o est aislados, se debe recordar que el sistema se refiere a los cuerpos y sus alrededores. Cua do dos susta cias i teracta e u proceso irreversible, el aum e to de la e tropa de u a parte del sistema es mayor que la dismi uci de la e tro pa de la otra parte. Por lo ta to, se puede co cluir que: el cambio e la e tropa del U iverso debe ser mayor que cero para u proceso irrev ersible e igual a cero para u proceso reversible E el fi , la e tropa del U iverso deber alca zar u valor mximo. E este pu to, el U iverso se e co trar e u estado de temperatura y de sidad u iforme. Todos los procesos fsicos, qumicos y biolgicos termi ar , ya que u estado de desorde perfec to sig ifica que o hay e erga dispo ible para hacer trabajo. Este te ebroso esta do de cosas se co oce como la muerte del calor del U iverso. 452

    

 

 

 

  

 

     

  

 

   

  

 

  

Cap. 15. Segu da ley de la termodi mica PR BLEMAS. 15.1 U a mqui a trmica absorbe 360 J de calor y realiza u trabajo de 25 J e cada cicl o. Calcular: a) la eficie cia de la mqui a, b) el calor liberado e cada ciclo. R : a) 6.94%, b) 335 J. U a mqui a trmica realiza 200 J de trabajo e cada ciclo y t ie e u a eficie cia de 30%. Para cada ciclo de la operaci calcular: a) el calor que absorbe, b) el calor que se libera. U a mqui a trmica tie e u a pote cia de sa lida de 5 kW y u a eficie cia de 25%. Si la mqui a libera 8000 J de calor e cada ciclo, calcular: a) el calor absorbido e cada ciclo, b) el tiempo que tarda e completar cada ciclo. U a mqui a trmica trabaja co u a eficie cia de 32% dura te el vera o, cua do el agua de mar usada para e friamie to est a 20 C. La pla ta ut iliza vapor a 350 C para accio ar las turbi as. Supo ie do que la eficie cia de l a pla ta cambia e la misma proporci que la eficie cia ideal Cul es la eficie cia de la pla ta e i vier o cua do el agua de mar se e cue tra a 10 C? R: 33%. U a c e tral elctrica uclear ge era 1200 MW y tie e u a eficie cia de 30 %. Si se util izara u ro cuyo caudal es 106 kg/s para liberar el exceso de e erga trmica, e cu to variara la temperatura promedio del ro? R: 0.95 K. El calor absorbido por u a mqui a es el triple del trabajo que realiza. a) Cul es su eficie cia trmica?, b) que fra cci del calor absorbido se libera a la fue te fra? R: a) 33.3%, b) 66.7%. U a mqui a co u a eficie cia de 20% se utiliza para acelerar u tre desde el reposo ha sta 5 m/s. Se sabe que u a mqui a ideal (de Car ot) co los mismos depsitos fros y calie te acelerara el mismo tre desde el reposo hasta u a velocidad de 6.5 m/s e mplea do la misma ca tidad de combustible. Si la mqui a emplea aire a 300 K como u depsito fro, e cue tre la temperatura del vapor que sirve como depsito calie te. R: 175 C. 453 15.2 15.3 15.4 15.5 15.6 15.7

 

    

 

 

 

 

 

 

   

  

 

 

  

 

Cap. 15. Segu da ley de la termodi mica 15.8 U a mqui a absorbe 1600 J de u a fue te calie te y libera 1000 J a la fue te fra e cada ciclo. Calcular: a) la eficie cia de la mqui a, b) el trabajo que realiza e cada ciclo, c) la pote cia de salida de la mqui a si cada ciclo dura 0.3s. R: a) 37.5%, b) 600 J, c) 2 kW. U a mqui a trmica opera e tre dos fue tes a temperatu ras de 20 C y de 300 C. Calcular la mxima eficie cia de esta mqui a. 15.9 15.10 La eficie cia de u a mqui a de Car ot es 30%. La mqui a absorbe 800 J de cal or por ciclo de u a fue te calie te a 500 K. Calcular: a) el calor liberado por ciclo, b) la temperatura de la fue te fra. R: a) 560 J, b) 350 K. 15.11 U a mqui a de Car ot tie e u a pote cia de salida de 150 kW. La mqui a opera e tre dos fue tes a temperaturas de 20 C y de 500 C. Calcular: a) la e erga calrica que absorbe po r hora, b) la e erga calrica que pierde por hora. 15.12 Se ha propuesto co struir u a ce tral de e erga que haga uso del gradie te vertical de temperatura del oca o , que opere e tre la temperatura de la superficie, de 20 C, y otra a u a profu di dad de cerca de 1 km, de 5 C. a) Calcular la eficie cia de esa ce tral. b) Si la pote cia de salida de la ce tral es 75 MW, calcular la e erga trmica que se extrae del oca o por hora. c) De acuerdo al resultado de a), pie sa que es posible esta ce tral de e erga? R: a) 5.1%, b) 5.3x1012 J. 15.13 U a mqui a trmica opera e u c iclo de Car ot e tre 80 C y 350 C. Absorbe 20000 J de calor de la fue te calie te por cada ciclo de 1 s de duraci . Calcular: a) la mxima pote cia de salida de esta mqui a, b) el calor liberado e cada ciclo. 15.14 U a de las mqui as ms eficie tes que se ha co struido opera e tre 430 C y 1870 C, co u a eficie cia de 42%. Calc ular: a) su eficie cia terica mxima, b) su pote cia de salida, si absorbe 1.4x105 J de calor cada segu do. R: a) 67.2%, b) 58.8 kW. 15.15 U gas ideal se lleva a travs de u ciclo de Car ot. La expa si isotrmica se produce a 250 C y la compresi isotrmica se produce a 454

  

 

 

 

 

 

 

 

 

 

 

 

 

   

Cap. 15. Segu da ley de la termodi mica 50 C. Si el gas absorbe 1200 J de calor dura te la expa si isotrmica, calcular: a) el calor liberado e cada ciclo a la fue te fra, b) el trabajo eto realizado po r el gas e cada ciclo. R: a) 741 J, b) 459 J. 15.16 El motor de u automvil, ope ra co el ciclo mostrado e la figura 15.7, llamado ciclo de tto idealizado. E u cili dro del motor, justo despus de la combusti (estado B), el gas est co fi a do a u volume de 50 cm3 y su presi es de 3x106 Pa. E el proceso adiabtico BC, el mbolo se mueve hacia fuera a u volume fi al de 300 cm3 mie tras el gas se ex pa de si perder calor. E el proceso CD el volume perma ece co sta te y la pre si descie de, de modo que e D es la mitad que e C. El proceso DA tambi es adia btico. Si la mezcla aire gasoli a pulverizada se comporta como gas ideal de cv= 5/2R y = 1.4, c lcul r ) l s si uiente r zones de presin: PB/PA, PC/PB, PD/PC y PA/PD, ) l s si uiente r zones de temper tur TB/TA, TC/TB, TD/TC y TA/TD, c) l eficienci del ciclo.

455

15.17 Dos mquin s trmic s tienen eficienci s e1 y e2. L s dos oper n de t l form que el c lor que li er l que tiene eficienci e1 es el c lor de entr d de l que tiene eficienci e2. Demuestre que l eficienci tot l est d d por e = e1 + e2 e1e2. 15.18 Cierto refri er dor que tiene un coeficiente de rendimiento i u l 5 y en c d ciclo sor e 140 J de c lor del depsito fro. C lcul r: ) el tr jo hecho so re l sust nci refri er nte en c d ciclo, ) el c lor li er do h c i el depsito c liente ( m iente).

"

"

"

Fi ur 15.7. Pro lem

15.16

 

 

 

 

  " 

"

 

 

 "

15.19 C lcul r el coeficiente de rendimiento de un refri er dor que oper con un eficienci de C rnot entre l s temper tur s -3 C y 27 C. R: 9. 15.20 C lcul r el coeficiente de rendimiento de un om de c lor que llev c lor del exterior -3 C h ci el interior de un c s 22 C. 15.21 C lcul r el tr jo que se requi ere, us ndo un refri er dor ide l de C rnot, p r remover 1 J de ener c lric de helio 4 K y li er rl l medio m iente de un h it cin 20 C. R: 72.2 J. 15. 22 Un refri er dor ide l es equiv lente un mquin de C rnot que oper l inv ers , donde el c lor QF se sor e de un fuente fr y el c lor QC se li er un fuente c liente. ) Demuestre que el tr jo que T TF . se ebe realizar para que funcione el refrigera or es W = C TF b) Demuestre que el coeficiente e ren imiento el refrigera or i eal TF es CR = . TC TF 15.23 Calcular el cambio e e ntropa cuan o un mol e plata (108 g) se fun e a 961 C. 15.24 Calcular el cambio e entropa cuan o: a) se fun e 1.5 kg e hielo a 1atm, b) se con ensa 1.5 kg e va por a 1atm. 15.25 Una congela ora hermtica tiene una temperatura inicial e 25 C y una presin e 1 atm. El aire se enfra espus hasta 18 C. Calcular el cambio e ent ropa si: a) el volumen se mantiene constante, b) la presin se mantuviera en 1 atm urante to o el enfriamiento. Analizar los resulta os y comparar. 15.26 Una herr a ura e hierro e 0.5 kg se saca e un horno a 1000 C y se sumerge en 4 kg e ag ua a 10 C. Calcular el cambio e entropa total si no se pier e calor al ambiente. R: 735.4 J/K. 15.27 Un trozo e aluminio e 100 g a una temperatura e 125 C se c oloca en lt e agua a 25 C. Calcular el aumento e entropa el sistema cuan o se a lcanza el equilibrio. R: 28 J/K. 456

" 

 #   "   "   

"

C p. 15. Se und

"

ley de l termodinmic

Cap. 15. Segun a ley e la termo inmica 15.28 Una avalancha e nieve con una masa e 100 kg, esliza colina abajo una i stancia vertical e 200 m. Calcular el cambio en la entropa si el aire e la mont aa est a 3 C. R: 7260 J/K. 15.29 Calcular la isminucin en la entropa e 1 mol e he lio, que se enfra a 1 atm es e una temperatura ambiente e 293 K hasta una tempe ratura final e 4 K (cP el helio es 21 J/mol K). 15.30 Calcular el cambio e en tropa cuan o 250 g e agua se calientan es e 20 C hasta 80 C. R: 195 J/K. 15.31 Un envase contiene 500 g e hielo a 0 C. Calcular el cambio e entropa el hielo al escongelarse completamente. 15.32 Calcular el cambio e entropa cuan o un mol e gas i eal monoatmico se calienta cuasiestticamente a volumen constante, e 300 K a 400 K. R: 3.6 J/K. 15.33 Calcular el cambio e entropa cuan o un kg e mercurio , que est al inicio a 100 C se calienta lentamente hasta 100 C. El calor e fusin el mercurio es 1.17x104 J/kg, su temperatura e fusin es 39 C y el calor especfico es 138 J/kg C. 15.34 Un mol e gas i eal monoatmico se lleva a travs el siguiente ciclo: una expansin isotrmica AB es e el punto A(10lt, 5atm) hasta el punto B(50 lt, 1atm), una compresin isobrica BC es e el punto B(50lt, 1atm) hasta el punto C (10lt, 1atm) y un aumento e presin isocoro CA es e el punto C(10lt, 1atm) hasta el punto A(10lt, 5atm). a) Dibujar el ciclo ABCA en el iagrama PV. Calcular: b ) el trabajo neto realiza o por el gas, c) el calor agrega o al gas, ) el calor libera o por el gas, e) la eficiencia el ciclo, f) el cambio e entropa el cic lo. R: b) 4100 J, c) 14200 J, ) 10100 J, e) 28.8 %. 15.35 Las superficies el S ol y e la Tierra estn aproxima amente a 5700 C y 20 C, respectivamente. Calcular e l cambio e entropa cuan o se transfieren 1000 J e energa trmica es e el Sol a la Tierra. 457

Cap. 15. Segun a ley e la termo inmica 15.36 Calcular los cambios e entropa el gas para ca a etapa el ciclo e la fig ura 15.7 y para el ciclo completo. Analizar los resulta os. 15.37 Un auto e 150 0 kg que se mueve a 20 m/s choca contra una pare e concreto. Si la temperatura el aire es 20 C, calcular el cambio e entropa. R: 1020 J/K. 15.38 Un recipiente trmicamente aisla o e 2 litros est ivi i o en os partes iguales (figura 15.6). El la o izquier o contiene 0.044 moles e hi rgeno y el erecho 0.044 moles oxig eno, ambos a temperatura ambiente y presin atmosfrica. Calcular el cambio e entro pa al eliminar la ivisin y ejar que los gases se mezclen. R: 507 J/K. 15.39 Un r ecipiente trmicamente aisla o, e 4.2 litros est ivi i o en os partes, una el o ble que la otra, como muestra la figura 15.8. El la o izquier o contiene hi rgeno y el erecho oxigeno, ambos a temperatura a 0 C y presin atmosfrica. Calcular el c ambio e entropa al eliminar la ivisin y ejar que los gases se mezclen. R: 43.7 J/K. Figura 15.8. Problema 15.39 15.40 Si fluyen 3200 J e calor e una fuente e calor a 500 K a otra fuente e 300 K, a travs e una varilla e metal con uctora, calcular la variacin e la entr opa e a) la fuente caliente, b) la fuente fra, c) la varilla e metal, ) total. 15.41 Un bloque e 2 kg que se mueve con una rapi ez inicial e 5 m/s se esliza sobre una mesa rugosa, hasta etenerse por la friccin. Suponien o que el aire y la mesa estn a la temperatura e 20 C, calcular la variacin e la entropa. R: 0.085 J/K. 458

Cap. 15. Segun a ley e la termo inmica 15.42 Un bloque e hielo e 6 kg a 0 C se eja caer en un lago a 27 C. Justamente espus e que se haya fun i o to o el hielo y justamente antes e que el agua el hielo se haya calenta o, calcular la variacin e la entropa e: a) el hielo, b) e l lago, c) total. R: a) 7340 J/K, b) 6680 J/K, c) 660 J/K. 15.43 Una mquina trmic a cclica opera entre os fuentes a temperaturas e 300 K y e 500 K. En ca a cicl o, la mquina absorbe 700 J e calor e la fuente caliente y realiza un trabajo e 160 J. Calcular la variacin e entropa en ca a ciclo para: a) ca a fuente, b) la mquina, c) total. 15.44 Si se mezclan 200 g e agua a 20 C con 300 g e agua a 75 C , calcular: a) la temperatura final e equilibrio e la mezcla, b) la variacin e entropa el sistema. R: a) 53 C, b) 7.34 J/K. 15.45 Un cubo e hielo e 18 gr a 0 C se calienta hasta que se convierte en vapor. Calcular: a) el aumento e entropa , b) la energa que se requiere para vaporizar el cubo e hielo. 15.46 Una mquina o pera en un ciclo entre las temperaturas 100 C y 180 C y emite 20000 J e calor por ciclo mientras realiza 1500 J e trabajo por ciclo. Compare la eficiencia e es ta mquina con la e una mquina reversible que opera entre las mismas temperaturas. R: 0.4 veces su valor. 459

APENDICES. A. ALGEBRA. Reglas para sumar, restar, multiplicar y ivi ir fraccion es, on e a, b, c y son cuatro nmeros: a c a bc = b b a c ac = b d bd a b ad = c d bc Para multiplicar y dividir potencias, se aplican las siguientes reglas, donde n y m son nmeros y x alguna variable: xn xm = xn+m xn = xnm m x Una potencia fraccionaria correspon e a una raz: x1 n = n x Cualquier canti a xn que es eleva a a una potencia m, es: (x ) Cua ra o e un binomio: Diferencia e cua ra os: n m = x nm Algunas frmulas tiles para factorizar una ecuacin son: (a + b )2 = a 2 + 2ab + b 2 a 2 b 2 = (a + b )(a b ) La forma general e una ecuacin cua rtica es: 461

ax 2 + bx + c = 0 on e x es la canti a esconoci a y a, b y c son factores numricos conoci os com o coeficientes e la ecuacin; tiene os soluciones a as por: x= 2 b b 2 4ac 2a Si b 4ac , las soluciones sern reales. Logaritmos. Si la variable x se expresa co mo potencia e una canti a a, e la forma x = ay el nmero a se llama base. El logaritmo e x con respecto a la base a es igual al exponente al cual se ebe elevar la base, que se escribe como: y = log a x En la prctica, las os bases mas usa as son la base 10, llama a logaritmo comn, y la base e = 2.718..., llama a logaritmo natural. Para el logaritmo comn y natural se utiliza respectivamente las notaciones: y = log x x = 10 y y = ln x x = e y Algunas propie a es e los logaritmos son las siguientes: log(xy) = log x + log y log(x/y) = log x log y log(xn) = n log x log 1 = ln 1 = 0 ln e = 1 ln ea = a 462

B. GEOMETRA La istancia x 2 , y 2 ) es:

entre os puntos cuyas coor ena as son ( x1 , y1 ) y (

= ( x 2 x1 ) 2 + ( y 2 y1 ) 2 Para calcular el ngulo en ra ianes, se sabe que la longitu el arco s (Fig. B.1) es proporcional al ra io r, para el valor e medido en radianes. s = r = s r Figura B.1 La ecuacin de una lnea recta (Fig. B.2) est dada por y = mx + b , donde b es la int erseccin con y y m la pendiente de la recta. La ecuacin de un crculo de radio R cen trado en el origen es: x + y = R 2 2 2 x2 y2 La ecuacin de una elipse con el origen como su centro (Fig. B3) es: 2 + 2 = 1 , donde a b a es la longitud del semieje mayor y b es la longitud del semieje menor. Figura B.2 Figura B.3 463

La ecuacin de la parbola cuyo vrtice est en y = b (Fig. B.4) es: y = ax + b . La ecu acin de una hiprbola rectangular (Fig. B.5) es: xy = cte 2 Figura B.4 Areas y volmenes. Forma Rectngulo lados a y b Circun erencia de radio r Tringulo base b, altura h Caja rectangular lados a, b, c Cilindro largo h, radio r Es era radio r Figura B.5 a b 2 1 bh 2 2(ab + bc + ca) 2( 2 + h) 4 2 A ea Volumen a b c 2 h 4 3 3 464

C. TRIGONOMETRA. La a te de las matemticas que se basa en las o iedades es ecia les de los t ingulos ectngulos se llama t igonomet a. Po definicin, un t ingulo ec to es el que contiene un ngulo de 90. Consid ese el t ingulo ecto de la figu a C.1, donde el lado a es o uesto al ngulo , el lado b es adjunto al ngulo y el lado c es la hipotenusa del tringulo. Las tres unciones trigonomtricas bsicas de inidas par a tales tringulos son las unciones seno (sen), coseno (cos) y tangente (tan). En trminos del ngulo , estas unciones se de inen por: sen la o opuesto a a = hipotenusa c lado adyacente a b = hipotenusa c lado opuesto a a = lado adyacente a b cos tan Figura C.1 El teorema ulo: e Pitgoras a la siguiente relacin entre los la os e un tringulo rectng

c2 = a2 + b2 De las efiniciones anteriores y el teorema e Pitgoras, se sigue que: sen 2 + cos 2 = 1 tan = sen cos Las unciones cotangente, secante y cosecante estn de inidas directamente de un t ringulo recto mostrado en la igura C.1 como: cot 1 tan sec 1 cos csc 1 sen

465

Algunas de las propiedades de las

unciones trigonomtricas son las siguientes:

sen = cos(90 ) sen( ) = sen cos = sen(90 ) , cot = tan(90 ) cos( ) = cos tan( ) = tan Las siguientes relaciones se aplican a cual uier tringulo, como el de la igura C .2: a + + = 180 Fi ur C.2 Ley de los cosenos

Ley de los senos 466

2 = 2 + c 2 2bc cos 2 2 2 = c = = sen sen sen

+ c 2ac cos 2 2 2 c =

"

2ab cos

L t l C.1 list l un s identid des tri onomtric s tiles. T l C.1 Al un s ide ntid des tri onomtric s. sen2 = 2 sen cos cos2 = cos2 sen2 2 tan 1 tan 2 1 sen 2 = (1 cos ) 2 2 tan 2 = sec = 1 + tan 2 2 tan cos 2 2 = 2 1 cos 1 + cos 1 (1 + cos ) 2 = cos A + cos B = 2 cos 1 ( A + B ) cos 1 ( A B ) 2 2 cos A cos B = 2sen 1 ( A + B )sen 1 (B A) 2 2

csc2 = 1 + cot2 467

sen( A B ) = senA cos B cos AsenB cos( A 2 sen 1 ( A B ) cos 1 ( A m B ) 2 2

B) = cos A cos B m senAsenB senA

"

"

"

"

senB =

D. DERIVADAS E INTEGRALES. Tabla D.1 Derivadas de algunas unciones. Nota: Las l etras a y n son constantes. d (a) = 0 dx d (ax n ) = nax n1 dx d ax (e ) = ae ax dx d (sin ax) = a cos ax dx d (cos ax) = asenax dx d x 1 arcsen = dx a a2 x2 d x a arctan = 2 dx a a + x2 d (u v) dv du =u +v dx dx dx d (tan ax) = a sec 2 ax dx d (cot ax) = a csc 2 ax dx d (sec x) = tan x sec x dx d (csc x) = cot x csc x dx d a (ln ax) = dx x m1 d x arccos = dx a a2 x2 d x a a rc cot = 2 dx a a + x2 d (u v) 1 du u dv = dx v dx v 2 dx Tabla D.2 Algunas integrales inde inidas. Nota: Las letras a, b, c y n son const antes. Una constante arbitraria se debe sumar a cada una de estas integrales. dx = x x n dx = x n +1 n +1 , n 1 dx 2 x = x dx 1 x2 = x x

( ) 32

e x dx = e x a2 x2 dx 1 x a 2 + x 2 = a arctan a 1 senaxdx = a cos ax 468 x x = arccos a a

= x2

1 2 x

a2 3

a2 = arcsen

x 2 2 2

a 2 dx = xdx x a dx

1 ax e a e ax ax xe dx = a 2 (ax 1) a x a dx = log a e ax dx = cos axdx = a senax tan axdx = a ln(cos ax) = a ln(sec ax) 1 1 1 cot axdx = a ln(senax) 1 ln axdx dx = ln x 1 a+x 2 2 a 2 = ln = x x

x ln ax x dx 1 a + bx = b ln(a + bx ) dx x 1 cx a + be cx = a ac ln a + be dx a 2 x 2 = 2a ln a x , a x > 0 xdx 1 2 2 a 2 x 2 = 2 ln a x dx 2 2 + x a

sec axdx = a ln(sec ax + tan ax) csc axdx = a ln(csc ax cot ax) x sen2ax 2 4a x sen2ax 2 cos ax x = 2 + 4a x 1 sen 2 ax = a cot ax x 1 cos 2 a x = a tan ax 1 1 sen 2 ax x = ( ) ( ) tan 2 ax x = 1 tan ax x a (a + bx )2 x = 1 b(a + bx )

cot 2 1 ax x = cot ax x a 1 a 2 x 2 a 1 a 2 x 2 a x x a2 x2 = a2 x2 1 2 a x2 3 arcsin ax x = x arcsin ax + 2 2 x a x x = ( ) 32 arccos ax x = x arccos ax 469

E. DATOS COMUNES EN EL SISTEMA SOLAR Y TERRESTRE. Tabla E1. Datos el sistema so lar. Cuerpo Mercurio Venus Tierra Marte Jpiter Saturno Urano Neptuno Plutn Luna Sol Mas a (kg) Ra io prom. (m) 3.18 x 10 4.88 x 1024 5.98 x 1024 6.42 x 1023 1.90 x 1027 5.68 x 1026 8.68 x 102 5 1.03 x 1026 1.4 x 1022 7.36 x 1022 1.99 x 1030 23 Perio o (s) 7.60 x 10 1.94 x 107 3.156 x 107 5.94 x 107 3.74 x 108 9.35 x 108 2.64 x 109 5.2 2 x 109 7.82 x 109 6 Distancia al Sol (m) 5.79 x 1010 1.08 x 1011 1.496 x 1011 2.28 x 1011 7.78 x 1011 1.43 x 1012 2.87 x 1012 4.50 x 1012 5.91 x 1012 2.43 x 10 6.06 x 106 6.37 x 106 3.37 x 106 6.99 x 107 5.85 x 107 2.33 x 107 2.21 x 107 1.5 x 106 1.74 x 106 6.96 x 108 6 Tabla E2. Valores e los atos fsicos comnmente utiliza os. Aceleracin ebi a a la grave a Rapi ez e la luz Presin atmosfrica estn ar Densi a el aire (20C y 1 atm) Densi a el agua (20C y 1atm) Rapi ez angular e la Tierr a Inclinacin eje terrestre Distancia prome io Tierra Luna Distancia prome io Tier ra Sol Ra io prome io e la Tierra Masa e la Tierra Masa e la Luna Masa el So l 9.80 m/s2 3 x 108 m/s 1.013 x 105 Pa 1.25 kg/m3 1 x 103 kg/m3 7.27 x 10 5 ra /s 23.5 3.84 x 108 m 1.496 x 1011 m 6.37 x 106 m 5.98 x 1024 kg 7.36 x 1022 kg 1.99 x 10 30 kg 470

1 metro 1 centmetro 1 kilmetro 1 pulga a 1 pie 1 milla Tabla F2. Masa. Kg 1 10 3 14.59 1.66 x 10 27 g 103 1 1.459 x 104 1.66 x 10 24 slug u 6.852 x 10 2 6.024 x 1026 6.852 x 10 5 6.024 x 1023 1 8.789 x 1027 1.137 x 10 28 1 1 kilogramo 1 gramo 1 slug (lb/g) 1 uni a e masa atmica

Tabla F3. Tiempo. s 1 60 3600 8.64 x 104 3.156 x 107 Min 1.667 x 10 2 1 60 1440 5.259 x 105 h 2.77 8 x 10 4 1.667 x 10 2 1 24 8.766 x 103 Da 1.157 x 10 5 6.994 x 10 4 4.167 x 10 2 1 365.2 ao 3.169 x 10 8 1.901 x 10 6 1.141 x 10 4 2.738 x 10 3 1 1 segun o 1 minuto 1 hora 1 a 1 ao 471

F. FACTORES DE CONVERSIN DE UNIDADES m Cm km 1 102 10 3 10 2 1 10 5 3 5 10 3.048x10 4 1609 1.609x105 1.609 pulg 3.281 3.281x10 2 3.281x103 8.333x10 2 78x10 5 1.894x10 4 1

DE MEDIDA. Tabla F1. Longitu . 10 1 2.54x10 2 2.54 2.54x10 5 0.3048 30.48 39.37 0.3937 3.937x104 1 12 6.336x104 pie 1 5280 mi 6.214x10 4 6.214x10 6 0.6214 1.5

Tabla F4. Fuerza. N 1 10 5 4.448 ina 105 1 4.448 x 105 lb 0.2248 2.248 x 10 6 1 1 Ne ton 1 ina 1 libra Tabla F5. Trabajo, energa y calor J erg pie Ib 1 107 0.7376 7 10 1 7.376x10 8 7 1.356 1.356x10 1 1.6x10 19 1.602x 10 12 1.182x10 19 4.186 4.186x107 3.087 3 10 1.06x10 1.055x10 7.779x102 3.6 x 10 6 3.6 x 1013 2.655x106 cal eV 6.242x1018 0.2389 6.242x1011 2.389x10 8 8.464x1018 0.3239 1 3.827x10 20 2.613x1019 1 21 6.585x10 2.520x102 2.247x1025 8.601x105 Bt u 9.481x10 4 9.481x10 11 1.285x10 3 1.519x10 22 3.968x10 3 1 3413x102 kWh 2.778x 10 7 2.778x10 14 3.766x10 7 4.450x10 26 1.163x10 6 2.930x10 4 1 1 joule 1 erg 1 pie Ib 1 eV 1 cal 1 Btu 1 kWh Tabla F6. Presin. dina/cm2 Atm cm Hg Ib/pulg2 Ib/pie2 Pa 1 10 9.869 x 10 6 7.501 x 10 4 1.45 x 10 4 2.089 x 10 2 1 pascal 1 2 10 1 9.869 x 10 7 7.501 x 10 5 1.45 x 10 5 2.089 x 10 3 1 dina/centmetro 1.013 x 105 1.013 x 106 1 76 14.7 2.116 x 103 1 atms era 1.3 33 x 103 1.333 x 104 1.316 x 10 2 1 0.1943 27.85 1 cm de mercurio* 3 4 2 2 6.89 5 x 10 6.895 x 10 6.805 x 10 5.171 1 144 1 libra/pulgada 47.88 4.788 x 102 4.725 x 10 4 3.591 x 10 2 6.944 x 10 3 1 1 libra/pie2 2 * A 0C y en un lugar donde la aceleracin debida a la gravedad sea su valor estndar, 9.80665 m/s . 472

473

G. LETRAS GRIEGAS. Tabla G1. Al abeto Griego . B mm D Eps Ch (J ) K pp L m d Mu Nu Om ron P s Zeta

0( $ $   & ' 

( ) 

You might also like